Case Studies in Infectious Disease [2 ed.] 0367696398, 9780367696399

Case Studies in Infectious Disease presents 40 case studies featuring the most important human infectious diseases world

453 170 74MB

English Pages 416 [431] Year 2023

Report DMCA / Copyright

DOWNLOAD FILE

Polecaj historie

Case Studies in Infectious Disease [2 ed.]
 0367696398, 9780367696399

Table of contents :
Cover
Half Title
Title
Copyright
Table of Contents
Preface
Author Affiliations
Acknowledgements
Pathogens by Type and Body Systems Affected
Digital Resources
Case 1 Aspergillus fumigatus
Case 2 Borrelia burgdorferi and related species
Case 3 Campylobacter jejuni
Case 4 Candida albicans
Case 5 Chlamydia trachomatis
Case 6 Clostridioides difficile
Case 7 Cytomegalovirus
Case 8 Echinococcus spp.
Case 9 Enteroviruses
Case 10 Epstein-Barr virus
Case 11 Escherichia coli
Case 12 Giardia intestinalis
Case 13 Helicobacter pylori
Case 14 Hepatitis B virus
Case 15 Hepatitis C virus
Case 16 Herpes simplex virus
Case 17 Histoplasma capsulatum
Case 18 Human immunodeficiency virus
Case 19 Influenza virus
Case 20 Leishmania spp.
Case 21 Leptospira spp.
Case 22 Listeria monocytogenes
Case 23 Mycobacterium leprae
Case 24 Mycobacterium tuberculosis
Case 25 Neisseria gonorrhoeae
Case 26 Neisseria meningitidis
Case 27 Norovirus
Case 28 Plasmodium spp.
Case 29 Respiratory syncytial virus
Case 30 Rickettsia spp.
Case 31 Salmonella typhi
Case 32 Schistosoma spp.
Case 33 Staphylococcus aureus
Case 34 Streptococcus pneumoniae
Case 35 Streptococcus pyogenes
Case 36 Toxoplasma gondii
Case 37 Trichophyton spp. (Dermatophytes)
Case 38 Trypanosoma spp.
Case 39 Varicella-zoster virus
Case 40 Wuchereria bancrofti
Glossary
Index

Citation preview

Case Studies in

Infectious Disease

9780367696399_Lydyard.indb 1

13/02/2023 07:31

Case Studies in

Infectious Disease

Second Edition

Peter M Lydyard

Michael F Cole

John Holton

William L Irving

Nina Porakishvili

Pradhib Venkatesan

Katherine N Ward

9780367696399_Lydyard.indb 3

13/02/2023 07:32

Second edition published 2023 by CRC Press 6000 Broken Sound Parkway NW, Suite 300, Boca Raton, FL 33487-2742 and by CRC Press 4 Park Square, Milton Park, Abingdon, Oxon, OX14 4RN CRC Press is an imprint of Taylor & Francis Group, LLC © 2023 Peter M Lydyard, Michael F Cole, John Holton, William L Irving, Nina Porakishvili, Pradhib Venkatesan and Katherine N Ward This book contains information obtained from authentic and highly regarded sources. While all reasonable efforts have been made to publish reliable data and information, neither the author[s] nor the publisher can accept any legal responsibility or liability for any errors or omissions that may be made. The publishers wish to make clear that any views or opinions expressed in this book by individual editors, authors or contributors are personal to them and do not necessarily reflect the views/opinions of the publishers. The information or guidance contained in this book is intended for use by medical, scientific or health-care professionals and is provided strictly as a supplement to the medical or other professional’s own judgement, their knowledge of the patient’s medical history, relevant manufacturer’s instructions and the appropriate best practice guidelines. Because of the rapid advances in medical science, any information or advice on dosages, procedures or diagnoses should be independently verified. The reader is strongly urged to consult the relevant national drug formulary and the drug companies’ and device or material manufacturers’ printed instructions, and their websites, before administering or utilizing any of the drugs, devices or materials mentioned in this book. This book does not indicate whether a particular treatment is appropriate or suitable for a particular individual. Ultimately it is the sole responsibility of the medical professional to make his or her own professional judgements, so as to advise and treat patients appropriately. The authors and publishers have also attempted to trace the copyright holders of all material reproduced in this publication and apologize to copyright holders if permission to publish in this form has not been obtained. If any copyright material has not been acknowledged please write and let us know so we may rectify in any future reprint. Except as permitted under U.S. Copyright Law, no part of this book may be reprinted, reproduced, transmitted, or utilized in any form by any electronic, mechanical, or other means, now known or hereafter invented, including photocopying, microfilming, and recording, or in any information storage or retrieval system, without written permission from the publishers. For permission to photocopy or use material electronically from this work, access www.copyright.com or contact the Copyright Clearance Center, Inc. (CCC), 222 Rosewood Drive, Danvers, MA 01923, 978-750-8400. For works that are not available on CCC please contact [email protected] Trademark notice: Product or corporate names may be trademarks or registered trademarks and are used only for identification and explanation without intent to infringe. ISBN: 978-0-367-72587-7 (hbk) ISBN: 978-0-367-69639-9 (pbk) ISBN: 978-1-003-15544-7 (ebk) DOI: 437827 Typesetting by Evolution Design and Digital Ltd

Instructor & Student Resources can be accessed at: www.routledge.com/cw/lydyard

9780367696399_Lydyard.indb 4

13/02/2023 07:32

Table of Contents

Preface

Author Affiliations Acknowledgements Pathogens by Type and Body Systems Affected Digital Resources

ix xiii

xv

xxii

xix

Case 1

Aspergillus fumigatus

Case 2

Borrelia burgdorferi and related species

13

Case 3

Campylobacter jejuni

21

Case 4

Candida albicans

27

Case 5

Chlamydia trachomatis

37

Case 6

Clostridioides difficile

51

Case 7

Cytomegalovirus

59

Case 8

Echinococcus spp.

Case 9

Enteroviruses

Case 10

Epstein-Barr virus

Case 11

Escherichia coli

Case 12

Giardia intestinalis

105

Case 13

Helicobacter pylori

115

Case 14

Hepatitis B virus

127

Case 15

Hepatitis C virus

137

Case 16

Herpes simplex virus

145

Case 17

Histoplasma capsulatum

155

Case 18

Human immunodeficiency virus

167

Case 19

Influenza virus

Case 20

Leishmania spp.

Case 21

Leptospira spp.

201

Case 22

Listeria monocytogenes

209

Case 23

Mycobacterium leprae

217

Case 24

Mycobacterium tuberculosis

227

Case 25

Neisseria gonorrhoeae

239

Case 26

Neisseria meningitidis

247

Case 27

Norovirus

257

Case 28

Plasmodium spp.

267

Case 29

Respiratory syncytial virus

277

Case 30

Rickettsia spp.

285

1

73

79

85

97

181

193

v

9780367696399_Lydyard.indb 5

13/02/2023 07:32

Table of Contents

Case 31

Salmonella typhi

295

Case 32

Schistosoma spp.

303

Case 33

Staphylococcus aureus

311

Case 34

Streptococcus pneumoniae

321

Case 35

Streptococcus pyogenes

329

Case 36

Toxoplasma gondii

339

Case 37

Trichophyton spp. (Dermatophytes)

347

Case 38

Trypanosoma spp.

355

Case 39

Varicella-zoster virus

365

Case 40

Wuchereria bancrofti

373

Glossary Index

393

425

Additional cases are available for download at www.routledge.com/cw/lydyard

Case e1

Bartonella bacilliformis

Case e2

Brucella

Case e3

Coxiella burnetti

Case e4

Dengue virus

Case e5

Enterococcus faecalis and E. faecium

Case e6

Mycobacterium abscessus

Case e7

SARS-CoV-2

Case e8

Streptococcus mitis

vi

9780367696399_Lydyard.indb 6

13/02/2023 07:32

Preface

Following the SARS-CoV-2 pandemic, microbiology has never been such an important topic to both students and indeed the general public. For this edition, we have updated the 40 cases which appeared in the first edition (2010), having acquired much more information in relation to the five core questions that the students are asked for each case. Again, each case fol­ lows the natural history of the infection from point of entry of the pathogen through pathogenesis, clinical presentation, diagnosis, and treatment. All features of this edition are iden­ tical to those of the first. There are 48 cases, 8 of which are

new, and the remainder thoroughly updated. Because of the size limitation of the book there are 40 cases in the book and 8 available on the website, where they will be regularly updated. As for the previous book, we consider this new edition to pro­ vide invaluable key information for both science courses and particularly for medical students who need instant access to specific medically important organisms. We hope that you enjoy the new edition The authors

vii

9780367696399_Lydyard.indb 7

13/02/2023 07:32

Preface to the First Edition

The idea for this book came from a successful course in a med­ ical school setting. Each of the forty cases has been selected by the authors as being those that cause the most morbidity and mortality worldwide. The cases themselves follow the natural history of infection from point of entry of the pathogen through pathogenesis, clinical presentation, diagnosis, and treatment. We believe that this approach provides the reader with a logical basis for understanding these diverse medically important organisms. Following the description of a case history, the same five sets of core questions are asked to encourage the student to think about infections in a common sequence. The initial set con­ cerns the nature of the infectious agent, how it gains access to the body, what cells are infected, and how the organism spreads; the second set asks about host defense mechanisms against the agent and how disease is caused; the third set enquires about the clinical manifestations of the infection and the complications that can occur; the fourth set is related to how the infection is diagnosed, and what is the differential diagnosis; and the fi nal set asks how the infection is man­ aged, and what preventative measures can be taken to avoid the infection.

In order to facilitate the learning process, each case includes summary bullet points, a reference list, a further reading list and some relevant reliable websites. Some of the websites con­ tain images that are referred to in the text. Each chapter con­ cludes with multiple-choice questions for self-testing with the answers given online. In the contents section, diseases are listed alphabetically under the causative agent. A separate table categorizes the patho­ gens as bacterial, viral, protozoal/worm/fungal and acts as a guide to the relative involvement of each body system affected. Finally, there is a comprehensive glossary to allow rapid access to microbiology and medical terms highlighted in bold in the text. All figures are available in JPEG and PowerPoint® format at www.routledge.com/cw/lydyard. We believe that this book would be an excellent textbook for any course in microbiology and in particular for medical stu­ dents who need instant access to key information about specific infect ions. Happy learning!! The authors March, 2009

viii

9780367696399_Lydyard.indb 8

13/02/2023 07:32

Author Affiliations

Peter M Lydyard, Emeritus Professor of Immunology, University College Medical School, London, UK, Honorary Professor of Immunology, School of Life Sciences, University of Westminster, London, UK and Professor of Immunology, University of Georgia, Tbilisi, Georgia.

Michael F Cole, Emeritus Professor of Microbiology and Immunology, Georgetown University Department of Microbiology & Immunology, Washington DC, USA.

John Holton, Department of Natural Sciences, University of Middlesex, London, UK and Department of Pathology, Darrent Valley Hospital, Dartford, UK.

William L Irving, Professor and Honorary Consultant

in Virology, University of Nottingham and Nottingham

University Hospitals NHS Trust, UK.

Nina Porakishvili, Principal Lecturer in Life Sciences,

University of Westminster, London, UK.

Pradhib Venkatesan, Consultant in Infectious Diseases,

Department of Infectious Diseases, Nottingham University

Hospitals City Campus, Nottingham, UK.

Katherine N Ward, Honorary Professor, Division of Infection

and Immunity, University College London, London, UK.

ix

9780367696399_Lydyard.indb 9

13/02/2023 07:32

Acknowledgements

In writing this book we have benefited greatly from the advice of many microbiologists and immunologists. We would like to thank the following for their suggestions in preparing this edition. William R. Abrams (New York University College of Dentistry, USA); Abhijit M. Bal (Crosshouse Hospital, UK); Keith Bodger (University of Liverpool, UK); Carolyn Hovde Bohach (University of Idaho, USA); Robert H. Bonneau (The Pennsylvania State University College of Medicine, USA); Dov L. Boros (Wayne State University, USA); Thomas J. Braciale (University of Virginia Health Systems, USA); Stephen M. Brecher (VA Boston Healthcare System, USA); Patrick J. Brennan (Colorado State University, USA); Christine M. Budke (Texas A&M University, USA); Neal R. Chamberlain (A.T. Still University of Health Sciences/KCOM, USA); Dorothy H. Crawford (University of Edinburgh, UK); Jeremy Derrick (University of Manchester, UK); Joanne Dobbins (Bellarmine University, USA); Michael P. Doyle (University of Georgia, USA); Sean Doyle (National University of Ireland); Gary A. Dykes (Food Science Australia); Stacey Efstathiou (University of Cambridge, UK); Roger Evans (Raigmore Hospital, UK); Ferric C. Fang (University of Washington School of Medicine, USA); Robert William Finberg (University of Massachusetts Medical School, USA); Joanne Flynn (University of Pittsburgh School of Medicine, USA); Scott G. Franzblau (University of Illinois at Chicago, USA); Caroline Attardo Genco (Boston University School of Medicine, USA); Geraldo Gileno de Sá Oliveira (Oswaldo Cruz Foundation, Brazil); John W. Gow (Glasgow Caledonian University, UK); Carlos A. Guerra (University of Oxford, UK); Paul Hagan (University of Glasgow, UK); Anders P. Hakansson (SUNY at Buffalo, USA); Tim J. Harrison (University College London, UK); Robert S. Heyderman (Liverpool School of Tropical Medicine, UK); Geoff Hide (University of Salford, UK); Stuart Hill (Northern Illinois University, USA); Stephen Hogg (University of Newcastle, UK); Malcolm J. Horsburgh (University of Liverpool, UK); Michael Hudson (University of North Carolina at Charlotte, USA); Karsten Hueffer (University of Alaska Fairbanks, USA); Paul Humphreys (University of Huddersfield, UK); Ruth Frances Itzhaki (University of Manchester, UK); Prof G. Janossy (University College, London, UK); Aras Kadioglu (University of Leicester, UK); A. V. Karlyshev (Kingston University, UK); Ruth A. Karron (Johns Hopkins University, USA); Stephanie M. Karst (Louisiana State University Health Sciences Center, USA); C. M. Anjam Khan (University of Newcastle, UK); Peter G.E. Kennedy (University of Glasgow, UK); Martin Kenny (University of Bristol, UK); H. Nina Kim (University

of Washington, USA); George Kinghorn (Royal Hallamshire Hospital, UK); Michael Klemba (Virginia Polytechnic Institute and State University, USA); Brent E. Korba (Georgetown University Medical Center, USA); Awewura Kwara (Warren Alpert Medical School of Brown University, USA); Jerika T. Lam (Loma Linda University, USA); Robert A. Lamb (Northwestern University, USA); Audrey Lenhart (Liverpool School of Tropical Medicine, UK); Michael D. Libman (McGill University, Canada); David Lindsay (Virginia Technical University, USA); Dennis Linton (University of Manchester, UK); Martin Llewelyn (Brighton and Sussex Medical School, UK); Diana Lockwood (London School of Hygiene & Tropical Medicine, UK); Francesco A. Mauri (Imperial College, UK); Don McManus (Queensland Institute of Medical Research, Australia); Keith R. Matthews (University of Edinburgh, UK); Ernest Alan Meyer (Oregon Health and Science University, USA); Manuel H. Moro (National Institutes of Health, USA); Kristy Murray (The University of Texas Health Science Center, USA); Tim Paget (The Universities of Kent and Greenwich at Medway, UK); Andrew Pekosz (Johns Hopkins University, USA); Lennart Philipson (Karolinska Institute, Sweden); Gordon Ramage (University of Glasgow, UK); Julie A. Ribes (University of Kentucky, USA); Alan Bernard Rickinson (University of Birmingham, UK); Adam P. Roberts (University College London, UK); Nina Salama (Fred Hutchinson Cancer Research Center and University of Washington, USA); John W. Sixbey (Louisiana State University Health Sciences CenterShreveport, USA); Deborah F. Smith (York Medical School University of York, UK); John S. Spencer (Colorado State University, USA); Richard Stabler (London School of Hygiene & Tropical Medicine, UK); Catherine H. Strohbehn (Iowa State University, USA); Sankar Swaminathan (University of Florida Shands Cancer Center, USA); Clive Sweet (University of Birmingham, UK); Clarence C. Tam (London School of Hygiene & Tropical Medicine, UK); Mark J. Taylor (Liverpool School of Tropical Medicine, UK); Yasmin Thanavala (Roswell Park Cancer Institute, USA); Christian Tschudi (Yale University, USA); Mathew Upton (University of Manchester, UK); Juerg Utzinger (Swiss Tropical Institute, Switzerland); Julio A. Vázquez (National Institute of Microbiology, Institute of Health Carlos III, Spain); Joseph M. Vinetz (University of California, San Diego, USA); J. Scott Weese (University of Guelph, Canada); Lee Wetzler (Boston University School of Medicine, USA); Peter Williams (University of Leicester, UK); Robert Paul Yeo (Durham University, UK); Qijing Zhang (Iowa State University, USA); Shanta M. Zimmer (Emory University School of Medicine, USA). x

9780367696399_Lydyard.indb 10

13/02/2023 07:32

Pathogens by Type and Body Systems Affected

Guide to the relative involvement of each body system affected by the infectious organisms described in this book: the organ­ isms are categorized into bacteria, viruses, and protozoa/fungi/worms.

Organism Bacteria

Resp

MS

GI

H/B

GU

Borrelia burgdorferi 4+

Chlamydia trachomatis

Skin

Syst

1+

1+

L/H

2+ 2+

Clostridium difficile

4+

2+

4+

4+

4+

4+

4+

4+

Helicobacter pylori

4+

4+

Leptospira spp.

4+

Listeria monocytogenes

2+

4+

Mycobacterium leprae Mycobacterium tuberculosis

CV

4+

Campylobacter jejuni

Escherichia coli

CNS

2+

2+

4+ 4+

4+

2+

Neisseria gonorrhoeae

4+

Neisseria meningitidis

2+ 4+

Rickettsia spp.

4+

4+

Salmonella typhi

4+

4+

Staphylococcus aureus

1+

Streptococcus pneumoniae

4+

Streptococcus pyogenes

1+

4+ 4+

2+

1+

4+

1+

4+

3+

4+ 3+

Viruses Coxsackie B virus

1+

Cytomegalovirus

2+

Enteroviruses

1+

1+

4+ 3+

2+

1+

1+

1+

4+

2+ 1+

3+

Epstein-Barr virus

Hepatitis B virus

4+

Hepatitis C virus

4+

Herpes simplex virus

4+

Human immunodeficiency virus

Influenza virus

4+ 1+

Norovirus

4+

2+

4+

4+

2+ 4+

2+

1+

4+

Parvovirus

2+

Respiratory syncytial virus

4+

Varicella-zoster virus

2+

3+

4+ 2+

2+

4+

xi

9780367696399_Lydyard.indb 11

13/02/2023 07:32

Pathogens by Type and Body Systems Affected

Protozoa/Fungi/Worms

Resp

Aspergillus fumigatus

4+

Candida albicans

1+

Echinococcus spp.

2+

MS

H/B

2+

GU

CNS

2+

CV

1+

Skin

Svst

1+

2+

4+*

1+

L/H

4+

Giardia intestinalis Histoplasma capsulatum

GI

4+

1+

3+

4+

Leishmania spp.

4+

Plasmodium spp.

4+

4+

Schistosoma spp.

4+

4+

4+ 4+

Trichophyton spp.

4+

Toxoplasma gondii

2+

Trypanosoma spp.

4+

4+

4+

4+

Wuchereria bancrofti

4+

Online Cases Organism

Resp

MS

GI

H/B

GU

CNS

Bartonella bacilliformis Brucella Coxiella burnetti

2+

Skin

3+

3+

Svst

L/H

2+

3+

1+

4+

Dengue virus

CV

4+

2+

1+

Enterococcus faecalis and E. faecium

3+

Mycobacterium abscessus

3+

2+

SARS-CoV-2

4+

1+

1+

Streptococcus mitis

2+

3+

1+

1+

1+ 1+

4+

The rating system (+4 the strongest, +1 the weakest) indicates the greater to lesser involvement of the body system. KEY:

Resp: Respiratory; MS: Musculoskeletal; GI: Gastrointestinal;

H/B: Hepatobiliary; GU: Genitourinary; CNS: Central Nervous System;

CV: Cardiovascular; Skin: Dermatological; Syst: Systemic; L/H: Lymphatic-Hematological; *: + mucosa

xii

9780367696399_Lydyard.indb 12

13/02/2023 07:32

Digital Resources

There are three digital resources that accompany this book. • Eight additional case studies. So that they can be regu­ larly updated, and to keep the size of the book reasonable, eight case studies are available to students as a digital resource, which can be downloaded at www.routledge. com/cw/lydyard. ⚬ Bartonella bacilliformis ⚬ Brucella ⚬ Coxiella burnetti ⚬ Dengue virus ⚬ Enterococcus faecalis and E. faecium ⚬ Mycobacterium abscessus

• Multiple choice questions. Each case study has a range of interactive MCQs to test understanding of the case. Answers are also provided enabling the reader to self-test. Students can access these MCQs by visiting www.routledge.com/cw/lydyard. • Figure slides – A library of all the figures from the book for instructors to use in presentations and lectures. Figures are available in two formats: PowerPoint and PDF. Instructors wishing to access this library of figures must register at the Instructor Resources Download Hub: w w w.routledgetextbooks.com/textbooks/instructor_ downloads/.

⚬ SARS-CoV-2 ⚬ Streptococcus mitis

xiii

9780367696399_Lydyard.indb 13

13/02/2023 07:32

1

Aspergillus fumigatus

A 68-year-old Caucasian man was diagnosed with B-cell chronic lymphocytic leukemia (B-CLL) and received chemo- and immuno­ therapy and attended the CLL clinic regularly. Ten years later, the patient presented with pneumonia symptoms and was examined by chest CT scan. The results were suggestive of aspergillosis and additional laboratory tests were performed. Positive Aspergillus serology allowed the doctors in the clinic to give a diagnosis of A. fumigatus pneumonia. The patient was not neutropenic and his condition improved following an 8-month course of itraconazole followed by voriconazole for 6 months. Two years later, the patient was diagnosed with pulmonary aspergillosis. The diagnosis was based on a CT scan, cytology results, and a history of prior infection (Figure 1.1). He was treated with amphotericin B, monitored by radiography, but he died 2 days later. An autopsy was performed and the diagnosis of invasive pulmonary aspergillosis was confirmed.

1. WHAT IS THE CAUSATIVE AGENT, HOW DOES IT ENTER THE BODY AND HOW DOES IT SPREAD A) WITHIN THE BODY AND B) FROM PERSON TO PERSON?

Figure 1.1 Chest X-ray showing that the fungus has invaded the lung tissue. There is a large cavity in the upper left lobe of the lung, with a fungus ball within the cavity. From Sovereign, ISM / Science Photo Library. With permission.

β(1–4)-glucan) leading to the establishment of a rigid cell wall. Glycosyltransferases bound to the membrane by a glycosylphosphatidyl inositol (GPI) anchor play a major role in the biosynthesis of the cell wall. Fungal cell composition affects its virulence and susceptibility to immune responses.

CAUSATIVE AGENT

Aspergillosis is caused by Aspergillus, a saprophytic, fi lamentous fungus found in soil, decaying vegetation, hay, stored grain, compost piles, mulches, sewage facilities, and bird excreta. It is also found in water storage tanks (for example in hospitals), fi re-proofi ng materials, bedding, pillows, ventilation and air conditioning, and computer fans. It is a frequent contaminant of laboratory media and clinical specimens and can even grow in disinfectants! Although Aspergillus is not the most abundant fungus in the world, it is one of the most ubiquitous. There are more than 100 species of Aspergillus. Although about 10 000 genes have been identified in the Aspergillus genome, none of the gene sets is shared with other fungal pathogens. The cell wall of A. fumigatus contains various polysaccharides (Figure 1.2). Newly synthesized β(1–3)­ glucans are modified and associated to the other cell wall polysaccharides (chitin, galactomannan (GM), and β(1–3)-,

Figure 1.2 Three-dimensional schematic representation of the Aspergillus fumigatus cell wall.

1

9780367696399_Lydyard.indb 1

13/02/2023 07:32

Case Studies in Infectious Disease

Of more than 100 species of Aspergillus, only a few are pathogenic, most of all A. fumigatus, but other species, including A. niger, A. terreus, A. flavus, A. clavatus, A. nidulans, and A. oryzae might also contribute to pulmonary allergic disorders. However, other than A. fumigatus, Aspergillus species do not normally grow at temperatures higher than 37°C, and therefore do not cause invasive disease (see Section 3). A. nidulans can cause occasional infections in children with chronic granulomatous disease. A. fumigatus is a primary pathogen of man and animals. It is characterized by thermotolerance: ability to grow at temperatures ranging from 15°C to 55°C, it can even survive temperatures of up to 75°C. This is a key feature for A. fumigatus, which allows it to grow over other aspergilli species and within the mammalian respiratory system. A. fumigatus is a fast grower. It reproduces by tiny spores known as conidia, formed on specialized conidiophores. A. fumigatus sporulates abundantly, with every conidial head producing thousands of conidia. The conidia released into the atmosphere ranges from 2–5μm in diameter and are small enough to fit in the lung alveoli. The spores are released through disturbances of the environment and air currents and become airborne both indoors and outdoors since their small size makes them buoyant. A. fumigatus can be identified by the morphology of the conidia and conidiophores: green-blue echinulate conidia are produced in chains basipetally from greenish phialides (Figure 1.3). A few isolates are nonpigmented and produce white conidia. A. fumigatus strains with colored conidia indicate the presence of accumulated metabolites and are more virulent than nonpigmented strains since they give the fungus an advantage to adapt to its environment.

ENTRY AND SPREAD WITHIN THE BODY

2

We normally inhale 100–200 Aspergillus conidial spores daily, but only susceptible individuals develop a clinical condition. The spores enter the body via the respiratory tract and lodge in the lungs or sinuses. Once inhaled, spores can reach distal areas of the lung due to their small size. Very rarely other sites of primary infection have been described such as the skin, peritoneum, kidneys, bones, eyes, and gastrointestinal (GI) tract, but these are not clinically important. Usually the invasion of other organs by A. fumigatus is secondary and follows its spread from the respiratory tract. When Aspergillus conidia enter the human respiratory system at body temperature they swell and develop into a different form, thread-like hyphae, which absorb nutrients required for the growth of the fungus. Some enzymes, particularly proteases, are essential for this fungal pathogen to invade the host tissue. Proteases are involved in the digestion of the lung matrix composed of elastin and collagen. In the case of infection of respiratory tissues, this contributes to the pathogenesis (see Section 3). Together, the hyphae can form a dense mycelium in the lungs. However, in the case of healthy immunocompetent individuals, the spores are prevented

9780367696399_Lydyard.indb 2

Figure 1.3 Microscopic morphology of Aspergillus fumigatus showing typical conidial heads. Conidiophores are short, smoothwalled, and have conical-shaped terminal vesicles, which support a single row of phialides. Conidia are produced in basipetal succession forming long chains. They are green and rough-walled to echinulate. Chains of spores can be seen emerging from phialides surrounding the head. Courtesy of Dr David Ellis, University of Adelaide, Australia.

from reaching this stage due to optimal immune responses (see Section 2), and there is some colonization but limited pathology. Due to the increase in aggressive immunosuppressive therapies and AIDS, the number of immunocompromised patients developing aspergillosis has grown significantly. Thus, severe and often fatal invasive infections with A. fumigatus have increased several times in developed countries and it has become the dominant airborne fungal pathogen. Since Aspergillus species are essentially ubiquitous in the environment worldwide, no geographic preference of the exposure to airborne conidia or spores has been reported.

PERSON-TO-PERSON SPREAD This organism is not spread from person to person.

2. WHAT IS THE HOST RESPONSE TO THE INFECTION AND WHAT IS THE DISEASE PATHOGENESIS? When the fungi colonize the respiratory tract, the clinical manifestation and severity of the disease depend on the efficiency of the immune responses, genetic predisposition and genetic polymorphism. In susceptible hosts, Aspergillus conidia germinate to form swollen conidia and then progress to hyphae, its invasive form. The main goal of the immune system is to recognize and kill Aspergillus conidia and to prevent its transition to the hyphal form.

MECHANICAL BARRIERS AND INNATE IMMUNITY In immunocompetent hosts, innate immunity to the inhaled spores begins with the epithelium of the respiratory tract. In larger airways, the epithelial layer contains goblet cells and ciliated cells. Goblet cells produce mucus which traps fungal

13/02/2023 07:32

Case 1: Aspergillus fumigatus

spores, and ciliated cells move the mucus up the airways where it can be coughed up or swallowed. The majority of the conidia are normally removed from the lungs through the ciliary action. However, A. fumigatus can produce toxic metabolites such as gliotoxin, which inhibit ciliary activity, and proteases which can damage the epithelial tissue. The mucociliary function is impaired in individuals with cystic fibrosis (CF) or asthma. In addition, increased viscosity of mucus in patients with CF severely affects ciliary movements. In the smaller airways, Type-II pneumocytes secrete surfactant proteins A and D. These act as opsonins that bind Aspergillus conidia to be targeted by neutrophils and macrophages. Because of the site of the infection by A. fumigatus, bronchoalveolar macrophages – the resident phagocytic cells of the lung, together with recruited neutrophils – are the major cells involved in the phagocytosis of A. fumigatus. While macrophages mostly attack conidia, neutrophils are more important for elimination of developing hyphae. Bronchoalveolar macrophages sense A. fumigatus through pathogen-associated membrane patterns (PAMPs) such as β-glucan, chitin, and GM, the conidia with their pattern recognition receptors (PRRs) including membrane-associated Toll-like receptors (TLR) TLR2 and TLR4. Endosomal TLR3 and TLR9 are also involved, hence their agonists are considered as adjuvants for Allergic Bronchopulmonary Aspergillosis (ABPA) therapy (see later). Inhaled conidia via GM bind some soluble receptors such as pentraxin-3 and lung surfactant protein D. This enhances phagocytosis and inflammatory responses. Phagosomes containing conidia fuse with endosomes followed by activation of NADPH oxidase-dependent killing and production of reactive oxygen species (ROS). Nonoxidative mechanisms are also essential for the digestion of phagocytosed conidia by macrophages. Swelling of the conidia inside the macrophage or in the bronchoalveolar space appears to be a prerequisite for fungal killing. Conidial swelling alters cell-wall composition and exposes fungal β-glucan. This further triggers fungicidal responses via mammalian β-glucan receptor dectin-1. In the cases when resident bronchoalveolar macrophages fail to control the fungus, conidia germinate into hyphae. Neutrophils and monocytes are then recruited from the circulation to phagocytose and kill hyphae. Neutrophils adhere to the surface of the hyphae, since hyphae are too large to be engulfed. They are often seen clustered around fungal hyphae. This triggers a respiratory burst, secretion of ROS, release of lysozyme, neutrophil cationic peptides, and degranulation of neutrophils. NADPH oxidase-independent killing through the release of lactoferrin, an iron-sequestering molecule, is important. In contrast to the slow and subefficient killing of conidia by macrophages, hyphal damage by neutrophils is rapid, possibly through a release of fungal cell-wall glycoproteins with the help of polysaccharide hydrolases. Defensins may also play a role in responses to A. fumigatus hyphae. The importance of neutrophils in protection against Aspergillus is illustrated by a development of invasive aspergillosis (IA) in immunodeficient patients with

9780367696399_Lydyard.indb 3

chemotherapy-induced neutropenia. Corticosteroid-based treatment, purine analogs (fludarabine) and some monoclonal antibody treatment (Campath 1H – anti-CD52) and immunosuppression lead to neutropenia and/or neutrophil dysfunction. Corticosteroids reduce the oxidative burst and superoxide anion release by neutrophils, thereby inhibiting hyphal killing. Platelets also play some role in protection against Aspergillus, by attaching to the cell walls of invasive hyphae and becoming activated, thus enhancing direct cell-wall damage of A. fumigatus and neutrophil-mediated fungicidal effect. Hence thrombocytopenia, which is associated with prolonged neutropenia during chemotherapy, increases the risk of infection by A. fumigatus. Invasion with A. fumigatus enhances the levels of serum fibrinogen, C-reactive protein, and other acutephase proteins. Resting conidia activate the alternative complement pathway and induce neutrophil chemotaxis and deposition of complement components on the fungal surface. Antigen-presenting dendritic cells (DC) exposed to hyphae in the lung migrate to the spleen and draining lymph nodes where they launch peripheral T helper (Th)-cell responses.

ADAPTIVE IMMUNITY T-Cell Responses T-cell responses against A. fumigatus are mostly confined to the CD4+ T cells. To a certain extent, their efficiency resides in the ability of Th1 cells to further enhance neutrophilmediated killing. A Th1 response, associated with a strong cellular immune component and increased levels of IFN- , granulocyte and granulocyte-macrophage colony-stimulating factors (G-CSF and GM-CSF), tumor necrosis factoralpha (TNF- ), interleukin-1 (IL-1), IL-6, IL-12, and IL-18, provides resistance to mycotic disease. Th1 pro-inflammatory signals recruit neutrophils into sites of infection. TNFenhances the capacity of neutrophils to damage hyphae; G-CSF, GM-CSF, and especially IFN- enhance monocyte and neutrophil activity against hyphae; while IL-15 enhances hyphal damage and IL-8 release by neutrophils. IL-8 recruits more neutrophils to sites of inflammation and mediates release of antimicrobial peptides. On the other hand, a Th2 response, which is associated with a minimal cellular component and an increase in antibody production, and secretion of IL-4, IL-5, and IL-10, appears to facilitate fungal invasion. Production of IL-4 by CD4+ T lymphocytes impairs neutrophil antifungal activity and IL-10 suppresses oxidative burst. Pathogenicity of ABPA, (see Section 3) is associated with a pulmonary eosinophilia – the result of production of Th2 cytokines. Aspergillus-specific CD4+ T-cell clones isolated from ABPA patients have a Th2 phenotype. It appears that a balance between beneficial Th1 and damaging Th2 types of immune responses is dependent upon the nature of antigens that prime DCs. Exposure of conidia to DCs leads to the activation of Th1 CD4+ T cells, while priming

3

13/02/2023 07:32

Case Studies in Infectious Disease

of DCs with hyphae enhances Th2-mediated pathways of CD4+ T-cell responses. Regulatory T cells may also be involved in determination of the Th1/Th2 bias. A role of CD8+ T cells in the resistance to A. fumigatus was found to be very limited since fungal gliotoxin suppresses granule exocytosis-associated cellular cytotoxicity.

Antibody Responses Humoral immunity to Aspergillus species is poorly character­ ized. Although even in severely immunocompromised patients the production of specific antibodies has been described, their protective role, if any, remains unclear. The antibody iso­ types produced are IgG1, IgG2, and IgA (particularly in bron­ chial lavage). Immune serum did not enhance phagocytosis of conidia in vitro but did induce macrophage-mediated kill­ ing. Neutralizing antibodies to proteases or toxins may also be beneficial to the host. Serum antibodies to A. fumigatus are often found in the absence of disease as a result of environ­ mental exposure. Serum samples from patients with ABPA (see Section 3) contain elevated levels of antigen-specific circulating antibodies, mainly of IgG and IgE isotypes, which participate in pathogenesis of ABPA. B cells secrete IgE spontaneously as a result of IL-4 production, while IL-5 recruits eosinophils. Eosinophilic infiltration and basophil and mast-cell degranulation in response to A. fumigatus antigens and IgE complex releases pro-inflammatory mediators. This leads to further chemotaxis of eosinophils and activated CD4+ lymphocytes to the site of the infection. Patients with aspergilloma (see Section 3), particularly those who recover from granulocytopenia, have increased levels of specific IgG and IgM, mostly against fungal carbohydrates and glycoproteins. Generally, the efficiency of host immune responses to A. fumigatus is a result of a dynamic interaction between fungal cell wall components and immune cells. Redundancy of host defense mechanisms may lead to the tissue-damaging inflammation favoring the invasive potential of the fungal cells and development of aspergillosis.

Pathogenesis The pathogenicity of A. fumigatus is usually based on (a) a direct attack to the host cells with the pathogen’s virulent factors and toxins; (b) evasion of the immune responses; (c) the hypersensitivity of innate and adaptive responses of patients. Toxins produced by A. fumigatus often represent secondary metabolites of fungi. They can affect the synthesis of DNA, RNA, and proteins, impair cellular functions of the host. Inhibiting phagocytic function of innate cells is critical for the survival of the fungus. Gliotoxin is the most potent toxin produced by A. fumigatus which can suppress T-cell responses, inhibit phagocytosis by macrophages and, most importantly – neutrophils by inhibiting NADPH and ROS production. A. fumigatus is often able to block phagocytosis by producing oxidoreductases and hydrophobic pigments – melanins such

as conidial dihydroxynaphthalene-melanin. Melanins are expressed on the conidial surface and protect the pathogen by quenching ROS. Another strategy of evading innate immune responses by A. fumigatus is production of a specific lipophilic inhibitor of the alternative complement pathway which enhances proteolytic cleavage of C3 complement component bound to conidia. Some of the host defense – such as recently discovered innate lymphoid cells (ILCs), might be involved in pathogenesis of ABPA (see Section 3) by enhancing pulmonary eosinophilia and mucus hypersecretion through the release of Th2 cytokines IL-5 and IL-13. ILC inhibition might reduce Aspergillus-induced inflammation. In patients with ABPA, immediate skin reactions are mediated mainly by type I hypersensitivity and IgE antibody. The late reaction (Arthus reaction) to Aspergillus antigens is the result of IgE-mediated mast-cell activation or immune complex formation (type III hypersensitivity) with complement activation. Immune complexes of specific IgG and A. fumigatus antigens trigger the generation of leukotriene C4 by mast cells which, in turn, promotes mucus production, bronchial constriction, hyperemia, and edema. Granuloma formation in the lung has also been reported since some patients have granulomatous bronchiolitis.

3. WHAT IS THE TYPICAL CLINICAL PRESENTATION AND WHAT COMPLICATIONS CAN OCCUR? The spectrum of pulmonary diseases caused by A. fumigatus is grouped under the name of aspergillosis. These conditions vary in the severity of the course, pathology, and outcome and can be classified according to the site of the disease within the respiratory tract, the extent of fungal invasion or colonization, and the immunological competence of the host. There are four main clinical types of pulmonary aspergillosis: ABPA, chronic necrotizing Aspergillus pneumonia (CNPA), IA, and pulmonary aspergilloma (Figure 1.4).

ALLERGIC BRONCHOPULMONARY ASPERGILLOSIS (ABPA) ABPA develops as a result of a Type-I hypersensitivity reaction to A. fumigatus colonization of the tracheobronchial tree. Estimating the frequency of ABPA is difficult due to the lack of standard diagnostic criteria (see Section 4). It often appears not as a primary pathology, but as a complication of other chronic lung diseases such as atopic asthma, CF, and sinusitis. Depending on definition criteria, ABPA occurs in approximately 2–19% of CF patients. The estimated prevalence of ABPA in asthmatic patients varies between 0.7% and 22%, and is likely to affect 4.8 million patients globally, while its prevalence in severe asthma is as high as 70%.

4

9780367696399_Lydyard.indb 4

13/02/2023 07:32

Case 1: Aspergillus fumigatus

Figure 1.4 Relative risk of Aspergillus infection. The two chest X-rays show examples of acute invasive and allergic pulmonary aspergillosis. The fungal ball (aspergilloma) that was removed from a lung and measures about 6cm in diameter is also shown. Hypersensitivity accompanies development of allergic aspergillosis, immunodeficiency leads to invasive aspergillosis, while aspergilloma can be observed in immunocompetent individuals. From the Aspergillus Website, https://www.aspergillus.org.uk and Dr Jennifer Bartholomew, School of Medicine, University of Manchester. With permission.

T he clinical course often follows as classic asthma but can also lead to a fatal destruction of the lungs. IgE- and IgG­ mediated Type I hypersensitivity and Type III hypersensitivity are the leading causes of pathology (see Section 2). Since ABPA presents as a bronchial asthma, the symptoms are similar to poorly controlled asthma and include wheezing, cough, fever, malaise, hemoptysis and weight loss. Additional symptoms include recurrent pneumonia, release of brownish mucoid plugs with fungal hyphae, and recurrent lung obstruction. In the case of secondary ABPA, unexplained worsening of asthma and CF is observed. It is essential to diagnose and treat ABPA at the onset of the disease, which can be traced to early childhood or even infancy. ABPA should be suspected in children with a history of recurrent wheezing and pulmonary infi ltrates. T he outcome of the disease depends on asthma control, presence of widespread bronchiectasis, and resultant chronic fibrosis of the lungs (Figure 1.5). Respiratory failure and fatalities can occur in patients in the third or fourth decade of life. The long-term prognosis of ABPA beyond 5 years is currently not well defi ned.

lung disease, previous thoracic surgery, chronic corticosteroid therapy or alcoholism. CNPA presents as a subacute pneumonia unresponsive to antibiotic therapy, which progresses and results in cavity formations over weeks or months. Symptoms include fever, cough, night sweats, and weight loss. Because it is uncommon, CNPA often remains unrecognized for weeks or months and causes a progressive cavitatory pulmonary infiltrate. It is often found at autopsy. The reported mortality rate for CNPA is 10–40% or higher if it remains undiagnosed.

CHRONIC NECROTIZING PULMONARY ASPERGILLOSIS (CNPA) CNPA is a subacute condition mostly developing in mildly immunocompromised patients and is commonly associated with underlying lung disease such as steroid-dependent chronic obstructive pulmonary disease (COPD), interstitial

Figure 1.5 High-resolution CT scan of chest demonstrating remarkable bronchial wall thickening in the context of longstanding ABPA. From the Aspergillus Website, https://www.aspergillus.org.uk and Dr Jennifer Bartholomew, School of Medicine, University of Manchester. With permission.

5

9780367696399_Lydyard.indb 5

13/02/2023 07:32

Case Studies in Infectious Disease

INVASIVE ASPERGILLOSIS (IA)

Exposure to A. fumigatus in immunocompromised individuals can lead to IA, which is the most serious, life-threatening condition. Due to the development of immunosuppression in transplantation and anticancer chemotherapy leading to severe immunodeficiency and the AIDS pandemic, the incidence of IA has increased. Leukemia or bone marrow transplant (BMT) patients are at particular risk. IA is responsible for approximately 30% of fungal infections in patients dying of cancer, and it is estimated that IA occurs in 10–25% of all leukemia patients, in whom the mortality rate is 80–90%, even when treated. It occurs in 5–10% of cases following allogeneic BMT and in 0.5–5% after cytotoxic therapy of blood diseases or autologous BMT. In solid-organ transplantation, IA is diagnosed in 19–26% of heart-lung transplant patients and in 1–10% of liver, heart, lung, and kidney recipients. Other patients at risk include those with chronic granulomatous disease (25–40%), neutropenic patients with leukemia (5–25%), and patients with AIDS, multiple myeloma, and severe combined immunodeficiency (about 4%). Drugs such as antimicrobial agents and steroids can predispose the patient to colonization with A. fumigatus and invasive disease. Four types of IA have been described. Clinical symptoms of the different types of IA depend on the organ localization and the underlying disease. • Acute or chronic pulmonary aspergillosis (lungs). • Tracheobronchitis and obstructive bronchial disease (bronchial mucosa and cartilage). • Acute invasive rhinosinusitis (sinuses). •

Disseminated disease (brain, skin, kidneys, heart, eyes).

IA starts with pneumonia, and then the fungus usually disseminates to various organs causing endocarditis, osteomyelitis, otomycosis, meningitis, vision obstruction, and cutaneous infection (Figure 1.6). Aspergillus-related endocarditis and wound infections may occur through cardiac surgery. In the developing world, infection with Aspergillus can cause keratitis – a unilateral blindness.

Figure 1.6 Bone infection caused by invasive aspergillosis. From the Aspergillus Website, http://www.aspergillus.org.uk and Dr Jennifer Bartholomew, School of Medicine, University of Manchester. With permission.

Symptoms are usually variable and nonspecific: fever and chills, weakness, unexplained weight loss, chest pain, dyspnea, headaches, bone pain, a heart murmur, decreased diuresis, blood in the urine or abnormal urine color, and straight, narrow red lines of broken blood vessels under the nails. Patients develop tachypnea and progressive worsening hypoxemia. IA is accompanied by increased sputum production (sometimes with blood), sinusitis, and acute inflammation with areas of ischemic necrosis, thrombosis, and infarction of the organs involved. Even before the arrival of the COVID-19 pandemic, a strong association between influenza infection and IA had been established. The latter contributed to further complications in up to 23% of patients with severe influenza. A high variability in clinical presentation was seen, ranging from tracheobronchitis to invasive and angioinvasive disease. With the COVID-19 pandemic throughout 2020, fewer than 200 cases of COVID-19-associated pulmonary aspergillosis (CAPA) were reported worldwide, mostly associated with underlying pulmonary disease, and characterized by high mortality. If not treated in a timely manner, these cases quickly developed into invasive forms. It must also be noted that patients with SARS-CoV-2 often develop other fungal infections, particularly candidemia following a prolonged stay in the ICU.

ASPERGILLOMA An aspergilloma, also known as a mycetoma or fungus ball, is a clump of fungus which populates a lung cavity. It occurs in 10–15% of patients with preexisting lung cavities due to conditions such as tuberculosis, CF, lung abscess, sarcoidosis, emphysematous bullae, and chronically obstructed paranasal sinuses. Although Aspergillus species are the most common, some Zygomycetes and Fusarium may also form mycetomas. In patients with AIDS, aspergilloma may occur in cystic areas resulting from prior Pneumocystis jiroveci pneumonia infection. The fungus invades, settles, and multiplies in a cavity mostly outside the reach of the immune system. The growth results in the formation of a ball shaped like a halfmoon (crescent). It consists of a mass of hyphae surrounded by a proteinaceous matrix, which incorporates dead tissue and mucus with sporulating structures at the periphery. Some cavities may contain multiple aspergilloma (Figure 1.7). Patients with aspergilloma do not manifest many related symptoms, and the condition may go on for many years undiagnosed, often discovered incidentally by chest X-ray or by CT scans. The most common, but still rare, symptom is hemoptysis. The patients may cough up the fungus elements, and sometimes chains of conidia can be seen in the sputum. Aspergilloma can lead to pleural thickening. Rarely, in immunocompromised individuals, asperg­ illomas can be formed in other body cavities. They may cause abscesses in the brain, or populate various face sinuses, ear canals, kidneys, urinary tracts, and even heart valves. Secondary aspergillomas may occur as a result of IA when

6

9780367696399_Lydyard.indb 6

13/02/2023 07:32

Case 1: Aspergillus fumigatus

The radiological diagnosis is supplemented by laboratory tests: peripheral blood eosinophilia (>10% or 1000 mm–3), immediate skin reactivity to A. fumigatus antigenic extracts within 15 ± 5 min, detection of precipitating IgG and IgM antibodies in >90% of cases, and elevated levels of total IgE in serum (>1000 ng/ml). Specific IgE antibodies against A. fumigatus are also normally measured. Isolation of A. fumigatus from sputum (Figure 1.8), expectoration of brown plugs containing eosinophils and Charcot−Leyden crystals, and a skin reaction occurring 6 ± 2 h after the application of antigen are used as a complementary diagnosis. Figure 1.7 Multiple aspergillomas. Gross pathology showing three fungus balls in one cavity. From the Aspergillus Website, http://www. aspergillus.org.uk and Dr Jennifer Bartholomew, School of Medicine, University of Manchester. With permission.

a solid lesion of IA erodes to the surface of the lung. These lesions can be detected by a chest CT scan and must be taken into account when further immunosuppressive therapy for relapsed IA is prescribed.

4. HOW IS THE DISEASE DIAGNOSED AND WHAT IS THE DIFFERENTIAL DIAGNOSIS? DIAGNOSIS OF ABPA

ABPA is a particularly difficult syndrome to diagnose since the symptoms are nonspecific. The current diagnostic approaches are known as the Rosenberg–Patterson criteria. The disease presents with bronchial asthma with transient pulmonary infiltrates and, at later stages, proximal bronchiectasis and lung fibrosis. Chest radiography is also not specific and shows various transient abnormalities: consolidation or collapse, thickened bronchial wall, and/or peripheral shadows. The following criteria are used for diagnosis of ABPA: asthma, a history of pulmonary infiltrates, and central bronchiectasis.

Figure 1.8 Microscopy of sputum. A typical example of a wet mount of a sputum sample from a patient with ABPA. From the Aspergillus Website, http://www.aspergillus.org.uk and Dr Jennifer Bartholomew, School of Medicine, University of Manchester. With permission.

DIAGNOSIS OF IA IA in the early stages is also difficult to diagnose. A safe diagnosis can only be made at autopsy with the histopathological evidence of mycelial growth in tissue. Differential diagnosis from the invasion of hyphae of other filamentous fungi such as Fusarium or Pseudallescheria is often difficult and requires immunohistochemical staining or in situ hybridization techniques. Clinical symptoms are usually nonspecific and require further laboratory tests. Criteria currently used for the diagnosis of IA are: a positive CT scan (see Figure 1.1), culture and/or microscopic evaluation, and the detection of Aspergillus antigens in the serum. A CT scan can demonstrate single or multifocal nodules, with and without cavitation, or widespread, often bilateral, infiltrates. However, the appearances are heterogeneous throughout the course of the disease, with the most specific being at the early stages and presenting a “halo” of hemorrhagic necrosis surrounding the fungal lesion or pleura-based lesions. In non-pulmonary forms of the disease such as cerebral aspergillosis, a CT scan, together with brain magnetic resonance imaging (MRI), can detect the extent of the disease and the bone invasion. The diagnostic value of the microscopic examination of sputum is limited due to the presence of airborne conidia of Aspergillus and the possibility of accidental contamination. However, in neutropenic or BMT patients, the predictive value of a sputum culture positive for A. fumigatus exceeds 70%. The presence of A. fumigatus in bronchoalveolar lavage fluid (BAL) samples from patients with leukemia and BMT is found in 50–100% of those who have definitive or probable aspergillosis. Nasal swabs of patients also have diagnostic value, although bronchoscopy is preferable due to the sterility of the clinical sample. For the same reason, percutaneous lung biopsy or aspirated material are the specimens of choice. However, invasive procedures in immunocompromised patients require careful consideration.

CELL CULTURE After the microscopic identification of A. fumigatus, cell culture may be critical in supporting the diagnosis of aspergillosis. The 7

9780367696399_Lydyard.indb 7

13/02/2023 07:32

Case Studies in Infectious Disease

specimen is usually inoculated onto a plate with Sabouraud glucose agar, inhibitory mold agar (IMA) or other appropriate medium with antibiotics – gentamicin or chloramphenicol. The plates are incubated at 30°C for up to 6 weeks with the cultures being examined at 3-day intervals.

Antigen Detection A highly specific (99.6%) and sensitive (1 ng ml–1) test for detection of Aspergillus GM has been developed for screening and early diagnosis of IA in serum, BAL, and cerebrospinal fluid (CSF). GM is a part of the Aspergillus cell wall (see Section 1), and can often be released into the patient’s bodily fluids. The detection of A. fumigatus GM by Enzyme-Linked Immunosorbent Assay (ELISA) becomes possible at an early stage of infection thus allowing timely initiation of therapy. In 65.2% of patients, GM can be detected in serum 5–8 days before the development of IA symptoms. In addition, ELISA can be used for monitoring the disease treatment. Positive results in two consecutive serum samples allows the diagnosis of IA. However, in some cases, false positive reactions can be observed due to cross-reactivity with nonspecific antigens derived from other fungi such as Rhodotorula rubra, Paecilomyces varioti, Penicillium chrysogenum and P. digitatum.

radiograph. The radiographic picture must be differentiated from other conditions such as cavitating neoplasm, blood clot, disintegrating hydatid cyst, and pulmonary abscess with necrosis. Clinical analysis should be coupled with serologic tests since a number of other fungi such as Candida, Torulopsis, Petriellidium, Sporotrichum, and Streptomyces can lead to the development of mycetoma. Since aspergilloma is a condition often observed in immunocompetent individuals, a laboratory diagnosis based on a humoral response is feasible. The most commonly used methods in clinical diagnosis are double immunodiffusion (Figure 1.9), immunoprecipitation, and counter-immunoelectrophoresis because they are simple, cheap, and easy to perform. It must be noted that patients undergoing corticosteroid treatment may become seronegative. For the polymerase chain reaction (PCR) analysis the most reliable and well-characterized antigens of A. fumigatus are RNase, catalase, dipeptidylpeptidase V, and the GM.

5. HOW IS THE DISEASE MANAGED AND PREVENTED?

DIAGNOSIS OF ASPERGILLOMA

ABPA

A definitive diagnosis of aspergilloma requires bronchoscopy, lung biopsy or resection, but this is rare. The diagnosis is usually made accidentally or specifically by chest radiography. A pulmonary aspergilloma appears as a solid ball of water density, sometimes mobile, within a spherical or ovoid cavity. It is separated from the wall of the cavity by the air space. Pleural thickening is also characteristic. A chest CT scan can sometimes detect aspergilloma with a negative chest

Although ABPA is a chronic condition, acute corticosteroid­ responsive asthma can occur and lead to fibrotic end-stage lung disease. The aim of treatment of ABPA is to suppress the immune reaction to the fungus and to control bronchospasm. For this, high doses of oral corticosteroids are used: 30–45 mg/ day of prednisolone or prednisone in acute phase and a lower maintenance dose of 5–10 mg/day. Oral triazole antifungal drugs are also effective against A. fumigatus. These include first-generation drugs such as itraconazole and fluconazole and second-generation antifungals – voriconazole, posaconazole, and isavuconazole. The administration of medications can be combined with removal of mucus plugs by bronchoscopic aspiration. Regular monitoring by X-rays, pulmonary function tests, and serum IgE levels are essential. Successive control of ABPA leads to a drop in IgE levels, while their increase indicates relapse. Novel treatments include immunotherapy, although these have not been studied in large-scale randomized trials. The following antibodies have been proposed: anti-IgE Omalizumab, Ligelizumab and Quilizumab; anti-IL-5 Mepolizumab and Resilizumab; anti-IL-5 receptor Benralizumab, and others. TLR agonists are also considered as adjuvants in allergen immunotherapy. Currently, the development of nanoparticle-based therapies is underway.

Figure 1.9 Double diffusion test for aspergillosis. The central well contains A. fumigatus antigen and wells at the top and bottom contain control antiserum. The three peripheral wells with precipitin bands contain sera from patients with A. fumigatus aspergilloma. More bands present in the upper right case are characteristic of aspergilloma. The well in the bottom left position is negative. From the Aspergillus Website, http://www.aspergillus.org.uk and Dr Jennifer Bartholomew, School of Medicine, University of Manchester. With permission.

INVASIVE ASPERGILLOSIS It is difficult to achieve a timely diagnosis of IA due to the rapid progression (1–2 weeks from onset to death) and waiting for the confirmation of diagnosis would put the patients at a greater risk of untreatable fungal burden. The decision to start

8

9780367696399_Lydyard.indb 8

13/02/2023 07:32

Case 1: Aspergillus fumigatus

antifungal treatment has therefore to be empirical and based more on the presence of risk factors such as long-term (12–15 days) severe neutropenia (< 100–500 mm–3). The antifungal regimen for the treatment of IA includes voriconazole, amphotericin B (deoxycholate and lipid preparations), and itraconazole. Voriconazole is particularly effective against IA and in reducing mortality. In addition, voriconazole, itraconazole, and amphotericin B exhibit a broad-spectrum activity against Aspergillus and the related hyaline molds. In the early treatment of CAPA, the most effect was achieved with isavuconasole demonstrating broadspectrum, predictable pharmacokinetics, high dose in lungs, few side-effects and few interactions with other drugs. Amphotericin B (AmB) has been used in antifungal therapy for more than 30 years, mostly under the name of Fungizone®, and remains a fi rst-line drug, although the overall success rate of AmB therapy for IA is only 34%. Despite some progress in antifungal therapy, mortality rates from IA remain very high: 86%, 66%, and 99% for pulmonary, sinus, and cerebral aspergillosis, respectively. CAPA cases were associated with high mortality. Few patients with severe persistent neutropenia survive, and neutrophil recovery is usually followed by resolution of aspergillosis. The duration of neutropenia is therefore an indication of a possible outcome of IA.

ASPERGILLOMA

Aspergilloma can be prevented by timely and effective management of diseases that increase the risk of its development, such as tuberculosis. Complication of aspergilloma by severe hemoptysis is an indication for surgery, although it is associated with high risks of morbidity and mortality. Surgical resection of aspergilloma is one of the most complex procedures in thoracic surgery, since prolonged chronic infection and inflammation lead to thickened fibrotic tissue, induration of the hilar structures, and obliteration of the pleural space. Postoperative complications include hemorrhage, bronchopleural fistula, a residual pleural space, and empyema. Surgery is restricted to patients with severe hemoptysis and adequate pulmonary function. On the other hand, in the absence of surgical intervention, the course of the disease remains unpredictable. An alternative treatment in patients with severe pulmonary dysfunction comprises topical therapy. This includes intracavitary instillation of an antifungal drug such as AmB, percutaneous injection into aspergilloma cavities of sodium and potassium bromide, and endobronchial instillation of ketoconazole via fiberoptic bronchoscopy. However, topical therapy is labor-intensive and nonapplicable for cases with multiple aspergilloma.

SUMMARY 1. WHAT IS THE CAUSATIVE AGENT, HOW DOES IT ENTER THE BODY, AND HOW DOES IT SPREAD A) WITHIN THE BODY AND B) FROM PERSON TO PERSON? ■ Aspergillus is a saprophytic fungus, and one of the most ubiquitous. Its natural habitat is soil, but it is also present around construction sites and indoors in water storage tanks, fire­ proofing materials, bedding, and ventilation and air-conditioning systems. ■ The cell wall of A. fumigatus contains various polysaccharides including a galactomannan core. ■ Of more than 100 species of Aspergillus, only a few are pathogenic: A. fumigatus, A. flavus, A. terreus, A. clavatus, and A. niger. ■ A. fumigatus is a primary pathogen of man and animals and causes all manifestations of aspergillosis. It is thermotolerant and grows at temperatures ranging from 15°C to 55°C; it can even survive temperatures of up to 75°C. This is a key feature that allows it to grow over other aspergilli species and within the mammalian respiratory system. ■ A. fumigatus sporulates abundantly, with every conidial head producing thousands of conidia. The conidia released into the atmosphere range from 2.5−3.0 μm in diameter. We normally inhale 100–200 spores daily, but only susceptible individuals develop a clinical condition. The spores enter the body via the respiratory tract and lodge in the lungs or sinuses. ■ Once inhaled, spores at body temperature develop into a different form – thread-like hyphae, which invade the host tissue. Together,

the hyphae can form a dense mycelium in lungs. However, in the case of healthy immunocompetent individuals, the spores are prevented from reaching this stage due to the optimal immune responses, and there is some colonization but limited pathology.

2. WHAT IS THE HOST RESPONSE TO THE INFECTION AND WHAT IS THE DISEASE PATHOGENESIS? ■ In immunocompetent hosts, fungal conidia may be cleared by ciliated epithelium of the terminal bronchioles and ingested by tissue macrophages or alveolar macrophages. While macrophages mostly attack conidia, neutrophils are more important for elimination of the next, hyphal form of the fungus. ■ Neutrophils adhere to the surface of the hyphae and trigger a respiratory burst, secretion of NADPH and reactive oxygen species (ROS), release of lysozyme, neutrophil cationic peptides, and lactoferrin. ■ In immunodeficient, particularly IA patients, corticosteroid­ based treatment, purine analogs (fludarabine) and some monoclonal antibody treatment lead to neutropenia and/or neutrophil dysfunction. Corticosteroids reduce oxidative burst and superoxide anion release by neutrophils, thereby inhibiting hyphal killing. ■ Th1 cytokines are important in neutrophil-mediated killing of hyphae. The Th2 response, which is associated with an increase in antibody production, seems to facilitate fungal invasion rather than protect. Continued...

9

9780367696399_Lydyard.indb 9

13/02/2023 07:32

Case Studies in Infectious Disease

....continued

■ ABPA pathogenesis is associated with elevated levels of antigen-specific circulating IgG and IgE. B cells secrete IgE spontaneously as a result of IL-4 production, while IL-5 recruits eosinophils. Development of Types I and II hypersensitivity leads to inflammation and tissue damage. ■ Patients with aspergilloma have increased levels of specific IgG and IgM, mostly against Aspergillus carbohydrates and glycoproteins. The pathogenicity of A. fumigatus is usually based on (a) a direct attack to the host cells with the pathogen’s virulent factors and toxins; (b) evasion of the immune responses; (c) the hypersensitivity of innate and adaptive responses of patients.

3. WHAT IS THE TYPICAL CLINICAL PRESENTATION AND WHAT COMPLICATIONS CAN OCCUR? ■ Pulmonary diseases caused by A. fumigatus are classified into four main clinical types: allergic bronchopulmonary aspergillosis, chronic necrotizing Aspergillus pneumonia, invasive aspergillosis, and pulmonary aspergilloma. ■ ABPA is the result of a hypersensitivity reaction to A. fumigatus colonization of the tracheobronchial tree. It often appears as a complication of other chronic lung diseases such as atopic asthma (0.5–2%), cystic fibrosis (7–35%), and sinusitis. ABPA occurs in up to 15% of asthmatic patients sensitized to A. fumigatus. ■ ABPA symptoms are similar to asthma and include wheezing, cough, fever, malaise, and weight loss. Additional symptoms include recurrent pneumonia, release of brownish mucoid plugs with fungal hyphae, and recurrent lung obstruction. In secondary ABPA, there is unexplained worsening of asthma and cystic fibrosis. ■ Chronic obstructive pulmonary disease (COPD) presents as a subacute pneumonia that is unresponsive to antibiotic therapy. Symptoms include fever, cough, night sweats, and weight loss. It develops mostly in mildly immunocompromised patients and is commonly associated with underlying lung disease. ■ Invasive aspergillosis (IA) is the most serious form of aspergillosis and normally occurs in immunocompromised individuals. There are four types of IA: acute or chronic pulmonary aspergillosis (lungs); tracheobronchitis and obstructive bronchial disease (bronchial mucosa and cartilage); acute invasive rhinosinusitis (sinuses); and disseminated disease (brain, skin, kidneys, heart, eyes). ■ IA symptoms are variable and nonspecific: fever and chills, weakness, unexplained weight loss, chest pain, shortness of breath, headaches, bone pain, a heart murmur, blood in the urine, decreased diuresis, and straight, narrow red lines of broken blood vessels under the nails. IA is accompanied by increased sputum production (sometimes with hemoptysis), sinusitis, and acute inflammation with ischemic necrosis, thrombosis, and infarction of the organs. ■ IA often accompanies severe influenza-based pulmonary distress. Since the beginning of the COVID-19 pandemic,

COVID-associated pulmonary aspergillosis has been identified which quickly develops into invasive forms and requires early treatment. ■ Aspergilloma occurs in 10–15% of patients with cavitating lung diseases such as tuberculosis, sarcoidosis, lung abscess, emphysematous bullae, cystic fibrosis, and paranasal sinuses. It is often discovered incidentally by chest X-ray or by computed tomography (CT) scans. The most common symptom is hemoptysis.

4. HOW IS THE DISEASE DIAGNOSED, AND WHAT IS THE DIFFERENTIAL DIAGNOSIS? ■ ABPA diagnostic criteria include: asthma, a history of pulmonary infiltrates, and central bronchiectasis supplemented by the laboratory tests: peripheral blood eosinophilia, immediate skin reactivity to A. fumigatus, precipitating IgG and IgM, elevated levels of total IgE in serum and specific IgE against A. fumigatus. Diagnosis of IA is based on a positive CT scan, cell culture and/ or microscopic evaluation, and detection of Aspergillus antigens in serum. ■ Since IA is life-threatening, early diagnosis using ELISA and PCRbased techniques is essential. Reliable antigens of A. fumigatus include RNase, catalase, dipeptidylpeptidase V, and the GM. GM detection by ELISA has become the basis for the most popular A. fumigatus diagnostic tests. ■ A definitive diagnosis of aspergilloma requires bronchoscopy, lung biopsy or resection. The diagnosis is usually made accidentally or specifically by chest radiography. Clinical analysis should be coupled with serologic tests.

5. HOW IS THE DISEASE MANAGED AND PREVENTED? ■ The aim of the treatment of ABPA is to suppress the immune reaction to the fungus and to control bronchospasm. High doses of oral corticosteroids are used. Antifungal drugs such as itraconazole are sometimes used. Regular monitoring by X-rays, pulmonary function tests, and serum IgE levels is essential. ■ Since it is difficult to achieve timely diagnosis of IA, the decision to start antifungal treatment has to be empirical and based on the presence of risk factors, particularly prolonged neutropenia. Antifungal treatments include voriconazole (particularly effective), amphotericin B (AmB), and itraconazole, which exhibit a broadspectrum activity against Aspergillus and the related hyaline molds. Itraconazole is prescribed to patients with AmB­ induced nephrotoxicity. Isavuconasole appears to be advantageous for the CAPA treatment. ■ Aspergilloma can be prevented by the treatment of diseases that increase its risk, such as tuberculosis. In the absence of surgical intervention, the course of the disease remains unpredictable. Itraconazole for 6–18 months is recommended for the oral treatment.

10

9780367696399_Lydyard.indb 10

13/02/2023 07:32

Case 1: Aspergillus fumigatus

FURTHER READING Barnes PD, Marr KA. Aspergillosis: spectrum of disease, diagnosis and treatment. Infect Dis Clin N Am, 20: 545–561, 2006. Thompson GRA, Young J-AH. Aspergillus Infections. NEJM, 385: 1496–1509, 2021.

REFERENCES Alastruey-Izquierdoa A, Cadranel J, Flick H, et al. Treatment of Chronic Pulmonary Aspergillosis: Current Standards and Future Perspectives. Respiration, 96: 159–170, 2018. Asano K, Hebisawa A, Ishiguro T, et al. New Clinical Diagnostic Criteria for Allergic Bronchopulmonary Aspergillosis/Mycosis and its Validation. J Allergy Clin Immunol, 147: 1261–1268, 2020. Chumbita M, Puerta-Alcalde P, Garcia-Pouton N, GarciaVidal C. COVID-19 and Fungal Infections: Etiopathogenesis and Therapeutic Implications. Rev Esp Quimioter, 34: 72–75, 2021. Gerson SL, Talbot GH, Hurwitz S, et al. Prolonged Granulocytopenia: The Major Risk Factor for Invasive Pulmonary Aspergillosis in Patients with Acute Leukemia. Ann Intern Med, 100: 345–351, 1984. Gu X, Hua Y-H, Zhang Y-D, et al. The Pathogenesis of Aspergillus fumigatus, Host Defense Mechanisms, and the Development of AFMP4 Antigen as a Vaccine. Polish J Microbiol, 70: 3–11, 2021. Herbrecht R, Letscher-Bru V, Oprea C, et al. Aspergillus Galactomannan Detection in the Diagnosis of Invasive Aspergillosis in Cancer Patients. J Clin Oncol, 20: 1898–1906, 2002. Hohl TM, Feldmesser M. Aspergillus fumigatus: Principles of Pathogenesis and Host Defense. Eukaryot Cell, 6: 1953– 1963, 2007. Koutsokera A, Corriveau S, Sykes J, et al. Omalizumab for Asthma and Allergic Bronchopulmonary Aspergillosis in Adults with Cystic Fibrosis. J Cyst Fibros, 19: 119–124, 2020. Latgé J-P, Chamilos G. Aspergillus fumigatus and Aspergillosis in 2019. Clin Microbiol Rev, 33: e00140–18, 2019.

Lewington-Gower E, Chan L, Shah A. Review of Current and Future Therapeutics in ABPA. Ther Adv Chronic Dis, 12: 1–17, 2021. Moss RB. Pathophysiology and Immunology of Allergic Bronchopulmonary Aspergillosis. Med Mycol, 43: S203–S206, 2005. Tekaia F, Latgé J-P. Aspergillus fumigatus: Saprophyte or Pathogen? Curr Opin Microbiol, 8: 385–392, 2005. Vanderbeke L, Spriet I, Breynaert C, et al. Invasive Pulmonary Aspergillosis Complicating Severe Influenza: Epidemiology, Diagnosis and Treatment. Curr Opin Infect Dis, 31: 471–480, 2018. Walsh TJ, Anaissie EJ, Denning DW, et al. Treatment of Aspergillosis: Clinical Practice Guidelines of the Infectious Diseases Society of America. Clin Infect Dis, 46: 327–360, 2008. Wassano NS, Goldman GH, Damasio A. Aspergillus fumigatus. Trends Microbiol, 28: 594–595, 2020.

WEBSITES American Lung Association, Aspergillosis Symptoms and Diagnosis, 2021: https://www.lung.org/lung-health-diseases/ lung-disease-lookup/aspergillosis/symptoms-diagnosis Aspergillus Website, The Fungal Research Trust, Copyright ©, 2007: http://www.aspergillus.org.uk John Hopkins Medicine, Shmuel Shoham, Aspergillus, 2020: https://www.hopkinsguides.com/hopkins/view/Johns_ Hopkins_ABX_Guide/540036/all/Aspergillus Mayo Clinic, Aspergillosis, 2022: https://www.mayoclinic. org/diseases-conditions/aspergillosis/symptoms-causes/ syc-203696198 Utah State University, Intermountain Herbarium: http:// herbarium.usu.edu

Students can test their knowledge of this case study by visiting the Instructor and Student Resources: [www. routledge.com/cw/lydyard] where several multiple choice questions can be found.

11

9780367696399_Lydyard.indb 11

13/02/2023 07:32

2

Borrelia burgdorferi and related species

A 45-year-old woman was on vacation in Cape Cod, Massachusetts and decided to attend an outdoor music festival. In order to get there, she walked 3 miles each way at night, through a dark, wooded, grassy area. Shortly thereafter, she developed nonspecific symptoms that included fever, headache, muscle aches, mild neck stiffness, and joint pain. She also noticed an oval “bull’s eye” rash on her right arm, which increased in size and cleared in the center. Over the ensuing month, she felt increasingly fatigued and developed facial paralysis (Bell’s palsy) (Figure 2.1), which precipitated a visit to her family physician. She couldn’t remember being bitten by a tick but based on her description of the rash and her other symptoms, her doctor suspected Lyme disease and took a blood sample for serology. Enzyme immunoassay and Western blot confirmed the presence of Borrelia burgdorferi-reactive antibodies. After confirming that the patient was not pregnant, she was pre­ scribed doxycycline 100 mg twice daily for 30 days.

1. WHAT IS THE CAUSATIVE AGENT, HOW DOES IT ENTER THE BODY AND HOW DOES IT SPREAD A) WITHIN THE BODY AND B) FROM PERSON TO PERSON? CAUSATIVE AGENT

The patient has Lyme disease. The causative agent of Lyme disease is Borrelia burgdorferi sensu lato (meaning in the broad sense). There are at least 20 genospecies within the Borrelia burgdorferi complex (B. burgdorferi sensu lato) worldwide. The three main pathogenic genospecies comprising this group are B. burgdorferi sensu stricto, B. garinii, and B. afzelii. Strains found in North America belong to B. burgdorferi sensu stricto whereas all three species are found in Europe and Asia. Borreliae are microaerophilic spirochetes (Figure 2.2) that are extremely difficult to culture because of their complex nutrient requirements. Thus, they are usually detected by the immune response they induce in blood of the infected person (see above and Section 4). The bacteria have a gram-negative wall structure and have a spiral mode of motility produced by axial fi laments termed endoflagella. In contrast to the usual type of flagella exhibited by gram-negative bacteria that are anchored in the cytoplasmic membrane and extend through

9780367696399_Lydyard.indb 13

Figure 2.1 Bell’s palsy: this is demonstrated by drooping at the left corner of the mouth, loss of the left naso-labial fold, and inability to completely close the left eye (not shown in image). Reprint permission kindly granted by Dr Charles Goldberg, MD, and Regents of the University of California.

the cell wall into the external environment of the cell, the endoflagella of borreliae are found within the periplasmic space contained between a semi-rigid peptidoglycan layer and a multi-layer, flexible outer-membrane sheath. Rotation of the endoflagella within the periplasmic space causes the borreliae to move in a cork-screw fashion (Figure 2.2). Borreliae lack lipopolysaccharide in their outer membrane (OM); instead the OM contains phospho- and glycolipids. Importantly, OM lipoproteins are phase variable during the infection

Figure 2.2 Borrelia burgdorferi is a spirochete: it is 0.2–0.3 micrometers (μm) wide and its length may exceed 15–20 μm. Courtesy of the Centers for Disease Control, Atlanta, Georgia. Image is found in the Public Health Image Library #6631.

13

13/02/2023 07:32

Case Studies in Infectious Disease

Figure 2.3 The global distribution of Ixodes spp. ticks able to transmit the agent of Lyme disease, Borrelia burgdorferi. Modified from http://geo.arc.nasa.gov/sge/health/sensor/disease/lyme.html

cycle. In addition, Borrelia species, instead of having circular chromosomes, have linear chromosomes and contain circular and linear plasmids, with some species containing more than 20 different plasmids.

TICKS – THE VECTORS OF B. BURGDORFERI The bacteria are maintained in an enzootic cycle involving hard-bodied ticks belonging to the Ixodes ricinus species complex and a wide range of reservoir vertebrate hosts. The global distribution of Ixodes species is shown in Figure 2.3. In the eastern US, the vector is primarily Ixodes scapularis and in the western US it is I. pacificus. I. ricinus and I. persulcatus are the vectors in Europe and Eurasia, respectively. These ticks have a 2-year life cycle (Figure 2.4). Adult ticks feed and mate on large animals, especially white-tailed deer, in the autumn and early spring. However, white-tailed deer are not considered reservoirs of B. burgdorferi because they do not support a sufficiently high level of spirochetes in their blood to infect ticks. Nevertheless, deer are important in tick reproduction and serve to increase tick numbers in an area and spread ticks into new areas. Female ticks then drop off the animals and lay eggs on the ground. By summer, the eggs hatch into larvae, which feed on mice and other small mammals and birds through to early autumn; then they become inactive until the following spring when they molt into nymphs. Nymphs feed on small rodents and other small mammals and birds during the late spring and summer and molt into adults in the autumn, completing the 2-year life cycle. Larvae and nymphs typically become infected with borreliae when they feed on infected small animals, particularly the white-footed mouse and chipmunks. The tick remains infected with the borreliae as it matures from larva to nymph or from nymph to adult. Infected nymphs and adult ticks then bite and transmit the bacteria to other small rodents, other animals, or humans

in the course of their normal feeding behavior. The ticks are slow feeders, requiring several days to complete a blood meal. Transfer of the borreliae from the infected tick to a vertebrate host probably does not occur unless the tick has been attached to the body for 36 hours or so. Ticks transmit Lyme disease to humans generally during the nymph stage, probably because nymphs are more likely to feed on a person and are rarely noticed because of their small size (< 2 mm) (Figure 2.5). Although tick larvae are smaller than nymphs, they rarely carry borreliae at the time of feeding

Figure 2.4 Tick life cycle. Courtesy of Carolyn Klass, Cornell University.

14

9780367696399_Lydyard.indb 14

13/02/2023 07:32

Case 2: Borrelia burgdorferi and related species

move toward the salivary glands. Once the borreliae enter the vertebrate host, they down-regulate OspA and express OspC, DbpA, and BBK32. The environmental cues for up- and down-regulation of Osps include nutrient availability, oxygen tension, temperature, and pH. The sequence of OspC is strainspecific, so the population of B. burgdorferi injected by the tick expresses a spectrum of antigenically distinct OspC proteins. Several salivary gland proteins are induced during tick feeding and one of these, Salp15, has immunosuppressive activity where the tick saliva is deposited in the skin. It has been shown that there is a specific interaction between tick Salp15 and OspC, both in vitro and in vivo.

ENTRY AND SPREAD WITHIN THE BODY

Figure 2.5 Appearance and relative sizes of adult male and female, nymph, and larval ticks including deer ticks (Ixodes scapularis), lone star ticks (Amblyomma americanum), and dog ticks (Dermacentor variabilis). Of those pictured, only the I. scapularis ticks are known to transmit Lyme disease. Courtesy of the Centers for Disease Control, Division of Vector-Borne Infectious Diseases, Fort Collins, Colorado.

and are probably not important in the transmission of Lyme disease to humans. While adult ticks can transmit borreliae they are less likely to do so than nymphs. This is because their larger size means that they are more likely to be noticed and removed from a person’s body within a few hours and they are most active during the cooler months of the year, when outdoor activity is limited. It should be noted that dogs, horses, cattle, deer, and other animals are also susceptible to Lyme disease. Borreliae express a number of outer-surface lipoproteins termed Osps (outer surface proteins) and they play an important role in the life cycle of the spirochete by interacting with intercellular and cellular components of its arthropod and vertebrate hosts. They are also important in the evasion of the host immune system (see Section 3). B. burgdorferi selectively expresses specific Osps during distinct phases of its life cycle and in specific tissue locations. OspA and OspB are expressed on spirochetes in unfed nymphs and adult ticks. Both OspA and OspB mediate adherence of the spirochetes to the cells of the tick mid-gut, which allows them to avoid endocytosis by tick enterocytes during digestion of the blood meal and subsequently allows their detachment when the tick takes a second blood meal so that the bacteria can enter the vertebrate host. In the mid-gut, during tick feeding, the bacteria up-regulate expression of OspC, which presages their

B. burgdorferi sensu lato spirochetes enter the tissues while the tick takes a blood meal. The bacteria may establish a localized infection in the skin at the site of the tick bite, producing a characteristic skin lesion known as erythema migrans (EM). In addition, they may disseminate via the bloodstream and/ or lymphatics. The organism demonstrates a tropism for the central nervous system (CNS), heart, joints, and eyes, all of which may become chronically infected, giving rise to neurologic disease, carditis, arthritis, and conjunctivitis (see Section 3). Even in the absence of systemic symptoms, it appears that as many as half of persons with EM have evidence of borreliae in the blood or cerebrospinal fluid (CSF). Furthermore, borreliae can also persist in skin and perhaps the CNS for years without causing symptoms.

PERSON-TO-PERSON SPREAD Lyme disease is not spread from person to person. It is possible in a woman who contracts Lyme disease during pregnancy for the borreliae to cross the placenta leading to infection of the fetus, but this rarely occurs. The consequence of fetal infection remains unclear. For this reason, the Centers for Disease Control and Prevention (CDC) maintains a registry of pregnant women with Lyme disease to accrue data on the effects of Lyme disease on the developing fetus.

EPIDEMIOLOGY Lyme disease is the most common tick-borne disease in North America and Europe. Although Lyme disease has now been reported in all 50 states in the US, almost all reported cases are confined to New England (Connecticut, Maine, Massachusetts, New Hampshire), the Mid-Atlantic region (Delaware, Maryland, New Jersey, Pennsylvania), the EastNorth Central region (Wisconsin), and the West NorthCentral region (Minnesota). A 2021 report, based on the use of commercial claims data for evaluating trends in Lyme disease diagnoses in the US between 2010–2018, gave an annual incidence of Lyme disease diagnoses per 100 000 enrollees of 49 to 88. However, all of these diagnoses may not have been proven to be Lyme disease. Although Lyme disease is common 15

9780367696399_Lydyard.indb 15

13/02/2023 07:32

Case Studies in Infectious Disease

in the US and Scandinavia and has been reported in other countries in Western and Eastern Europe, Japan, China, and Australia, it is not a common disease in the UK, with about 3000 cases per year being reported in England and Wales in the most recent data. The incidence of Lyme disease in Europe is estimated to be a little over 232 000 cases.

2. WHAT IS THE HOST RESPONSE TO THE INFECTION AND WHAT IS THE DISEASE PATHOGENESIS? Because the spirochete is delivered to the host via the bite of a tick, it bypasses the physical barrier of the intact skin and the antimicrobial factors in sweat.

INNATE IMMUNITY Components of the innate immune system are brought into play to combat the spirochetes at the site of their delivery by the interaction of microbe-associated molecular patterns (MAMPs) on the surface of the borreliae with Toll-like receptors (TLRs) on epithelium and leukocytes. B. burgdorferi activates both the innate and classical pathways of the complement cascade but is resistant to complementmediated lysis because OspE and other proteins on its surface bind the complement control glycoprotein, factor H, which inactivates complement factor C3b. The membrane attack complex (MAC) can also be inactivated in a similar manner. Resident macrophages in the area of the inoculum are able to bind, phagocytose, and kill borreliae without the need for opsonization by complement or antibody. Binding may be mediated by the mannose-binding receptor and/or the MAC-1 receptor. Polymorphonuclear leukocytes (PMNs) can also kill borreliae without opsonization.

ADAPTIVE IMMUNITY

16

There is little doubt that IgM and IgG antibodies play the principal role in the clearance of B. burgdorferi. Furthermore, these antibodies do not need to be complement fixing. Murine IgG and IgM monoclonal antibodies have been developed that are bactericidal for borreliae in the absence of complement. The finding that mice deficient in /β T cells or deficient in /βand / T cells can clear spirochetemia indicates that T cells are not required for spirochete clearance. Lyme arthritis appears to result from a constellation of factors that include production of pro-inflammatory cytokines and immune complexes, and arthritis is linked to HLA-DR4 and HLA-DR2. How do the borreliae evade these host defense mechanisms? First of all, as mentioned earlier, Salp15 salivary gland protein induced during tick feeding has immunosuppressive activity where the tick saliva containing the borreliae is deposited in the skin. Salp15 inhibits the IgG antibody response by blocking CD4+ T-cell activation and so may protect B. burgdorferi by suppressing production of neutralizing antibodies. This mechanism may be particularly

9780367696399_Lydyard.indb 16

important in Lyme-endemic areas where infected ticks frequently feed on their primary hosts that may possess preexisting antibodies against B. burgdorferi. Furthermore, borreliae can stimulate interleukin-10 (IL­ 10) production by macrophages, mast cells, and Th2 CD4+ T cells. IL-10 inhibits synthesis of pro-inflammatory cytokines and suppresses antigen presentation to CD4+ helper T cells by antigen-presenting cells. B. burgdorferi undergoes antigenic variation and modulates the expression of Osps on the cell surface during infection. VlsE is an example of an Osp that undergoes antigenic variation. The vls locus of B. burgdorferi is located on a 28-kb linear plasmid and consists of an expression site (vlsE) and 15 silent vls cassettes. Another candidate may be OspE, since the gene for this Osp has two hypervariable domains and repeat regions that allow recombination with other genes, which may result in the formation of new antigens.

PATHOGENESIS The tissue injury in Lyme disease is mediated by inflammation induced by B. burgdorferi. The manner in which the bacterium induces inflammation in the host is not fully understood. Spirochetemia results in the invasion of tissues such as the heart and joints and the host responds with a vigorous inflammatory response. The role of B. burgdorferi MAMPs–TLR interactions in the induction of pro-inflammatory cytokines has been questioned by results of knockout experiments, which indicate that TLR-2-deficient mice or mice deficient in TLR signaling molecules develop arthritis and have a much larger burden of B. burgdorferi. It has been suggested that chemokines produced at the site of infection may be more important in the influx of inflammatory cells to the site of infection.

3. WHAT IS THE TYPICAL CLINICAL PRESENTATION AND WHAT COMPLICATIONS CAN OCCUR? Whether the disease develops following B. burgdorferi infection depends on the balance between the pathogen and the host’s immune response. There are three potential outcomes of the borrelia–host interaction. The spirochete may be cleared without any manifestations of disease, the only indicator of infection being that the individual is seropositive. Alternatively, the spirochete establishes in the skin and, after a variable incubation period ranging from a few days to a month, produces a characteristic spreading rash termed EM. The rash begins as a small macule (a visible change in the color of the skin that cannot be felt) or papule (a small, solid and usually conical elevation of the skin), which then expands, ranging in diameter to between 5 and 50 cm (Figure 2.6). The rash has a flat border and central clearing so that it resembles a “bull’s­ eye”. EM is probably caused by the inflammatory response to the spirochete infection. From the initial focus of infection in

13/02/2023 07:32

Case 2: Borrelia burgdorferi and related species

the skin, the spirochete spreads throughout the body. Systemic spread of the spirochete results in malaise, headaches, chills, joint pain, myalgia, lymphadenopathy, and severe fatigue. This phase may last for up to one month. Unless treated, over two-thirds of infected individuals manifest neurologic and cardiac symptoms. These manifestations may occur as early as one month or as late as 2 years or more post-infection. Neurologic sequelae include meningitis, encephalitis, and peripheral nerve neuropathy, particularly seventh cranial nerve palsy (Bell’s palsy). Cardiac sequelae include heart block, myopericarditis, and congestive heart failure. Neurologic and cardiac sequelae may be followed by arthralgia and arthritis. About two-thirds of patients with untreated infection will experience intermittent bouts of arthritis, with severe joint pain and swelling. Large joints are most often affected, particularly the knees. These manifestations may last for months to years with little evidence of bacterial invasion. The manifestations of Lyme disease are related to the particular genospecies of Borrelia involved. In Europe B. garinii is associated with neurologic disease, while B. afzelii is associated with a dermatologic manifestation known as acrodermatitis chronica atrophicans, a progressive fibrosing skin process. The existence of an entity termed “chronic Lyme disease” or post-treatment Lyme disease is controversial. The term chronic Lyme disease is used in North America and in Europe as a diagnosis for patients with persistent pain, neurocognitive

symptoms, fatigue, either separately or together, with or without clinical or serologic evidence of previous early or late Lyme disease. However, persistent joint swelling lasting as long as several years is seen in about 10% of adult patients with Lyme arthritis following appropriate antibiotic therapy. The inability to detect borreliae in joint aspirates or tissues using polymerase chain reaction (PCR) has led to a proposed autoimmune etiology.

OTHER DISEASES TRANSMITTED BY HARD-BODIED TICKS Hard-bodied ticks belonging to the I. ricinus species complex are also vectors for the infectious agents Ehrlichia phagocytophila, Babesia microti, and tick-borne encephalitis (TBE) virus. E. phagocytophila is a small intracellular gramnegative coccobacillus that parasitizes neutrophils (human granulocytic ehrlichiosis – HGE). Once taken up into phagosomes, the bacteria prevent fusion with lysosomes and replicate forming membrane-enclosed masses termed morulae. Disease presents as a flu-like illness with leukopenia and thrombocytopenia. Most infected individuals require hospitalization and severe complications are not infrequent. B. microti is an intracellular sporozoan parasite that causes babesiosis. The infectious stage, termed pyriform (pearshaped) bodies, enter the bloodstream and infect erythrocytes. Within the erythrocyte, trophozoites replicate by binary fission forming tetrads. The erythrocytes lyse releasing merozoites, which may infect new red blood cells. Disease presents as a flu-like illness leading to hemolytic anemia and renal failure. Hepatomegaly and splenomegaly may be observed in advanced disease. TBE virus is a member of the Flaviviridae. The disease spectrum ranges from a mild febrile illness to meningitis, encephalitis or meningoencephalitis. Chronic or permanent neuropsychiatric sequelae are observed in as many as 20% of infected patients.

4. HOW IS THE DISEASE DIAGNOSED, AND WHAT IS THE DIFFERENTIAL DIAGNOSIS?

Figure 2.6 This 2007 photograph depicts the pathognomonic erythematous rash in the pattern of a “bull’s-eye”, which manifested at the site of a tick bite on this Maryland woman’s posterior right upper arm. She subsequently manifested Lyme disease. Lyme disease patients who are diagnosed early and receive proper antibiotic treatment usually recover rapidly and completely. A key component of early diagnosis is recognition of the characteristic Lyme disease rash called erythema migrans. This rash often manifests itself in a “bull’s­ eye” appearance and is observed in about 80% of Lyme disease patients. Courtesy of the Centers for Disease Control, Atlanta, Georgia. Image is found in the Public Health Image Library #9874. Additional photographic credit is given to Dr James Gathany who took the photo in 2007.

In patients with signs and symptoms consistent with Lyme disease, a diagnosis is confirmed by antibody detection tests. The current recommendation from the US Centers for Disease Control and Prevention (CDC) (https://www.cdc.gov/ lyme/diagnosistesting/) is for a two-test approach consist­ ing of a sensitive enzyme immunoassay (EIA) or immuno­ fluorescence assay (IFA) followed by a Western immunoblot. Similar tests are used worldwide. All specimens positive or equivocal by the EIA or IFA should be tested by a standard­ ized Western immunoblot. Specimens negative by the EIA or IFA do not require further testing unless clinically indicated. The EIA or IFA can be performed either as a total Lyme titer or as separate IgG and IgM titers. If a Western immunoblot 17

9780367696399_Lydyard.indb 17

13/02/2023 07:32

Case Studies in Infectious Disease

is performed during the first 4 weeks of disease onset, both IgM and IgG immunoblots should be performed. A positive IgM test result alone is not recommended for use in determin­ ing active disease in persons with illness of greater than one month’s duration because the likelihood of a false-positive test result for a current infection is high for these persons. If a patient with suspected early Lyme disease has a negative serology, serologic evidence of infection is best obtained by testing paired acute- and convalescent-phase serum samples. Serum samples from persons with disseminated or late-stage Lyme disease almost always have a strong IgG response to B. burgdorferi antigens. For an IgM immunoblot to be considered positive, two of the following three bands must be present: 24 or 21 kDa (OspC) (the apparent molecular mass of OspC is dependent on the strain of B. burgdorferi being tested), 39 kDa (BmpA), 41 kDa (Fla). For an IgG immunoblot to be considered positive, five of the following 10 bands must be present: 18 kDa, 21 or 24 kDa (OspC, see above), 28 kDa, 30 kDa, 39 kDa (BmpA), 41 kDa (Fla), 45 kDa, 58 kDa (not GroEL), 66 kDa, and 93 kDa. The different genospecies of B. burgdorferi sensu lato in Europe make the serologic diagnosis of Lyme disease more complex and the standardization of tests, particularly the Western blot, more difficult.

5. HOW IS THE DISEASE MANAGED AND PREVENTED? MANAGEMENT

Routine use of antibiotics or serologic testing after a tick bite is not recommended. Persons who remove attached ticks (see below) should be monitored for up to one month for signs and symptoms of Lyme and other tick-borne diseases. Persons developing EM or other illness should seek medical attention. The recommended antimicrobial regimens and therapy for patients with Lyme disease is shown in Table 2.1. More information on antibiotic therapy can be found at https:// www.cdc.gov/lyme/treatment/index.html and https://www. idsociety.org/practice-guideline/lyme-disease/.

PREVENTION The best method of preventing Lyme disease is to avoid tickinfested areas. If this is not feasible then the following are recommended: • wear light-colored clothing so that ticks can be more easily spotted; • tuck trouser cuffs into socks or boots and tuck shirts into trousers so that ticks cannot crawl under clothing;

DIFFERENTIAL DIAGNOSIS The following conditions should be considered in the differential diagnosis: calcium pyrophosphate deposition disease, fibromyalgia, gonococcal arthritis, gout, meningitis, psoriatic arthritis, rheumatoid arthritis, syncope, systemic lupus erythematosus (SLE), and urticaria.

• wear a long-sleeved shirt and hat; • use DEET (meta-N,N-diethyl toluamide) tick repellent on skin; • spray clothing and boots with permethrin;

Table 2.1 Recommended treatment for adults with Lyme borreliosis Manifestation

Antibiotic

Treatment duration (days)

Erythema migrans, borrelial lymphocytoma or

Doxycycline

10

Amoxicillin

14

acrodermatitis chronica atrophicansa

Lyme meningitis, Cranial neuropathy or radiuclopathy Lyme encephalomyelitis Cardiac Lyme disease

Lyme arthritis

Cefuroxime axetil

14

Phenoxymethylpenicillin

14

Azithromycinb Doxycyclinec

5–10 14

Ceftriaxoned

14

Ceftriaxone Doxycycline Amoxicillin Cefuroxime axetil Ceftriaxone Doxycycline Amoxicillin Cefuroxime axetil Ceftriaxonee

14–28 14–21 14–21 14–21 14–21 28 28 28 14–28

a Treatment duration for borrelial lymphocytoma is 14 days for β-lactam and tetracycline antibiotics; the duration for acrodermatitis chronica atrophicans is 21–28 days. b For patients who are unable to take β-lactams or tetracyclines. c For ambulatory patients. d For hospitalized patients. e Used when there is only a minimal response to oral antibiotics. Adapted from the Infectious Diseases Society of America (IDSA) treatment guidelines. (From Steere A, Strle F, Wormser G et al. (2016) Nat Rev Dis Primers 2, 16090. https://doi.org/10.1038/nrdp.2016.90. With permission from Nature Springer.)

18

9780367696399_Lydyard.indb 18

13/02/2023 07:32

Case 2: Borrelia burgdorferi and related species

• check the entire body for ticks every day; • remove any attached ticks as soon as possible since it takes about 36 hours of attachment before borreliae are transmitted.

VACCINES Currently, no vaccine is available for the prevention of Lyme disease. However, in December 1998, the US Food and Drug Administration licensed the LYMErix™ vaccine against Lyme disease for human use. In February 2002, the vaccine was withdrawn from the market, reportedly because of poor sales but more likely because of the fear of inducing autoimmunity. LYMErix™ contained lipidated recombinant OspA from

B. burgdorferi sensu stricto. The vaccine was targeted for use in persons aged 15–70 years at high risk of exposure to infected ticks. Interestingly, OspA is not expressed by spirochetes in infected humans, but anti-OspA IgG antibodies in human blood are taken up by the infected tick during feeding and kill the borreliae in the tick hind-gut, preventing transmission. Despite withdrawal from the market, research on modified OspA-based vaccines and other spirochaete protein-based vaccines has continued. With modified multivalent OspA­ based vaccines lacking a T-cell epitope thought to have autoreactive potential. Phase I and phase II trials have confirmed that the vaccine is safe and induces significant antibody responses but no attempt has been made to bring the vaccine to market.

SUMMARY 1. WHAT IS THE CAUSATIVE AGENT, HOW DOES IT ENTER THE BODY, AND HOW DOES IT SPREAD A) WITHIN THE BODY AND B) FROM PERSON TO PERSON? ■ Lyme disease is the most common tick-borne disease in North America and Europe. ■ There are three genospecies within the Borrelia burgdorferi complex: B. burgdorferi (sensu lato), B. burgdorferi (sensu stricto), B. garinii, and B. afzelii. ■ Strains found in North America belong to B. burgdorferi sensu stricto, whereas the other two species are found in Europe and Asia. ■ The bacteria are maintained in an enzootic cycle involving hardbodied ticks belonging to the Ixodes ricinus species complex and a wide range of reservoir vertebrate hosts. ■ Because the ticks are slow feeders, transfer of the borreliae from the infected tick to a vertebrate host probably does not occur unless the tick has been attached to the body for about 36 hours. ■ Ticks transmit Lyme disease to humans generally during the nymph stage. ■ Borreliae are microaerophilic spirochetes that are extremely difficult to culture because of their complex nutrient requirements. ■ B. burgdorferi sensu lato are detected by the immune response that they induce in blood of the infected person. ■ The bacteria have a gram-negative wall structure and have endoflagella within the periplasmic space. ■ Borrelia species have linear chromosomes and contain circular and linear plasmids with some species containing more than 20 different plasmids. ■ Borreliae are adept at evading host immunity by varying the lipoproteins on their outer surface. ■ The Osps play an important role in the life cycle of the spirochete by interacting with intercellular and cellular components of its arthropod and vertebrate hosts.

■ B. burgdorferi selectively expresses specific Osps during distinct phases of its life cycle and in specific tissue locations. ■ B. burgdorferi sensu lato establishes a localized infection in the skin at the site of the tick bite, producing a characteristic skin lesion known as erythema migrans (EM). ■ In addition, it may disseminate via the bloodstream and/or lymphatics to the CNS, heart, joints, and eyes, all of which may become chronically infected. ■ Lyme disease is not spread person to person. ■ Rarely, borreliae may cross the placenta leading to infection of the fetus.

2. WHAT IS THE HOST RESPONSE TO THE INFECTION AND WHAT IS THE DISEASE PATHOGENESIS? ■ Because the spirochete is delivered to the host via the bite of a tick, it bypasses the physical barrier of the intact skin and the antimicrobial factors in sweat. ■ The innate pathways of the complement cascade, macrophages, and dendritic cells are the first line of defense in the skin. ■ B. burgdorferi activates both the innate and classical pathways of the complement cascade but is resistant to complement­ mediated lysis. ■ Macrophages and polymorphonuclear leukocytes can phagocytose and kill borreliae without the need for opsonization by complement or antibody. ■ IgM and IgG antibodies play the principal role in the clearance of B. burgdorferi. ■ B. burgdorferi undergoes antigenic variation and modulates the expression of Osps on the cell surface during infection. ■ The pathogenesis of Lyme disease is not well understood. ■ Tissue injury results from the inflammatory response mounted against the borreliae. ■ Chemokines may be the principal mediators of inflammation in Lyme disease.

Continued...

19

9780367696399_Lydyard.indb 19

13/02/2023 07:32

Case Studies in Infectious Disease

...continued

3. WHAT IS THE TYPICAL CLINICAL PRESENTATION AND WHAT COMPLICATIONS CAN OCCUR? ■ Lyme disease can be divided into three stages: (1) erythema migrans and some associated symptoms; (2) intermittent arthritis, cranial nerve palsies and nerve pain, atrioventricular node block, and severe malaise and fatigue; (3) prolonged arthritis, chronic encephalitis, myelitis, and parapareses (partial paralysis of the

■ All specimens positive or equivocal by the EIA or IFA should be tested by a standardized Western immunoblot. Specimens negative by the EIA or IFA do not require further testing. ■ The following conditions should be considered in the differential diagnosis: calcium pyrophosphate deposition disease, fibromyalgia, gonococcal arthritis, gout, meningitis, psoriatic arthritis, rheumatoid arthritis, syncope, systemic lupus erythematosus, and urticaria.

lower limbs). ■ The manifestations of Lyme disease are related to the particular genospecies of Borrelia involved.

4. HOW IS THE DISEASE DIAGNOSED, AND WHAT IS THE DIFFERENTIAL DIAGNOSIS? ■ In patients with signs and symptoms consistent with Lyme disease, a diagnosis is confirmed by antibody detection tests comprising an enzyme immunoassay (EIA) or immunofluorescence assay (IFA) followed by a Western immunoblot.

FURTHER READING Goering R, Dockrell H, Zuckerman M, Chiodini PL. Mims’ Medical Microbiology and Immunology, 6th edition. Elsevier, Philadelphia, 2018. Murray PR, Rosenthal KS, Kobayashi GS, Pfaller MA. Medical Microbiology, 9th edition. Elsevier, Philadelphia, 2021.

REFERENCES Hyde JA. Borrelia burgdorferi Keeps Moving and Carries on: A Review of Borrelial Dissemination and Invasion. Front Immunol, 8: 114, 2017. Kullberg, BJ, Vrijmoeth HD, van de Schoor F, Hovius JW. Lyme Borreliosis: Diagnosis and Management. BMJ, 369: m1041, 2020. Kurokawa C, Lynn GE, Pedra JHF, et al. Interactions Between Borrelia burgdorferi and Ticks. Nat Rev Microbiol, 18: 587–600, 2020. Margos G, Fingerle V, Reynolds S. Borrelia bavariensis: Vector Switch, Niche Invasion, and Geographical Spread of a Tick-Borne Bacterial Parasite. Front Ecol Evol, 7: 401, 2019.

5. HOW IS THE DISEASE MANAGED AND PREVENTED? ■ The preferred oral regimens for erythema migrans and early disease are amoxicillin 500 mg three times daily, doxycycline 100 mg twice daily or cefuroxime 500 mg twice daily for 14 days. ■ Meningitis or radiculopathy require parenteral ceftriaxone 2g IV for 14 days. ■ Late disease can be treated for 28 days with the oral regimen, except for central or peripheral nervous system disease, which should be treated using the parenteral regimen.

Ozdenerol E. GIS and Remote Sensing Use in the Exploration of Lyme Disease Epidemiology. Int J Environ Res Public Health, 12: 15182–15203, 2015. Schwartz AM, Kugeler KJ, Nelson CA, et al. Use of Commercial Claims Data for Evaluating Trends in Lyme Disease Diagnoses, United States, 2010–2018. Emerg Infect Dis, 27: 499–507, 2021. Steere AC, Strle F, Wormser GP, et al. Lyme Borreliosis. Nat Rev Dis Primers, 2: 16090, 2016.

WEBSITES European Centre for Disease Prevention and Control, Tick maps: https://www.ecdc.europa.eu/en/disease-vectors/ surveillance-and-disease-data/tick-maps Lyme Disease Association, Inc.: http://www.lymedisease association.org WHO, Vector-borne diseases: https://www.who.int/news­ room/fact-sheets/detail/vector-borne-diseases Students can test their knowledge of this case study by visiting the Instructor and Student Resources: [www. routledge.com/cw/lydyard] where several multiple choice questions can be found.

20

9780367696399_Lydyard.indb 20

13/02/2023 07:32

3

Campylobacter jejuni

A 35-year-old man had been feeling unwell for a few days with nonspecific aches and pains in his joints and a slight headache. He put this down to a barbecue he had attended a few days previously, where he had also consumed a considerable amount of alcohol. The following day, he felt rather worse with severe colicky abdominal pain and he developed bloody diarrhea, going to the lavatory 10 times during the day. This persisted overnight and he attended his local hospital’s Emergency Department. On examination, he was dehydrated and rather pale. He was admitted to hospital for intravenous (IV) rehydration and blood and

1. WHAT IS THE CAUSATIVE AGENT, HOW DOES IT ENTER THE BODY AND HOW DOES IT SPREAD A) WITHIN THE BODY AND B) FROM PERSON TO PERSON? CAUSATIVE AGENT

Campylobacter jejuni is a slender, motile non-spore-forming curved gram-negative bacterium (Figure 3.1) measuring 0.2–0.5 μm wide by 0.5–5.0 μm long, with a single unsheathed polar flagellum. It is resistant to complement. C. jejuni also produces superoxide dismutase (sodB), an enzyme which catalyzes the breakdown of superoxide radicals, thus protecting the organism from an inflammatory response. It has a complex respiratory chain and can use other electron acceptors instead of oxygen giving it phenotypic versatility. Its genome has been sequenced. Several species of Campylobacter exist, two of which cause the majority of human disease: C. jejuni and C. coli. A small number of other “emerging” campylobacters have also been linked to human disease (Table 3.1). Two closely related genera are: Arcobacter and Helicobacter (see Case study 12). C. jejuni has a non-typical gram-negative cell wall structure with lipo-oligosaccharide (LOS) rather than lipopolysaccharide (LPS). C. jejuni is subdivided into Penner serogroups based on the antigenic variation of the heat stable capsular polysaccharide antigens (HS) or Lior serogroups based on the heat labile capsular and LOS antigens (HL). Although serogrouping is being replaced by molecular typing methods, a distribution of the serotypes is valuable

feces samples were sent for culture. He was started on antibiotics and over the subsequent few days he improved with lessening of the symptoms and was discharged home. Some weeks later, he began to develop weakness in his feet, which gradually affected his legs. He contacted his primary care physician who admitted him to hospital once again. Over the subsequent few days, the paralysis affected his upper leg muscles, and gradually over the ensuing weeks slowly resolved with treatment.

for developing a vaccine and as certain Penner/Lior groups are associated with immune-mediated diseases (see below). Campylobacter can be typed by molecular methods such as restriction fragment length polymorphism (RFLP), multilocus enzyme electrophoresis (MLEE) and whole genome sequencing (WGS), although the problem with this emerging technology is understanding the natural genome diversity of campylobacter species. The organism is microaerophilic, requiring an atmosphere of 5–10% oxygen and 10% CO2, growing on campylobacter

Figure 3.1 Campylobacter jejuni: a silver stain showing the “seagull” morphology of Campylobacter spp. Courtesy of the Centers for Disease Control, Atlanta, Georgia. Image is found in the Public Health Image Library #6654. Additional photographic credit is given to Robert Weaver, PhD, who took the photo in 1980.

21

9780367696399_Lydyard.indb 21

13/02/2023 07:32

Case Studies in Infectious Disease

selective agar (Brucella blood agar with added trimethoprim, vancomycin and polymyxin B) within 2–5 days. The two main human pathogens are thermotolerant and can grow at 42°C, which is used as a selective condition for clinical isolates. Other campylobacters have been linked with human disease. C. fetus can cause intestinal or systemic infections in immunocompromised individuals or those who work with animals. C. concisus and C. showae, part of the oral flora, have been associated with inflammatory bowel disease, although not proven. C. curvus is associated with gastroenteritis, abscesses, and bacteremias. C. hyointestinalis found in the intestine of pork, is an opportunist pathogen in humans causing diarrheal disease. C. sputorum has been isolated from gastroenteritis, abscesses, and blood (Table 3.1). C. jejuni is found widely in the animal kingdom and is part of the normal flora of many food-source animals, for example, poultry, lamb, and pig. The commonest meat source is poultry. Campylobacteriosis is thus a zoonosis, with infection being acquired principally from eating poorly cooked meats. Each year in the EU approximately 23 million people become ill from eating contaminated food. Infection can also be acquired from raw milk contaminated at source, contaminated water sources, or from close contact with animals such as in children’s zoos, infected dogs or bird excreta. Sexual contact is also a risk factor since MSM have 14 times the chance of acquiring Campylobacter compared to controls. Those who were positive for Campylobacter were also 74 times more likely to have Shigella.

ENTRY INTO THE BODY Entry of the organism is via the mucosal surfaces of the intestine where it mostly remains. The infectious dose is about 500–1000 organisms and colonization affects the small and large intestine, with the terminal ileum and colon affected most often. Mesenteric adenitis regularly occurs. The shape and motility of the organisms enable them to penetrate the mucus layer where they adhere to the enterocytes and are internalized utilizing microfilament/microtubule-dependent mechanisms. Important in the internalization is the campylobacter invasion antigen (Cia), which is secreted by a flagellar secretion apparatus (the evolutionary forerunner of the type III secretion apparatus).

Table 3.1 Some species of Campylobacter associated with human disease Campylobacter

C. jejuni

C. coli C. curvus C. lari C. fetus C. conciscus C. showae C. hyointestinalis C. sputorum C. upsaliensis

EPIDEMIOLOGY

Campylobacteriosis is the most common bacterial GI infectious disease globally. The true global incidence is unknown but, in industrialized countries, the annual incidence is up to 9.3/100 000 population with all ages being susceptible. Globally, the three common Penner types are HS4c, HS2, and HS1/4cc. In Europe, HS4c and HS2 are equally common (17.3% and 15.3%), in Asia 8.9% and 11.8%, in N. America 23.5% and 10.7%, in Oceania 17.4% and 18.2%, and in Africa 7.0 % and 6.2% respectively. In all regions, HS1/44c is the least common and varies between 10.5% and 4.2%. Globally, eight serotypes represented over 50% of the isolates. In the UK in 2017, the number of reported cases was 56 729 (96.57/100 000) increasing from 49 891 (90.97/100 000) in 2008 with a peak in 2012 of 65 044 (114.88/100 000). In the EU, it has been the most reported GI disease since 2005 affecting 220 000 individuals in 2019. There is a seasonality to infection, with most cases in different countries occurring in the spring– summer and declining in the winter months. In the UK, the highest rate of laboratory-confirmed cases occurs in June/ July with the highest number of cases in the 50–59 age group. Confirmed disease is notifiable in Austria, Denmark, Finland, Germany, Italy, Sweden, and Norway. In the UK, a case of food poisoning or a case of diarrhea with blood are notifiable; both can relate to campylobacter but also to other infections. Isolation of campylobacter is notifiable by the laboratory.

SPREAD WITHIN THE BODY C. jejuni rarely causes a bacteremia, and remains in the gastrointestinal (GI) tract, causing a terminal ileitis and colitis. Systemic infections are more commonly associated with Campylobacter fetus, which principally causes diseases of the reproductive system in sheep or cattle and infections in immunocompromised humans. C. fetus possesses an S-layer (protein microcapsule), which is anti-phagocytic and may explain its tendency to cause septicemia.

PERSON-TO-PERSON SPREAD 22

Direct person-to-person spread is uncommon.

9780367696399_Lydyard.indb 22

2. WHAT IS THE HOST RESPONSE TO THE INFECTION AND WHAT IS THE DISEASE PATHOGENESIS? INNATE IMMUNITY

Following invasion of the enterocytes, an acute IL-8 induced inflammatory reaction occurs with infiltration of the lamina propria by granulocytes. A variety of pathogen recognition receptors on and in the enterocytes are triggered, for example TLRs, and this leads to the production of pro-inflammatory

13/02/2023 07:32

Case 3: Campylobacter jejuni

cytokines that play a role in generating the acute inflammation reaction.

ADAPTIVE IMMUNITY Soon after infection, the host mounts an antibody response, which peaks at about 2 weeks and declines over the following weeks. The main antigens are flagella, outer-membrane proteins (OMPs), and LOS. Since antibodies to OMP and LOS cross-react with myelin components, this can lead to the development of Guillain-Barré or the Miller Fisher syndromes (see below). Little is known about the role of the cell-mediated response. However, CD4+ T-helper cells are involved in the production of IgG and IgA antibodies to the bacterial antigens. Secretory IgA in both the intestinal secretions and in maternal milk are important in protection against further infection and against children in early life getting severe disease.

PATHOGENESIS Following infection, ulceration of the epithelium with formation of crypt abscesses occurs. Motility is essential for initial colonization of the GI tract and a variety of surface proteins are important for: adhesion – CadF, FlpA which bind to fibronectin cell; invasion – CapC an autotransported protein involved in adhesion and inflammation; ciaB (a protein similar to a Type III secretion protein) which is internalized by the gastric cell; iamA (invasion-associated marker, function unknown) and Cas9 which appears to regulate the campylobacter virulence factors. The bacteria also produce a

cytolethal distending toxin (cdtA, cdtB, cdtC) which is present in most strains of C. jejuni and a similar toxin is found in other bacteria. The toxin induces cell-cycle arrest leading to apoptosis. The cdtA and cdtC components form the binding subunit which attaches to lipid rafts in the cell membrane. The cdtB unit is the active toxic component having DNAse and phosphatidylinositol-3, 4, 5-triphosphate (PIP3) phosphatase activity which, after internalization by endocytosis, enters the nucleus and causes DNA damage, cell-cycle arrest, cell distention, and apoptosis. The toxin also affects cell signaling and induces a pro-inflammatory response by up-regulating cytokines IL-6 IL-8. C. jejuni also has a Type VI secretion system (T6SS) with a single hemolysin-coregulated protein (Hcp) the possession of which is associated with bloody diarrhea. The protein is important in adhesion and invasion of intestinal cells. In animal studies, IFN producing innate lymphoid cells (ILCs) induce intestinal pathology caused by C. jejuni. This distinct population of cells NK1.1-T-bet+ ILCs accumulate in the intestine after infection with campylobacter. They develop from ROR t+ progenitors and promote the pro-inflammatory response. Neurologic complications (e.g. Guillain-Barré syndrome, Miller-Fisher syndrome) may occur, caused by antibodies to the LOS and cross-reacting with GM1 gangliosides at the nodes of Ranvier, affecting Schwann cells and ion channels leading to a conduction block (Figure 3.2). The cause of the fluid loss is due to a direct effect of the organism affecting the sodium channel proteins and indirectly due to the immune response generated by the organism.

Figure 3.2 Putative mechanism for development of Guillain-Barré syndrome. Antibodies raised against the lipo-oligosaccharide (LOS) component of the cell wall of some Campylobacter strains cross-react with GM1 gangliosides in the myelin sheath of the nerve, due to a structural similarity, leading to damage, loss of nerve conduction, and paralysis.

9780367696399_Lydyard.indb 23

23

13/02/2023 07:32

Case Studies in Infectious Disease

Campylobacter inhibits expression of the β and subunits of the sodium channel in the colonic epithelial cells leading to sodium malabsorption. In addition, the organism causes the redistribution of tight junction proteins. Occludin is cleaved by HtrA protease (high temperature requirement protein A) and is displaced to the cytoplasm with ZO1 to the lateral plasma membrane along with claudins 1, 3, 4, 5, and 8. The organism gains access to the basolateral membrane and entry into the epithelial cells via the fibronectin receptor inducing cellular apoptosis leading to epithelial barrier porosity for fluids.

3. WHAT IS THE TYPICAL CLINICAL PRESENTATION AND WHAT COMPLICATIONS CAN OCCUR? The incubation period is commonly 1–3 days but can be as long as 8 days. Infection may be symptomless in a proportion of individuals. A typical clinical presentation is one of generalized systemic upset with fever and myalgia accompanied by abdominal pain and diarrhea. Diarrhea may range from a few loose motions to profuse and watery or grossly bloody. The symptoms may last from one or two days to over a week in about 20% of patients and are generally self-limiting, although relapses may occur in about 10% of patients. In severe cases of enterocolitis, toxic megacolon may occur and rare cases of cholecystitis or pancreatitis have been reported. Three major extraintestinal complications are recognized. The commonest complication is reactive arthritis, which occurs in about 1% of cases and occurs several weeks after the initial infection. It is more common in subjects who are HLA B27. The hemolytic uremic syndrome may also follow infection and usually comes on within a few days. Infection may be followed by the Guillain-Barré or the Miller Fisher syndrome variant. Although a rare complication, because campylobacteriosis is so common, an infection with C. jejuni is the cause of these immune pathologies in about 50% of cases. Other extraintestinal complications that have occasionally been reported are IgA nephropathy and interstitial nephritis.

4. HOW IS THIS DISEASE DIAGNOSED AND WHAT IS THE DIFFERENTIAL DIAGNOSIS? Campylobacter enteritis is confirmed by the isolation of the organism from the feces using a charcoal-based selective medium and culturing under microaerobic conditions at 42°C. Both a polymerase chain reaction (PCR) and an immunochromatographic assay for antigen detection are also available. In resource-limited countries, a rapid means of provisional diagnosis is to look at a wet preparation of

feces under the microscope, where the rapid motion of the Campylobacter can be seen – in contrast to Shigella dysentery where there is no obvious microbial movement as Shigella are nonmotile. Pus cells are also prominent. C. jejuni may also be detected using the PCR and immunochromatographic assays which can give a rapid (15 min) turnaround time.

DIFFERENTIAL DIAGNOSIS It is not possible to make a microbiological diagnosis from the clinical presentation, so all the other causes of gastroenteritis are in the differential diagnosis such as Shigella, Salmonella, Escherichia coli, and so forth. Additionally, as the presentation may be severe with bloody diarrhea, inflammatory bowel disease may also be confused with campylobacteriosis.

5. HOW IS THE DISEASE MANAGED AND PREVENTED? MANAGEMENT

The disease is self-limiting and, in most cases, no therapy is required. In severe cases, particularly if it lasts for several days, the patient may have to be admitted to hospital for rehydration. Antibiotics should also be given in severe cases. Erythromycin or ciprofloxacin are the two most frequently used, although for both agents, antibiotic-resistant isolates are increasing in prevalence due to overuse of antibiotics in both human and veterinary practice. In the presence of macrolide- or quinolone­ resistant isolates, alternative agents such as tetracycline (except in children) or clindamycin may be used. Campylobacter is not susceptible to β-lactams, with the exception of co-amoxiclav. If systemic spread occurs, gentamicin can be used but is not useful for the enteric infection.

PREVENTION Preventive measures rely mainly on adequate food hygiene and establishing disease-free animal stocks. In order to reduce contamination of chickens, a variety of approaches have been adopted such as acidification of feed water, feed supplements such as prebiotics and bacteriophages and biosecurity measures. Important areas of cross contamination in kitchens are sinks, after washing chickens in running water, lack of hand washing, and cutting boards. An established vaccine does not yet exist, although several trial vaccines for chickens and humans are in trial. The most effective chicken vaccines are based on either total outermembrane proteins or cysteine ABC transporter substrate binding protein, but a capsule-conjugated vaccine seems to be promising. The two leading contenders for a human vaccine are a capsule-conjugate vaccine and a DNA-based vaccine.

24

9780367696399_Lydyard.indb 24

13/02/2023 07:32

Case 3: Campylobacter jejuni

SUMMARY 1. WHAT IS THE CAUSATIVE AGENT, HOW DOES IT ENTER THE BODY, AND HOW DOES IT SPREAD A) WITHIN THE BODY AND B) FROM PERSON TO PERSON? ■ Campylobacter is microaerophilic. ■ The cell wall has an atypical gram-negative structure. ■ The antigenic variation of the capsular polysaccharide is the basis for the Penner groups. ■ It is one of the commonest causes of gastroenteritis globally. ■ Infection occurs at any age, with peaks in children and young adults. ■ Infection is a zoonosis. ■ The commonest food source is poultry. ■ The organism is located in the small and large intestine.

2. WHAT IS THE HOST RESPONSE TO THE INFECTION AND WHAT IS THE DISEASE PATHOGENESIS? ■ Motility and the organism’s spiral shape are important in colonization. ■ The infectious dose is about 500–1000 organisms. ■ Cell invasion involves Campylobacter invasion antigens. ■ Invasion involves the microtubules. ■ Innate immunity leads to pro-inflammatory cytokines and acute inflammation. ■ Campylobacter produces a cytolethal distending toxin, which causes cell-cycle arrest and apoptosis.

FURTHER READING Fischer GH, Paterek E. Campylobacter. StatPearls, Treasure Island, 2022. Ketley JM, Konkel ME. Campylobacter: Molecular & Cellular Biology. Garland Science, Abingdon, 2005. Nachamkin I, Blaser MJ. Campylobacter. Blackwell Publishing, Oxford, 2000.

REFERENCES Bucker R, Krug SM, Moos V, et al. Campylobacter jejuni Impairs Sodium Transport and Epithelial Barrier Function via Cytokine Release in Human Colon. Mucosal Immunol, 11: 474–485, 2018. Cardoso MJ, Ferreira V, Truninger, M, et al. CrossContamination Events of Campylobacter spp. in Domestic Kitchens Associated with Consumer Handling Practices of Raw Poultry. Int J Food Microbiol, 338: 108984, 2021. Desvaux M, Hebraud M, Henderson IR, Pallen MJ. Type III Secretion: What’s in a Name? Trends Microbiol, 14: 157–160, 2006.

9780367696399_Lydyard.indb 25

■ Histologically, there is epithelial ulceration and infiltration by granulocytes. ■ Antibodies produced in response to the organism can be protective but some can cross-react with myelin.

3. WHAT IS THE TYPICAL CLINICAL PRESENTATION AND WHAT COMPLICATIONS CAN OCCUR? ■ The incubation period is 1–8 days, usually 1–3 days. ■ Presents with constitutional symptoms. ■ Gastrointestinal symptoms are colicky abdominal pain and diarrhea, which may contain blood. ■ Complications include megacolon, reactive arthritis, hemolytic uremic syndrome, and Guillain-Barré syndrome.

4. HOW IS THIS DISEASE DIAGNOSED, AND WHAT IS THE DIFFERENTIAL DIAGNOSIS? ■ Diagnosis is by culture on selective medium incubated microaerobically. ■ Differential diagnosis is other bacterial causes of diarrhea and dysentery including Shigella, Salmonella, E. coli, and inflammatory bowel disease.

5. HOW IS THE DISEASE MANAGED AND PREVENTED? ■ Most infections require no treatment. ■ In severe cases, rehydration and antibiotics may be required. ■ Prevention is largely by adequate food processing and hygiene.

Dingle KE, Colles FM, Wareing DRA, et al. Multilocus Sequence Typing for Campylobacter jejuni. J Clin Microbiol, 39: 14–23, 2001. Elgamoudi BA, Andrianova EP, Shewell LK, et al. The Campylobacter jejuni Chemoreceptor Tlp10 has a Bimodal Ligand-Binding Domain and Specificity or Multiple Classes of Chemo-Effectors. Sci Signal, 14: eabc8521, 2021. Harrer A, Bücker R, Boehm M, et al. Campylobacter jejuni Enters Gut Epithelial Cells and Impairs Intestinal Barrier Function Through Cleavage of Occludin by Serine Protease HtrA. Gut Pathol, 11: 4, 2019. Karenlampi R, Rautelin H, Hanninen ML. Evaluation of Genetic Markers and Molecular Typing Methods for Prediction of Sources of Campylobacter jejuni and C. coli Infections. Appl Environ Microbiol, 73: 1683–1685, 2007. Konkel ME, Monteville MR, Rivera-Amill V, Joens LA. The Pathogenesis of Campylobacter jejuni-Mediated Enteritis. Curr Issues Intest Microbiol, 2: 55–71, 2001. Lu T, Marmion M, Ferone M, et al. On Farm Interventions to Minimise Campylobacter spp. Contamination in Chicken. Br Poult Sci, 62: 53–67, 2021. Muraoka WT, Korchagina AA, Xia Q, et al. Campylobacter Infection Promotes IFN -Dependent Intestinal Pathology via 25

13/02/2023 07:32

Case Studies in Infectious Disease

ILC3 to ILC1 Conversion. Mucosal Immunol, 14: 703–716, 2021. Nylen G, Dunstan F, Palmer SR, et al. The Seasonal Distribution of Campylobacter Infection in Nine European Countries and New Zealand. Epidemiol Infect, 128: 383–390, 2002. Parkhill J, Wren BM, Mungall E, et al. The Genome Sequence of the Food Borne Pathogen Campylobacter jejuni Reveals Hypervariable Sequences. Nature, 403: 665–668, 2000. Payot S, Bolla JM, Corcoran D, et al. Mechanisms of Fluoroquinolone and Macrolide Resistance in Campylobacter spp. Microb Infect, 8: 1967–1971, 2006. Prokhorova TA, Nielsen PN, Petersen J, et al. Novel Surface Polypeptides of Campylobacter jejuni as Travellers Diarrhoea Vaccine Candidates Discovered by Proteomics. Vaccine, 24: 6446–6455, 2006.

Solomon T, Willison H. Infectious Causes of Acute Flaccid Paralysis. Curr Opin Infect Dis, 16: 375–381, 2005.

WEBSITES Centers for Disease Control and Prevention (CDC), Morbidity and Mortality Weekly Report, 2007: http://www. cdc.gov/mmwr/preview/mmwrhtml/mm5614a4.htm Centre for Infections, Health Protection Agency, HPA Copyright, 2008: http://www.hpa.org.uk/infections/topics_az/ campy/menu.htm Students can test their knowledge of this case study by visiting the Instructor and Student Resources: [www. routledge.com/cw/lydyard] where several multiple choice questions can be found.

26

9780367696399_Lydyard.indb 26

13/02/2023 07:32

4

Candida albicans

A 38-year-old man presented to his GP complaining of acute pain, numbness and tingling of the lower extremities and evidence of ischemia. There were no recent flare ups of fever, chills, sweating, respiratory, gastroenteritis, or urinary tract infection. The patient’s notes, however, revealed that as a result of endocarditis, he had previously suffered from a mycotic aneurysm and underwent aortic valve replacement 17 months prior to the visit. A second episode of endocarditis occurred post operation, but since no organism was recovered and the patient was treated empirically with a combination of vancomycin and ceftriaxone. The patient was also on long-term anticoagulation therapy (warfarin) due to associated deep vein

1. WHAT IS THE CAUSATIVE AGENT, HOW DOES IT ENTER THE BODY AND HOW DOES IT SPREAD A) WITHIN THE BODY AND B) FROM PERSON TO PERSON? CAUSATIVE AGENT

Candida is a commensal fungus found in the gastrointestinal (GI) tract, genitourinary tract, mouth, and skin. In healthy individuals, it does no harm. However, in immunocompromised people Candida can cause a range of pathologic conditions from mild superficial infections to life-threatening invasive candidiasis (IC) which comprises candidemia (bloodstream infection), disseminated and deep-seated candidiasis. The Candida species most frequently isolated from severe IC are C. albicans, C. glabrata, C. parapsilosis, and C. tropicalis. C. auris has emerged as a global pathogen over the past decade, with the majority of cases associated with high mortality. The shift toward other non-albicans Candida species is thought to be the result of selective pressure by first-line drugs effectively used against invasive infections caused by C. albicans, in particular, fluconazole and echinocandins (see Section 5). However, C. albicans remains the most frequent clinically isolated species, arguably the most successful, and the focus of this case. In response to environmental pressure, C. albicans appears in two reversible morphologic forms: unicellular budding yeast and a filamentous form (hyphae and pseudohyphae). Yeast cells have a round-to-oval morphology, while hyphae consist of firmly attached branched tubular cells. Pseudohyphae

thrombosis (DVT). Laboratory tests showed elevated white blood cell (WBC), neutrophilia with hypersegmented neutrophils and anemia. Clinical chemistry results revealed decreased serum iron, increased ferritin, increased lactate dehydrogenase and abnormal coagulation. The patient was referred to his local hospital. Surgical intervention to correct occlusion of the distal aortae revealed white debris identified as Candida albicans. Aortic vegetation was determined to be the source of the infection. The patient was successfully treated with antifungal therapy: combination of amphotericin B and flucytosine followed by prolonged suppression with fluconazole.

resemble both yeasts and hyphae (Figure 4.1). Under certain conditions, C. albicans can undergo additional morphologic transitions to tochlamydospores, gray cells, white/opaque cells and a particular GUT phenotype characteristic of GI transition. Among the factors regulating the morphogenesis between yeast and hyphal forms are temperature and pH. Higher temperatures (37°C) and neutral pH favor hyphal morphology, while the yeast form is supported by lower temperatures (less than 30°C) and acidic pH (95% of IC. However, PCR essays can yield false-negative results in the case of a low number of fungal cells available in the sample as well as problems with DNA extraction and false positivity – due to sample contamination. There are currently no FDA-cleared PCR assays for Candida.

T2 Candida Nanodiagnostic Panel The FDA-approved automated T2 Candida nanodiagnostic panel assesses Candida directly in whole blood using magnetic resonance technology. The method is applicable for C. albicans/C. tropicalis, C. glabrata/C. krusei, and C. parapsilosis. Fluorescence in situ hybridization assay using peptide nucleic acid probes (PNA FISH) targets species-specific RNA in yeast and can identify C. albicans in a blood culture in three hours. A latex agglutination test is based on the detection of heat-labile Candida protein antigens in the serum such as a 47-kDa fragment of Hsp90, Eno1, Mp65 or Sap1/2. It has a

low sensitivity since, as mentioned above, Candida antigens rapidly disappear from the bloodstream. Candida mannans can also be measured using this test. Loop-mediated isothermal amplification (LAMP) is a relatively new technique based on the usage of specific primers for the yeast DNA targeting the ITS2 gene of C. albicans. This is a highly sensitive method allowing one to differentiate C. albicans samples from other yeast species. The LAMP method is as specific and precise as common diagnostic methods, but is faster, easier deployable, more sensitive, and can be used in the field. Differential diagnosis of candidiasis include the following: • Cutaneous candidiasis - Dermatitis (contact, allergic), folliculitis. • GI tract candidiasis – Esophagitis due to herpes simplex virus, herpes zoster, induced by radiation, gastroesophageal reflux disease. • Respiratory candidiasis – Bacterial pneumonia, viral pneumonia, tracheitis, Aspergillus pneumonia, pulmonary cryptococcosis. • For IC and/or candidemia, differential diagnosis should be considered from abdominal abscess, bacterial sepsis, and septic shock.

5. HOW IS THE DISEASE MANAGED AND PREVENTED? MANAGEMENT

Three classes of antifungal drugs are used as first-line treatment of IC: polyenes (amphotericin B), azoles (fluconazole), and echinocandins (anidulafungin, caspofungin, and micafungin). Amphotericin B and fluconazole target fungal sterol ergosterol, while anidulafungin affects the fungal cell wall through inhibition of β-1,3-glucan synthase. Due to the superior fungicidal activity and positive safety record, the echinocandins are now the first-line antifungal treatment for IC in both neutropenic and non-neutropenic critical patients. For most adults with IC, the initial recommended antifungal treatment is an echinocandin given IV. Fluconazole can be used as an alternative as initial therapy for those patients who are not critically ill and who are considered unlikely to have a fluconazole-resistant Candida infection. Other treatments include voriconazole and amphotericin B formulations. The treatment should continue for 2 weeks after clearance of Candida from the bloodstream and resolution of symptoms. For neonatal candidiasis, the recommended primary treatment is amphotericin B deoxycholate or fluconazole for

32

9780367696399_Lydyard.indb 32

13/02/2023 07:32

Case 4: Candida albicans

2 weeks after clearance of Candida from the bloodstream and resolution of symptoms. Other forms of IC, such as infections in the bones, joints, heart, or central nervous system (CNS), usually need to be treated for a longer period of time. However, some types of Candida are becoming increasingly resistant to the first-line and second-line treatment with fluconazole and the echinocandins. Therefore, new antifungal drugs have recently been introduced such as tetrazoles, encochleated amphotericin B, terpenoid derivative ibrexafungerp, rezafungin, the novel echinocandin with an enhanced half-life, and other drugs which are in the process of becoming licensed. For candidiasis of the mouth, throat, or esophagus, clotrimazole, miconazole, or nystatin are usually applied to the inside of the mouth for 7 to 14 days. For severe infections, fluconazole is taken orally or IV. The treatment for candidiasis in the esophagus is usually fluconazole. For vaginal candidiasis, an antifungal medicine is applied inside the vagina or a single dose of fluconazole is taken orally. For unresponsive or more severe infections, the doses of oral fluconazole can be increased or boric acid, nystatin, or flucytosine applied inside the vagina.

PREVENTION Candidiasis in the mouth and throat can be prevented through maintaining good oral health, rinsing the mouth or brushing

teeth after using inhaled corticosteroids, particularly for immunocompromised individuals. Wearing cotton underwear might help reduce the chances of getting Candida infection as well as taking antibiotics strictly as prescribed. In hospital environments, it is important to adhere to hand hygiene, and follow recommendations for placement and maintenance of central venous catheters. Patients who may benefit from antifungal prophylaxis are solid-organ transplant recipients, stem-cell transplant recipients with neutropenia, high-risk ICU patients, and those with chemotherapy-induced neutropenia. There are currently no vaccines available for the prevention of Candidiasis. However, there are some encouraging results. Two anti-Candida vaccines have reached the stage of clinical trials. • PEV7 contains recombinant aspartyl-proteinase 2 (Sap2), (an enzyme secreted by C. albicans), delivered via virosomes. • NDV-3 contains the recombinant N terminus of C. albicans agglutinin-like sequence 3 protein Als3p, which is a cell-surface adhesin and invasin. Aluminum hydroxide (Alum) is used as adjuvant. An alternative strategy would be to produce a vaccine based on common antigens expressed by multiple genera of fungi in order to target a broad range of mycoses.

SUMMARY 1. WHAT IS THE CAUSATIVE AGENT, HOW DOES IT ENTER THE BODY, AND HOW DOES IT SPREAD A) WITHIN THE BODY AND B) FROM PERSON TO PERSON? ■ Candida is a commensal fungus found in the GI tract, genitourinary tract, mouth, and skin. In healthy individuals, it does no harm. However, in immunocompromised people Candida can cause a range of pathologic conditions from mild superficial infections to life-threatening invasive candidiasis (IC). ■ C. albicans appears in two reversible morphologic forms: a unicellular budding yeast and filamentous form (hyphae and pseudohyphae). ■ As an opportunistic infection, C. albicans is capable of rapidly colonizing almost all anatomic sites in the body. In order to reach internal organs, Candida yeast adheres to endothelial cells by expressing a range of adhesion molecules. The yeast-to-hyphal transition of C. albicans is essential for the pathogen’s further spread. ■ The use of broad-spectrum antibiotics, immunosuppressive therapies

such

as

cancer

chemotherapy,

and

organ

transplantation, increases the probability of the development of invasive candidiasis. ■ Currently, C. albicans is a leading cause of hospital-acquired infections with 47% mortality in intensive care unit (ICU) patients.

2. WHAT IS THE HOST RESPONSE TO THE INFECTION AND WHAT IS THE DISEASE PATHOGENESIS? ■ Innate immunity is essential for preventing infection by the opportunist C. albicans. The phagocytosis of C. albicans by macrophages and polymorphonuclear neutrophils (PMNs) is critical for the effective clearance of C. albicans from the host tissues. ■ Fungal β-glucan is recognized by Dectin-1, a C-type lectin pathogen recognition receptor (PRR) expressed by phagocytic cells, leading to the activation of phagocytosis and the release of pro-inflammatory cytokines, reactive oxygen species (ROS) and reactive nitrogen (RNS) species. ■ The adaptive immune response against fungal commensals in the normal skin and gut is mainly through Th17 cells and through a small proportion of fungus-specific Th1 and Th2 cells. Continued...

33

9780367696399_Lydyard.indb 33

13/02/2023 07:32

Case Studies in Infectious Disease

...continued

■ Specific antibodies directed against fungal cell-wall components increase upon fungal infection and can impair the adherence and inhibit invasion of human oral epithelial cells by C. albicans through blocking induced endocytosis.

3. WHAT IS THE TYPICAL CLINICAL PRESENTATION AND WHAT COMPLICATIONS CAN OCCUR? ■ Candidiasis of the mouth and throat (oropharyngeal candidiasis or thrush) and esophagus presents as white patches or sores on the inner cheeks, tongue, roof of the mouth, and throat, accompanied by a cotton-like feeling in the mouth, loss of taste, and pain while eating. ■ Symptoms of candidiasis in the esophagus include pain and difficulty in swallowing. ■ Vaginal candidiasis usually first presents as a thick white or yellow vaginal discharge (leukorrhea) with itching and redness of the female genitalia (vagina and vulva), pain when urinating and during sexual intercourse. ■ The most common form of invasive candidiasis (IC) is candidemia, a bloodstream infection. ■ Systemic candidiasis infection can occur in the heart, kidney, bones, and other internal organs. The risk factors include: primary or acquired immunodeficiency, neutropenia, stem-cell transplantation, prolonged intensive care unit stay, central venous catheters, parenteral nutrition, being on hemodialysis due to the renal failure, and taking broad-spectrum antibiotics. ■ The most common symptoms of IC are fever and chills that don’t improve after antibiotic treatment for suspected bacterial infections. IC is often associated with high rates of morbidity and mortality, as well as increased length of hospital stays.

4. HOW IS THE DISEASE DIAGNOSED, AND WHAT IS THE DIFFERENTIAL DIAGNOSIS? ■ Blood cultures are still the gold standard method for the diagnosis of candidemia despite the variable sensitivity (21–71%) in the first stages of infection.

FURTHER READING Ostrosky-Zeichner L (ed). Fungal Infections, An Issue of Infectious Disease Clinics of North America. Elsevier, Philadelphia, 2021. Pappas PG, Lionakis MS, Arendup MC, et al. Invasive Candidiasis. Nat Rev Disease Primers, 4: 18026, 2018.

REFERENCES

34

Bojang E, Ghuman H, Kumwenda P, Hall RA. Immune Sensing of Candida albicans. J Fungi, 7: 119, 2021. Chow EWL, Pang LM, Wang Y. From Jekyll to Hyde: The Yeast–Hyphal Transition of Candida albicans. Pathogens, 10: 859, 2021.

9780367696399_Lydyard.indb 34

■ Serum assays include detection of the most abundant cell­ wall components mannan and 1,3-β-D-glucan (BDG). Antibody based assays have also been used for detecting specific serum IgG antibody levels to mannan. The sensitivity and specificity of the combined mannan/antimannan assay can reach 85%. ■ Various sources of Candida DNA can be used for PCR: blood, serum, plasma, cerebrospinal fluid and different tissues. The DNA targets include multi-copy broad-range panfungal genes as well as Candida-specific genes. However, PCR essays can yield false-negative results in the case of a low number of fungal cells in the sample. ■ The FDA approved automated T2 Candida nanodiagnostic panel assesses Candida directly in whole blood using magnetic resonance technology. ■ Loop-mediated isothermal amplification (LAMP) is a highly sensitive method allowing one to differentiate C. albicans samples from other yeast species.

5. HOW IS THE DISEASE MANAGED AND PREVENTED? ■ For most adults with IC, the initial recommended antifungal treatment is an echinocandin given intravenously. Fluconazole can be used as an alternative as initial therapy for those patients who are not critically ill and who are considered unlikely to have a fluconazole-resistant Candida infection. Other treatments include voriconazole and amphotericin B formulations. The treatment should continue for 2 weeks after clearance of Candida from the bloodstream and resolution of symptoms. ■ For candidiasis of the mouth, throat, or esophagus, clotrimazole, miconazole, or nystatin are usually applied to the inside of the mouth for 7 to 14 days. For severe infections, fluconazole is taken orally or intravenously. ■ For vaginal candidiasis, an antifungal medicine is applied inside the vagina or a single dose of fluconazole is taken orally. ■ There are currently no vaccines available for the prevention of Candidiasis.

Clancy CJ, Nguyen MH. Diagnosing Invasive Candidiasis. J Clin Microbiol, 56: e01909–17, 2018. Griffiths JS, Camilli G, Kotowicz NK, et al. Role for IL-1 Family Cytokines in Fungal Infections. Front Microbiol, 12: 633047, 2021. Hameed S, Hans S, Singh S, et al. Revisiting the Vital Drivers and Mechanisms of β-Glucan Masking in Human Fungal Pathogen, Candida albicans. Pathogens, 10: 942, 2021. Murphy SE, Bicanic T. Drug Resistance and Novel Therapeutic Approaches in Invasive Candidiasis. Front Cell Infect Microbiol, 11: 759408, 2021. Oliveira LVN, Wang R, Specht CA, Levitz SM. Vaccines for Human Fungal Diseases: Close but still a Long Way to Go. NPJ Vaccines, 6: 33, 2021.

13/02/2023 07:32

Case 4: Candida albicans

Scheffold A, Bacher P, LeibundGut-Landmann S. T-cell Immunity to Commensal Fungi. Curr Opin Microbiol, 58: 116–123, 2020. Tortorano AM, Prigitan A, Morroni G, et al. Candidemia: Evolution of Drug Resistance and Novel Therapeutic Approaches. Infect Drug Resist, 14: 5543–5553, 2021. Wich M, Greim S, Ferreira-Gomes M, et al. Functionality of the Human Antibody Response to Candida albicans. Virulence, 12: 3137–3148, 2021.

WEBSITES Centers for Disease Control and Prevention, Fungal Diseases, 2021: https://www.cdc.gov/fungal/diseases/candidia sis/thrush/index.html Web MD, What is Candidiasis? 2021: https://www.web md.com/skin-problems-and-treatments/g uide/what-is­ candidiasis-yeast-infection Students can test their knowledge of this case study by visiting the Instructor and Student Resources: [www. routledge.com/cw/lydyard] where several multiple choice questions can be found.

35

9780367696399_Lydyard.indb 35

13/02/2023 07:32

5

Chlamydia trachomatis

A 19-year-old woman was seen by her doctor for a routine gynecologic examination and complained about some mid-cycle bleeding. She had been with her current boyfriend for a year, had a pregnancy termination 2 years previously, and was taking birth control pills. Internal examination revealed a mucopurulent discharge at the external cervical os (Figure 5.1). The cervix was friable and bled easily. The doctor suspected chlamydial infection and collected an endocervical swab specimen for a Chlamydia test. The woman returned for the results and was told that the test for Chlamydia was positive. The doctor prescribed a course of doxycycline for 1 week and explained to the patient that this treatment is sufficient and effective in more than 95% of cases, and no repeat testing to prove the eradication of this infection is necessary. However, due to the high risk of re-infection in sexually active young adults, the doctor recommended her to return for a follow-up visit in 6 months. He further stated that a timely cleared chlamydial infection does not normally lead to infertility, although around a 10% probability of ectopic pregnancy remains. The woman was given a leaflet on chlamydial infection and on other sexually transmitted infections (STIs). The patient was counseled regarding safe-sex practices. The doctor also advised her to contact the local genitourinary clinic to be tested for other STIs including HIV and made all necessary arrangements according to the national guidelines.

1. WHAT IS THE CAUSATIVE AGENT, HOW DOES IT ENTER THE BODY AND HOW DOES IT SPREAD A) WITHIN THE BODY AND B) FROM PERSON TO PERSON? CAUSATIVE AGENT

This patient was infected with Chlamydia trachomatis, which belongs to the Family Chlamydiaceae of the Order Chlamydiales. The Chlamydiaceae Family consists of two genuses – Chlamydia and Chlamydophila. Bacteria of this Family are obligate intracellular human and animal pathogens. The term “Chlamydia” is derived from the word “chlamys,” which means cloak (Khlamus) in Greek, an appropriate name reflecting the cloak-like chlamydial inclusion around the host cell nucleus (see below).

As a part of the disease management, the doctor carried out all the appropriate actions recommended in the national guidelines to notify the patient’s partner and to advise him to visit a genitourinary clinic or to see his doctor for Chlamydia and other STI tests. The couple were advised to abstain from sex until both were cleared of the infection.

Figure 5.1 Mucopurulent cervical discharge caused by chlamydial infection, showing ectopy and edema. Courtesy of the Seattle STD/HIV Prevention Training Center, University of Washington, Seattle.

Chlamydiaceae are some of the most widespread bacterial pathogens in the world and there are several species that infect a variety of hosts based on a wide range of tissue tropism. Two species, Chlamydia trachomatis and Chlamydophila pneumoniae, are human pathogens and are responsible for various diseases that represent a significant economic burden. Chlamydophila psittaci and C. pecorum are mainly bird/animal pathogens, although zoonotic transmission of the former to humans can occur resulting in the disease psittacosis. Chlamydiaceae species share some important structural features. Like other gram-negative bacteria, they have inner and outer membranes, but have a specific lipopolysaccharide (LPS) that differs from that of other bacteria. The extracellular osmotic stability of Chlamydiaceae is provided by several complex disulfide cross-linked membrane proteins, the main ones being a 40 kDa major outer-membrane protein (MOMP, a product of the ompA gene); a hydrophilic cysteine-rich 60 kDa protein (OmcA); and a low molecular weight cysteine-rich 37

9780367696399_Lydyard.indb 37

13/02/2023 07:32

Case Studies in Infectious Disease

lipoprotein (OmcB). The Chlamydiaceae are thought to have little or no muramic acid, the hallmark constituent of peptidoglycan (PG). No transfer RNAs were identified in chlamydial cells, thus confirming the parasitism of this bacterium. There are two human biological variants (biovars) of C. trachomatis: trachoma and lymphogranuloma venereum (LGV), and one biovar C. pneumoniae infecting mice, and causing mouse pneumonitis (which will not be discussed here). Fifteen serologic variants (serovars) have been identified in the trachoma biovar (A–K, Ba, Da, Ia, and Ja). These mostly infect columnar and squamo-columnar epithelial cells of mucous membranes (see below). Serovars D–K, Da, Ia, and Ja typically infect genitourinary tissues, but were also found in the mucous membranes of the eye conjunctiva and epithelial tissues in the neonatal lung. Serovars A, B, Ba, and C generally infect the conjunctiva and cause trachoma. The LGV biovar consists of four serovars (L1, L2, L2a, and L3), which predominantly infect monocytes and macrophages passing through the epithelial surface to regional lymphoid tissue. Proteomic analysis of the pathogen is very difficult since Chlamydiaceae species are all obligate intracellular parasites and cannot grow in a cell-free system. As a result, most of the data obtained on the structural and functional proteins and

38

biochemical pathways utilized by Chlamydiaceae are derived from gene sequencing and indirect evidence. The genome of C. trachomatis (serovars A, D, and L2) has been fully sequenced. Chlamydiaceae species have a more complicated biphasic developmental life cycle (Figure 5.2) than other bacteria in that they have two different forms, a metabolically inert infectious elementary body (EB) and a larger noninfectious reticulate body (RB). Interestingly, the term “elementary body” belongs to the virology world and is derived from the time when Chlamydiaceae were initially considered to be viruses. The EB form of the bacterium survives outside the host cell whereas the RB form lives and replicates in a specialized vacuole of the host cell called an inclusion (another virology-derived term).

ENTRY INTO THE BODY The infectious EB form of the majority of Chlamydia strains is typically 0.2–0.3 μm in diameter. MOMP makes up 60% of its cell wall. Due to their rigid outer membrane, EBs are able to survive outside the eukaryotic host cells. C. trachomatis infecting genital tissue usually does so through small abrasions in the mucosal surfaces. The EBs infect nonciliated columnar, cuboidal or transitional epithelial cells, but can also infect macrophages. The infection process is multivalent. EBs bind to the epithelial cells directly via cellular proteoglycans

Figure 5.2 Biphasic developmental cycle of C. trachomatis. From Brunham R & Rey-Ladino J (2005) Nat Rev Immunol 5:149-161. https://doi. org/10.1038/nri1551. With permission from Springer Nature.

9780367696399_Lydyard.indb 38

13/02/2023 07:32

Case 5: Chlamydia trachomatis

with heparan sulfate (HS) moieties (HSPGs) mainly through electrostatic interactions. Depending on the serovar, the degree of HS involvement varies. Additionally, C. trachomatis EB form is able to adhere to and enter cells indirectly through binding with fibroblast growth factor 2 (FGF2). This complex then interacts with the FGF2 receptor, which mediates EB internalization into cells. The EB-FGF2 complex may involve synergistic interactions with the EB membrane bacterial protein OmcB, which, in turn, interacts with HSPGs. This facilitates chlamydial entry through phagocytosis, receptormediated endocytosis, and pinocytosis. Early intracellular phase (0–2 hours after infection). Once inside the cell, EB-containing vacuoles move toward the perinuclear region. The vacuole membrane phospholipids promote homotypic fusion of EB vacuoles with each other, but not with lysosomes – an important feature that helps the pathogen to avoid intracellular destruction. The homotypic fusion is specific to C. trachomatis only and not to other Chlamydia species. This results in the formation of a single fusion vacuole (a nascent inclusion) containing several EBs. Some of these vacuoles may contain different serovars such as F and E, leading to the possibility for genetic exchange to occur. The inclusions then move toward the microtubule organization center where they are supplied with nutrients via the host-cell Golgi apparatus. Bacterial proteins are directly secreted into the host cell cytosol. Inclusion development (2–40 hours after infection). The EB forms now undergo a lengthy and complex development using host-cell ATP and nutrients as a source of energy. With a reduced genome, C. trachomatis is dependent on its host for survival and hijacks host-cell metabolism particularly glycolytic enzymes, aldolase A, pyruvate kinase, and lactate dehydrogenase which are enriched at the C. trachomatis inclusion membrane during infection. The increased requirement for glutamine, important for the growth of C. trachomatis in infected cells is achieved by reprogramming the glutamine metabolism. Still remaining in the inclusion, the EB forms now transform into RB forms, which are typically 0.8–1.0 μm in diameter. RBs multiply by binary fission so that the resulting inclusions may contain 500–1000 progeny RBs and occupy up to 90% of the cell cytoplasm. After several rounds of replication, RB forms revert to the infectious EB forms. In tissue culture, the productive infectious cycle of Chlamydia lasts about 48–72 hours depending on the serovar. Eventually, the EBs are released to infect other adjacent cells. C. trachomatis is able to regulate host cell apoptosis throughout the early and productive growth stages. Nonreplicating forms of Chlamydia inhibit apoptosis through interfering with the TP53 tumor suppressor gene. However, late in the life cycle, the pathogen produces a caspase-independent pro-apoptotic Bax protein, which facilitates apoptosis of the host cell thus freeing the secondary EB forms. Under conditions unfavorable for a pathogen, such as the presence of interferon- (IFN- ), lack of nutrients or drug

9780367696399_Lydyard.indb 39

treatment (see Section 5) Chlamydia may enter a nonreplicating mode called persistence (Figure 5.2). During persistent infection, the developmental cycle is lengthened or aborted and RB forms are produced that do not divide or differentiate back into the EB forms. Persistent infection with C. trachomatis may lead to serious clinical conditions that are difficult to treat. However, dormant forms can revert to metabolically active forms if the unfavorable conditions are removed.

SPREAD WITHIN THE BODY EBs of C. trachomatis infect cervical columnar epithelial cells, but the bacteria can spread by ascending into the endometrium and the fallopian tubes, causing pelvic inflammatory disease (PID), ectopic pregnancy, and infertility (see Section 3). Sexually transmitted C. trachomatis serovars D–K can also lead to conjunctivitis through autoinoculation or ocular– genital contact.

PERSON-TO-PERSON SPREAD Genital Tract Infections C. trachomatis (serovars D–K) is sexually transmitted in vaginal fluid or semen containing the EB form, through vaginal intercourse but occasionally via oral and anal sex. These serovars can also be vertically transmitted from mother to child during birth through an infected birth canal, causing conjunctivitis (ophthalmia neonatorum) or chlamydial pneumonia. C. trachomatis (biovar LGV) is also sexually transmitted. In some parts of Africa, Asia, South America, and the Caribbean it is largely found in heterosexuals. In outbreaks in industrialized countries, the cases are mostly confined to men who have sex with men (MSM) with multiple sexual partners.

Ocular Infections C. trachomatis (serovars A–C) is found predominantly in areas of poverty and overcrowding. Infection can be transmitted from eye-to-eye by fingers, shared cloths or towels, by eyeseeking flies, and by droplets (coughing or sneezing). The latter route is possible because C. trachomatis can exist in the nasopharynx and external nasal exudates of children with trachoma. Importantly, the undiagnosed and untreated children can contribute to a so-called “age-reservoir effect” responsible for the continuous transmission within the community. These must be identified and treated to prevent further spread of the pathogen in communities.

EPIDEMIOLOGY C. trachomatis (serovars D–K) is the leading bacterial cause of STIs, with over 50 million new cases occurring yearly worldwide and 4 million new cases each year in the US. The highest infection rates are detected in African Americans, American Indian/Alaska Natives, and Hispanics. Two-thirds of new chlamydial infections occur among youth aged 15–24

39

13/02/2023 07:32

Case Studies in Infectious Disease

Figure 5.3 World distribution of trachoma according to the WHO. C. trachomatis (biovar trachoma) is endemic in large areas of Africa and the Middle East, and focal areas of disease are found in India, South-East Asia, and Latin America. As per 2020 WHO data, trachoma affects some 140 million people worldwide, and about 1.9 million people suffer visual loss and blindness. From Belland R, Ojcius D & Byrne G (2004) Focus: Chlamydia Nat Rev Microbiol 2:530. https:doi.org/10.1038/nrmicro931. With permission from Springer Nature.

years. In general practice, around 1 in 20 sexually active women aged less than 25 years may be infected. Among MSM worldwide incidence of rectal chlamydial infection range from 3.0% to 10.5%, and pharyngeal chlamydial infection from 0.5% to 2.3%. Interestingly, there was a 30% decline in Chlamydia cases as well as in all major STIs in England in 2020 compared to 2019, most likely due to the COVID-19 lockdowns. Every year C. trachomatis (serovars A–C) is a major cause of 500 000 cases of trachoma worldwide (Figure 5.3). Active trachoma affects some 85 million people, more than 10 million have trichiasis (turned-in eyelashes that touch the eye globe, Figure 5.4), and about 6 million people suffer visual loss and

blindness. Active disease is most commonly seen in children and, in adults, the prevalence of trichiasis is about three times higher in women than in men. Trachoma is endemic in large areas of Africa and the Middle East, and focal areas of disease are found in India, South-West Asia, Latin America, and Aboriginal communities in Australia. The disease is generally found in clusters in certain communities or even households, indicating the existence of local risk factors in addition to the generally accepted poverty and lack of water and sanitation. C. trachomatis (biovar LGV – serotypes L1–L3) causes sexually transmitted disease that is prevalent in parts of Africa, Asia, South America, the Caribbean, and increasingly in Europe and the US. Humans are the only natural host, with MSM being the major reservoir of the disease. In the US, the incidence is 300–500 cases per year.

2. WHAT IS THE HOST RESPONSE TO THE INFECTION AND WHAT IS THE DISEASE PATHOGENESIS? INNATE IMMUNITY

40

Figure 5.4 Trachomatous trichiasis: at least one eyelash rubs on the eyeball. Courtesy of the Centers for Disease Control, Atlanta, Georgia. Image is found in the Public Health Image Library #4076. Additional photographic credit is given to Joe Miller, PhD, who took the photo in 1976.

9780367696399_Lydyard.indb 40

Both innate and adaptive immune responses are induced during C. trachomatis infection. However, these responses are often insufficient, providing only partial clearance of the pathogen from the body. This can lead to protracted chronic infections and chronic inflammation contributing to pathogenesis.

13/02/2023 07:32

Case 5: Chlamydia trachomatis

Innate immune responses. Chlamydial infection initially induces an influx of polymorphonuclear cells and macrophages into the infection site as a part of acute inflammation. This is facilitated by the release by the infected epithelial cells of cytokines and chemokines, particularly interleukin-8 (IL­ 8), which is a powerful neutrophil attractant. Infiltration with neutrophils and macrophages is followed by accumulation of B cells, T cells, and dendritic cells (DCs) in submucosal areas launching T-cell responses and antibody production. Chlamydial PAMPs (pathogen-associated molecular patterns) are recognized by multiple PRRs (pattern recognition receptors), particularly TLR2 which was shown to be activated by MOMP and heat shock protein 60 (Hsp60), during the productive infection, although the direct ligand for TLR2 in this infection is still unknown. Antibody responses. A humoral response is invoked resulting in production of mucosal secretory IgA and circulating IgM and IgG antibodies. These antibodies are mostly specific for MOMP and Hsp60. Anti-chlamydial IgG antibodies are also found in ocular secretions in trachoma patients. The effectiveness of these antibodies in damaging or blocking further entry of EBs to the cells adjacent to the infected ones is unclear. However, it appears, that IgG antibodies bound to the bacterial MOMP invoke antibodydependent cellular cytotoxicity (ADCC) by natural killer (NK) cells. Cellular responses. It is likely that at least some professional antigen-presenting cells (APCs) at the site of infection engulf EBs, process and present Chlamydia peptides via the major histocompatibility complex (MHC) class II-mediated pathway leading to the activation of CD4+ T cells. DCs in response to the infection produce IL-12 and drive Th1-cell development and hence production of interferon (IFN- ) – the major inhibitory cytokine for Chlamydia (see below). Recent data suggests that in order to counteract this, C. trachomatis up-regulates expression of PD-L1 on the DCs present in the uterus. Its interaction with PD-1 receptor on T cells might lead to blocking of T-cell responses. That both Chlamydia-specific CD4+ and CD8+ T cells are involved in controlling C. trachomatis infection is indicated by their expansion with memory phenotype in the endocervix during the infection. Patients with C. trachomatis infection had the highest levels of T-cell recruiting cytokines out of major STI, a factor associated with higher risk of HIV co-infection which may occur in patients with untreated Chlamydia infection. In epithelial cells, which do not express MHC class II molecules, recognition of Chlamydia peptides may occur via the MHC class I presentation pathway, which activates CD8+ T cells. These can recognize proteins present in the host-cell cytosol or cytosolic domains of membrane proteins. MOMP is one of the potential antigenic targets for CD8+ CTLs. However, no evidence has been found so far of CD8+ T-cell-mediated killing of the infected cells that would disrupt pathogen replication and intracellular survival. It appears, therefore, that the production of the Th1 cytokine IFN- is one of the most powerful anti-chlamydial

9780367696399_Lydyard.indb 41

T-cell-mediated immune mechanisms. IFN- inhibits the growth of Chlamydia in cell culture and, in experimental models, disruption of its production enhances host susceptibility to Chlamydia infection. It is thought that IFNcan potentially limit C. trachomatis infection by the following mechanisms: • activation of macrophages and their phagocytic potential; • up-regulation of the expression of MHC molecules by professional and nonprofessional APCs; • expression of indoleamine 2,3-dioxygenase (IDO), a host enzyme that degrades intracellular tryptophan essential for C. trachomatis growth; • up-regulation of inducible nitric oxide synthase (iNOS), which catalyzes the production of nitric oxide (NO) and other reactive nitrogen intermediates, that can enhance damage to intracellular pathogens; • down-regulation of transferrin receptor on infected cells, resulting in an intracellular iron deficiency that may limit C. trachomatis replication. Even although IFN- plays a role in protective immunity to C. trachomatis infection, its overall effect is limited. Infection does not stimulate long-lasting immunity and repeated re-infections are common, which results in a prolonged inflammatory response and subsequent tissue damage (see Section 3). One of the reasons for poor anti-chlamydial immunity is the ability of the pathogen to evade immune responses.

HOW DOES C. TRACHOMATIS EVADE THE HOST IMMUNE RESPONSES? This bacterium employs several strategies to evade the host immune response. • Its intracellular location protects it from antibodies and complement. • It down-regulates the expression of MHC class I molecules on the surface of infected cells, blocking recognition and MHC class I-restricted CD8+ T-cell­ mediated cytotoxicity. The pathogen uses a protease-like activity factor (CPAF) able to degrade host transcription factors required for MHC gene activation. • Fusion of the pathogen-containing phagosome with host cell lysosomes is prevented. • Chlamydia-infected macrophages induce apoptosis of T cells, by paracrine effects and tumor necrosis factor(TNF- ).

PATHOGENESIS Tissue damage is the result of the host inflammatory response to the persistent infection as well as direct damage to infected cells by the bacteria. Various species of Chlamydia produce cytotoxins that can deliver immediate cytotoxicity of host cells if infected with large doses of the pathogen.

41

13/02/2023 07:32

Case Studies in Infectious Disease

With unsuccessful initial elimination by the innate and adaptive immune systems leading to persistence of the pathogen, the site of infection becomes infiltrated with macrophages, plasma cells, and eosinophils. The continuous production of cytokines and chemokines results in development of lymphoid follicles and tissue scarring due to fibrosis. It is believed that IFN- production by T cells at the site of infection is reduced as the bacterial load decreases. This, in turn, supports replication of C. trachomatis and the inflammatory process resumes, entering into a vicious circle. Chronic inflammation leads to many of the clinical symptoms seen with C. trachomatis. Trachoma is characterized by the conjunctival lymphoid follicle formation, which contain germinal centers consisting predominantly of B lymphocytes, with CD8+ T lymphocytes in the parafollicular region. The inflammatory infiltrate contains plasma cells, DCs, macrophages, and polymorphonuclear leucocytes. Inflammatory infiltrates taken from patients with scar tissue are characterized by the expansion of CD4+ T lymphocytes.

3. WHAT IS THE TYPICAL CLINICAL PRESENTATION AND WHAT COMPLICATIONS CAN OCCUR? Chlamydia is known as the “silent epidemic”, since it may not cause any symptoms sometimes for months or years before being discovered. There are several conditions caused by various biovars and serovars of C. trachomatis (Table 5.1).

UROGENITAL INFECTIONS Most people with C. trachomatis infection do not have any symptoms and are unaware of the infection. However, when symptoms develop, treatment is urgently required to prevent complications.

Males. When the symptoms develop, the patient may suffer from nongonococcal urethritis (NGU), which can result in discharge from the penis or pain and burning sensation when urinating. If not treated, it can lead to inflammation near the testicles with considerable pain. Spread to the testicles may cause epididymitis and, rarely, sterility. Chlamydia causes more than 250 000 cases of epididymitis in the US each year. Post-gonococcal urethritis may occur in men infected with both Neisseria gonorrhoeae and C. trachomatis who receive antibiotic treatment effective solely for gonorrhoea. MSM men could also develop rectal infection. Females. The incubation period is usually 1–3 weeks after which the symptoms of urethritis and cervicitis may develop: dysuria and pyuria, cervical discharge or vaginal spotting, and lower abdominal pain. Physical examination reveals yellow or cloudy mucoid discharge from the os (see Figure 5.1). If untreated, Chlamydia may spread through the uterus to the fallopian tubes, causing salpingitis. In the US, chlamydial infection is the leading cause of first trimester pregnancyrelated deaths. Women infected with Chlamydia have a threeto five-fold increased probability of acquiring HIV due to the increased behavioral risk. Complications. More than 4 billion US dollars are spent annually on the treatment of the most common and severe complication of the sexually transmitted C. trachomatis PID. Over 95% of women with uncomplicated and effectively treated chlamydial infection will not develop tubal infertility. Chlamydia can also cause subclinical inflammation of the upper genital tract, so called “subclinical PID”. Some patients develop perihepatitis, or “Fitz-Hugh-Curtis Syndrome”, an inflammation of the liver capsule and surrounding peritoneum, causing pain in the right upper quadrant. More than 85% of women with PID remain fertile. However, an approximate 10% risk of ectopic pregnancy – a potentially life-threatening condition – remains after both clinical and subclinical PID due to the permanent damage to

Table 5.1 C. trachomatis serovars and associated human diseases Serovars

Human disease

Method of spread

Pathology

A, B, Ba, and C

Ocular trachoma

Hand to eye, fomites, and eye-seeking flies

D, Da, E, F, G, H, I, Ia, J, Ja, and K

Oculogenital disease

Sexual and perinatal

L1, L2, and L3

Lymphogranuloma venereum

Sexual

Conjunctivitis and conjunctival and corneal scarring Cervicitis, urethritis, endometritis, pelvic inflammatory disease, tubal infertility, ectopic pregnancy, neonatal conjunctivitis, and infant pneumonia Submucosa and lymph node invasion, with necrotizing granuloma and fibrosis

Reproduced with permission from Brunham RC and Rey-Ladino J. Immunology of Chlamydia infection: implications for Chlamydia trachomatis vaccine. (2005). Nature Reviews Immunology, 5: 149–161.

42

9780367696399_Lydyard.indb 42

13/02/2023 07:32

Case 5: Chlamydia trachomatis

the fallopian tubes, uterus, and surrounding tissues. Women with PID often suffer later from abdominal pain and may require a hysterectomy. It is difficult to give a precise prognosis of infertility until a patient tries to have a child. In both sexes, an asymptomatic infection may be present in either the throat or rectum if the patient has had oral and/ or anal intercourse.

INCLUSION CONJUNCTIVITIS This condition is caused by C. trachomatis (serovars D–K) being associated with genital infections. It is often transferred from the genital tract to the eye by contaminated hands. The main symptom is a sensation of a foreign body in the eye, redness, and irritation. Other symptoms include mucosal discharge later replaced by purulent discharge, large lymphoid follicles, and papillary hyperplasia of conjunctiva, corneal infiltrates, and vascularization. Corneal scarring is rare and happens mostly in the chronic stage followed by epithelial keratitis. Ear infection and rhinitis can accompany the ocular disease.

INFANT PNEUMONIA Infants vertically infected with C. trachomatis (serovars D–K) from their mother at birth can develop pneumonia presented by staccato cough and tachypnea often preceded by conjunctivitis.

LYMPHOGRANULOMA VENEREUM (LGV) The causative agent is C. trachomatis biovar LGV. The first symptom of the infection is the development of a primary lesion – a small painless papule or ulcer at the site of infection, often the penis or vagina. Several weeks after the primary lesion, patients develop painful inguinal and/or femoral lymphadenopathy. In the case of extragenital infection, the lymphadenopathy can occur in the cervix. Patients develop a fever, headache, and myalgia followed by inflammation of the draining lymph nodes. As a result, the lymph nodes become enlarged and painful and may eventually rupture. Elephantiasis of the genitalia, more often in women, can develop due to obstruction of the lymphatics. In females, lymphatic drainage occurs usually in perianal sites and can involve proctitis and recto-vaginal fistulae. In males, proctitis develops from anal intercourse or from lymphatic spread from the urethra.

OCULAR INFECTIONS Trachoma is the most serious of the eye infections caused by C. trachomatis. The word “trachoma” in Greek means rough (trakhus) and reflects the roughened appearance of the conjunctiva. Repeated re-infection with the ocular serovars A, B, Ba, and C results in chronic keratoconjunctivitis. Following infection there is an incubation period of 5–12 days, after which, the symptoms start to appear and include a

mild conjunctivitis and eye discharge. Initial infection is often self-limiting and heals spontaneously. A repeated infection, however, leads to the development of chronic inflammation (see Section 2), characterized by swollen eyelids and swelling of lymph nodes in front of the ears. Years of re-infection and chronic inflammation may result in fibrosis and in scarring in the upper subtarsal conjunctiva. Scarring is more frequent in young adults, particularly in women. The progress of scarring over many years causes distortion of the lid margin and the lashes turn inward and rub against the cornea. This is called trichiasis (Figure 5.4). If untreated, persistent trauma can result in ulceration of the cornea, corneal opacity, and blindness. Inflammation and scarring in the eye may block the natural flow of tears, which represent an important first line of defense against bacteria. This can facilitate secondary re-infection with C. trachomatis as well as infection with other bacteria or fungi.

OCULAR LYMPHOGRANULOMA VENEREUM Ocular infection with C. trachomatis biovar LGV can lead to conjunctivitis and preauricular lymphadenopathy.

REACTIVE ARTHRITIS This can occur in men and women following symptomatic or asymptomatic chlamydial infection, sometimes coupled with urethritis and conjunctivitis and painless mucocutaneous lesions. Formerly, this complication was referred to as Reiter’s Syndrome.

4. HOW IS THE DISEASE DIAGNOSED, AND WHAT IS THE DIFFERENTIAL DIAGNOSIS? CLINICAL DIAGNOSIS OF GENITAL INFECTIONS The following clinical indicators are used for the diagnosis and screening of chlamydial infection in women: • less than 25 years of age, sexually active; • more than one sexual partner; • mucopurulent vaginal discharge (Figure 5.1); • burning sensation when passing urine; • friable cervix or bleeding after sex or between menstrual periods; • lower abdominal pain, or pain during sexual intercourse. Because of the possibility of multiple STIs, all patients with any STI should be evaluated for chlamydial infection and offered an HIV test. 43

9780367696399_Lydyard.indb 43

13/02/2023 07:32

Case Studies in Infectious Disease

CLINICAL DIAGNOSIS OF OCULAR INFECTIONS Examination of an eye for the clinical signs of trachoma involves careful inspection of the lashes and cornea, then aversion of the upper lid and inspection of the upper tarsal conjunctiva using binocular loupes. Clinical diagnosis is best made based on investigation of the history of living in a trachoma-endemic environment, in combination with clinical signs.

CLINICAL DIAGNOSIS OF LYMPHOGRANULOMA VENEREUM The clinical symptoms may initially be unclear since they overlap with those of other STIs. Some men may have had treatment for a range of conditions including inflammatory bowel disease, Crohn’s disease, and so forth. Diagnosis is largely based on the history of the disease, physical examination, and laboratory tests. The clinical course of LGV is divided into three stages. 1. Primary painless lesion, which develops after incubation of

3–30 days. A papule or ulcer can be found on the genitalia (glans of the penis, vaginal wall, labia, or cervix) or, in some cases, in the oral cavity. 2. Secondary lesion/lymphadenitis. It is a regional dissemination causing ing uinal and femoral lymphadenopathy and possibly bubo formation that ulcerates and discharges pus. Lymphadenopathy is usually unilateral involving the retroperitoneal lymph nodes in women and the inguinal lymph nodes in men. 3. Tertiary stage/genito-ano-rectal syndrome. The majority of patients will recover from the second stage without sequelae. However, a few may develop proctitis and fibrosis that may result in chronic genital ulcers or fistulas, rectal strictures, and genital elephantiasis. Early symptoms of LGV proctocolitis include anal pruritus and discharge, fever, rectal pain, and tenesmus.

For conjunctival specimens, epithelial cells are collected by rubbing a dry swab over the everted palpebral conjunctiva. The following tests can be made on the samples collected. Cytology is used to detect the inclusion bodies in stained cell scrapings (Figure 5.5), but this method lacks sensitivity and is time-consuming. Cell culture. For many years, cell culture has been the gold standard for the diagnosis of C. trachomatis and is very specific for this pathogen, but NAATs are more sensitive and represent a new gold standard (see below). Cultured cells are stained with Giemsa or iodine, or by fluorescent antibodies and examined for the presence of iodine-staining inclusion bodies. Iodine stains glycogen, which is only found in the inclusions of C. trachomatis, not in other Chlamydia species. NAATs are now recommended to replace culture techniques. They include first of all the polymerase chain reaction (PCR), but also transcription-mediated amplification (TMA). NAATs are characterized by high sensitivity (> 89%) and specificity (95–99.5%) for endocervical, urethral, and conjunctival samples. NAATs are the only testing techniques currently recommended by the English National Chlamydia Screening Programme. Certain NAAT test platforms have been cleared by FDA for non-genital sites. NAATs can be used for detection of the bacterium as well as for the quantification of the bacterial load. The wider use of NAATs worldwide in screening strategies is, however, limited by the costs and complexity of the equipment required. Direct hybridization probe tests have been used in the past, but now are largely replaced by NAATs. They include Gen-Probe PACE 2 test, which uses a synthetic singlestranded DNA probe complementary to the chlamydial rRNA region, and Digene HC-II-CT-II probe based on enzyme immunoassay (EIA) for the detection of the RNA probe binding to the single-stranded bacterial DNA.

LABORATORY DIAGNOSIS Sample collection. For genital infections, swabs are collected from the cervix or vagina of women or the urethra of men. Self-collection of swabs should be taken from the throat or rectum if there is a possibility of infection there. With the use of nucleic acid amplification tests (NAATs) (see below) a noninvasive urine test can be used for screening for Chlamydia with sufficient sensitivity instead of swabs. It is particularly useful in asymptomatic cases where genital examination and sampling may not be justified. For suspected LGV infections in the primary stage, a swab of the lesion can be taken. In the secondary stage, bubo pus, saline aspirates of the bubo, swabs of the rectum, vagina, urethra, urine, serum, or biopsy specimens of the lower gastrointestinal (GI) tract are used.

Figure 5.5 Chlamydial inclusions, which may contain 100–500 RB progeny. From Mabey DCW, Solomon AW & Foster A (2003) The Lancet 362: 223-229. https://doi.org/10.1016/S0140-6736(03)13914-1. With permission from Elsevier.

44

9780367696399_Lydyard.indb 44

13/02/2023 07:32

Case 5: Chlamydia trachomatis

Antibody-based laboratory techniques include direct fluorescent antibody (DFA) detection and enzyme-linked immunosorbent assay (ELISA). The latter can be used for determination of the serovars. Usually diagnostic antibodies target group-specific LPS or serovar-specific outer-membrane proteins (OMPs). However, ELISA-based technology is widely used in screening programs and is of great help in reducing the overall chlamydial infection and incidence of PID and NGU.

DIFFERENTIAL DIAGNOSIS Differential diagnosis of genital chlamydial infection includes gonorrhea, Ureaplasmaurealyticum, Mycoplasma genitalium, M. hominis, and Trichomonas vaginalis, as well as urinary tract infections and bacterial vaginosis. Some other conditions such as periurethral abscess, endometriosis, urethral/vaginal foreign body, other causes of PID, prostatitis, and epididymo­ orchitis must also be ruled out. In the case of trachoma, a differential diagnosis can be made with the following conditions: adult inclusion conjunctivitis; other bacterial infections, especially with Moraxella species and Streptococcus pneumoniae; viral infections including adenovirus, herpes simplex virus, and molluscum contagiosum; Pediculosis palpebrarum; toxic conjunctivitis, which is secondary to topical drugs or eye cosmetics; Axenfield’s follicular conjunctivitis; Parinaud’s oculoglandular syndrome; and vernal conjunctivitis. For LGV and inguinal adenopathy, differential diagnosis of the genital ulcer includes chancroid, herpes, syphilis, and donovanosis. For proctitis, the differential diagnosis includes inflammatory bowel disease. The differential diagnosis is mostly based on the laboratory findings, particularly PCR analysis, supplemented by clinical observations.

5. HOW IS THE DISEASE MANAGED AND PREVENTED? MANAGEMENT

In 2021, the CDC introduced new guidelines for the treatment of all STIs.

Urogenital Infections For urogenital infections, the antibiotic doxycycline is the drug of choice and 100 mg is taken twice daily for 7 days. This is effective in 95% of cases but can interfere with the contraceptive pill and can cause photosensitivity and stomach irritation. Doxycycline is also available in a delayed-release 200-mg tablet formulation, which requires once-daily dosing for 7 days. An alternative CDC-recommended regimen includes azithromycin, 1g orally in a single dose or levofloxacin 500 mg

orally once daily for 7 days. Azithromycin is a derivative of erythromycin and is characterized by improved bioavailability and ability to maintain high tissue concentrations, particularly at sites of inflammation. Azithromycin has also been proven to be neonatally safe. Erythromycin is no longer recommended due to the GI side effect. In the past, it was given to the infected woman if pregnant or lactating, as 500 mg four times a day for 7 days or twice daily for 14 days. Pregnant women are given a single 1g dose of azithromycin orally or amoxicillin 500 mg orally 3 times a day for 7 days. Neonates – erythromycin base or ethylsuccinate, 50 mg/kg body weight/day orally, divided into 4 doses daily for 14 days. The same regimen is used for infants. However, in addition, they can be prescribed an azithromycin suspension, 20 mg/kg body weight/day orally, 1 dose daily for 3 days. Since genital chlamydial infections often have no symptoms, particularly in women, the sexual partner could have become infected months ago. In case of multiple sexual partners, all of them should be tested for chlamydial infection and treated if positive. The patients should also be tested for other STIs. Having sex is not recommended during treatment and for at least a week after the completion of the treatment, particularly if both partners are infected (see Case Study). A high prevalence of C. trachomatis infection has been observed post-treatment, due to re-infection caused by failure of sex partners to receive treatment or initiation of sexual activity with a new infected partner, indicating a need for improved education and treatment of sex partners. Repeat infections confer an elevated risk for PID and other complications among women. As a preventive measure, men and women who have been treated for chlamydia should be retested approximately 3 months after treatment, or whenever persons next seek medical care during the first year after initial treatment.

Trachoma For the treatment of active trachoma, two antibiotic regimens are currently recommended: tetracycline ointment applied twice daily for 6 weeks or one 20 mg kg–1 dose of azithromycin can be used instead. It must be noted that the application of the ointment is inconvenient in children. Azithromycin is also effective for treating extraocular reservoirs of chlamydial infection, although antibiotic resistance could eventually develop. Currently, annual mass treatment for 3 years is recommended by the WHO in districts and communities where the prevalence of follicular trachoma in children aged 1–9 years is equal to or greater than 10%. Surgical correction of trichiasis to fix eyelid deformities and to prevent vision loss, coupled with post-operational one dose of azithromycin, is applied. Due to the severe damage of eyes in trichiasis, cornea transplantation is not considered.

45

9780367696399_Lydyard.indb 45

13/02/2023 07:32

Case Studies in Infectious Disease

PREVENTION

VACCINE

Prevention of genital chlamydial infections involves safe sexual practices, using a condom during sexual intercourse, and prompt treatment of infected patients and their sexual partners. It is recommended to have a Chlamydia test under the following conditions:

Due to its high incidence, relapses, prolonged bouts of infection, and significant morbidity, C. trachomatis is an important target for vaccine development. A safe vaccine administered prior to adolescence and effective through child-bearing age would have a significant impact on the spread of the disease. The attempts of using inoculation of whole inactivated bacteria, however, led to the exacerbation of inflammatory disease and subsequent re-infection. This shifted the focus of vaccine development toward MOMP-based subunit vaccines. Monoclonal antibodies to MOMP neutralize C. trachomatis infection in vitro and in vivo. The results of clinical trials were reported in 2019 on a multivalent vaccine, CTH522 incorporating MOMP proteins. The vaccine gave promising results by significantly increasing the levels of antigen-specific mucosal IgG and IgA, antibody neutralization, and the production of IFN . Further human clinical trials are required to fi ne-tune this vaccine, which is likely to include recombinant protein combination with adjuvants. Since 2013, vaccine development against C. trachomatis has been intensified and various delivery platforms have been suggested, such as nanoparticles, Hepatitis B core antigen and Neisseria lactamicaporin B as career molecules, adenoviral and mRNA vectors.

• after sex with a new or casual partner; • immediately if symptoms occur (see Section 3); • if a sexual partner has Chlamydia or symptoms of Chlamydia. The best way to avoid becoming infected with Chlamydia and other STIs is to have sexual contact or to be in a long-term, mutually monogamous relationship with a partner who is not infected. Flies acting as physical vectors for transmission of C. trachomatis transmit ocular infection, particularly in children in areas with poor sanitation. The presence of cattle pens has been associated with trachoma in some African countries. Crowded living conditions in the family unit constitute another factor increasing the risk of trachoma. Therefore, special socioeconomic measures have been recommended to reduce the risk of transmission of ocular C. trachomatis infection. These include increasing access to water, fly control interventions, education, and improved living conditions.

SUMMARY 1. WHAT IS THE CAUSATIVE AGENT, HOW DOES IT ENTER THE BODY, AND HOW DOES IT SPREAD A) WITHIN THE BODY AND B) FROM PERSON TO PERSON? ■ Chlamydia trachomatis belongs to the family Chlamydiaceae, which contains two genuses: Chlamydia and Chlamydophila. Bacteria of this family are obligatory intracellular human and animal pathogens. ■ Chlamydia

trachomatis

and

Chlamydophila

pneumoniae

0.2–0.6 μm in diameter and resistant to unfavorable environmental conditions outside of their eukaryotic host cells. ■ EBs bind to the epithelial cells directly via cellular proteoglycans with heparan sulfate (HS) moieties (HSPGs). Additionally, C. trachomatis EB form is able to adhere to and enter cells indirectly through binding with fibroblast growth factor 2 (FGF2). This complex then interacts with the FGF2 receptor, which mediates EB internalization into cells. ■ EB entry is followed by translocation of EB-containing endosomes

are human pathogens. C. trachomatis can cause sexually

to the perinuclear region and their homotypic fusion with each

transmitted urogenital infections, neonatal conjunctivitis and

other forming a fusion vacuole – a nascent inclusion. EBs

pneumonia, ocular trachoma, and urogenital infection associated

accumulate in an inclusion where they use a supply of nutrients

with lymphadenopathy and lymphadenitis (LGV). C. pneumoniae can cause bronchitis, sinusitis, and pneumonia, and accelerates atherosclerosis. ■ All Chlamydiaceae species have specific lipopolysaccharides

via Golgi apparatus and mature into the RB form. ■ C. trachomatis is dependent on its host for survival and hijacks host-cell metabolism particularly glycolytic enzymes, aldolase A, pyruvate kinase and lactate dehydrogenase, which are enriched

and envelope proteins: 40 kDa major outer-membrane protein

at the C. trachomatis inclusion membrane during infection. The

(MOMP), a hydrophilic cysteine-rich 60 kDa protein, and a low

increased requirement for glutamine, important for the growth of

molecular weight cysteine-rich lipoprotein.

C. trachomatis in infected cells is achieved by reprogramming the

■ C. trachomatis contains two human biological variants (biovars):

glutamine metabolism.

trachoma and LGV, and one biovar infecting mice. Fifteen

■ The RB form (up to 1.5 μm in diameter) multiplies by binary fission.

serologic variants (serovars) have been identified in trachoma

After several rounds of replication during 48–72 hours, the RB

biovar. MOMP confers serovar specificity.

form reverts to the infectious EB form. Eventually, the EBs are

■ Chlamydiaceae species are characterized by a biphasic

released through the extrusion of the inclusions by reverse

developmental cycle. The infectious EB form is typically

endocytosis or through apoptosis or are released after cell lysis. Continued...

46

9780367696399_Lydyard.indb 46

13/02/2023 07:32

Case 5: Chlamydia trachomatis

...continued

■ C. trachomatis (serovars D–K) is sexually transmitted in the EB form, usually through vaginal intercourse but occasionally can be transmitted by oral and anal sex. Spread to the fallopian tubes in females, it can lead to pelvic inflammatory disease (PID). It can also lead to conjunctivitis through autoinoculation. Genital infection of C. trachomatis (serovars D–K) is the leading bacterial cause of sexually transmitted infection with over 50 million new cases occurring yearly worldwide. ■ Ocular infection of C. trachomatis (serovars A–C) is spread in areas of poverty and overcrowding. Every year C. trachomatis is a major cause of 500 000 cases of trachoma worldwide, approximately 85 million people worldwide have active trachoma, more than 10 million have trichiasis (inturned eyelashes that

production in the host cells, enhancing production of nitric oxide, and down-regulating transferrin receptor on the host cells. ■ Infection does not stimulate long-lasting immunity and repeated episodes of infection are common. Re-infection results in an inflammatory response and subsequent tissue damage. ■ One of the reasons for poor anti-chlamydial immunity is the abilit y of the pathogen to evade or block host immune responses by intracellular location, down-regulation of MHC class I molecules, prevention of the formation of phagolysosome, and induction of apoptosis of cytotoxic T cells. ■ Pathogenesis is a result of a direct killing of infected cells and of chronic inflammation. The chronic inflammation induced by C. trachomatis infection leads to episodes of PID or conjunctivitis resulting in tubal infertility or blindness, respectively.

touch the globe), and about 6 million people suffer visual loss and blindness. ■ Infection can be transmitted from eye-to-eye by fingers, shared cloths or towels, by eye-seeking flies, and by droplets (coughing or sneezing). ■ C. trachomatis (biovar LGV) causes sexually transmitted disease that is prevalent in Africa, Asia, and South America. Humans are the only natural host.

2. WHAT IS THE HOST RESPONSE TO THE INFECTION AND WHAT IS THE DISEASE PATHOGENESIS? ■ Both innate and adaptive immune responses are induced during C. trachomatis infection, but they are usually inefficient at controlling the infection. This may lead to the partial clearance of the pathogen from the body resulting in bacterial persistence, chronic inflammation, tissue damage, and severe clinical symptoms. ■ Chlamydial PAMPs are recognized by multiple PRRs, particularly TLR2 which was shown to be activated by MOMPs and heat shock protein 60 (Hsp60), during the productive infection. Infection with Chlamydia induces production of secretory IgA and circulatory IgM and IgG antibodies mostly directed to MOMP and Hsp60. IgG antibodies bound to the bacterial MOMP invoke antibody-dependent cellular cytotoxicity (ADCC) by natural killer (NK) cells. ■ Dendritic cells (DCs) in response to the infection release IL-12 and drive Th1 cell development and hence production of interferon (IFN- ) – the major inhibitory cytokine for Chlamydia (see below). Recent data suggests that to counteract this, C. trachomatis up-regulates expression of PD-L1 on the DCs in the uterus. Its interaction with PD-1 receptor on T cells might lead to blocking of T-cell responses. ■ One of the most powerful anti-chlamydial T-cell-mediated immune mechanisms is production of the Th1 cytokine IFN- . IFNcan limit C. trachomatis infection by activating macrophages, up-regulating expression of MHC, suppressing tryptophan

3. WHAT IS THE TYPICAL CLINICAL PRESENTATION AND WHAT COMPLICATIONS CAN OCCUR? ■ Urogenital infection in men causes urethritis, which can produce a discharge from the penis or pain and burning sensation when urinating. If not treated, it can lead to epididymitis and, rarely, sterility. ■ Urogenital infection in women leads to urethral discharge, dysuria and pyuria, urethritis, and cervical discharge and friability. If untreated, the chlamydial infection may spread through the uterus to the fallopian tubes, causing salpingitis, infertility, or ectopic pregnancy. ■ A severe complication of the sexually transmitted C. trachomatis is PID. ■ Inclusion conjunctivitis is associated with genital infections transferred from the genital tract to the eye by contaminated hands. The main symptom is a sensation of a foreign body in the eye, redness, irritation. ■ Infants infected with C. trachomatis vertically from their mother at birth can develop pneumonia, which presents as a tachypnea, staccato cough and is often preceded by conjunctivitis. ■ The first symptom of LGV is the development of a primar y lesion, often in the penis or vagina. Several weeks after the primary lesion, patients develop painful inguinal and/or femoral lymphadenopathy, fever, headache, and myalgia followed by inflammation of the draining lymph nodes.

4. HOW IS THE DISEASE DIAGNOSED, AND WHAT IS THE DIFFERENTIAL DIAGNOSIS? ■ Clinical diagnosis of genital chlamydial infection in women is based on age, sexual activity, more than one sexual partner, mucopurulent vaginal discharge, burning when passing urine, friable cervix or bleeding after sex or between menstrual periods, lower abdominal pain, or pain during sexual intercourse. ■ Clinical diagnosis of LGV is based on the clinical presentation and the course of the disease assisted by differential diagnostic and laboratory tests. continued...

47

9780367696399_Lydyard.indb 47

13/02/2023 07:32

Case Studies in Infectious Disease

...continued

■ Eye examination for the clinical signs of trachoma involves careful inspection of the lashes and cornea, then reversion of the upper lid and inspection of the upper tarsal conjunctiva. A grading system is used by the WHO for the assessment of the prevalence and severity of trachoma. ■ Nucleic acid amplification tests (NAATs), including polymerase chain reaction (PCR), are now increasingly used for diagnosis and differential diagnosis of chlamydial infections, although their wider use in screening strategies worldwide is limited by the costs and complexity of the equipment required. ■ Cell culture has traditionally been the gold standard for the diagnosis of C. trachomatis and is specific for this pathogen. However, it is now replaced by NAATs. ■ Antibody-based laboratory techniques – immunofluorescence and enzyme-linked immunosorbent assay (ELISA) – are widely used for determining the serovars in screening programs.

5. HOW IS THE DISEASE MANAGED AND PREVENTED? ■ For urogenital infections, the antibiotic doxycycline (Doryx®, Vibramycin®) is given 100 mg twice daily for 7 days. Alternative CDC-recommended regimen include azithromycin, 1g orally in a single dose or levofloxacin. 500 mg orally once daily for 7 days. Pregnant women are given a single 1g dose of azithromycin orally or amoxicillin 500 mg orally 3 times/day for 7 days. Neonates –

FURTHER READING Goering RV, Dockrell H, Zuckerman M, Chiodini P. Mims’ Medical Microbiology and Immunology, 6th edition. Mosby, Edinburgh, 2018. Murray PR, Rosenthal KS, Pfaller MA. Medical Microbiology, 9th edition. Elsevier Mosby, Philadelphia, 2021.

REFERENCES Corsaro D, Greub G. Pathogenic Potential of Novel Chlamydiae and Diagnostic Approaches to Infections due to these Obligate Intracellular Bacteria. Clin Microbiol Rev, 19: 283–297, 2006. Dautry-Varsat A, Subtil A, Hackstadt T. Recent Insights into the Mechanisms of Chlamydia Entry. Cell Microbiol, 7: 1714–1722, 2005. Ende RJ, Derre I. Host and Bacterial Glycolysis During Chlamydia trachomatis Infection. Infect Immune, 88: e00545– 20, 2020. Gambhir M, Basáñez M-G, Turner F, et al. Trachoma: Transmission, Infection, and Control. Lancet Infect Dis, 7: 420–427, 2007.

erythromycin base or ethylsuccinate 50 mg/kg body weight/ day orally, divided into 4 doses daily for 14 days. The same regimen is used for infants. In addition, they can be prescribed an azithromycin suspension, 20 mg/kg body weight/day orally, 1 dose daily for 3 days. ■ All sexual partners should be tested for chlamydial infection and treated if positive. Patients should also be tested for other sexually transmitted infections. ■ Environmental improvement includes removing the risk factors for transmission of infection such as flies, the presence of cattle pens, and crowded living conditions. ■ Safe sexual practices, using a condom during sexual intercourse, and prompt treatment of infected patients and their sexual partners can prevent genital infections with C. trachomatis. ■ Repeat infections confer an elevated risk for PID and other complications among women. As a preventive measure, men and women who have been treated for chlamydia should be retested approximately 3 months after treatment, or whenever persons next seek medical care during the first year after initial treatment. ■ The major target for vaccine development currently is the multivalent MOMP-based vaccine in combination with adjuvants. For the delivery platforms nanoparticles, Hepatitis B core antigen and Neisseria lactamicaporin B as career molecules, adenoviral and mRNA vectors are considered.

McClarty G, Caldwell HD, Nelson DE. Chlamydial Interferon Gamma Immune Evasion Influences Infection Tropism. Curr Opin Microbiol, 10: 47–51, 2007. Murray SM, McKay PF. Chlamydia trachomatis: Cell Biology, Immunology and Vaccination. Vaccine, 39: 2965– 2975, 2021. Starnbach MN. The Well Evolved Pathogen. Curr Opin Microbiol, 54: 33–36, 2020. West SK. Milestones in the Fight to Eliminate Trachoma. Ophthalmic Physiol Opt, 40: 66–74, 2020. Wright HR, Taylor HR. Clinical Examination and Laboratory Tests for Estimation of Trachoma Prevalence in a Remote Setting: What are they Really Telling us? Lancet Infect Dis, 5: 313–320, 2005.

WEBSITES Centers for Disease Control and Prevention, Atlanta, 2022: https://www.cdc.gov/std/chlamydia/STDFact-Chlamydia.htm National Institutes of Health, Department of Health and Human Services, USA: www.nih.com UK Health Security Agency, English National Chlamydia Screening Programme, 2022: https://assets.publishing.service.

48

9780367696399_Lydyard.indb 48

13/02/2023 07:32

Case 5: Chlamydia trachomatis

gov.uk/government/uploads/system/uploads/attachment_ data/file/759846/NCSP_Standards_7th_edition_update_ November_2018.pdf Wikipedia – The Free Encyclopedia, Chlamydia: https:// en.wikipedia.org/wiki/Chlamydia World Health Organization, WHO guidelines for the treatment of Chlamydia trachomatis, 2016: https://www.who. int/publications/i/item/978-92-4-154971-4

Students can test their knowledge of this case study by visiting the Instructor and Student Resources: [www. routledge.com/cw/lydyard] where several multiple choice questions can be found.

49

9780367696399_Lydyard.indb 49

13/02/2023 07:32

6

Clostridioides difficile

An elderly woman, of no fixed abode, arrived at the hospital’s emergency department after having fallen. She was admitted to hospital for fixation of a fracture of the hip. Shortly after admission, she developed signs of a chest infection and was started on a cephalosporin, which she remained on for a week. Subsequently, she developed profuse watery diarrhea and abdominal pain. A fecal sample was sent to the laboratory to test for the toxins of Clostridioides difficile, which proved positive and she was commenced on oral vancomycin. Despite treatment, the diarrhea persisted and it also failed to respond to a course of metronidazole. The condition of the patient worsened and a sigmoidoscopy was performed, revealing that she had pseudomembranous colitis (Figure 6.1). Her clinical condition deteriorated and she developed toxic megacolon and an emergency colectomy was performed. The patient died shortly after the operation.

Figure 6.1 Endoscopic view (A) of a patient with pseudomembranous colitis showing plaques of pseudomembranes. Histologically (B) the pseudomembranes are composed of mushroom-shaped collections of neutrophils, cellular debris, and fibrin. From David M Martin, MD / Science Photo Library. With permission. Courtesy of Jae Lim, Calvin Oyer and Murray Resnick, editors and photographic contributors to the Digital Pathology website associated with Brown Medical School, Providence, Rhode Island.

CAUSATIVE AGENT

acids, and sporulation. Toxin A and B production occurs maximally in stationary phase and during nutrient limitation. Carbohydrate limitation results in a switch to amino-acid fermentation which, in turn, results in a shortage of amino acids and toxin production. Low levels of biotin also increase toxin production. Additionally, C. difficile has a thiolactone quorum-signaling system, which regulates toxin expression independent of TcdC.

Clostridioides difficile (previously Clostridium difficile [CD]) is an anaerobic spore-forming motile gram-positive rod measuring 0.5–1.0 × 3.0–16.0 μm (Figure 6.2). It produces irregular white colonies on blood agar (Figure 6.3). Toxigenic strains of CD produce three toxins, TcdA, TcdB, and CDT. TcdA and B are carried on a pathogenicity locus (PaLoc) which is a mobile element and can be transferred to non-toxigenic strains converting them to toxigenic strains. Both TcdA and B are glycosyl transferases. Regulation of the production of the toxins is by TcdR (Group V sigma factor) and inhibition is by TcdC (an anti-sigma factor) which acts directly on TcdR. TcdR also activates its own production in a positive feedback loop. A further protein, TcdE, is found on the PaLoc which is involved with secretion of the toxins (Figure 6.4) TcdC hyper-virulent mutants have been described. These strains produce increased amounts of toxins A and B and are caused by a single nucleotide deletion. Toxin production is influenced by several environmental factors such as sugars, amino

Figure 6.2 Gram stain of C. difficile showing that it is a grampositive rod. The spore can be clearly seen as a subterminal clear area in most of the bacilli. Courtesy of the Centers for Disease Control, Atlanta, Georgia. Image is found in the Public Health Image Library #3876. Additional photographic credit is given to Dr Gilda Jones who took the photo in 1980.

1. WHAT IS THE CAUSATIVE AGENT, HOW DOES IT ENTER THE BODY AND HOW DOES IT SPREAD A) WITHIN THE BODY AND B) FROM PERSON TO PERSON?

51

9780367696399_Lydyard.indb 51

13/02/2023 07:32

Case Studies in Infectious Disease

electrophoresis (MLEE) and analysis of variable number of tandem repeats (VNTR) may also be used. Phylogenomic studies of C . difficile using micro-arrays demonstrate five clades: a hypervirulent clade, a defective trehalose metabolism clade, a TcdA-negative/TcdB-positive clade, and two other human and animal clades. Differences between the clades are related to virulence, ecology, antibiotic resistance, motility, adhesion, and metabolism.

ENTRY INTO THE BODY

Figure 6.3 Colonies of C. difficile showing the typical appearance of white colonies with crenated edges. Under UV light, the colonies fluoresce a greenish-yellow color. Courtesy of the Centers for Disease Control, Atlanta, Georgia. Image is found in the Public Health Image Library #3876. Additional photographic credit is given to Dr Gilda Jones who took the photo in 1980.

The binary toxin CDT is found at a different location on the genome. The toxin consists of two genes cdtA and ctdB and is an ADP-ribosyltranferase. Production of the toxin is regulated by cdtR, which is one of the common strains of CD (027) also up-regulated TcdA and B. Several methods of typing C . difficile are available, the most frequently used being pulse field gel electrophoresis, and ribotyping (Figure 6.5), although multilocus enzyme

Following the ingestion of spores from contaminated feces or a contaminated environment, the organism colonizes mainly the large intestine of the GI tract. The dynamics of the GI microbiome is important in C . difficile infections as a dysbiosis caused by antibiotics predisposes to colonization. Treatment of C . difficile disease can therefore be ameliorated by fecal transplants. Eradication of colonization by C . difficile relies on a normal intestinal microbiota. Colonization resistance is provided by bacteria in the Ruminococcaceae and Lachnospiraceae which produce short-chain fatty acids (SCFA) such as butyric acid. Many studies have demonstrated that the microflora in an individual is rather stable, although variations have been shown in relation to diet and stress. C. difficile is present in the intestines of 2–3% of asymptomatic adults, but may increase to 30% in hospital, and in between 60% and 70% of neonates. The reason why neonates do not often get disease is unclear but may be related to the nature of the neonate microbiome or toxin not binding to cells.

SPREAD WITHIN THE BODY The organism remains within the GI tract and only rarely is extra-intestinal disease reported.

TcdR

TcdR

Figure 6.4 This figure shows the arrangement of the operon for toxins A and B of C. difficile. The expression of the toxin genes (TcdA and TcdB) is controlled by positive (TcdR) and negative (TcdC) regulators, which respond to changes in the environment. The operon also carries a gene, TcdE, which produces a protein thought to assist in the release of the toxins across the bacterial cell wall. Toxin A is a 308 kDa protein and toxin B is a 270 kDa protein. Each protein has an enzyme active domain E (glucosyltransferase); a domain responsible for binding B, which consists of multiple oligopeptide repeat units; and a domain T that is involved in translocation of the toxin across the host cell membrane.

52

9780367696399_Lydyard.indb 52

13/02/2023 07:32

Case 6: Clostridioides difficile

Figure 6.5 Ribotyping of Clostridium difficile – photograph of a gel showing some different common ribotypes of C. difficile (001, 027, etc.). The pattern of bands produced by the restriction enzyme is characteristic for the different ribotypes and the patterns can be used to determine if there is cross infection or a common source outbreak where the bands would be identical. Because some ribotypes are currently very common, e.g. 027, a more discriminatory typing system, e.g. variable number of tandem repeats (VNTR) may be used to subdivide the common ribotypes. Courtesy of the Windeyer Institute of Medical Sciences, University College London, UK.

PERSON-TO-PERSON SPREAD

Patients with diarrhea, especially if it is severe or accompanied by incontinence, may unintentionally spread the infection to other patients in hospital. In addition, the ability of C. difficile to form spores enables it to survive for long periods in the environment, for example, on floors and around toilets, and protects it against heat and chemical disinfectants. The spores can survive for 5 months on inanimate surfaces and are resistant to most disinfectants. Acquisition of C. difficile is probably as a spore rather than the vegetative organism. Colonization in hospitals is directly related to the floor area in single rooms: for every15 m2, there is an OR of 3.0. Effective agents for whole room decontamination are UV light and H 2O2 mist.

EPIDEMIOLOGY C . difficile is a well-recognized hospital “superbug” that has received considerable media attention and generated much public concern. In the US, there were around 500 000 cases annually with 15 000 deaths at a cost to the healthcare system of $4.8billion (2015 data). In the UK, cases have been subject to mandatory reporting since 2007 when the infection rate was 110/100 000. In 2018, the infection rate was 20/100 000 recording all cases over the age of 2 years. This represented 13 286 cases of which 4739 were acquired in hospital. T he standard typing method is PCR sequencing of the 16-23S inter-spacer region (ribotype, RT). Although important in typing C . Difficile, more discrimination can be achieved with multilocus variable number tandem repeats (MLVA) and whole genome sequencing (WGS). Investigation of outbreaks by specific r ibotypes has indicated widely separate subtypes or, in some cases, closely related subtypes. In the UK, the two most prevalent ribotypes

9780367696399_Lydyard.indb 53

are RT002 and RT015. The hypervirulent ribotype RT027 was very common in 2008–2009 (40% ) but by 2018 had decreased about 1% of the total and it remains uncommon. RT220 is another more virulent isolate that is linked to high C-reactive protein levels and a high mortality but is not a common isolate. Some types are particularly associated with outbreaks such as RT001 and TR106 but again their prevalence has decreased since 2008. Certain ribotypes are also geographically linked. In Europe (2012–2013), the common ribotypes were RT027, RT 001/072,RT014/020. A study in 2018 using WGS demonstrated specific country clades: RT356 and RT018 were found in Italy, RT176 in the Czech Republic and Germany, RT001/072 in Spain, Slovakia, and Germany, RT027 in Poland, Romania, Hungary, Italy, and Germany. Other ribotypes did not have specific country clades: RT078, RT015, RT002, RT014 and RT020. Overall, two distinct routes of spread have been identified, hospital-acquired and non-hospital acquired, the latter probably transmitted by several routes such as food or feco-oral. Recognizing a “One Health” approach, a study on prevalence of C . difficile in dogs and Zoo animals demonstrated 17% of dog feces, 56% for dog paws and 22% of environmental samples from Parks were positive for toxigenic C . difficile found associated with humans. Eighteen percent of Zoo animals were also colonized.

2. WHAT IS THE HOST RESPONSE TO THE INFECTION AND WHAT IS THE DISEASE PATHOGENESIS? IMMUNE RESPONSES

Colonization is initially with a spore. Once germinated, the organism expresses a number of factors that stimulate the 53

13/02/2023 07:32

Case Studies in Infectious Disease

Figure 6.6 Mode of action of C. difficile toxins A/B. This figure shows the action of the toxins A and B on the host cell. The toxins are taken up by receptor-mediated endocytosis after binding to specific glycoprotein receptors. The toxins inhibit the normal functioning of G proteins Rho, Rac, and Cdc42 by glucosylating them. This inactivation affects the actin cytoskeleton which, in turn, affects cell morphology and the integrity of the tight junctions between the cells, thereby increasing fluid loss. The toxins also stimulate the release of cytokines, e.g. IL-8, which recruits PMNs and establishes an inflammatory response and the production of reactive oxygen radicals. The toxins increase the release of substance P, a peptide that enhances fluid loss from the intestines and decreases the exocytosis of the protective mucus barrier.

immune system such as the flagella and surface proteins. The organism also produces a number of toxins (TcdA, TcdB, CDT) that are also immunogenic. Some surface proteins are pathogen-associated molecular patterns (PAMPs) and trigger a pro-inflammatory response along with an inhibitory response (IL-1β, IL-6, TNF- , IL-10, and IL-12, IL-23). Both T and B cells are generated. Immunoglobulin A (IgA) is produced along with IgM and IgG. High levels of both IgA and IgM at the initial infection are related to a low risk of recurrence and a low level of IgA is also linked to prolonged disease as well as a high risk of recurrence. High levels of IgG protect against C . difficile disease but if diarrhea develops, high levels of IgG at day 12 are linked to a low risk of recurrence. Specific deficiencies of IgG2 and 3 are also a risk factor for recurrences. Studies on IgG and IgA antibodies against TcdA and TcdB are contradictory, some showing a neutralizing effect, and others not. Despite this, humanized monoclonal antibodies directed against the toxins TcdA and TcdB have been developed as an effective immunotherapy.

PATHOGENESIS The toxins TcdA and TcdB are A-B type toxins (Figure 6.6). Both toxins consist of a glucosyltransferase domain (GTD), an

auto-processing domain (APD) a translocation domain (TD), and a C-terminal repetitive binding domain (CROP). The CROP of TcdA is homologous to the carbohydrate binding region of streptococcal glycosyltransferase and binds to various carbohydrate moieties on the cell membrane. The binding of TcdB is to the poliovirus receptor-like 3 (PVRL3) expressed on colonic epithelial cells. The toxins are internalized by receptormediated endocytosis, but with slightly different mechanisms. TcdA uptake is independent of clathrin whereas TcdB is mediated by clathrin. In the endosome, the toxins undergo a conformational change caused by the low pH environment. The proteins from the TD of the toxins insert into the membrane forming a pore through which the active enzyme component (glucosyltransferase) and the auto-processing component enter the cytosol. The glucosyltransferase enzyme is released from rest of the toxin by proteolytic cleavage by the auto-processing component. The GT enzyme then inactivates the GTPases Rho and Ras which leads to disruption of the actin cytoskeleton, apoptosis or necrosis (Figure 6.6). Both TcdA and B toxins induce a strong inflammatory response involving a number of factors including interleukin (IL)-8, IL-6, tumor necrosis factor- (TNF- ), leptin, and substance P. TcdA stimulates the vanilloid receptor, VR1,

54

9780367696399_Lydyard.indb 54

13/02/2023 07:32

Case 6: Clostridioides difficile

which leads to the release of substance P from sensory neurons (and increases the expression of its receptor – neurokinin 1-R) which, in turn, leads to the release of neurotensin and mast-cell degranulation. TcdA also increases the release of endocannabinoids, which increase the release of more substance P via stimulation of VR1. Both toxins also induce an inflammatory response with infiltration of polymorphonuclear leukocytes (PMNs) into the lamina propria by up-regulation of IL-8 secretion from sensory neurons and colonocytes. The polymorphism within the IL-8 gene may be related to the presentation of more severe disease. The role of biofilm development in the GI tract, along with its role in pathogenesis and antibiotic resistance, is an area of further study.

3. WHAT IS THE TYPICAL CLINICAL PRESENTATION AND WHAT COMPLICATIONS CAN OCCUR? C . difficile is associated with antibiotic-associated diarrhea (CDAD) at one extreme to pseudomembranous colitis (PMC) at the other. The latter has a significant mortality. C. difficile is largely linked to hospital admission and has been associated with hospital outbreaks. Infection has usually occurred in those aged over 65 years and usually in association with antibiotics, bowel surgery or chemotherapeutic agents. The illness may follow antibiotics taken briefly or even several weeks previously. Recently, community cases of C. difficile­ associated disease have been reported in a younger age group who have not necessarily been taking antibiotics. These cases have often been caused by hypervirulent strains such as ribotype 027. Patients present with colicky abdominal pain, bloating, and watery diarrhea with an almost characteristic smell for C . difficile. In severe cases (PMC), there may be blood in the feces and the colon may perforate. Toxic megacolon may occur with ileus, and colectomy may be required. Thus C. difficile with little or no diarrhea, and pain predominating, may be an ominous sign. Severe damage to the large bowel can result in rupture or perforation. T he rare patients with extra-intestinal disease almost always have long-term carriage of the organism or severe GI disease. Diseases reported include bacteremia, abscess, a nd a prosthetic hip infection. The significance of the isolate is questionable as, in abscesses, they are often part of a mixed infection.

4. HOW IS THE DISEASE DIAGNOSED, AND WHAT IS THE DIFFERENTIAL DIAGNOSIS? The routine diagnosis of C . difficile-associated disease is the detection of toxins in the feces by enzyme immunoassays

9780367696399_Lydyard.indb 55

(ELISA). An additional useful assay is the detection of glutamate dehydrogenase in the feces as a surrogate marker for the organism. Culture is also available and is important for the investigation of suspected outbreaks where the isolates can be ribotyped. The organism may be grown on cycloserine/ cefoxitin/egg yolk agar and a variety of other more sensitive media, always anaerobically producing 2–3 mm gray-white colonies. Polymerase chain reaction (PCR) for toxin-related genes is also available. PMC is diagnosed by colonoscopy where raised yellowish plaques may be seen on the mucosa. If severe disease is suspected, limited sigmoidoscopy may be used as colonoscopy may result in perforation. The differential diagnosis includes other infective causes of diarrhea, inflammatory bowel disease, and diverticulitis.

5. HOW IS THE DISEASE MANAGED AND PREVENTED? MANAGEMENT

For patients who are on antibiotics and develop diarrhea, one should stop the antibiotics if at all possible. Such a decision is on a case-by-case basis. Mild disease is not associated with a raised white cell count, and increased frequency of type 5–7 stools. Moderate disease is associated with a raised white cell count but less than 15 x 109. Severe disease has a white cell count of greater than 15 x 109, a temperature of 38.5˚C or greater, and a rising creatinine with clinical evidence of colitis such as pain and or ileus. For a fi rst attack, mild-to-moderate disease is treated with oral metronidazole 400 mg TDS for 10–14 days. Severe disease is treated with oral vancomycin 125 mg QDS for 10–14 days or Fidaxomicin 200 mg BD for 10–14 days. Fidaxomicin reduces the episode of recurrence but is expensive and the decision to use Fidaxomicin should be based on a cost-benefit analysis. If there is no response, and colitis is suspected (and the initial treatment was with vancomycin), one can use vancomycin 500 mg QDS with IVI metronidazole 500 mg TDS or Fidaxomicin 200 mg BD. With a worsening situation, intracolonic vancomycin 500 mg 4 hourly and IVI immunoglobulin 400 mg/kg. Irrespective of the antibiotic treatment, there was a similar reduction in the alpha diversity, (richness of the microbiome in relation to the number of different bacterial species) however, the beta diversity (comparative richness of the microbiome) differed according to the antibiotic used. Each antibiotic (with the exception of fidaxomicin) was associated with an increase in pathogenic fungi and a functional increase in xenobiotic metabolism which may perpetuate a dysbiosis. If a surgical resolution is needed, a total colectomy or loop ileostomy with lavage have been performed. Recurrence of C . difficile-associated disease occurs in about 25% of cases and, if not severe, is treated with Fidaxomicin 200 mg BD. Alternative treatments are a tapering course of oral vancomycin such as 125 mg qds week 1, 125 mg tds week 2,

55

13/02/2023 07:32

Case Studies in Infectious Disease

125 mg week 3 , 125 mg week 4, 125 mg on alternate days week 5, 125 mg every third day week 6, although other schedules have also been used. Similarly, many other treatments have been tried such as probiotics, rifamycin with little evidence of success. Fecal transplant (from a close family member) seems to be very effective in cases of multiple recurrences. Currently, several fecal microbial transplant (FMT) biotics are available which require assessment for cost-benefit analysis. Actoxumab (a human monoclonal antibody to toxin A) and bezlotoxumab (a human monoclonal antibody to toxin B), are two mAb that reduce recurrence by binding to the CROP domain and inhibiting binding of the toxins. The use of a triple mutant of adeno-associated virus (AAV6.2FF) that expresses actoxumab or bezlotoxumab may prevent relapses of the disease.

PREVENTION

In hospitals and nursing homes, prevention is by strict control-

of-infection procedures. Contact precautions for any patients

in whom CDAD is suspected are essential. The patient should be isolated in a single room with dedicated toilet facilities and the use of personal protective equipment (PPE), that is gloves and apron. If more than one patient has the illness, the patients should be nursed in a dedicated ward. A decision whether to close the ward/unit must be made on an individual basis but if there is evidence of widespread cross-infection then the unit should be closed. Horizontal and “high touch” surfaces and other items within the immediate environment, particularly toilet facilities, should be regularly cleaned with a hypochlorite disinfectant. All items and rooms after discharge of the patient(s) should be cleaned in the same manner. Attention to hand-washing practices is important and should be undertaken with soap and water rather than alcohol, as the latter has no effect on the spores of C . difficile. In fact, the spores of C . difficile are highly resistant to many of the generic disinfectants. There is currently no vaccine although research is ongoing.

SUMMARY 1. WHAT IS THE CAUSATIVE AGENT, HOW DOES IT ENTER THE BODY, AND HOW DOES IT SPREAD A) WITHIN THE BODY AND B) FROM PERSON TO PERSON? ■ Clostridium difficile is a gram-positive sporulating anaerobe. ■ Several typing methods are available including ribotyping and PFGE. ■ The spore is acquired by feco-oral spread or from the environment and infects colonocytes of the host. ■ Up to 70% of neonates may be colonized without disease. ■ Asymptomatic colonization may occur in up to 30% of patients in hospital. ■ A hypervirulent strain (ribotype 027) first isolated in Canada produces large amounts of toxin and has spread across Europe, North America, and globally. ■ C. difficile produces three toxins: toxins A and B and sometimes a binary toxin. ■ Important risk factors for disease are increased age (> 60 years) and taking antibiotics. ■ The organism only rarely causes extra-intestinal disease.

2. WHAT IS THE HOST RESPONSE TO THE INFECTION AND WHAT IS THE DISEASE PATHOGENESIS? ■ Antibodies to toxins A and B are produced following C. difficile diarrhea, some of which are neutralizing. ■ The toxins bind to host receptors on colonocytes and are taken up into the cell. ■ Toxins A/B induce disaggregation of the actin cytoskeleton by inactivating GTPase proteins. ■ Impairment of tight junctions and cell death occurs leading to fluid loss and a localized inflammatory response.

■ Both the toxins and the inflammatory response itself are thought to be involved in tissue damage.

3. WHAT IS THE TYPICAL CLINICAL PRESENTATION AND WHAT COMPLICATIONS CAN OCCUR? ■ The patient presents with abdominal pain and diarrhea. ■ Frequently, the patient is aged > 60 years, in hospital, and is on antibiotics. ■ Community-acquired disease in younger patients who have not been on antibiotics is relatively rare and associated with hypervirulent strains. ■ The disease may be mild-to-moderate CDAD or severe PMC. ■ Complications include perforation or toxic megacolon.

4. HOW IS THE DISEASE DIAGNOSED, AND WHAT IS THE DIFFERENTIAL DIAGNOSIS? ■ The disease is routinely diagnosed by detection of the toxins in the feces using enzyme immunoassays. ■ Culture is important for typing the organism in identifying and managing outbreaks. ■ PMC is diagnosed by colonoscopy or sigmoidoscopy. ■ Differential diagnosis includes other causes of gastroenteritis, diverticulitis and inflammatory bowel disease, and antibiotic­ associated diarrhea not due to C. difficile.

5. HOW IS THE DISEASE MANAGED AND PREVENTED? ■ The first-line treatment is oral metronidazole or vancomycin for 14 days. ■ Alternative treatments include agents to bind the toxin, probiotics, or fecal enema. ■ In hospital, prevention is by standard control of infection methods.

56

9780367696399_Lydyard.indb 56

13/02/2023 07:32

Case 6: Clostridioides difficile

FURTHER READING Cimolai N. Laboratory Diagnosis of Bacterial Infections. CRC Press, New York, 2001. Mandell GL, Bennet JE, Dolin R. Principles & Practice of Infectious Diseases, 6th edition, Vol 3. Elsevier/Churchill Livingstone, Edinburgh, 1249–1259, 2005. Mayhall GC. Hospital Epidemiology & Infection Control, 3rd edition. Lippincott Williams & Wilkins, Philadelphia, 623–631, 2004. Murphy K, Weaver C. Janeway’s Immunobiology, 9th edition. Garland Science, New York/London, 2016.

REFERENCES Alam MJ, McPherson J, Miranda J, et al. Molecular Epidemiology of Clostridioides difficile in Domestic Dogs and Zoo Animals. Anaerobe, 59: 107–111, 2019. Ausiello CM, Cerquetti M, Fedele G, et al. Surface Layer Proteins from Clostridium difficile Induce Inflammatory and Regulatory Cytokines in Human Monocytes and Dendritic Cells. Microbes Infect, 8: 2640–2646, 2006. Balaji A, Ozer EA, Kociolek LK. Clostridioides difficile Whole-Genome Sequencing Reveals Limited Within-Host Genetic Diversity in a Pediatric Cohort. J Clin Micro, 57: e00559–19, 2019. Bruxelle J-F, Péchiné S, Collignon A. Immunization Strategies Against Clostridium difficile. Adv Exp Med Biol, 1050: 197–225, 2018. Ciftci Y, Girinathan BP, Dhungel BA, et al. Clostridioides difficile SinR' Regulates Toxin, Sporulation and Motility Through Protein-Protein Interaction with SinR. Anaerobe, 59: 1–7, 2019. Durai R. Epidemiology, Pathogenesis and Management of Clostridium difficile Infection. Dig Dis Sci, 52: 2958–2962, 2007. Fawley J, Napolitano LM. Vancomycin Enema in the Treatment of Clostridium difficile Infection. Surg Infect, 20: 311–316, 2019. Khoruts A, Staley C, Sadowsky MJ, et al. Faecal Microbiota Transplantation for Clostridioides difficile: Mechanisms and Pharmacology. Nat Rev Gastroenterol Hepatol, 18: 67–80, 2021. Lamendella R, Wright JR, Hackman J, et al. Antibiotic Treatments for Clostridium difficile Infection are Associated

with Distinct Bacterial and Fungal Community Structures. mSphere, 3: e00572–17, 2018. Matamouros S, England P, Dupuy B. Clostridium difficile Toxin Expression Is Inhibited by the Novel Regulator TvdC. Mol Microbiol, 64: 1274–1288, 2007. McMaster-Baxter NL, Musher DM. Clostridium difficile: Recent Epidemiological Findings and Advances in Therapy. Pharmacotherapy, 27: 1029–1039, 2007. Mounsey A, Lacy Smith K, Reddy VC, Nickolich S. Clostridioides difficile Infection: Update on Management. Am Fam Physician, 101: 168–175, 2020. Owens RC. Clostridium difficile Associated Disease: Changing Epidemiology and Implications for Management. Drugs, 67: 487–502, 2007. Peled N, Pitlik S, Samra Z, et al. Predicting Clostridium difficile Toxin in Hospitalized Patients with Antibiotic Associated Diarrhoea. Infect Control Hosp Epidemiol, 28: 377–381, 2007. Smith JA, Cooke DL, Hyde S, et al. Clostridium difficile Toxin Binding to Human Intestinal Epithelial Cells. J Med Microbiol, 46: 953–958, 1997. Summers BB, Yates M, Cleveland KO, et al. Fidaxomicin Compared with Oral Vancomycin for the Treatment of Severe Clostridium difficile-Associated Diarrhea: A Retrospective Review. J Hosp Pharm, 55: 268–272, 2020. Voth DE, Ballard JD. Clostridium difficile Toxins: Mechanism of Action and Role in Disease. Clin Microbiol Rev, 18: 247–263, 2005. Vuotto C, Donelli G, Buckley A. Clostridium difficile Biofilm. Adv Exp Med Biol, 1050: 97–115, 2018. Wetzel D, McBride SM. The Impact of pH on Clostridioides difficile Sporulation and Physiology. Appl Environ Microbiol, 86: e02706–19, 2020.

WEBSITES GOV.UK, Clostridioides difficile: guidance, data and analysis, 2014: http://www.hpa.org.uk/infections/topics_az/ clostridium_difficile/C_diff_faqs.htm Students can test their knowledge of this case study by visiting the Instructor and Student Resources: [www. routledge.com/cw/lydyard] where several multiple choice questions can be found.

57

9780367696399_Lydyard.indb 57

13/02/2023 07:32

Cytomegalovirus

7

*

Vp* Figure 7.1 Fetal hydrops noted at growth scan (30 wk 6d). Lateral ventricles of fetal brain measuring 12 mm (Vp * to *) showing ventriculomegaly. A 26-year-old healthy primigravida woman was booked into hospital early in the second trimester. At her first trimester antenatal screen, she was Rhesus positive, rubella immune, and negative for HIV, hepatitis B and C and syphilis. She underwent a routine morphology scan at 19-weeks’ gestation. The biparietal diameter (BPD) and head circumference (HC) were on the 5th centile, the abdominal circumference (AC) on the 57th centile with an estimated fetal weight (EFW) of 304 g, on the 40th centile. Fetal anatomy appeared normal except for low-grade echogenic bowel. In view of these results, she was advised to have interval growth scans at 28 and 34 weeks. The planned third trimester growth scan was performed at 30 weeks 6 days, and showed the BPD on the 35th centile, HC on the 44th centile, AC >95th centile, femur length (FL) on the 10th centile and the EFW was estimated as >95th centile (~2218 g). Several abnormalities were noted. The fetus had developed gross hydrops; gross ascites, pericardial effusions of 6 mm on both sides of the fetal cardia, and ventriculomegaly

of 12 mm was noted (Figure 7.1). A Doppler scan of the middle cerebral artery suggested moderate to severe fetal anemia. At 31-weeks’ gestation, various possibilities were discussed with the patient, including aneuploidy, structural abnormalities, viral infection, or a genetic syndrome. Urgent tests were organized for CMV antibody, and amniocentesis to test for CMV DNA by polymerase chain reaction (PCR) and chromosomal abnormalities by microarray. Because of the possibility of severe fetal anemia, cordocentesis and intrauterine fetal transfusion were also performed. Pre-transfusion full blood count revealed a hemoglobin of 79 g/dl, a hematocrit of 0.25% and a platelet count of 29.9 × 109/L (thrombocytopenia). After intrauterine transfusion, the hemoglobin increased to 100 g/dl, the hematocrit to 0.30 and the platelet count was 30 × 109/L. Results of requested tests were positive CMV IgG and IgM with high IgG avidity, and on amniotic fluid no chromosomal abnormalities but a high CMV DNA load. The patient was told that primary CMV infection was the most likely diagnosis. Continued...

59

9780367696399_Lydyard.indb 59

13/02/2023 07:32

Case Studies in Infectious Disease

...continued

Given the severe hydrops, ascites, pericardial effusion, and pancytopenia with probable bone-marrow involvement, as well as ventriculomegaly, there was a poor perinatal prognosis with a high chance of early neonatal death or significant long-term neurologic impact. The late termination was discussed but the patient wished for all reasonable measures for perinatal resuscitation to be taken. Use of CMV-specific immunoglobulin and antivirals were also considered, but at 32 weeks’ gestation, the patient spontaneously ruptured membranes, went into labor and the baby was delivered by cesarean section. The neonate was admitted into the neonatal

1. WHAT IS THE CAUSATIVE AGENT, HOW DOES IT ENTER THE BODY AND HOW DOES IT SPREAD A) WITHIN THE BODY AND B) FROM PERSON TO PERSON? CAUSATIVE AGENT

The case above is an example of cytomegalic inclusion disease of the newborn. Cytomegalovirus (CMV), also known as human cytomegalovirus (HCMV), is so named because of the characteristic owl’s eye or cytomegalic inclusions it produces in the nuclei of infected cells (Figure 7.2). CMV is a beta herpesvirus, one of eight human herpesviruses (see Table 7.1, Epstein-Barr virus case). The Herpesviridae are characterized by a double-stranded (ds) linear DNA genome. CMV has the largest human herpesvirus genome of 236 kb encoding more than 200 proteins. The genome structure is similar to herpes simplex virus in that long and short unique sequences are bound by left and right terminal repetitive sequences. Individual genes of CMV are numbered sequentially in the unique long, unique short or terminal repeat regions. For example, UL128 refers to the 128th gene in the unique long region and pUL128 to

intensive care unit for ongoing supportive care. On day four of life, a cranial ultrasound scan revealed severe periventricular leukomalacia, ventriculitis and diffuse calcifications. The poor long­ term outlook was explained to the parents and the decision was taken to withdraw care. Adapted with permission from Mathias CR & Joung SJS. Diagnostic challenges in congenital cytomegalovirus infection in pregnancy: A case report. Case Rep Women’s Health, 2019, 22: e00119. doi: 10.1016/j.crwh.2019.e00119

its protein product (see Table 7.1 for other genes and gene products). Using next-generation sequencing, extensive CMV genome-wide variability has recently been recognized even within a single individual such as congenitally infected and immunocompromised patients. This variability potentially affects virus pathogenesis, but no definitive studies have been reported. Virus structure is that of a typical herpesvirus (Figure 7.3). The space between the capsid and envelope is filled by the tegument or matrix, of which a major component in CMV is pp65 but there are dozens of other viral tegument proteins. Embedded in the CMV envelope are many viral glycoproteins including those important for cell entry, namely glycoprotein B (gB), gH, gL, gO, pUL128, pUL130, and pUL131A. In common with all herpesviruses, CMV persists for life after primary infection. In latency, the viral genome remains in the cell’s nucleus as a circular episome in the absence of lytic replication and virus production. The site of latency is primarily CD34+ hematopoietic progenitor cells. Upon cellular differentiation into monocytes and dendritic cells, the virus reactivates. Given the diversity of CMV genotypes, re-infection does occur but, in clinical practice, given the difficulty of distinguishing between reactivation of latent virus (endogenous) and re-infection (exogenous), the term CMV reactivation is applied to both scenarios.

VIRUS REPLICATION

60

Figure 7.2 Photomicrograph of a section of lung tissue from an infant with congenital CMV infection. A large cytomegalic inclusion cell is shown on the lower left. Courtesy of the Centers for Disease Control, Atlanta, Georgia. Image is found in the Public Health Image Library #22212. Additional photographic credit is given to Roger A. Feldman, who took the photo in 1976.

9780367696399_Lydyard.indb 60

CMV can infect a remarkably broad cell range within its host, including parenchymal and connective tissue cells of virtually any organ, as well as hematopoietic and myeloid cells. Lytic virus replication occurs predominantly in epithelial cells, endothelial cells, fibroblasts, and smooth muscle cells. Host-cell entry is initiated when gB attaches to heparin sulphate. Binding to various cellular receptors by one of two gH/gL complexes on the virion is critical. The trimer gH/ gL/gO mediates entry into fibroblasts and gB fusion at the plasma membrane. Whereas the pentamer gH/gL/pUL128/ pUL130/pUL131A mediates entry into epithelial/endothelial and myeloid cells, and gB fusion at the endocytic membrane. Tegument proteins are released upon entry into the cytoplasm and function to ensure successful infection by modulating both the initial host response to infection and transcription of immediate early genes. The nucleocapsid is transported to

13/02/2023 07:32

Case 7: Cytomegalovirus

Figure 7.3 Thin section electron micrograph of herpesviruses budding after replication in an infected cell. Lipid virus envelope with embedded glycoproteins surrounds the tegument (red arrow), within which is the nucleocapsid.

Table 7.1 Selected CMV genes and gene products Gene

Gene product

Nucleic acid metabolism UL97

Protein kinase confers ganciclovir sensitivity

UL122

Immediate early protein – IE2, responsible for regulation of viral gene expression and

UL54

DNA polymerase

UL51/UL56/UL89

Terminase complex enables viral DNA packaging

virus replication

Structural proteins UL83

Phosphoprotein 65 (pp65), major tegument protein, also known as ppUL83 or lower matrix protein. Provides scaffold for optimal tegumentation. See also immunomodulatory molecules below

UL86

Major capsid protein

Virus attachment/membrane fusion proteins UL55

Glycoprotein B (gB), fusion protein, major target of neutralizing antibodies

UL75

Glycoprotein H (gH), target of neutralizing antibodies

UL115

Glycoprotein L (gL)

UL74

Glycoprotein O (gO), unique to CMV, forms a trimer with gH/gL critical for cell entry to fibroblasts

UL128/UL130/UL131A

pUL128/pUL130/pUL131A, unique to CMV, forms a pentamer with gH & gL critical for entry to epithelial/endothelial and myeloid cells

Immunomodulatory molecules US2

gpUS2 inserts into ER membrane and redirects human leukocyte antigen (HLA) class I to the cytosol for proteasomal degradation

US3

gpUS3 inhibits maturation and transport of peptide complexes with HLA class I

US6

gpUS6 inhibits TAP-mediated ER peptide transport

US10

gpUS10 degrades HLA G

US11

gpUS11 dislocates and degrades HLA class I from the ER

UL83

pp65 suppresses interferon response by inhibiting DNA sensing pathways, e.g. cGAS, IFI16

UL111A

Viral homolog of interleukin (IL)-10 regulates CD4+ T cell response

ER = endoplasmic reticulum

61

9780367696399_Lydyard.indb 61

13/02/2023 07:32

Case Studies in Infectious Disease

the nuclear pore and the genome enters the nucleus. Gene expression occurs sequentially in immediate early, early, and late phases. The linear viral DNA is circularized and copied by rolling circle replication to produce long concatemers consisting of linearly linked copies of the viral genome. The concatemer is cut into separate genome copies by the viral terminase complex and packaged into a newly formed capsid.

ENTRY AND SPREAD WITHIN THE BODY In primary infection, CMV usually enters the body by replication in oropharyngeal or genital mucosal epithelia. Infection spreads to endothelial cells and leukocytes. Viremia facilitates systemic infection and presumably seeds secretory organs such as salivary glands and kidneys as well as the bone marrow where the virus becomes latent in CD34+ cells. Infection of ubiquitous cell types such as fibroblasts and smooth muscle cells provides the platform for virus proliferation.

PERSON-TO-PERSON SPREAD After primary infection, CMV is shed asymptomatically in saliva and urine for many months in children and for a shorter time in adults. Notably, children with congenital CMV infection whether symptomatic or asymptomatic shed large amounts of virus of the order of 106 genomes/mL of urine or saliva for years. Shedding recurs periodically in all infected persons especially those with immunodeficiency or immunosuppression. Virus is also found in breast milk, cervical secretions, blood, and semen. In childhood, CMV is acquired by contact with saliva or urine from infected children or on fomites. Children in day care nurseries transmit the virus to each other and to suscep­ tible adult carers. Later in life, sexual transmission is a major route of infection; seroprevalence approaches 100% in men who have sex with men and sex workers. Another major route of transmission is from mother to child both prenatally when virus infects and crosses the placenta and perinatally through cervical secretions, breast milk, and saliva. CMV can also be transmitted in leukocytes by blood trans­ fusion from seropositive donors with a risk of 2.5%/unit of blood. Solid-organ and hematopoietic cell transplant recipi­ ents can acquire CMV from seropositive donors.

EPIDEMIOLOGY OF CMV INFECTION AND CONGENITAL CMV INFECTION

62

In developing countries, CMV is usually acquired in childhood and nearly 100% of young adults are seropositive. In developed countries, seroconversion progresses with age, but seroprevalence is higher in lower socioeconomic groups; overall about 40% of adolescents have been infected and seroprevalence increases by approximately 1% per year thereafter. More than one million congenital CMV infections are pre­ dicted to occur each year worldwide but detailed data from

9780367696399_Lydyard.indb 62

developing countries is lacking. The best population-based estimates of congenital CMV infection come from the US, where 0.4%–0.7% of babies (approximately 28 000) are born each year with congenital CMV infection. Of these, 12.7% may be symptomatic at birth, and 13.5% of these developed symptoms on follow-up, making congenital CMV infection the most common viral cause of sensorineural hearing loss and neurodevelopmental delay. The prevalence of congenital CMV infection in Europe is similar.

2. WHAT IS THE HOST RESPONSE TO THE INFECTION AND WHAT IS THE DISEASE PATHOGENESIS? HOST RESPONSE

Mammalian cytomegaloviruses are species specific and so human CMV infection cannot be studied in animals. Additionally, the host response has been difficult to dissect since human infection is mostly asymptomatic. Much of the information comes from murine CMV infections but many mechanisms have also been confirmed with human CMV. On first encounter with the virus, both innate and adaptive immunity responses play a role in restricting viral replication. Adaptive immunity in healthy individuals controls viremia, and productive infection in the face of virus reactivation or re-infection.

INNATE IMMUNITY CMV infection of host cells triggers a rapid innate response through recognition of viral proteins and DNA by a variety of pattern recognition receptors (PRRs). These include Tolllike receptor (TLR) 2 on the surface of antigen-presenting cells such as dendritic cells and macrophages. TLR2 binds CMV envelope proteins, gB and gH, and activates the NF-κB pathway that directs the production of inflammatory cytokines, such as interleukin (IL)-6, IL-8, IL-12, and IFN­ β. TLR3 and TLR9 recognize CMV DNA in the endosome and nucleotide-binding oligomerization domain (NOD)-like receptors (NLRs) perform the same action in the cytoplasm. Type I interferons (IFN- /β), together with natural killer (NK) cells, are very important in controlling primary infection with herpesviruses; this is amply demonstrated in patients with NK-cell deficiencies who suffer severe herpesvirus infections. NK cells target virus infected cells by recognizing reduced levels of major histocompatibility complex (MHC) class I, together with elevated levels of infection-induced stress ligands; they do not express either immunoglobulin or T-cell receptor genes. These cells have been shown to inhibit human CMV transmission in fibroblasts, endothelial and epithelial cells by production of IFN- , and induction of IFN-β in infected cells. Notably, although NK cells are part of the innate immune response, under certain circumstances they exhibit immunologic memory, a feature usually confined to adaptive

13/02/2023 07:32

Case 7: Cytomegalovirus

responses. Studies of the antiviral response to human CMV have demonstrated clonally expanded NK cells based on their avidity to certain antigens, which leads to lifelong memory and readiness to respond to re-infection or reactivation. Murine CMV experiments have shown that other innate effector cells are also important in controlling viral infection. Mast cells are activated and produce chemokines that recruit CD8+ T cells (see later) to infection sites. Neutrophils are recruited by NK-cell produced IL-22. Finally, invariant NKT (iNKT) cells help to control murine CMV infection and therefore may be active against human CMV. In humans, iNKT cells express an invariant T-cell receptor that recognizes glycolipids presented by the non-classical MHC class I molecule, CD1d.

for by the viral genome and microRNAs. These include the following: 1. Latency. Maintenance of the latent viral genome in the

2.

3.

ADAPTIC IMMUNITY – ANTIBODY RESPONSES

4.

When assessed using cultures of human fibroblasts, gB and gH were found to be the targets of neutralizing antibodies directed against human CMV infection. However, it is now clear that the most important neutralizing antibodies are those targeted against the pentameric complex which block virus replication in endothelial/epithelial cells but not in fibroblasts.

5.

ADAPTIVE IMMUNITY – T-CELL RESPONSES MHC class I restricted CD8+ cytotoxic cells (T-cell receptor composed of and β chains) are the single most important immune defense to give lifelong control over human CMV. Both CMV-specific CD4+ and CD8+ T cells dominate the memory compartments of seropositive subjects (often exceeding 10%). CD4+ T cells that recognize epitopes of CMV in the context of MHC class II are also important for helping generate cytotoxic CD8+ T cells but also some are cytotoxic for virus-infected cells themselves. Lastly, it is likely that T cells are also involved; unlike β T cells, these cells have a receptor composed of and chains that can recognize antigens directly without the need for antigen processing and resemble antibodies in recognizing whole unprocessed molecules.

HUMAN CMV IMMUNE EVASION MECHANISMS Remarkably, each CMV species appears to have coevolved within the mammalian species with which it is associated today. Such species-specific evolution produced different solutions to the problem of evading host immune effector mechanisms. Human CMV encodes more genes for immune evasion than it does to produce the virion. Mechanisms by which human CMV evades both primary and secondary immune responses are mainly the result of proteins coded

absence of lytic replication and virus production allows evasion or delaying of the immune response. The interferon response is disabled at multiple points in the viral life cycle. For example, pp65 (Table 7.1) interferes with DNA sensing pathways. Regulation of NK-cell activation. Some UL genes of CMV have been identified that suppress NK-cell recognition and activation through up-regulation of NK-cell inhibitory receptors and inactivating NK-cell signaling. In addition, by suppressing co-stimulatory molecules expression, they inhibit both CD8+ T-cell and NK-cell mediated antibody dependent cellular cytotoxicity. Targeting antigen presentation by classical and nonclassical human leukocyte antigen (HLA) class I pathways. Several human CMV coded glycoproteins (gpUS2, gpUS3, gpUS6, gpUS10, and gpUS11) down-regulate these pathways using several different mechanisms to suppress the action of CD8+ T cells (Table 7.1). Viral homolog of IL-10 (Table 7.1). This promotes down-regulation of HLA class II molecules and reduces presentation of viral epitopes to CD4+ T cells and consequent activation.

PATHOGENESIS Mammalian cytomegaloviruses are species specific a nd so CMV cannot be studied in animal models. Pathology is presumably produced by direct viral cytopathic effects, although indirect effects produced by viral-encoded proteins or the host immune response remain a possibility. CMV primary infection or reactivation in a healthy individual is rarely a cause of morbidity; subclinical infection occurs frequently in the normal host but is controlled by a robust immune response. Thus, a surfeit of virus-encoded immune evasion mechanisms is not sufficient to completely evade immunosurveillance. Uncontrolled viral replication and disease occurs both in the setting of impaired T-cell function and when CMV crosses the placenta and infects the fetus. Primary infection in pregnancy carries a 40% risk of the virus crossing the placenta and infecting the fetus; virus transmission is highest in the late 2nd and 3rd trimester. Conversely, intrauterine transmission in the late 1st trimester and early 2nd trimester is associated with a greater likelihood of clinical abnormalities at birth and long-term neurodevelopmental abnormalities. Notably, the incidence of symptomatic congenital infection is much lower, although not absent, in maternal CMV reactivation or re-infection. Although maternal preexisting neutralizing antibody in

63

9780367696399_Lydyard.indb 63

13/02/2023 07:32

Case Studies in Infectious Disease

brain encephalitis

lung pneumonitis

liver hepatitis

CMV MONONUCLEOSIS

eye retinitis esophagus esophagitis heart myocarditis kidneys nephritis

colon colitis

spinal cord and nerves Myelitis polyradiculopathy neuropathy

Although primary infection is usually clinically silent in the immunocompetent, it may present as a mononucleosis syndrome in a young adult. Classically, the syndrome comprises any combination of fever, tiredness, sore throat, lymphadenopathy, hepatosplenomegaly, and rash, accompanied by lymphocytosis and altered liver function tests. The disease is clinically indistinguishable from infectious mononucleosis caused by primary Epstein-Barr virus (EBV) infection (see Section 4, differential diagnosis). Complications include CMV pneumonitis, which is usually mild with no treatment necessary. Guillain-Barré syndrome, associated with the emergence of anti-ganglioside antibodies, is rare. There is ascending muscle weakness in the legs and loss of tendon reflexes, accompanied by varying extents of sensory loss.

AIDS AND CMV

Figure 7.4 Sites of CMV organ disease.

seropositive women can reduce intrauterine transmission, the neurodevelopmental consequences of infected infants are strikingly similar to those in infants infected in utero after primary maternal infection.

3. WHAT IS THE TYPICAL CLINICAL PRESENTATION AND WHAT COMPLICATIONS CAN OCCUR? Severe CMV disease mostly occurs in the immunocompro­ mised, namely those with advanced HIV disease (AIDS), solid-organ and hematopoietic stem-cell transplants, and in congenital CMV infection. Because of its broad cell tropism, CMV infection can affect many different organs (Figure 7.4). The disease pattern varies according to the patient’s condition (see below).

_7_4 CMV

64

CMV is the most frequent viral opportunistic infection in patients with AIDS. Disease is usually due to virus reactivation as CMV seropositivity has been noted in more than 90% of HIV-infected men who have sex with men. CMV retinitis (Figure 7.5) is, by far, the most common outcome usually occurring when the CD4 cell count falls to less than 50 cells/mm3. The peripheral retina first shows retinal necrosis, which progresses to more central areas. By the time the patient notices visual disturbance, the retinal infection may be advanced. Blindness occurs within 4 to 6 months. Gastrointestinal (GI) disease is also common. Esophageal CMV infection can cause painful ulceration with dysphagia whereas CMV colitis causes diarrhea. Less frequently, the nerve roots emerging from the spinal cord can become inflamed with a polyradiculopathy. Notably, pneumonitis is not seen; when CD4 counts are below 50 cells/mm3, virus may be detectable in lung washings, but there are no pulmonary infiltrates. Nowadays, CMV end-organ disease is relatively infrequent and observed only in untreated HIV patients or those with poorly controlled HIV infection.

Figure 7.5 Cytomegalovirus (CMV) retinitis. (A) Early disease with retinal involvement along blood vessels. (B) Extensive retinal damage, and retinal hemorrhages. (C) CMV retinitis with papillitis. From Cytomegalovirus (CMV) in: Bennett JE, Dolin R, Blaser MJ, eds. Mandell, Douglas and Bennett’s Principles and Practice of Infectious Diseases, 8th Edition. Elsevier Saunders, 2015. With permission from Elsevier.

9780367696399_Lydyard.indb 64

13/02/2023 07:32

Case 7: Cytomegalovirus

SOLID-ORGAN TRANSPLANT RECIPIENTS

Patients receiving a solid-organ transplant are heavily immunosuppressed to prevent organ rejection. CMV may be transmitted from infected leukocytes in blood perfusing the donor organ, but endothelial cells may also be a source of latent or persistent infection. The risk of CMV disease is greatest in a seronegative recipient (R-) receiving a graft from a seropositive donor (D+) as compared with a seropositive recipient (R+) with re-infection or reactivation of latent virus. The clinical spectrum of CMV infection in the early posttransplant period ranges from the CMV syndrome to organ- and life-threatening end-organ disease. The CMV syndrome includes fever, fatigue, low white blood counts, reactive lymphocytes, elevated hepatic transaminases, and CMV viremia. End-organ disease includes esophagitis, colitis, hepatis, and rarely interstitial pneumonitis or CMV retinitis. Disease varies according to the organ transplanted, for example, nephritis in renal transplant recipients, pancreatitis in pancreas transplants, hepatitis after liver transplant, pneumonitis in lung and heart-lung transplants, myocarditis in heart transplants.

HEMATOPOIETIC STEM-CELL TRANSPLANT RECIPIENTS Patients receiving an allogeneic hematopoietic stem-cell transplant have their hematopoietic cells ablated before receiving donor (D) cells to reconstitute their immune system which takes at least several months. In contrast to solid-organ transplant recipients (R), those at highest risk of disease are the D-R+ cohort; on receipt of a transplant, the patient’s endogenous CMV reactivates as the donor cells lack immune experience of CMV. Transmission from a D+ stem-cell donor to an R- recipient is an intermediate risk because few cells, 90% protective efficacy in sheep. Despite efforts, Echinococcus infection continues to be a problem in many countries.

SUMMARY 1. WHAT IS THE CAUSATIVE AGENT, HOW DOES IT ENTER THE BODY, AND HOW DOES IT SPREAD A) WITHIN THE BODY AND B) FROM PERSON TO PERSON?

2. WHAT IS THE HOST RESPONSE TO THE INFECTION AND WHAT IS THE DISEASE PATHOGENESIS?

■ The causative agent is Echinococcus granulosus or Echinococcus multilocularis. These are tapeworms, or cestodes. ■ Adult Echinococcus tapeworms live in the intestine of dogs or foxes and shed eggs in the feces. Dogs and foxes are the definitive hosts. ■ The eggs are accidentally swallowed by other species. Once swallowed, oncospheres emerge from the eggs, penetrate the intestinal wall, and spread to other parts of the body where they form cysts. ■ Dogs are infected when they eat meat or offal containing cysts. This happens for instance on farms, where they might eat offal from sheep or cattle. ■ Species such as sheep or cows are therefore referred to as intermediate hosts. Humans are usually an end host.

■ Little is known about the host response to the oncospheres when they penetrate the intestinal wall. ■ Once the oncospheres develop into cysts in tissues, there is a surrounding inflammatory response triggered by complement activation, involving macrophages and then a CD4+ and CD8+ T-lymphocyte response. ■ Eventually, an acellular and fibrous layer develops around the cyst with down-regulation of the immune and inflammatory response. ■ The pathogenesis of disease depends on the size of cysts. Large cysts can pose a physical problem in the infected organ. ■ If cysts rupture and leak, an IgE-mediated response can be triggered. Continued...

76

9780367696399_Lydyard.indb 76

13/02/2023 07:32

Case 8: Echinococcus spp.

...continued

3. WHAT IS THE TYPICAL CLINICAL PRESENTATION AND WHAT COMPLICATIONS CAN OCCUR? ■ Small cysts may remain asymptomatic. ■ Hydatid cysts due to E. granulosus are mostly found in the liver or lung or both. The cysts of E. multilocularis are primarily found in the liver. ■ Large hydatid cysts in the liver can cause pain, may obstruct the flow of bile, and may rupture into the biliary tree. ■ Disease due to E. multilocularis can be steadily progressive. ■ Cyst rupture and leakage of proteinaceous contents can trigger an anaphylactic reaction.

4. HOW IS THE DISEASE DIAGNOSED, AND WHAT IS THE DIFFERENTIAL DIAGNOSIS? ■ Diagnosis is based on imaging such as ultrasound or CT scan. Typical radiologic features may be present to diagnose hydatid disease.

FURTHER READING Murphy K, Weaver C. Janeway’s Immunobiology, 9th edition. Garland Science, New York/London, 2016. Stojkovic M, Gottstein B, Junghanns T. Echinococcosis. In: Farrar J, Hotez P, Junghanns T, et al. (eds). Manson’s Tropical Diseases, 23rd edition. Elsevier Saunders, London, 795–819, 2014.

■ Additional tests include serology. ■ Cross-reactions with other tapeworm infections affect the specificity of serology. Sensitivity ranges from 60% to 80%. ■ The differential diagnosis is from other causes of cystic lesions.

5. HOW IS THE DISEASE MANAGED AND PREVENTED? ■ Treatment options depend on cyst size, cyst compartmentaliza­ tion, and features of cyst viability. ■ The main chemotherapeutic agent is albendazole. ■ Surgical resection may be performed if necessary and if feasible. ■ Some experienced centers also undertake a procedure called PAIR, which involves percutaneous puncture of cysts, aspiration of contents, injection of ethanol, and then reaspiration. ■ Prevention requires breaking the life cycle by treating dogs and preventing the consumption of offal by dogs. To avoid accidental infection, humans must maintain high standards of hand hygiene.

Neumayr A, Toia G, de Bernardis C, et al. Justified Concern or Exaggerated Fear: The Risk of Anaphylaxis in Percutaneous Treatment of Cystic Echinococcosis – a Systematic Literature Review. PLos Negl Trop Dis, 5: e1154, 2011. Zhang W, Wen H, Li J, et al. Immunology and Immunodiagnosis of Cystic Echinococcosis: An Update. Clin Dev Immunol, 2012: 101895, 2012.

WEBSITES REFERENCES Bakhtiar NM, Spotin A, Mahami-Oskouei M, et al. Recent Advances on Innate Immune Pathways Related to HostParasite Cross-Talk in Cystic and Alveolar Echinococcosis. Parasit Vectors, 3: 232, 2020. Brunetti E, Kern P, Vuitton DA. Writing Panel for the WHO­ IWGE. Expert Consensus for the Diagnosis and Treatment of Cystic and Alveolar Echinococcosis in Humans. Acta Trop, 114: 1–16, 2010. McManus DP, Zhang W, Li J, Bartley PB. Echinococcosis. Lancet, 362: 1295–1304, 2003.

Centers for Disease Control and Prevention (CDC), Parasites – Echinococcosis, 2020: https://www.cdc.gov/para­ sites/echinococcosis/index.html World Health Organization, Echinococcosis, 2012: https:// www.who.int/health-topics/echinococcosis Students can test their knowledge of this case study by visiting the Instructor and Student Resources: [www. routledge.com/cw/lydyard] where several multiple choice questions can be found.

77

9780367696399_Lydyard.indb 77

13/02/2023 07:32

9

Enteroviruses

A 22-year-old medical student walked into an Emergency Department complaining of a headache that had begun 3 hours before and seemed to be getting worse. He felt generally unwell, but without any other specific symptoms. On examination, he was febrile (38°C), and was noted to have neck stiffness. He complained when a bright light was shone into his eyes. The casualty officer made a diagnosis of acute meningitis and admitted him to the medical ward. A lumbar puncture was performed and 2 hours later the following results were received: ■ cerebrospinal fluid (CSF) – clear and colorless;

1. WHAT IS THE CAUSATIVE AGENT, HOW DOES IT ENTER THE BODY, AND HOW DOES IT SPREAD A) WITHIN THE BODY AND B) FROM PERSON TO PERSON? CAUSATIVE AGENT

Coxsackie viruses belong to the Enterovirus genus within the family of viruses known as the Picornaviridae. “Pico” means small, so this translates as the small RNA viruses. The Picornaviridae carry a genome of positive sense single-stranded RNA in an unenveloped viral particle. The other important human pathogens within this family are the rhinoviruses, the commonest cause of the common cold, the parechoviruses and hepatitis A virus. Enteroviruses are small, icosahedral in shape, and approximately 25–30 nm in diameter. They are non-enveloped, and the virions are relatively simple, consisting of a protein capsid surrounding a single-stranded, positive sense RNA genome of around 7500 bases. The enterovirus genus contains a large number of human pathogens. Historically, their nomenclature has been somewhat eclectic, with a number of polioviruses, Coxsackie A and B viruses (named after the place of first isolation and split into two because of different effects in experimental mice), and ECHO viruses (enteric cytopathic human orphan viruses). More recently, the genus has been divided on the basis of genetic phylogeny based on the VP1 capsid region into 10 species, of which seven – Enterovirus A, B, C, D (containing over 100 serotypes) and Rhinovirus A, B, C – (with over 150 serotypes) are known to infect humans.

9780367696399_Lydyard.indb 79

■ ■ ■ ■

white cell count 120 × 106/l, 75% mononuclear cells, no red cells; CSF protein 4.2g/l (normal 1.5–4.0 g/l); CSF glucose 4.3 mmol/l, blood glucose 5.7mmol/l; Gram stain – no organisms seen. A throat swab and a sample of feces were also sent to the laboratory. The following day, the ward received a telephone call from the microbiology laboratory informing them that the CSF and throat swab were positive for an enterovirus. Three days later, the patient was feeling much better and was ready to be discharged home. Two weeks later, a report from the reference laboratory confirmed that the virus present in the CSF was coxsackie B4.

Any new isolates are designated as EV- (or RV) followed by the letter denoting the species, and the next consecutive number. All these viruses are serologically distinct, that is infection with any one enterovirus induces antibody production that does not cross-neutralize any of the other enteroviruses. The more common/important human enteroviruses are listed in Table 9.1.

ENTRY AND SPREAD WITHIN THE BODY The term enterovirus reflects the fact that these viruses enter and leave the human host via the enteric tract (otherwise known as the gastrointestinal (GI) tract). Thus, they are swallowed in contaminated food or water, being able to survive the highly acidic Ph of the stomach (non-enveloped viruses are in general more hardy than enveloped ones). Initial replication in the small bowel wall results in viral excretion in the feces, which may persist for several days. At some stage, virus passes through the small bowel wall and into the host bloodstream. This viremic stage allows access of the virus to many different cells and tissues of the body, resulting in a number of different clinical manifestations (see below). In addition, some enteroviruses, for example EV-D68, appear to infect via the respiratory tract, while EV-D70 is found almost exclusively in conjunctival and throat swabs.

PERSON-TO-PERSON SPREAD Spread between individuals mostly arises through contamination of food and water, for example, through inadequate hand hygiene after defecation. Enterovirus contamination of water supplies through inadequate disposal

79

13/02/2023 07:32

Case Studies in Infectious Disease

Table 9.1 The human enteroviruses Old classification: Polioviruses (serotypes 1, 2, and 3) Coxsackie A viruses (23 serotypes, numbered A1–A24, as coxsackie A23 is the same virus as echovirus 9) Coxsackie B viruses (serotypes B1–B6) Echoviruses (31 serotypes, numbered 1–33, as echoviruses 10 and 28 have been reclassified into other virus families) Enterovirus serotypes (EV 68–71) New classification: Enterovirus species

Types known to infect humans

Enterovirus A

Coxsackievirus A16 EV-A71

Enterovirus B

Coxsackievirus A9 Coxsackie B viruses 1-6 All echoviruses

Enterovirus C

Coxsackievirus A1 Polioviruses 1-3 EV-C95 et seq

Enterovirus D

EV-D68

of sewage can give rise to outbreaks of infection. Outbreaks may also occur in closed communities such as boarding schools or households.

EPIDEMIOLOGY Enteroviral meningitis may arise at any age but is most common in infants and young children. There is a seasonality to enterovirus infections, most arising in the summer months in temperate climates. Enterovirus infections are common worldwide. The current epidemiology of poliovirus infection is described below under Section 5 (Prevention).

2. WHAT IS THE HOST RESPONSE TO THE INFECTION AND WHAT IS THE DISEASE PATHOGENESIS? IMMUNE RESPONSES

Innate immune responses are crucial in determining the fate of many virus infections. Initial recognition of viral entry is via pattern recognition receptors (PRRs), which bind pathogenassociated molecular patterns (PAMPs). There are five PRRs, including Toll-like receptors (TLRs), retinoic acid-inducible gene 1 (RIG-I)-like receptors, and nucleotide-binding and oligomerization domain (NOD)-like receptors (NLRs). Binding of these receptor families to fragments of viral DNA, RNA or proteins triggers signaling pathways leading ultimately to activation of NF-κB and AP-1 transcription factors which, in turn, generate rapid pro-inflammatory responses, including type-1 interferon production.

In order to evade these initial antiviral mechanisms, viruses have evolved a number of countermeasures, and enteroviruses are no exception. The enteroviral genome encodes a number of non-structural proteins, including at least two with protease function – proteases 2A and 3C. These have been shown in in-vitro systems, and in animal models, to mediate a number of cleavages of key host antiviral proteins – the EV-71 2A protein cleaves melanoma differentiation-associated protein 5 (MDA5), RIG-I, and the mitochondrial antiviral signaling protein (MAVS), all of which are integral to the signaling pathways leading to pro-inflammatory and interferon responses. It further reduces the effect of any type-1 interferon that is generated by cleaving the type-1 interferon receptor, thereby blocking the expression of interferon-stimulated genes. More recently, EV-71 proteases C3 and 2A have been shown to antagonize the activation of the NOD-like receptor family pyrin domain containing 3 (NLRP3) inflammasome which is responsible for IL-1β production in response to infection. Study of the interactions between enteroviruses and these complex innate immune recognition signaling pathways is not only generating new knowledge about the function and mechanisms of those pathways but may also identify new targets for antiviral drugs. Natural killer (NK) cells appear to be important in the early response to coxsackie B viruses, not through cytotoxicity but rather through their production of interferon (IFN)- . In fact, mice lacking the IFN-g receptor die quickly after challenge with coxsackie B viruses. Antibody responses against enteroviruses appear 7–10 days after infection. Those directed against the capsid proteins may be neutralizing.

80

9780367696399_Lydyard.indb 80

13/02/2023 07:32

Case 9: Enteroviruses

T-cell responses are also made but the humoral immune response appears to be more important for this group of viruses than the cell-mediated response, as evidenced by the following: 1. In patients with antibody deficiency, enterovirus

infection may become chronic, for example, persistence of live poliovirus vaccines given to patients with hypogammaglobulinemia, with a particular risk of serious CNS manifestations, for example, chronic enteroviral encephalitis. 2. Enteroviruses are not recognized as particularly serious pathogens in the context of patients with cell-mediated immune deficiencies, for example, those with HIV infection, or transplant recipients. IgA might play a role in protection by preventing entry via the intestinal epithelium. Coxsackie B viruses and other enteroviruses have a number of strategies to prevent immune responses. These include: down-regulation of surface MHC class I molecules to thwart CD8+ T-cell responses and inhibition of dendritic cell function (seen with echoviruses but not coxsackie B viruses).

PATHOGENESIS Evidence has been accumulating that coxsackie B viruses and other enteroviruses may play a role in autoimmune disease. Enteroviruses have been implicated in the etiology of type-I diabetes through “molecular mimicry”. There is some evidence that antibodies to coxsackie viruses cross-react with human islet cell antigens and it has been reported that some epitopes of coxsackie B viruses cross-react at the T-cell level with glutamic acid decarboxylase (GAD). Enteroviruses are thought to be the main etiologic agents of viral myocarditis, which is a common cause of idiopathic dilated cardiomyopathy, a severe pathologic condition that often requires heart transplantation.

of poliovirus. Most infections with polioviruses are asymptomatic; a small percentage (up to 10%) may present with a febrile illness with, in some patients, evidence of CNS involvement such as headache, photophobia, or neck stiffness. Paralytic poliomyelitis occurs in around 1 in 1000 children or 1 in 75 adults. Infection of nerve cells supplying muscle tissue (collectively referred to as lower motor neurons, whose cell bodies lie within the anterior horn of the spinal cord) results in death of those cells and flaccid paralysis of the relevant muscle (Figure 9.1). This is potentially lifethreatening, especially if muscles involved with respiration are affected. Very occasionally, poliomyelitis-like illness can arise from infections with other enteroviruses, for example, coxsackie viruses, or even with completely unrelated viruses, for example, Japanese encephalitis or West Nile viruses.

MENINGITIS As in our specific case, meningitis typically presents with a fever, irritability and/or lethargy, especially in young children, neck stiffness (although the absence of this symptom/sign does not exclude a diagnosis of meningitis, especially in young children), and an aversion to bright light – photophobia. Complications of enteroviral meningitis are unusual in an immunocompetent child or adult. Spread of virus from the meninges to the brain, resulting in meningoencephalitis, may occur rarely. This may be heralded by an abrupt deterioration in mental state, or the onset of seizures. In patients with immunodeficiencies, particularly those associated with impaired antibody production, meningoencephalitis is much more common, and may become chronic.

3. WHAT IS THE TYPICAL CLINICAL PRESENTATION AND WHAT COMPLICATIONS CAN OCCUR? As mentioned above, the clinical manifestations of enteroviral infection are protean, although, despite the name enterovirus, these viruses do not cause gastroenteritis. The most common outcome is probably no disease at all, that is, asymptomatic infection. Febrile illnesses with nonspecific rashes are common in children, and some at least are due to enterovirus infections. Coxsackie A viruses may give rise in children to herpangina, or hand, foot, and mouth disease (vesicles in the mouth and on the hands and feet). Conjunctivitis is particularly associated with EV70. The disease poliomyelitis has been known for centuries and arises from infection with one of the three serotypes

Figure 9.1 This image shows muscle wasting (right lower limb) arising from poliomyelitis. From Science Photo Library, with permission.

81

9780367696399_Lydyard.indb 81

13/02/2023 07:32

Case Studies in Infectious Disease

MUSCLE INFECTIONS

Coxsackie viruses can also infect muscle tissue itself, giving rise to Bornholm’s disease (with involvement of chest muscles) or myocarditis. Enteroviruses are also the commonest cause of viral pericarditis.

RESPIRATORY INFECTIONS Within the respiratory system, enteroviruses occasionally cause upper respiratory tract manifestations such as the common cold or, especially in young babies, can involve the lower respiratory tract, presenting with bronchiolitis or even pneumonia.

NEONATAL INFECTIONS Enteroviruses are particularly feared pathogens in neonates, in whom they may cause devastating disseminated infections, resulting in multisystem life-threatening disease including myocarditis, hepatitis, and encephalitis. Furthermore, patient-to-patient spread within a neonatal ward has been reported on a number of occasions.

4. HOW IS THE DISEASE DIAGNOSED, AND WHAT IS THE DIFFERENTIAL DIAGNOSIS? DIAGNOSIS

82

Meningitis is a medical emergency, and appropriate diagnostic tests must be initiated as soon as possible to expedite effective therapy (if available). Initial investigations should include blood cultures and a lumbar puncture (although this is contraindicated if there is evidence of raised intracranial pressure) to obtain a sample of CSF. Laboratory examination of the CSF usually allows distinction between bacterial and viral meningitis. In the former, CSF protein is grossly raised, CSF glucose is reduced to less than half of the blood sugar level, and the predominant cellular infiltrate is with polymorphonuclear leucocytes (PMNs) (Figure 9.2A). Gram staining of centrifuged CSF, that is the pellet, will often reveal the presence of bacteria. In viral meningitis, CSF protein is only marginally raised, CSF glucose is usually normal, and the predominant cellular infiltrate is with mononuclear cells, that is lymphocytes and monocytes (Figure 9.2B) (although an early predominance of PMNs can sometimes be seen very early in the course of viral meningitis). A Gram stain will be negative. Mumps virus used to be the commonest cause of viral meningitis, but with the introduction of effective mumps vaccines (given universally precisely because of this manifestation of infection) enteroviruses now account for the majority of cases of viral meningitis. Herpes simplex virus (HSV) can also cause meningitis, usually in young women with extensive genital herpes infection. Confirmation that viral meningitis is due to an enterovirus, as opposed to other

9780367696399_Lydyard.indb 82

viral causes, can be made in a number of ways. The most sensitive approach is to detect the presence of enteroviral RNA in the CSF sample by a genome amplification technique such as the polymerase chain reaction (PCR). By sequencing the PCR product, it is possible to determine which particular enterovirus is the culprit – this is essential if patients present with flaccid paralysis, in order to demonstrate or rule out infection with polioviruses. However, genome detection assays may miss infection with novel enteroviruses if there is sequence divergence at the oligonucleotide primer binding sites. Many (but not all) enteroviruses may also be isolated from CSF in routine tissue culture. If CSF is not available, then alternative samples are a throat swab or a sample of feces. Detection of enterovirus in feces may be possible up to 2 weeks after initial clinical presentation. Serologic assays are available for detection of an antibody response to an enterovirus. IgM anti-enterovirus antibodies are indicative of a recent infection, but this test is usually only available within a reference laboratory. An alternative approach is to demonstrate a rise in IgG anti-enterovirus antibody titers in a sample taken 1 or 2 weeks after the initial presentation (known as a convalescent sample) compared with those in a sample taken at the time of acute illness. Thus, this is not a technique that allows the diagnosis to be made when the patient first presents acutely ill. Other clinical manifestations may be evident on examination of the patient, depending on the particular causative agent, for example, meningococcal meningitis is

Figure 9.2 Staining (Giemsa) of CSF deposit showing: (A) polymorphonuclear cells typical of bacterial meningitis; (B) mononuclear cells typical of viral meningitis.

13/02/2023 07:32

Case 9: Enteroviruses

often associated with a spreading, nonblanching purpuric rash (see Case 25).

DIFFERENTIAL DIAGNOSIS Other causes of viral meningitis include mumps virus, HSV, and some arboviruses.

5. HOW IS THE DISEASE MANAGED AND PREVENTED? MANAGEMENT

Enteroviral meningitis will usually resolve spontaneously, and the vast majority of patients will make a complete recovery. There are no licensed antiviral drugs with activity against enteroviruses, so management is supportive only.

PREVENTION There are two types of vaccines available for the prevention of poliovirus infection (but neither offers any protection against infection with other enteroviruses): (a) live attenuated vaccines, known as Sabin vaccines after the person who first developed them, which may contain one, two or all three poliovirus serotypes. These are administered orally (i.e. mimicking the natural route of infection), which therefore

induces immunity within the gut mucosa, as well as within the bloodstream. While these are effective and inexpensive, the two main drawbacks are the necessity of maintaining a cold chain so that the viruses remain viable, and the potential for these viruses to revert to pathogenicity as they are excreted from the bowel in feces. Such vaccine-derived polioviruses (VDPV) can circulate and cause disease; (b) inactivated vaccines (generically referred to as Salk vaccines), again containing one, two or all three poliovirus serotypes. These are administered by intramuscular (IM) injection and may therefore not induce as good a mucosal immune response as Sabin vaccines but are safer as there is no risk of reversion to virulence. The World Health Organization (WHO) is currently co-ordinating a campaign for the elimination of poliovirus infection, which has resulted in a decrease of over 99% in the numbers of cases from an estimated 350 000 in 1988 to 175 in 2019. Wild type polioviruses 2 and 3 have officially been certified as eradicated. Wild type poliovirus 1 currently is only circulating in Pakistan and Afghanistan. However, VDPV type 2 viruses are circulating in 30 countries, including the UK as of August 2022, giving rise to over 400 cases of polio in 2021. The dramatic changes in poliovirus circulation and epidemiology has led to the modification of the traditional trivalent forms of polio vaccines, most notably with the development of monoand bi-valent vaccines such as a more genetically stable novel OPV2 vaccine (nOPV2) designed to combat the circulating VDPV2.

SUMMARY 1. WHAT IS THE CAUSATIVE AGENT, HOW DOES IT ENTER THE BODY, AND HOW DOES IT SPREAD A) WITHIN THE BODY AND B) FROM PERSON TO PERSON? ■ The enteroviruses are non-enveloped positive single-stranded RNA viruses belonging to the family Picornaviridae. ■ The enterovirus genus is now classified into 10 species, seven of which (four enterovirus and three rhinovirus) contain types which infect humans. The EV types (A, B, C and D) contain over 100 individual enterovirus types, including poliovirus types 1–3, coxsackie A viruses (23 types), coxsackie B viruses (6 types), echoviruses (31 types), and enteroviruses numbered from 68 to over 100. ■ Virus entry is usually via the gastrointestinal tract. Initial replication occurs within the small bowel, associated with excretion of virus in the feces. ■ This is followed by viremic spread to various target organs. ■ Human-to-human spread is via the feco–oral route.

2. WHAT IS THE HOST RESPONSE TO THE INFECTION AND WHAT IS THE DISEASE PATHOGENESIS? ■ Enteroviruses have evolved several mechanisms for evasion of the innate immune response. ■ Antibody responses are essential to control enterovirus infections.

■ In antibody-deficient individuals, infection may become chronic and can be life-threatening.

3. WHAT IS THE TYPICAL CLINICAL PRESENTATION AND WHAT COMPLICATIONS CAN OCCUR? Enteroviruses cause a wide range of diseases, including the following: ■ Febrile illness with or without skin rashes. ■ Respiratory tract infections (e.g. EV-D68). ■ Herpangina (usually coxsackie A viruses). ■ Hand, foot, and mouth disease (usually coxsackie A viruses). ■ Conjunctivitis (e.g. EV-D70). ■ Meningitis. ■ Encephalitis (in antibody-deficient patients). ■ Myocarditis and pericarditis. ■ Bornholm’s disease (epidemic pleurodynia). ■ Disseminated multisystem disease in the newborn. ■ Poliomyelitis.

4. HOW IS THE DISEASE DIAGNOSED, AND WHAT IS THE DIFFERENTIAL DIAGNOSIS? Enteroviral meningitis is diagnosed on the basis of typical CSF findings (mononuclear cell infiltrate, protein raised only marginally, glucose normal, gram-stain negative) plus any of the following: ■ Genome amplification (e.g. PCR) within CSF. ■ Isolation of the virus in tissue culture from CSF, throat swab or feces. Continued...

83

9780367696399_Lydyard.indb 83

13/02/2023 07:32

Case Studies in Infectious Disease

...continued

■ IgM antibodies, or rising titers of IgG antibodies. ■ Other causes of viral meningitis include mumps virus, herpes simplex virus, some arboviruses.

5. HOW IS THE DISEASE MANAGED AND PREVENTED? ■ Enterovirus infections, including meningitis, are almost always selflimiting in immunocompetent hosts and resolve spontaneously.

FURTHER READING Barer M, Irving W, Swann A, Perera N. Medical Microbiology, 19th edition. Elsevier, Philadelphia, 2019. Murphy K, Weaver C. Janeway’s Immunobiology, 9th edition. Garland Science, New York/London, 2016. Richman DD, Whitley RJ, Hayden FG. Clinical Virology, 4th edition. ASM Press, Washington, DC, 2016.

REFERENCES Autore G, Bernardi L, Perrone S, Esposito S. Update on Viral Infections Involving the Central Nervous System in Paediatric Patients. Children, 8: 782, 2021. Brown DM, Zhang H, Scheuermann RH. Epidemiology and Sequence-Based Evolutionary Analysis of Circulating Non-Polio Enteroviruses. Microorganisms, 8(12): 1856, 2020. Chumakov K, Ehrenfeld E, Agol VI, Wimmer E. Polio Eradication at the Crossroads. Lancet Global Health, 9: e1172– 1175, 2021. Elrick MJ, Pekosz A, Duggal P. Enterovirus D68 Molecular and Cellular Biology and Pathogenesis. J Biol Chem, 296: 100317, 2021. Zhang Y, Li J, Li Q. Immune Evasion of Enteroviruses Under Innate Immune Monitoring. Front Microbiol, 9: 1866, 2018.

■ There are no licensed antiviral drugs with proven activity against enteroviruses. ■ Poliovirus infection can be prevented by use of either live attenuated (Sabin) or inactivated (Salk) vaccines. The WHO poliovirus eradication campaign has achieved considerable success, although wild-type poliovirus 1 still circulates in Pakistan and Afghanistan, while vaccine-derived polioviruses also continue to circulate.

WEBSITES All the Virology on the WWW Website, Tulane University, 1995: http://www.virology.net/garryfavweb13.html#entero Centers for Disease Control and Prevention, USA: http:// w w w.cdc.gov/ncidod/dv rd/revb/enterov irus/non-polio_ entero.html GOV.UK, Polio: the green book, chapter 26, 2013: https:// w w w.gov.uk/government/publications/polio-the-green­ book-chapter-26 GOV.UK, Polio: guidance, data and analysis, 2013: https:// w w w.gov.u k /gover nment/col lect ions/pol io-g u ida nce­ data-and-analysis Virology Online, Enteroviruses Slide Set: http://virology­ online.com/viruses/Enteroviruses.htm

Students can test their knowledge of this case study by visiting the Instructor and Student Resources: [www. routledge.com/cw/lydyard] where several multiple choice questions can be found.

84

9780367696399_Lydyard.indb 84

13/02/2023 07:32

10

Epstein-Barr virus

A 20-year-old woman went to her doctor complaining of a sore throat. It started 4 days previously with associated episodes of fever and chills. On examination, her doctor noticed that her tonsils and uvula were red and slightly swollen and that she had cervical lymphadenopathy. Suspecting a bacterial infection, her doctor prescribed a course of ampicillin and suggested that she took a couple of days off work. The patient returned a week later with worsening symptoms. She was now very lethargic with a temperature of 38.5°C and a widespread maculopapular rash as shown in Figure 10.1.

On the second visit to her doctor, she had patches of white exudate on the tonsils (Figure 10.2), petechial hemorrhages on the soft palate, and generalized lymphadenopathy. The doctor could also palpate an enlarged spleen and noticed that she was slightly tender over the right hypochondrium. He suspected that she might have infectious mononucleosis and sent her for hematological and antibody tests.

Figure 10.1 Infectious mononucleosis: ampicillin-induced rash. Note the resemblance to measles. With permission from Elsevier and the original author. This photo was published in ‘A Colour Atlas of Infectious Diseases’ by Emond, Welsby and Rowland.

Figure 10.2 Infectious mononucleosis: follicular exudate on the tonsil, which is very swollen; the uvula is red and edematous. With permission from Elsevier and the original author. This photo was published in ‘A Colour Atlas of Infectious Diseases’ by Emond, Welsby and Rowland.

1. WHAT IS THE CAUSATIVE AGENT, HOW DOES IT ENTER THE BODY AND HOW DOES IT SPREAD A) WITHIN THE BODY AND B) FROM PERSON TO PERSON?

A fter primary infection, herpesviruses remain latent in the body persisting for life, for EBV the site of latency is the B lymphocyte. EBV-infected cells can exist in two states. In lytic replication, the resultant virions have a linear DNA genome. In latency, the viral genome is a circular episome and is replicated in dividing cells by cellular enzymes; there is

CAUSATIVE AGENT

T he patient has the clinical symptoms of primary EpsteinBarr virus (EBV) infection – infectious mononucleosis or glandular fever, as it is commonly called due to concomitant swollen lymph nodes (lymphadenopathy). The virus has an outer envelope with an inner icosahedral protein capsid and a linear double-stranded (ds) DNA genome of about 172 kb. The EBV envelope has viral glycoproteins on its surface including gp350, gp42, gB, gH, and gL. EBV belongs to the Herpesviridae, of which there are nine viruses identified to date as human pathogens (Table 10.1).

9780367696399_Lydyard.indb 85

Table 10.1 Human herpesviruses Alpha herpesvirus

Herpes simplex virus 1 Herpes simplex virus 2 Varicella-zoster virus

Beta herpesvirus

Cytomegalovirus Human herpesviruse 6A and 6B Human herpesvirus 7

Gamma herpesvirus

Epstein-Barr virus Human herpesvirus 8, also known as Kaposi sarcomaassociated herpesvirus

85

13/02/2023 07:32

Case Studies in Infectious Disease

Table 10.2 Patterns of EBV latent gene expression and disease Latency program

EBV gene product

EBV-associated disease

EBNA1

EBNA2

EBNA3

LMP1

LMP2

0

-

-

-

-

-

None

I

+

-

-

-

-

BL, gastric carcinoma

II

+

-

-

+

+

NPC, Hodgkin lymphoma, T-cell lymphoma

III

+

+

+

+

+

Infectious mononucleosis, posttransplant LPD, PCNSL

BL, Burkitt lymphoma; EBNA, Epstein-Barr nuclear antigen; LMP, latent membrane protein; LPD, lymphoproliferative disease; NPC, nasopharyngeal carcinoma; PCNSL, primary central nervous system lymphoma Table 10.3 EBV latent gene products and their functions EBV gene product

Function

EBNA1

EBV gene maintenance; EBNA1 is only expressed in dividing cells to ensure proper segregation of the EBV episomal genome to both daughter cells

EBNA2

Activates expression of EBV, and host B cell genes including c-myc, & CD21

EBNA3

Represses tumor suppressor gene expression and promotes B lymphocyte transformation into LCLs – see below for LCLs in “Entry and spread within the body”

LMP1

Initiates B-cell activation and proliferation and inhibits apoptosis; mimics CD40 signaling – constitutively activates pathways that mimic the growth and survival signals given to B cells by CD4+ T lymphocytes. Essential for transformation into LCLs.

LMP2

Mimics B-cell receptor signaling.

LCL, lymphoblastoid cell line

limited gene expression and no infectious particle production. Table 10.2 shows the various patterns of latency, distinguished by different latency antigens and Table 10.3 the functions of the latency gene products. The latent state is implicated in several cancers, neoplasms (Table 10.2, and see Section 3).

VIRUS REPLICATION For entry into B cells, gp350 binds to the cellular receptor for the complement component C3d (CR2/CD21). Viral gp42 then binds to the major histocompatibility complex (MHC) class II on the cell surface, which serves as co-receptor. Fusion between the virus envelope and the host-cell membrane follows mediated by gH/gL and gB and the virus is endocytosed. EBV replication in epithelial cells is well established in vitro and gp350 may also be important for virus attachment to CD21. Another possibility is that the viral glycoprotein, BMFR2, binds to cellular integrins. In contrast to B cells, epithelial cells lack MHC class II and entry at the plasma membrane is gp42 independent. EBV undoubtedly has clinically significant tropism for epithelial cells in vivo, as seen in cancer of the nasopharynx and oral hairy leukoplakia of the tongue in HIV patients, where unchecked lytic replication occurs in the squamous epithelium of the tongue. EBV virions produced by B cells have reduced gp42 because it binds to MHC class II during budding from the infected cell; the consequence is that the virus infects epithelial cells more efficiently than other B cells. Conversely, virions derived from

epithelial cells lacking MHC Class II are better at infecting B lymphocytes as they have high levels of gp42. This reciprocal tropism is proposed to enhance EBV shuttling between B lymphocytes and the oral epithelium. Natural killer (NK) cells and T cells may occasionally be the targets of EBV during primary infection. CD21 has recently been established as the entry receptor for T cells but the receptor for NK cells is unknown.

ENTRY AND SPREAD WITHIN THE BODY In primary infection, EBV transmitted in saliva enters the oral cavity, where CD21 is expressed on tonsil epithelium making it a likely site for virus replication rather than non-tonsillar sites. This is logical because tonsils are the site of mucosa­ associated lymphoid tissues with consequent circulating B lymphocytes. Although final proof is lacking, it is generally accepted that EBV first replicates in oral epithelial cells and then infects mucosal B cells. Alternatively, the virus may infect B cells in the tonsillar crypts directly. While some oropharyngeal B cells undergo lytic viral replication infection producing virions, most become latently infected (latency program III); these B lymphoblasts have entered the cell cycle and proliferate continuously in a process termed transformation or immortalization, which serves to expand rapidly the pool of infected B lymphocytes. These cells can be propagated in vitro indefinitely. The ability of EBV to convert peripheral blood B cells into immortalized

86

9780367696399_Lydyard.indb 86

13/02/2023 07:32

_3

Individuals infected with EBV (%)

Case 10: Epstein-Barr virus

Spread of EBV is usually through intimate contact between an infected and an uninfected person – often through kissing. Hence, infectious mononucleosis is known as the “kissing disease”. In children, the virus is mainly spread by fingers contaminated with saliva and other close contact. EBV-seronegative individuals can also acquire EBV by blood transfusion or organ transplant.

100 75 50 25 0

0

5

10

15

20

25

30 40 50 60 70+ Age (years)

Africa

Developed countries - less well off

Southeast Asia

Developed countries - affluent classes

Figure 10.3 Comparison of the ages at which individuals in different populations become infected with Epstein-Barr virus (EBV). From Epstein MA & Rickinson AB (2010) Virology https://doi. org/10.1002/9780470015902.a0002318.pub2. With permission from John Wiley & Sons.

lymphoblastoid cell lines is widely used in genomic studies as a means of preserving DNA from volunteer donors for future use. Virus and infected cells pass via the afferent lymphatics into the cervical lymph nodes where further B cells are infected. These cells reach the bloodstream via the thoracic duct and spread to the spleen and other lymphoid tissues where EBV causes B-lymphocyte proliferation, lymphadenopathy, and splenomegaly. The incubation period of infectious mononucleosis (i.e., time from initial infection to development of symptoms) is usually 4–8 weeks. Even after resolution of symptoms, the virus is not cleared from the body and remains latent throughout life with occasional reactivation into a lytic cycle producing virions transmissible to a new host. It is not fully understood how an initial growth transforming infection in latency program III results in selective colonization of the nonproliferating memory B-cell compartment in latency program 0. ThorleyLawson and colleagues have postulated a model applied to normal B-cell differentiation which provides an attractive explanation. To enter the resting state and become a memory cell, an uninfected B-cell receptor must bind its cognate antigen and receive appropriate signals from helper T cells in the germinal centers of lymphoid tissue. During EBV latent infection III, the viral LMP proteins mimic all these steps, such that the infected B cell could differentiate into a memory cell in the absence of external cues.

PERSON-TO-PERSON SPREAD EBV is present in saliva throughout life. EBV-infected memory cells can differentiate into plasma cells seeding lytic infection at oropharyngeal mucosal sites. However, it is unclear whether the final source of salivary virus is infected B or epithelial cells. Whatever the mechanism, most, if not all EBV-infected persons shed low levels of virus throughout their life with the potential to infect others.

9780367696399_Lydyard.indb 87

EPIDEMIOLOGY OF EBV INFECTION AND INFECTIOUS MONONUCLEOSIS Globally, most adults are infected with EBV (95–98%; Figure 10.3). At least 50% of those who first acquire the virus in the second or third decade develop some symptoms of infectious mononucleosis (see Section 3). The incidence of infectious mononucleosis shows no consistent seasonal peak, and the disease is rare in older adults. In developing countries, almost all children have acquired the virus by 2–4 years of age, and the infection is usually asymptomatic or so mild that it is undiagnosed. In developed countries with high standards of living and hygiene, the time of infection is delayed for many individuals, more markedly among the affluent than the less well off. Among the very rich, as many as 50% may reach adolescence or young adulthood without having encountered the virus and will undergo delayed primary infection with a high risk that this will be accompanied by the symptoms of infectious mononucleosis.

2. WHAT IS THE HOST RESPONSE TO THE INFECTION AND WHAT IS THE DISEASE PATHOGENESIS? There is a paucity of information concerning the role that oropharyngeal mucosal immunity plays in host defense against EBV. Mucosal surfaces are protected by a common immune system and about two-thirds of B and T cells in the body are found both within the epithelium and in the submucosa (lamina propria).

MUCOSAL INNATE IMMUNITY The oral mucosa is flushed by saliva, which protects, hydrates, and lubricates the epithelium. Mucins, high molecular weight glycoproteins serve to aggregate and remove microorganisms and several antimicrobial peptides, such as defensins, have antiviral activity. Rather than being protective, defensins and transmembrane mucins might promote infection by inducing local inflammation. However, oral epithelial cells do not generally secrete high levels of pro-inflammatory cytokines and infection is resolved with minimal inflammatory tissue destruction. Secretory immunoglobulin A is the principal immunoglobulin isotype present in external secretions. IgA is a non-inflammatory immunoglobulin that does not activate complement. Its role at barrier epithelia is immune exclusion. Two studies from the early 1990s reported that salivary IgA

87

13/02/2023 07:32

Case Studies in Infectious Disease

antibody responses to gp350 were detected in a minority (12–19%) of healthy virus carriers and in a higher proportion (49%) of nasopharyngeal carcinoma patients. In vitro IgA specific for gp350 promoted EBV infection of the otherwise refractory epithelial cells with virus entering the cell through secretory component-mediated IgA transport. However, EBV coated in IgA no longer infected B lymphocytes. Such an immune-induced shift in EBV tissue tropism might possibly be involved in the pathogenesis of cancer of the nasopharynx, a malignancy of epithelial origin (see Section 3, Non-B-cell malignancies of epithelial origin). The mouth has keratinized squamous epithelium on the palate, dorsum of the tongue, and attached gingivae as well as non-keratinized squamous epithelium elsewhere. These epithelial cells desquamate continuously taking with them any adherent microorganisms. Importantly, the oropharynx has a ring of mucosa-associated lymphoid tissues, known as Waldeyer’s ring, comprising the pharyngeal, tubal, palatine, and lingual tonsils as well as small collections of lymphatic tissue distributed throughout the pharyngeal mucosa. EBV must penetrate the oral mucosal epithelium to infect B cells. M cells are present in the epithelium overlying Waldeyer’s ring; these serve to capture and transcytose microparticles, such as viruses, and might deliver EBV to B cells. Alternatively, EBV may enter via sites of microscopic injury, be picked up by intraepithelial dendritic cells that extend their dendrites onto the mucosal surface or be endocytosed by epithelial cells triggered by interaction between viral surface glycoproteins

and lectin-like molecules on the epithelial cell surface. Once EBV reaches the submucosa, it is subject to the innate and lectin complement pathways. However, gamma herpesviruses encode homologs of regulators of complement activation that help them evade the complement system. Upon entering the mucosal epithelium, EBV will be exposed to intraepithelial lymphocytes (IELs), among which are NK cells. These cells target and kill infected cells with decreased surface expression of MHC class I due to EBV infection. Notably NK cell frequency is higher in early childhood than in adulthood; this may explain why robust CD8+ T-cell responses in adults and young adults are required to control EBV in infectious mononucleosis (see later, Adaptive Immunity) whereas primary infection is usually asymptomatic in young children. Other key IELs are invariant NK T (iNKT) cells and T cells. In humans, iNKT cells express an invariant T-cell receptor that recognizes glycolipids presented by the non­ classical MHC class I molecule CD1d. A small percentage of these cells bear CD8 and can directly lyse EBV-infected cells as well as producing interferon to enhance Th1 responses. Notably, patients with X-linked lymphoproliferative disease, (see Section 3, XLPD) lack iNKT cells, which may explain their susceptibility to EBV. The T-cell receptors of T cells may also exert important anti-EBV responses; they recognize antigen directly without a requirement for antigen processing enabling them to function in the recognition of damaged or stressed host cells.

anti-VCA IgM

anti-VCA IgG HA anti-EBNA IgG

anti-EA(D)

Figure 10.4 Basic pattern of serum antibodies to Epstein-Barr virus-associated antigens before, during, and after primary infection. HA, heterophile antibody; EA(D), diffuse form of early antigen; VCA, virus capsid antigen; EBNA, Epstein-Barr nuclear antigen. Incubation period 4–8 weeks. Adapted from ‘Principles and Practice of Clinical Virology’ by Arie J. Zuckerman et al. (2004). With permission from John Wiley and Sons.

88

9780367696399_Lydyard.indb 88

13/02/2023 07:32

Case 10: Epstein-Barr virus

Figure 10.5 Diagrammatic representation of virus–cell interactions and of virus-induced CD8+ T-cell responses in primary EBV infection, as seen in infectious mononucleosis patients, and in healthy virus carriers. Red arrows denote transmission of virus; black arrows denote movement of cells (where the arrow is dotted, this reflects uncertainty as to the level of cell movement via this route); broad shaded arrows denote effector T-cell function. Transf. B = transformed B. From Hislop AD, Taylor GS, Sauce D & Rickinson AB (2007) Annual Review of Immunology 25:587-617. With permission from Annual Reviews.

ADAPTIVE IMMUNITY – ANTIBODY RESPONSE

• Early antigens

EA, expressed early in lytic replication

infection but are often detectable at varying levels thereafter. Antibodies to EBNAs arise approximately 4 weeks after onset of symptoms and persist for life as do IgG antibodies to VCA. EBV-neutralizing antibodies appear late in infectious mononucleosis and persist at stable titers for life. These antibodies block entry of EBV into epithelial cells and/or B cells. These include anti-gp350, anti-gH/gL, and anti-gp42, but this latter cannot neutralize epithelial cell infection.

• Late antigens

Structural proteins – virus capsid antigens (VCA) and envelope, gp350

ADAPTIVE IMMUNITY – T-CELL RESPONSES

• Latency antigens

EB nuclear antigens (EBNAs), latent membrane proteins (LMPs)

Humoral responses to EBV lytic and latent antigens are seen in infectious mononucleosis. These antigens include the following:

Primary EBV infection also induces heterophile antibodies, which are not EBV-specific a nd characterized by broad reactivity with antigens of other animal species (see Section 4, How is this disease diagnosed); they are the result of EBVinduced polyclonal B-cell activation with differentiation into plasma cells. Figure 10.4 shows the typical sequence of appearance of EBV-specific a nd heterophile antibodies in a primary response. IgM to VCA is the fi rst to appear followed by IgG. Antibodies to EA are highest during the acute phase of

Figure 10.5 shows the virus–cell interactions and the virusinduced CD8+ T-cell response in primary EBV infection as seen in infectious mononucleosis patients and healthy virus carriers. T cells initially respond to the EBV-infected cells by proliferation, fi rstly CD4+ T cells and then CD8+ T cells which outnumber the B cells by a factor of about 50 to 1. These T cells appear in the circulation as “atypical lymphocytes”, a diagnostic feature (see Section 4). During the early response to the virus, up to 50% of the CD8+ T cells in the circulation can be specific for EBV antigens. Infectious mononucleosis is an immunopathologic disease with the immune response itself being responsible for the clinical symptoms and pathogenesis. 89

9780367696399_Lydyard.indb 89

13/02/2023 07:32

Case Studies in Infectious Disease

The symptoms result from the intense immunologic activity of CD8+ T cells specifically recognizing EBV-encoded peptides presented by infected B cells, accompanied by cytokine release and a strong inflammatory response. At first, CD8+ T cells specific for lytic antigens predominate, but with convalescence, a shift occurs toward T cells that recognize latent antigens, particularly EBNA3, expressed by transformed B cells. In infectious mononucleosis, the cytolytic CD8+ T cells control the infection by killing most of the virus-infected cells (transformed B cells) but high-level shedding of virus in the throat of infectious mononucleosis patients continues. This might be because of the poor recruitment of the CD8+ T cells to the tonsillar tissue. A few EBV-infected B cells enter a resting state as members of a long-lived memory cell pool, with about 1–50 cells per million circulating B cells in latency program 0. Following recovery from the primary infection, cytotoxic memory CD8+ T cells persist in the blood at a frequency of 0.2–2% of cytotoxic CD8+ T cells, which increases with age. Some individuals aged over 60 years can have up to 40% of their CD8+ T cells specific for EBV antigens. The importance of T-cell immunity toward EBV is highlighted by the high frequency of EBV-associated malignancies in patients with congenital immunodeficiency syndromes, in patients with HIV/AIDS, or who receive T-cell immunosuppression posttransplant (see Section 3).

VIRUS IMMUNE EVASION EBV has many strategies that help it evade the immune system and avoid attack by CD8+ T cells. These include the following: 1. A few resting memory B cells containing the latent virus

evade the immune response by down-regulation of all EBV antigens (latency program 0). However, in any dividing memory cell, EBV must express EBNA1 to ensure that its genome is replicated. To prevent targeting of this key protein, EBNA1 contains a sequence of expanded glycine­ alanine repeats, not necessary for its function in genome maintenance, capable of inhibiting its proteasomal processing. So that EBNA1 peptides cannot be presented on class I MHC molecules and can elude immune surveillance. 2. The envelope protein gp42 is actively shed as a soluble truncated molecule during lytic infection and binds directly to MHC class II/peptide complexes and inhibits activation of the CD4+ T cells required for generation of CD8+ cytolytic T cells. 3. BCRF1 is secreted during the EBV lytic cycle. This protein is a viral homolog of the cytokine IL-10, which is an immune modulator. Inhibition of Th1- cell function is the result promoting a shift toward T-helper 2 (Th2) differentiated CD4+ effectors; these cells can provide B-cell help but do not promote the CD8+ responses needed to kill EBVinfected cells. BCRF1 also inhibits the activity of NK cells.

3. WHAT IS THE TYPICAL CLINICAL PRESENTATION AND WHAT COMPLICATIONS CAN OCCUR? Around 98% of patients presenting with infectious mononucleosis have a triad of symptoms – pharyngitis, pyrexia, and enlargement of cervical lymph nodes and tonsils. Myalgia is common as is fatigue, which may persist for months after the initial presentation. Splenomegaly is frequent and occurs in up to 100% in patients, usually resolving 4 weeks after symptom onset; splenic rupture is a rare but potentially life-threatening complication. Other complications that are less common include encephalitis, Guillain-Barré syndrome, upper airway obstruction (due to massive lymphoid hyperplasia of the tonsil) – tracheostomy may be required to relieve this, abdominal pain, and liver involvement including hepatomegaly and jaundice. Older patients are less likely to have sore throat and lymphadenopathy but more likely to have liver involvement. Platelet abnormalities are relatively common with patients having mild thrombocytopenia. Autoantibodies induced by EBV infection are believed to be the cause of the thrombocytopenia and, in some cases, this can develop into the more severe idiopathic thrombocytic purpura with hemorrhagic consequences. Aplastic anemia (pancytopenia) has also been described following EBV infection with a similar autoimmune etiology proposed. Rashes occur in about 5% of patients and may be macular, petechial, scarlatiniform, urticarial, or erythema multiforme. Also, 90–100% of patients with infectious mononucleosis who receive ampicillin develop a pruritic maculopapular rash usually 7–10 days after administration. This was seen in the patient described here (Figure 10.1). It is not caused by an allergy to penicillin but rather a transient hypersensitivity reaction since transient ampicillin-binding antibodies have been detected in patients with infectious mononucleosis.

OTHER CLINICAL CONSEQUENCES OF EBV INFECTION As already indicated, recovery is the norm after infectious mononucleosis. However, in addition to the complications already mentioned above, there are several serious clinical consequences that can result from EBV infection. X-linked lymphoproliferative disease (XLPD) is a rare familial condition affecting young boys characterized by extreme susceptibility to EBV. Primary infection results in a particularly severe form of infectious mononucleosis with a large expansion of infected B cells together with T cells infiltrating the organs of the body, especially the liver. Patients with this condition have several abnormalities of the immune system including defects in NK-cell activity, a lack of iNKT cells, overactivity of Th1 cells, and poor Th2 responses leading

90

9780367696399_Lydyard.indb 90

13/02/2023 07:32

Case 10: Epstein-Barr virus

to overexpansion of CD8+ T cells and overproduction of Th1 inflammatory cytokines and hence macrophage activation and hemophagocytosis. The primary defect has been mapped to a small cytoplasmic protein, SAP, involved in NKand T-lymphocyte signaling. In 60% of cases, the disease is fatal. Patients with XLPD generally die of acute liver necrosis or multi-organ failure. Survivors are at high risk of developing B-cell lymphoma. Chronic active EBV (CAEBV) infection is another rare complication of primary EBV infection with a genetic predis­ position. There is histologic evidence of major organ involve­ ment such as interstitial pneumonia, hemophagocytosis, uveitis, lymphadenitis, or persistent hepatitis. It is extremely rare in the US and UK but more frequent in Asia and South America. Diagnostic tests show markedly elevated antibodies to EBV lytic antigens (VCA IgG and EA IgG) and raised EBV DNA levels in the blood. The prognosis for these patients is poor, with most dying of progressive pancytopenia and hypo­ gammaglobulinemia or NK/T cell nasal lymphoma within a few years. The pathogenesis of CAEBV is not well understood but is probably the result of an immune defect that permits the proliferation of EBV-infected T or NK cells.

NEOPLASTIC DISEASES Several malignancies are associated with one of the three general patterns of latent EBV gene expression (Table 10.2). In lymphoproliferative disease (LPD) many of the EBV transforming genes (latency program III) are expressed but immune surveillance is avoided due to immunocompromise; there are few if any host cell mutations. On the other hand, Burkitt lymphoma (BL), nasopharyngeal carcinoma (NPC), and gastric carcinoma arise in the immunocompetent, latency programs I or II mean fewer EBV genes are expressed and other host cell mutations contribute to pathogenesis.

EBV LPD – latency program III. Patients at risk of LPD and lymphoma are those with congenital, acquired or iatrogenic immunodeficiencies, for example, XPLD, HIV, post-organ transplant respectively. They have elevated EBV DNA levels in blood and saliva. EBV causes 90% of LPD in immunosuppressed transplant patients – posttransplant lymphoproliferative disease (PTLD). HIV patients are at risk of primary central nervous system lymphoma. Other B-cell malignancies. EBV has also been associated with some cases of Hodgkin lymphoma in apparently immunocompetent patients.

NON-B-CELL MALIGNANCIES OF EPITHELIAL ORIGIN NPC – latency program II, a malignancy of epithelial cells – relatively rare (less than 1 per 100 000 in most populations) except in southern China, where there is an annual incidence of more than 20 cases per 100 000. There is also a high incidence in isolated northern populations such as Inuit and Greenlanders, while the incidence is moderate in North Africa, Israel, Kuwait, the Sudan, and parts of Kenya and Uganda. The rate of incidence generally increases from age 20 to around 50 and men are twice as likely to develop NPC than women. Evidence indicates that genetic and environmental factors have a role in tumor development. In the US, Chinese Americans comprise most NPC patients, together with workers exposed to fumes, smoke, and chemicals, implying a role for chemical carcinogenesis. There is also an association between eating highly salted foods and the incidence of NPC. Gastric carcinoma – 10% are of mucosal epithelial origin and associated with EBV.

OTHER EBV MALIGNANCIES These include some T- and NK-cell lymphomas.

B-CELL MALIGNANCIES BL – latency program I. This disease is endemic in Central Africa and New Guinea with an annual incidence of 6–7 cases per 100 000 and a peak incidence at 6 or 7 years of age. BL is most frequently found in the “lymphoma belt”, a region extending from West to East Africa. The region is characterized by high temperature and humidity, ideal for the malaria parasite, probably the reason why malaria was suspected early to be associated with BL. In Uganda, the association of BL with EBV is very strong (97%) but is weaker outside the “lymphoma belt”; (85% in Algeria; only 10–15% in France and the US). Persons in regions with endemic disease with high titers of EBV IgG VCA antibodies are at risk of BL and the EBV genome is found in lymphoma cells. In addition, BLs contain a chromosomal translocation involving the c-myc gene, a potent oncogene, and an immunoglobulin heavy or light chain locus resulting in continuous cell proliferation due to the unregulated expression of c-myc.

4. HOW IS THE DISEASE DIAGNOSED, AND WHAT IS THE DIFFERENTIAL DIAGNOSIS? Diagnosis of infectious mononucleosis is made on presentation with pharyngitis, pyrexia, and cervical lymphadenopathy and is confirmed by laboratory tests.

HEMATOLOGIC TESTS The white blood cell count is moderately raised by the second week and a blood film will contain “atypical lymphocytes” up to 10–20% of the blood mononuclear cells, hence mononucleosis (Figure 10.6). Atypical lymphocytes are characterized by the presence of bean-shaped or lobulated nuclei and may persist in the blood for several months. They are cytotoxic CD8+ T cells, produced to control the EBV-induced B-cell proliferation. Atypical lymphocytes are not pathognomonic for infectious 91

9780367696399_Lydyard.indb 91

13/02/2023 07:32

Case Studies in Infectious Disease

antibodies are often absent in children, especially under 4 years. Detection of EBV-specific antibodies is rarely necessary for the diagnosis of infectious mono-nucleosis because 90% of cases are heterophile antibody positive. The basic pattern of serum antibodies in primary EBV infection is shown in Figure 10.4. The most useful antibody assay is that which detects IgM against the Epstein-Barr VCA. Anti-VCA of the IgM class develops early in the illness and declines rapidly over the following 3 months. Enzyme immunoassays (ELISAs) can be used to quantitate IgM anti-VCA, but detection can also be by indirect immunofluorescence assay using EBVinfected lymphoblastoid cell lines as the antigenic substrate. Patients also seroconvert to IgG anti-VCA antibodies in primary infection and these serum antibodies persist for life; testing for IgG anti-VCA antibodies can therefore be used to determine whether an individual has been infected with EBV. Antibodies against EBNA are absent early in infectious mononucleosis but persist indefinitely thereafter. They may therefore be helpful in differentiating EBV reactivation from primary infection. Patients with NPC have elevated levels of serum IgA directed against EBV VCA and EA, which may occur several years before the onset of disease. A screening program for these antibodies has been instituted in southern China to allow early detection and treatment of the cancer.

DIFFERENTIAL DIAGNOSIS Figure 10.6 Photomicrographs of peripheral blood films stained with Giemsa (×40; insets ×100). (A) From a patient with infectious mononucleosis; numerous large atypical mononuclear cells are present (arrows and insets) in addition to the normal cells. (B) From a normal individual showing four neutrophils (detail in left inset) and one lymphocyte (arrow and right inset) among a mass of red cells. From Encyclopaedia of Life Sciences, Infectious Mononucleosis, by MA Epstein, University of Oxford. Doi: 10.1038/npg.els0002318. Permission by John Wiley and Sons. All rights reserved.

mononucleosis and can be observed with other infections (see differential diagnosis below).

ANTIBODY TESTS Heterophile antibodies (see Section 2, Adaptive immunity – antibody) are directed against glycolipid antigens on sheep, horse, or ox erythrocytes, agglutination of which forms the basis of the Paul-Bunnell or Monospot tests. Heterophile

Patients with streptococcal pharyngitis or one of several other infections can present with sore throat, fatigue, and lymphadenopathy (Table 10.4). In the case presented here, the doctor attempted treatment with ampicillin on the supposition of a streptococcal infection. Unfortunately, the patient responded adversely to this antibiotic with a rash, due to an idiosyncratic antibody response peculiar to infectious mononucleosis (see Section 2). Primary CMV infection, toxoplasmosis and primary HIV infection share many clinical and laboratory features of infectious mononucleosis, including splenomegaly, hepatomegaly, lymphocytosis, atypical lymphocytosis, and even (but rarely) false-positive results in the heterophile antibody test. Distinguishing clinically between infectious mononucleosis caused by EBV infection and an infectious mononucleosis-like syndrome caused by toxoplasmosis or

Table 10.4 Infectious mononucleosis: differential diagnosis Diagnosis

Key distinguishing features

Streptococcal pharyngitis

Absence of splenomegaly or hepatomegaly; fatigue less prominent

Other viral pharyngitis, (e.g. adenoviruses, patients with enteroviruses)

Patients are less likely to have lymphadenopathy, tonsillar exudates, or fever than in streptococcal pharyngitis or infectious mononucleosis

Cytomegalovirus (CMV)

Sore throat less severe, lymphadenopathy may be minimal or absent. Confirm by specific antibody tests for primary CMV infection

Toxoplasmosis

Sore throat less severe; confirm by specific antibody tests

Primary HIV infection

Mucocutaneous lesions, rash, nausea, vomiting, diarrhea and weight loss

92

9780367696399_Lydyard.indb 92

13/02/2023 07:32

Case 10: Epstein-Barr virus

CMV may not be possible, or perhaps not useful, since the management of these syndromes is the same. However, diagnostic testing is needed in pregnant women because toxoplasmosis and CMV infections may cause congenital infection resulting in severe damage to the unborn child. If primary HIV infection is suspected, diagnostic tests should also be performed. The differential diagnosis also includes leukemia and lymphoma although these are less likely.

5. HOW IS THE DISEASE MANAGED AND PREVENTED? MANAGEMENT

There is usually no treatment for infectious mononucleosis other than supportive care. Nonsteroidal anti-inflammatory drugs (NSAIDs) or paracetamol (acetaminophen) are given for fever and myalgias; aspirin should not be given to children because it may cause Reye’s syndrome. Patients are recommended to rest and return to usual activities based on their energy levels. Although the exact period of increased risk is unknown, patients are advised to avoid contact sports or other activities that may lead to splenic rupture for several weeks. Corticosteroids may be considered in patients with significant pharyngeal edema that might cause or threaten respiratory compromise.

PREVENTION EBV is a major global health problem. Although it is associated with about 200 000 cancers annually worldwide, a vaccine is not

available. It would be useful to protect adolescents and young adults against the debilitating consequences of infectious mononucleosis. In addition, vaccination might reduce the frequency of EBV-positive Hodgkin lymphoma, which is increased in the first 5 years after infectious mononucleosis. An important target for vaccine is the prevention of BL and NPC in at-risk populations. A vaccine is also needed to prevent LPD in seronegative individuals with XLPD or those who receive a transplant from an EBV seropositive donor. The principal target of EBV-neutralizing antibodies is the major virus surface glycoprotein gp350 and several vaccine candidates based on this molecule have been developed. In 2007, phase I and II studies of a purified gp350 vaccine showed protection from symptoms of infectious mononucleosis but did not prevent asymptomatic infection with EBV, consistent with the possibility of at least reducing cases of infectious mononucleosis. More recently, presentation of gp350 on nanoparticles enhanced the levels of neutralizing antibody in monkeys compared with soluble gp350 and focused the immune response on the CD21 receptor binding site of gp350. An alternative or complementary strategy is to induce potent T-cell responses to control primary infection, reduce the viral load and thus potentially reduce the risk of EBVassociated malignancy. This is based on the observation that elevated EBV viral loads in blood predict development of EBVassociated LPD in transplant recipients and are present at the onset of NPC. A phase I trial has been completed in Australia with a single EBNA3 EBV epitope. In this trial, two placebo recipients became EBV-infected, and one had symptomatic mononucleosis; four vaccine recipients acquired EBV infection but none were symptomatic. To generate a broad-based cytotoxic T-cell response, a vaccine containing multiple EBV epitopes will be necessary.

SUMMARY 1. WHAT IS THE CAUSATIVE AGENT, HOW DOES IT ENTER THE BODY, AND HOW DOES IT SPREAD A) WITHIN THE BODY AND B) FROM PERSON TO PERSON? ■ Epstein-Barr virus (EBV) is a gamma herpesvirus with an outer envelope surrounding an icosahedral capsid enclosing the linear ds DNA genome. ■ EBV-infected cells can exist in two states. In lytic replication, the resultant virions have a linear DNA genome. In latency, the viral genome is a circular episome, there is limited gene expression and no infectious particle production. ■ There are different patterns of EBV gene expression, latency programs 0, I, II, III associated with various neoplasms. ■ For entry into B cells, gp350 on the viral envelope binds to a complement receptor CD21 (CR2) on the surface of B cells. Another envelope glycoprotein, gp42, is responsible for the fusion between the virus envelope and the host-cell membrane. ■ EBV can also infect epithelial cells but entry into the cell is gp42 independent.

■ During primary infection, the virus probably infects pharyngeal squamous epithelial cells and undergoes a lytic cycle producing many more virions that directly infect B cells in transit through the tonsils. ■ EBV causes proliferation of the B cells (B-cell transformation). The ability of EBV to convert peripheral blood B cells into immortalized lymphoblastoid cell lines (LCLs) is widely used in genomic studies. ■ Virus and infected cells pass via the afferent lymphatics into the cervical lymph nodes and from thence via the thoracic duct to the spleen and other lymphoid tissues where EBV causes lymphocyte proliferation, lymphadenopathy, and splenomegaly. ■ Even after resolution of symptoms, the virus is not cleared from the body and remains latent for life in memory B cells with occasional reactivation into a lytic cycle producing virions transmissible to a new host.

Continued...

93

9780367696399_Lydyard.indb 93

13/02/2023 07:32

Case Studies in Infectious Disease

...continued

■ EBV virions are shed into the saliva at high level for several months and persist at low level for life. Spread of the infection is usually through intimate contact with an uninfected person often through kissing, and in children probably mainly by fingers contaminated with saliva and other close contact. ■ Globally, most adults are infected with EBV (95–98%). In developing countries, most children become infected with the virus early in life. In developed countries, at least 50% of adolescents and young adults undergo primary infection with EBV and develop some symptoms of infectious mononucleosis.

2. WHAT IS THE HOST RESPONSE TO THE INFECTION AND WHAT IS THE DISEASE PATHOGENESIS? ■ At the start of the infection, EBV must penetrate the oral mucosal epithelium to infect B cells. This occurs at Waldeyer’s ring, comprising pharyngeal, tubal, palatine and lingual tonsils. M cells are present in the overlying epithelium serving to capture and transcytose microparticles such as viruses and might deliver EBV to B cells. Alternatively, the virus might be picked up by intraepithelial dendritic cells. ■ After entry, EBV is exposed to intraepithelial lymphocytes including natural killer (NK) cells, invariant NK T (iNKT) cells and T cells. These cells are key components of the innate immune response against EBV. ■ EBV antibodies produced at different times as viral antigens are recognized by the immune system of the host. These include early antigens, EA; late antigens such as virus capsid antigens (VCA); and latency antigens including EBV nuclear antigens (EBNAs) and latent membrane proteins (LMPs). ■ IgM antibody to VCA is the first to appear followed by IgG. Antibodies to the latency antigens, for example EBNA, are seen later. ■ Heterophile antibodies (not EBV-specific) also arise due to polyclonal B-cell activation (B-cell transformation) with consequent differentiation into plasma cells. These IgM antibodies are directed against horse and sheep glycolipid antigens on red blood cells. ■ Cytotoxic CD8+ T cells are the main cells that control EBV infection. During primary infection up to 50% of the CD8+ T cells in the circulation can be specific for EBV antigens. ■ The symptoms of infectious mononucleosis are attributable to the intense immunologic response to polyclonally activated B cells mounted by CD8+ T cells. ■ Cytotoxic T cells can control the primary infection but a few B cells containing latent EBV genome enter a resting state as members of a long-lived memory cell pool. ■ Strategies that help EBV to evade the immune system include switching off the expression of viral antigens in resting B memory cells; shedding of gp42, which binds to MHC class II/peptide complexes and inhibits CD4+ T-cell activity; an EBV late protein, BCRF1, is an immunomodulator that inhibits T helper 1 cell activity and hence cytolytic CD8+ T cells.

3. WHAT IS THE TYPICAL CLINICAL PRESENTATION AND WHAT COMPLICATIONS CAN OCCUR? ■ Globally, most individuals infected with EBV do not show clinical symptoms of infectious mononucleosis. Nearly all patients with the disease have a triad of symptoms – pharyngitis, pyrexia, and cervical lymph node and tonsillar enlargement. Hepatocellular enzymes are mildly increased in most cases. ■ Complications include splenic rupture and upper airway obstruction. Encephalitis and Guillain-Barré syndrome occur rarely. ■ Older adults are less likely to have sore throat and lymphadenopathy but more likely to have hepatitis and jaundice. ■ Patients with X-linked lymphoproliferative disease (XLPD), a rare familial condition affecting young boys, have extreme sensitivity to EBV. In 60% of cases, primary infection is fatal due to macrophage activation, hemophagocytosis, and destruction of all lymphoid tissue. Survivors are at risk of B-cell lymphoma. ■ Chronic active EBV (CAEBV) is rare in the US and UK but more frequent in Asia and South America. The prognosis is poor with most dying of progressive pancytopenia and hypogammaglobulinemia or NK/T cell lymphoma within a few years. ■ Several malignancies are associated with EBV latent infection. ■ Burkitt lymphoma (BL) is a malignant tumor associated with EBV. BL has limited EBV gene expression and is characterized by a chromosomal translocation involving the c-myc oncogene. ■ BL is endemic to central parts of Africa and New Guinea, with an annual incidence of 6–7 cases per 100 000 and a peak incidence at 6 or 7 years of age. In African countries such as Uganda, the association of BL with EBV is very strong (97%) but is weaker elsewhere in the world (85% in Algeria; only 10–15% in France and the US). ■ In the absence of a T-cell immune response in XPLD or HIV infection or post-organ transplant, EBV infection can result in life-threatening lymphoproliferative disease (LPD). There is uncontrolled B-cell proliferation as most, if not all, EBV latency genes are expressed. ■ Non-B-cell

malignancies

associated

with

EBV

include

nasopharyngeal carcinoma (NPC) and gastric carcinoma, both of epithelial origin. The virus is also associated with some cases of Hodgkin lymphoma as well as with T- and NK-cell lymphomas. ■ NPC is relatively rare (less than 1 per 100 000 in most populations) except in southern China, where there is an annual incidence of more than 20 cases per 100 000. The rate of incidence generally increases from age 20 to around 50 and men are twice as likely to develop NPC as women. Environmental factors have a role in tumor development.

4. HOW IS THE DISEASE DIAGNOSED, AND WHAT IS THE DIFFERENTIAL DIAGNOSIS? ■ Diagnosis of infectious mononucleosis requires persistent pharyngitis with pyrexia and cervical lymphadenopathy confirmed by laboratory tests. continued...

94

9780367696399_Lydyard.indb 94

13/02/2023 07:32

Case 10: Epstein-Barr virus

...continued

■ The white blood count is moderately raised, and a blood film contains “atypical lymphocytes” up to 10–20% of the blood mononuclear cells. These are cytotoxic CD8+ T cells. ■ Tests for “heterophile antibodies” such as the Paul-Bunnell or Monospot tests are positive. ■ EBV-specific IgM anti-VCA can be detected by indirect immunofluorescence or ELISA. ■ Differential diagnosis includes streptococcal pharyngitis, toxoplasmosis, and primary CMV or HIV infection.

5. HOW IS THE DISEASE MANAGED AND PREVENTED?

■ Patients are recommended to rest and return to usual activities based on their energy levels. Sports activities should be avoided because of the risk of splenic rupture. ■ Corticosteroids may be considered in patients with significant pharyngeal edema that might cause respiratory compromise. ■ Vaccines targeting the membrane gp350 have already been trialed (phase I and phase I/II) and shown to be safe protecting against symptoms of infectious mononucleosis but not asymptomatic EBV infection. ■ A proposed alternative approach is to induce potent T-cell responses to control primary infection, reduce the viral load and thus potentially reduce the risk of EBV-induced malignancy. A phase I trial has been completed in Australia with a single

■ Usually, no treatment is given other than good supportive care.

EBNA3 EBV epitope. To generate a broad-based cytotoxic T-cell

■ Non-steroidal anti-inflammatory drugs (NSAIDs) or paracetamol

response, a vaccine containing multiple EBV epitopes will be

(acetaminophen) may be given to relieve fever and sore throat.

FURTHER READING Gewurz BE, Longnecker RM, Cohen JI. Epstein-Barr virus. In: Howley PM and Knipe DM (eds). Fields Virology, 7th edition. Wolters Kluwer, Philadelphia, 2022. Goering RV, Dockrell HM, Zuckerman M, Chiodini PL. Medical Microbiology and Immunology, 6th edition. Elsevier, Oxford, 2019. Johannsen EC, Kaye KM. Epstein-Barr Virus (Infectious Mononucleosis, Epstein-Barr Virus–Associated Malignant Diseases, and Other Diseases). In: Bennett JE, Dolin R, Blaser MJ, (eds). Mandell, Douglas and Bennett’s Principles and Practice of Infectious Diseases, 8th edition. Elsevier Saunders, Philadelphia, 2015. Rickinson AB, Epstein MA. Epstein-Barr virus. In: Firth J, Conlon C, Cox T, editors. Oxford Textbook of Medicine, 6th edition. Oxford University Press, Oxford, 2020.

REFERENCES Alari-Pahissa E, Ataya M, Moraitis I, et al. NK Cells Eliminate Epstein-Barr Virus Bound to B Cells Through a Specific Antibody-Mediated Uptake. PLoS Pathog, 17: e1009868, 2021. Balfour HH Jr. Epstein-Barr Virus Vaccine for the Prevention of Infectious Mononucleosis: And What Else? J Infect Dis, 196: 1724–1726, 2007. Bharadwaj M, Moss DJ. Epstein-Barr Virus Vaccine: A Cytotoxic T-Cell-Based Approach. Expert Rev Vaccines, 1: 467–476, 2002. Borza CM, Hutt-Fletcher. Alternate Replication in B Cells and Epithelial Cell Switches Tropism of Epstein-Barr Virus. Nat Med, 8: 594–599, 2002.

necessary.

Chung BK, Tsai K, Allan LL, et al. Innate Immune Control of EBV-Infected B Cells by Invariant Natural Killer T Cells. Blood, 122: 2600–2608, 2013. Elliott SL, Suhrbier A, Miles JJ, et al. Phase I Trial of a CD8+ T-Cell Peptide Epitope-Based Vaccine for Infectious Mononucleosis. J Virol, 82: 1448–1457, 2008. Hislop AD, Taylor GS, Sauce D, Rickinson AB. Cellular Responses to Viral Infection in Humans: Lessons from Epstein-Barr Virus. Ann Rev Immunol, 25: 587–617, 2007. Jangra S, Yuen K-S, Botelho MG, Jin D-J. Epstein–Barr Virus and Innate Immunity: Friends or Foes? Microorganisms, 7: 183, 2019. Kanekiyo M, Bu W, Joyce MG, et al. Rational Design of an Epstein-Barr Virus Vaccine Targeting the Receptor-Binding Site. Cell, 162: 1090–1100, 2015. Moutschen M, Leonard P, Sokai EM, et al. Phase I/II Studies to Evaluate Safety and Immunogenicity of a Recombinant gp350 Epstein-Barr Virus Vaccine in Healthy Adults. Vaccine, 25: 4697–4705, 2007. Münz C. Epstein–Barr Virus-Specific Immune Control by Innate Lymphocytes. Front Immunol, 8: 1658, 2017. Sixbey JW, Yao QY. Immunoglobulin A-Induced Shift of Epstein-Barr Virus Tissue Tropism. Science, 255: 1578–1580, 1992. Thorley-Lawson DA, Gross A. Persistence of the EpsteinBarr Virus and the Origins of Associated Lymphomas. N Engl J Med, 350: 1328–1337, 2004. Yao QY, Rowe M, Morgan AJ, et al. Salivary and Serum IgA Antibodies to the Epstein-Barr Virus Glycoprotein gp340: Incidence and Potential for Virus Neutralization. Intl J Cancer, 48: 45–50, 1991.

95

9780367696399_Lydyard.indb 95

13/02/2023 07:32

Case Studies in Infectious Disease

WEBSITES Centers for Disease Control and Prevention (CDC), National Center of Infectious Diseases, 2008: https://www. cdc.gov/epstein-barr/index.html

Students can test their knowledge of this case study by visiting the Instructor and Student Resources: [www. routledge.com/cw/lydyard] where several multiple choice questions can be found.

96

9780367696399_Lydyard.indb 96

13/02/2023 07:32

11

Escherichia coli

Figure 11.1 E. coli cultured from the urine. This shows the growth of lactose-fermenting (yellow colonies) E. coli on a medium containing lactose and a pH indicator. In routine practice, urine is cultured on cystine lactose electrolyte-deficient (CLED) medium. Cystine is a growth factor for some organisms; lactose is incorporated to differentiate utilization of lactose from non-utilization (e.g. Pseudomonas); and the electrolyte deficiency inhibits the swarming of Proteus, which may obscure bacterial colonies.

1. WHAT IS THE CAUSATIVE AGENT, HOW DOES IT ENTER THE BODY AND HOW DOES IT SPREAD A) WITHIN THE BODY AND B) FROM PERSON TO PERSON?

CAUSATIVE AGENT

Uropathogenic E. coli (UPEC) is the major cause of urinary tract infections (UTIs) in anatomically normal, unobstructed urinary tracts. Escherichia coli is a motile non-sporing gramnegative bacillus approximately 0.6–1μm wide and 2–3 μm long (Figure 11.2). It has a typical gram-negative cell wall with an outer hydrophobic membrane containing lipopolysaccharide (LPS). It has peritrichous flagella and possesses fi mbriae, which are important in adhesion (Figure 11.3). Eighty percent of all UTIs are caused by E. coli in the community and about 60% in hospital. The LPS is composed of lipid A, an inner core of polysaccharide linked to the lipid A by ketodeoxyoctonate (KDO) and an outer variable polysaccharide (Figure 11.4). The general arrangement of LPS is similar in most gram-negative bacteria, with the highest variability appearing in the outer “O” antigen component. The inner lipid A component comprises different fatty acids (such as hexanoic (6:0), dodecanoic (12:0),

9780367696399_Lydyard.indb 97

A 50-year-old worker at a school cafeteria went to her primary care physician complaining of tiredness, shaking chills, a pain in her loin, and a burning sensation on passing urine, which she was doing more frequently than normal. On examination, her doctor noted that she seemed a bit pale and that she had some suprapubic tenderness. He tested her urine with a dipstick and found she had a positive result for nitrite, pus cells, and protein. The doctor took a blood and urine sample for confirmation and sent them to the local hospital laboratory. A diagnosis of pyelonephritis was made and she was started on antibiotics. The following day, the laboratory results were available. The full blood count showed a neutrophilia and a pure culture of Escherichia coli was grown from the urine (Figure 11.1).

tetradecanoic (14:0), hexadecanoic (16:0), and octadecanoic (18:0), with their monoenoic equivalents) linked by ester amide bonds to KDO via N-acetylglucosamine. This part of LPS is the endotoxin – which is phosphorylated – and can have dramatic effects on the clotting and kallikrein cascades, complement activation, and cytokine production. The core region comprises 6- and 7-carbon sugars (glucose, galactose, heptose) linked to the outer variable region comprising 6-carbon sugars (e.g. glucose, mannose, tyvelose, rhamnose).

Figure 11.2 E. coli organisms. This is a Gram stain of E. coli showing the red (gram-negative) appearance. Courtesy of the Centers for Disease Control, Atlanta, Georgia. Image is found in the Public Health Image Library #3211.The photo was taken in 1979.

97

13/02/2023 07:32

Case Studies in Infectious Disease

isolates are generally very similar. Furthermore, there was no clustering according to site of isolation. Additionally, several genes were associated with bacteremia such as papA and G (adhesion); traJ (invasion); sat, tosA (toxins). A further genome study of enterotoxigenic E. coli (ETEC) was undertaken and revealed 21 global ETEC lineages and showed that colonization factors and toxins were commonly found in these isolates challenging the suggestion that such factors are usually found on plasmids. This study also showed that no specific lineage correlated with any specific disease. Other agents that cause UTI are other members of the Enterobacteriaceae such as Enterobacter and Klebsiella; Enterococci; Staphylococcus saprophyticus in young women; Proteus spp.; and Pseudomonas spp.

ENTRY INTO THE BODY Figure 11.3 The hair-like fimbriae on E. coli. This electronmicrograph shows the hair-like fimbriae on E. coli, which are important for adhesion. Fimbriae were first demonstrated by Prof. Duguid at the University of Dundee in the 1960s. Courtesy of the photographer Dennis Kunkel. Copyright Dennis Kunkel Microscopy, Inc.

The polysaccharide sequences of the variable region may be branched and repeated to give long chains projecting from the surface of the bacterium. Not all E. coli strains cause UTI and, in fact, E. coli (mainly of the K12 strain) is part of the normal flora of the intestine (i.e. a commensal) that usually does not cause any problems. However, other diseases caused by E. coli are: gastrointestinal (diarrheagenic E. coli with different pathogenic mechanisms e.g. VeroToxin E. coli O157.H7 Table 11.1); septicemia, pneumonia, meningitis (mainly neonatal cases with E. coli K1 capsule). Whole genome sequencing (WGS) of extra-intestinal pathogenic E. coli reveals that sequential UTI isolates are usually genetically different whereas sequential bloodstream

98

The GI tract in man is colonized by E. coli within hours to a few days after birth. The organism is ingested in food or water or obtained directly from other individuals handling the infant. Urinary tract infection with E. coli is acquired from the person’s own microflora (Endogenous) as opposed to being acquired from another person, animal or the environment (Exogenous). Enterotoxigenic E. coli are acquired from contaminated food.

SPREAD WITHIN THE BODY In females, if the E. coli organisms from the feces have the appropriate virulence factors (see below), they can attach to the epithelium of the vagina in the introitus, which then becomes colonized. The organisms can enter the urinary tract via the ureter and may eventually be introduced into the bladder where cystitis can develop. Once in the bladder, and in the presence of vesicourethral reflux, the organism may

Figure 11.4 The structure of E. coli lipopolysaccharide (LPS). This illustrates the structure of the LPS found in the outer membrane of gram-negative bacteria. It consists of a lipid A component consisting of β-hydroxy fatty acids attached to carbohydrate, a core carbohydrate component consisting of monomers of different carbohydrates, e.g. heptoses and ketodeoxyoctonic acid. This inner region of the LPS is virtually identical in all gram-negative bacteria, although the fatty acids may vary in different genera of bacteria. The outer part of the LPS consists of repeating units consisting of carbohydrates, e.g. mannose, glucose, galactose, rhamnose, etc., as illustrated by the different shading in the figure. This part is highly variable even within the same species of bacteria and is called the variable region or the “O” antigen. Variation between different species also exists in the degree of phosphorylation of LPS. The differences in the fatty acids and level of phosphorylation affect the endotoxin activity of the LPS.

9780367696399_Lydyard.indb 98

13/02/2023 07:32

Case 11: Escherichia coli

Table 11.1 The different pathovars of E. coli Strain

Designation

Disease

E. coli K12

K12

Commensal

Enteropathogenic E. coli

EPEC

Gastroenteritis

Enterotoxigenic E. coli

ETEC

Gastroenteritis

Enteroinvasive E. coli

EIEC

Gastroenteritis

Enterohemorrhagic E. coli

EHEC (O157)

Gastroenteritis, hemolytic uremic syndrome

Enteroaggregative E. coli

EAEC

Gastroenteritis

Diffusely adherent E. coli

DAEC

Gastroenteritis

E. coli K1

K1

Neonatal meningitis

Uropathogenic E. coli

UPEC

Pyelonephritis

spread up the ureter to the renal pelvis and pyelonephritis may ensue as it did in this patient. Some cases of pyelonephritis are hematogenous, the organism localizing in the kidneys from the blood, for example, Staphylococcus aureus, although this route is uncommon with E. coli, as the ascending route of infection predominates.

PERSON-TO-PERSON SPREAD Person-to-person spread is not relevant for E. coli causing UTI, although enteropathogenic E. coli that causes gastroenteritis can be transmitted by feco–oral spread.

EPIDEMIOLOGY OF UTI Because of the anatomic differences in the length of the urethra between the male and female, the female generally is far more likely to have a UTI compared with the male, except in neonates where it occurs more commonly in boys. In neonates, the prevalence is about 1–2%. In schoolchildren, the prevalence is 4.5% in girls and 0.5% in boys and where it occurs in the latter it frequently signifies renal tract congenital abnormalities. In adulthood, up to 40% of females will have a UTI at some time in their lives. Sexual intercourse and pregnancy are risk factors for developing a UTI. In men, the prevalence is less than 0.1%. A risk factor in males is an enlarged prostate and the prevalence of UTI in elderly men is up to 10%. In post-menopausal women, the lack of estrogen is a risk factor, as the microbial flora changes from one mainly of lactobacilli to one with gram-negative bacteria. In both males and females, a significant risk factor in hospitals is catheterization. In fact, it is known that catheter-associated infection is the most frequent hospital-acquired infection.

2. WHAT IS THE HOST RESPONSE TO THE INFECTION AND WHAT IS THE DISEASE PATHOGENESIS? Some virulence characteristics of E. coli are shown in Table 11.2 and are often carried on plasmids or on pathogenicity islands (PAIs). These PAIs are DNA sequences recognized as

having a GC content that differs from the average GC content of the organism and occur due to horizontal transfer from other organisms. UPEC have specific virulence characteristics that allow them to cause infection in the renal tract. These virulence characteristics include (a) specific fimbriae (e.g. P-fimbriae) that bind to the P-blood group antigen present on uroepithelial cells; (b) specific capsular antigens, for example, K1 associated with serum resistance; (c) the production of hemolysin; (d) cytotoxic necrotizing factor; and (e) secreted autotransported toxin (SAT) (f) siderophore receptors and the TonB iron uptake mechanism. The P-fimbriae are composed of the main fimbrial protein (PapA) and the adhesin (PapG). PapG adhesin has several allelic variants, with allele II variant binding to globoside and causing most cases of pyelonephritis and allele III variant binds Forssmann antigen found mainly in children and women causing cystitis or asymptomatic bacteriuria. The GI variant binds globotriaosylceramide. P-fimbriae binding is not inhibited by mannose, but UPEC can bind to mannose receptors on the epithelial cells of the urinary tract by type I fimbriae, which binds mannosyl conjugates, and uroplakins 1a, 1b and which are present in most E. coli strains and are composed of the Fim-A structural protein and the Fim-H adhesin. S-fimbriae bind to sialyllactose residues found on the renal tubules and glomeruli. Expression of Dr fimbriae, which bind to decay accelerating factor (a natural complement inhibitory protein found on epithelial cells) and collagen IV is associated with cell invasion and related to cystitis in children and pyelonephritis in pregnant women as well as gastroenteritis. Presence of this adhesin also re-attaches dislodged epithelial cells back to the epithelial surface by E. coli attached to the cell. Other adhesins are secreted components involved in adhesion and biofilm formation. Curli (a type of fimbriae) are secreted amyloid, which with phosphoethanolamine cellulose, also secreted by E. coli strains, provide firm adhesion present in high stress flow conditions. 1. These adhesins with different binding specificities have

also been identified and expression of certain of these

adhesins is thought to determine whether the E. coli moves

99

9780367696399_Lydyard.indb 99

13/02/2023 07:32

Case Studies in Infectious Disease

Table 11.2 Virulence characteristics of E. coli Virulence characteristic

Strain

Binds

Effect/disease

Type I fimbriae

Most strains

Mannose

Pyelonephritis/gastroenteritis

P-fimbriae

UPEC

Gal 1–4gal

Pyelonephritis

S-fimbriae

UPEC

Sialyl2 – 3lactose

Pyelonephritis

Dr-fimbriae

UPEC

DAF

Cystitis/pyelonephritis

Nonfimbrial adhesins

Most strains

Various ligands

Pyelonephritis/gastroenteritis

CFA I/II

ETEC

Gastroenteritis

AAF type III

EAggEC

Gastroenteritis

K-antigen K12

Commensals

Antiphagocytic and serum resistance

K1

UPEC

Pyelonephritis/neonatal meningitis

Adhesins

Toxins Hemolysin

Most



Cytotoxic/antiphagocytic

Siderophores

Most



Sequester iron

LT (heat-labile toxin)

ETEC



Fluid loss/traveler’s diarrhea

ST (heat-stable toxin)

ETEC



Fluid loss/traveler’s diarrhea

Verotoxin, EspP

ETEC



Dysentery/hemolytic uremic syndrome

SAT

UPEC



Cytotoxic/pyelonephritis

PET/PIC

EAggEC



Gastroenteritis

EspC

EPEC



Gastroenteritis

DAF, decay accelerating factor; CFA, colonization factor; SAT, secreted autotransported toxin; PET, plasmid­ encoded toxin; PIC, protein involved in intestinal colonization; Esp, E. coli secreted protein

2.

3.

4.

5.

100

further up the tract to infect the kidney. Individuals who are of the nonsecretor genotype (do not secrete blood group antigens into, e.g. saliva, urine, mucus) are more prone to binding of E. coli and thus to UTI. UPEC protect themselves from the host defenses with the K-capsule, which is antiphagocytic and also provides resistance to complement (serum resistance). UPEC also produces a number of toxins, which are also secreted by other E. coli pathovars. The hemolysin is a pore-forming toxin and is cytotoxic. Cytotoxic necrotizing factor affects intracellular signaling by modifying the Rho GTP-binding proteins, actin polymerization and is cytotoxic. UPEC also secrete an autotransporter toxin (SAT): a member of the SPATE (serine protease autotransporters of Enterobacteriaceae) group of toxins, which is also cytotoxic.

Binding of E. coli to the uroepithelial cells in the bladder induces widening of the junctions between the squamous epithelium, exposing the underlying basal cells to which the organism can readily bind, and infection also

9780367696399_Lydyard.indb 100

increases desquamation of the superficial uroepithelium. Adhesion of bacteria also induces an acute inflammatory response stimulating the synthesis of Th1 cytokines by the uroepithelium, interleukin (IL)-1 and IL-6 with IL-8, which recruits granulocytes and then macrophages and the patient develops a temperature. Production of these cytokines is probably related to the binding of the LPS to Toll-like receptors (especially TLR4) on the uroepithelium. In the renal medulla, the high ammonium concentration, osmolarity, and low pH all contribute to immune-paresis favoring the organism. The host secretes defensins from the uroepithelium and Tamm Horsfall protein, which binds E. coli, aiding its removal. Binding of the organism also results in exfoliation contributing to the E. coli that are found in the urine. The host develops a normal acquired immune response to the presence of the organism in the upper urinary tract, with the production of immunoglobulins of all isotypes. Infection of the bladder has a much-reduced antibody response. Cell-mediated immunity to infection appears to be of little importance in the urinary tract with the exception of cytokine secretion. T he high numbers of granulocytes that accumulate in some parts of the renal tract cause damage to the host in a bystander effect through release of oxygen free radicals and proteolytic

13/02/2023 07:32

Case 11: Escherichia coli

enzymes. The E. coli invade the uroepithelium forming intra­ cellular bacterial colonies and inducing necrosis of the cells leading to further inflammation. The role of the adaptive immune response in pyelonephri­ tis is poorly understood, although sIgA antibodies are pro­ duced and may inhibit the binding of the organism.

3. WHAT IS THE TYPICAL CLINICAL PRESENTATION AND WHAT COMPLICATIONS CAN OCCUR? The clinical presentation of UTI depends on the age of the person. In neonates, the symptoms are nonspecific with vomiting, fever, and a “floppy” infant. In older children and adults, localized symptoms occur. Cystitis presents with frequency, dysuria, suprapubic pain, and fever. The urine may contain blood. In cases of pyelonephritis, the patient will present with fever, rigors, loin pain, frequency, and dysuria and hematuria. Elderly patients may present with a typical picture or with fever, incontinence, dementia or signs suggestive of a chest infection. A “colicky” pain radiating from the loin to the groin is suggestive of renal stones – which may occur in the absence of infection, although renal calculi are a risk for infection and often associated with Proteus, which has an enzyme (urease) that can hydrolyze urea. Complications include renal scarring, septicemia, papillary necrosis which, if bilateral, can lead to renal failure, parenchymal abscess, and perinephric abscess. There is no clear relationship between pyelonephritis and the sequential development of chronic interstitial nephritis and hypertension.

4. HOW IS THE DISEASE DIAGNOSED, AND WHAT IS THE DIFFERENTIAL DIAGNOSIS? The organism can be isolated by culturing a mid-stream specimen of urine (see Figure 11.1) and, in 30% of cases, the organism will also be found in the blood culture. Work by Kass showed that if the number of organisms in the urine was greater than 105 bacteria per ml then this correlated well with clinical disease and this figure is considered as a “significant” bacteriuria. However, this study was in asymptomatic healthy women and it is recognized that UTIs can occur with fewer organisms in the urine. The presence of a single species of bacteria in the urine is also taken as evidence of significance but again UTIs can occur when more than one species is present. Other organisms can present with signs and symptoms of a UTI but not grow on the medium routinely used yet have

a high white cell count in the urine. This is called a sterile pyuria. Two organisms that present like this are Mycobacterium tuberculosis and Actinotignum spp. For the former, the urine should be grown on Lowenstein-Jennson medium and for the latter it should be grown on blood agar under micro-aerobic conditions. A number of automated methods for assessing bacteriuria are available, such as turbidometric, bioluminescence, electrical impedance, flow cytometry, and radiometric tests. Microscopy is generally not useful in the diagnosis of infection, although the presence of white cell casts is suggestive but not diagnostic of pyelonephritis. The differential diagnosis of pyelonephritis depends on the context and age of the patients. Dermatologic conditions such as shingles (before the appearance of the rash) and musculoskeletal injury may give rise to loin pain but without urinary symptoms or a fever in the case of musculoskeletal injury. Renal vein thrombosis can give rise to severe pain and fever; renal abscess, papillary necrosis, and urolithiasis will give rise to pain and fever. The presence of a “sterile” pyuria can indicate renal tuberculosis, urolithiasis or neoplasm.

5. HOW IS THE DISEASE MANAGED AND PREVENTED? There is no strong reason for treatment of asymptomatic bacteriuria in either non-pregnant women or the elderly. On the other hand, in pregnant women and children, particularly in the latter if there are renal congenital abnormalities allowing vesico-ureteric reflux, then treatment is required even in the absence of symptoms. If not treated, renal scarring or even renal failure may occur. The choice of antimicrobial agent to treat UTI is often empirical but is usually based on the recognition that the commonest infectious agent is E. coli and a knowledge of the local antibiotic resistance patterns. Cystitis can be treated with 3 days of appropriate antibiotics. Trimethoprim, nitrofurantoin or amoxycillin are often used. In cases of pyelonephritis, as there is parenchymal disease, a 2-week course of antibiotics is required, for example, cefuroxime or a third-generation cephalosporin, an aminoglycoside or a fluoroquinolone depending on the resistance pattern. Because of the spread of extended-spectrum β-lactamases (ESBLs), particularly in bacteria such as Enterobacter, no β-lactam antibiotic is effective (except the carbapenems) and a different antibiotic class should be used, for example, a fluoroquinolone. Frequent relapses of UTIs may require prophylaxis, usually with nitrofurantoin or trimethoprim taken at night, so high concentrations of the antibiotic are present in the bladder overnight.

101

9780367696399_Lydyard.indb 101

13/02/2023 07:32

Case Studies in Infectious Disease

SUMMARY 1. WHAT IS THE CAUSATIVE AGENT, HOW DOES IT ENTER THE BODY, AND HOW DOES IT SPREAD A) WITHIN THE BODY AND B) FROM PERSON TO PERSON? ■ The commonest infecting agent for urinary tract infections is E. coli. ■ Other agents causing UTI are enterococci, Proteus, Staph. saprophyticus, Pseudomonas, and other enteric bacteria. ■ E. coli has a typical gram-negative cell wall structure with an outer lipopolysaccharide (LPS) membrane. ■ LPS consists of an inner lipid A component, a common core region of polysaccharides, and an outer variable region of polysaccharides. ■ The infection is endogenous, the source of the bacteria being

3. WHAT IS THE TYPICAL CLINICAL PRESENTATION AND WHAT COMPLICATIONS CAN OCCUR? ■ Infection may be asymptomatic. ■ In pregnant women and children, asymptomatic infection can have serious consequences. ■ Cystitis presents with frequency of micturition, dysuria, and suprapubic pain. ■ Pyelonephritis presents with loin pain, fever, rigors, frequency of micturition, and dysuria. ■ Neonates have a nonspecific presentation with fever, vomiting, and failure to thrive. ■ Elderly patients may have fever, incontinence, and dementia. ■ Complications are septicemia, renal scarring, renal abscess, and renal failure.

the fecal flora. ■ Organisms gain entry to the bladder from the perineum and in the presence of vesico-ureteric reflux may infect the renal parenchyma. ■ UTI is more common in females because of the short urethra.

2. WHAT IS THE HOST RESPONSE TO THE INFECTION AND WHAT IS THE DISEASE PATHOGENESIS? ■ UPEC have several adhesins that bind to uroepithelial cells. ■ UPEC produce a number of toxins that increase desquamation of uroepithelial cells and are also cytotoxic. ■ UPEC are protected from the innate host defense system by the K1 capsular antigen, which is antiphagocytic. ■ In the renal tract, the immune defenses are relatively inactive due to the osmolarity and the concentration of ammonium ions. ■ Binding of UPEC to uroepithelial cells stimulates the production of IL-8 and the recruitment of granulocytes to the renal tract. ■ The role of the adaptive immune system in protection is uncertain although IgA and IgG are produced.

FURTHER READING Murphy K, Weaver C. Janeway’s Immunobiology, 9th edition. Garland Science, New York/London, 2016. Rogers MC, Peterson ND. E. coli Infections: Causes, Treatment & Prevention. Nova Science, 2011. Samie, A. Escherichia coli – Recent Advances on Physiology, Pathogenesis and Biotechnological Applications. In Tech Open, 2017. Zaslau S. Bluprints Urology. Blackwell Publishing, Oxford, 2004.

REFERENCES

102

Barnhart MM, Chapman MR. Curli Biogenesis and Function. Annu Rev Microbiol, 60: 131–147, 2006.

9780367696399_Lydyard.indb 102

4. HOW IS THE DISEASE DIAGNOSED, AND WHAT IS THE DIFFERENTIAL DIAGNOSIS? ■ Microscopy is generally unhelpful although a sterile pyuria may indicate renal tuberculosis. ■ Laboratory diagnosis is by culture. ■ A significant bacteriuria is a single species in numbers greater than 105 organisms per ml of urine. ■ Differential diagnosis includes musculoskeletal injury, renal vein thrombosis, urolithiasis.

5. HOW IS THE DISEASE MANAGED AND PREVENTED? ■ Asymptomatic bacteriuria in pregnant women or children should be treated. ■ Antibiotic use depends upon the local resistance pattern. ■ Three days treatment with an appropriate antibiotic is usually sufficient for uncomplicated cystitis. ■ Two weeks of treatment with the same antibiotic is required for pyelonephritis.

Browne P, Ki M, Foxman B. Acute Pyelonephritis Among Adults: Cost of Illness and Considerations for the Economic Evaluation of Therapy. Pharmacoeconomics, 23: 1123–1142, 2005. Katchman EA, Milo G, Paul M, et al. Three Day Versus Longer Duration of Antibiotic Treatment for Cystitis in Women: Systematic Review and Metaanalysis. Am J Med, 118: 1196–1207, 2005. Kau AL, Hunstad DA, Hultgren SJ. Interaction of Uropathogenic Escherichia coli with Host Uroepithelium. Curr Opin Microbiol, 8: 54–59, 2005. Larcombe J. Urinary Tract Infection in Children. Clin Evid, 14: 429–440, 2005. MacDonald RA, Levitin H, Mallory GK, Kass EH. Relation Between Pyelonephritis and Bacterial Counts in the Urine. N Engl J Med, 256: 915–922, 1957.

13/02/2023 07:32

Case 11: Escherichia coli

Neumann I, Fernanda Rojas M, Moore P. Pyelonephritis in Non-Pregnant Women. Clin Evid, 14: 2352–2357, 2005. Sheffield JS, Cunningham FG. Urinary Tract Infection in Women. Obstet Gynecol, 106: 1085–1092, 2005.

Medscape, Chronic Pyelonephritis, 2019: http://www. emedicine.com/med/topic2841.htm Todar’s Online Textbook of Bacteriology, Pathogenic E. coli (page 1), © Kenneth Todar, 2008: http://textbookofbacteriol­ ogy.net/e.coli.html

WEBSITES American Urological Association: http://www.auanet.org/ Centers for Disease Control and Prevention, E. coli (Escherichia coli): https://www.cdc.gov/ecoli/index.html EcoCyc, E. coli Database: https://ecocyc.org European Association of Urology: http://www.uroweb.org/

Students can test their knowledge of this case study by visiting the Instructor and Student Resources: [www. routledge.com/cw/lydyard] where several multiple choice questions can be found.

103

9780367696399_Lydyard.indb 103

13/02/2023 07:32

12

Giardia intestinalis

A 24-year-old man went on a 3-month backpacking trip across India. He drank bottled water and reportedly ate well-cooked food in hotels and restaurants. While in India, his stools were looser than normal. In the week before his return he developed frequent watery, non-bloody diarrhea. This settled enough for him to fly home. He immediately went to his doctor and a stool culture grew Campylobacter. His bowels improved over 10 days without treatment but 2 weeks after his return he developed more

1. WHAT IS THE CAUSATIVE AGENT, HOW DOES IT ENTER THE BODY, AND HOW DOES IT SPREAD A) WITHIN THE BODY AND B) FROM PERSON TO PERSON? CAUSATIVE AGENT

Giardia is a protozoan consisting of six species that infect mammals, amphibians, and birds. The only species that infects humans is Giarda intestinalis (alt. lamblia and duodenalis). It has two stages to its life cycle (Figure 12.1): (a) a trophozoite (feeding and pathology-causing stage) that is flagellated (with four pairs of flagellae), pear-shaped, with two nuclei, a ventral “sucking” disk, and median bodies. It also has a rigid cytoskeleton composed of microtubules and microribbons. It measures 9–21μm long by 5–15 μm wide; (b) a cyst, with a highly resistant wall that enables it to remain viable outside the body of the host for long periods. The cyst is smoothwalled and oval in shape, measuring 8–12 μm long by 7–10 μm wide. The genome of the parasite has five chromosomes of different sizes and variation of the number of genome equivalents (ploidy) occurs during the life cycle: trophozoite >cyst>excyzoite>trophozoite. The genome ploidy is important in gene regulation and differentiation. Giardia is the oldest known extant eukaryote, having prokaryotic characteristics: no mitochondria and metabolism similar to prokaryotes. G. intestinalis is a multispecies complex with at least eight recognized assemblages or genotypes (A–H) based on a number of clinical samples. The two genotypes A and B are the major human pathogens. Molecular analysis of 2800 samples indicates that genotype B accounts for around 58%

diarrhea, with loss of appetite, bloating, and flatulence. For the first time, his stools failed to flush away completely in the toilet and were particularly offensive in smell. He began to lose weight. His doctor requested three stool specimens for culture and also microscopy for ova, cysts, and parasites. One out of three specimens contained Giardia cysts. He was treated with a course of metronidazole and his symptoms improved.

giardiasis cases with genotype A found in around 37% of cases worldwide. There are, however, differences in percentages of these in different countries. For example in Spain, there were 27.4% of A and 72% of B recorded in Madrid. A and B genotypes were equally represented in Rioja. In Spain, children also appear to be more commonly infected with the B phenotype than adults.

ENTRY INTO THE BODY Cysts are ingested from contaminated water or food and having passed through the stomach, begin to open up at one end (excystation), releasing trophozoites into the intestine lumen, which then divide within 12 hours (Figure 12.1). They settle in the small intestine (predominantly in the midjejunum). A minimum of 10–25 cysts are necessary to produce an infection. The trophozoites attach to the intestinal wall through their ventral “sucking” disk and feed on nutrients (Figure 12.2). They increase in number by binary fission and colonize large areas of epithelial surface causing diarrhea and damage to the epithelium (see Section 2). At regular intervals and following detachment and movement down toward the colon (and probably exposure to biliary secretions), some of the trophozoites become encysted (encystation), with each trophozoite forming a single cyst. Both trophozoites and cysts pass out of the body in the feces.

SPREAD WITHIN THE BODY Penetration of the epithelial surface by the trophozoites is very rare, as is migration of the trophozoites to systemic sites. Invasion of the gallbladder, pancreas, and urinary tract have been reported but the trophozoites normally remain in the intestine/colon and do not cross the mucosal barrier.

105

9780367696399_Lydyard.indb 105

13/02/2023 07:32

Case Studies in Infectious Disease

Figure 12.1 The life cycle of G. intestinalis. Both cysts and trophozoites are found in feces (1). The cysts are hardy and can survive 2–3 months or more in cold water. Cysts in contaminated water, food, or by the fecal-oral route (hands or fomites) cause infection (2). In the small intestine, the cysts give rise to trophozoites (each cyst producing two trophozoites) (3). The trophozoites multiply by binary fission and remain in the lumen of the small bowel where they can be free in the mucus or attached to the epithelial cells by their ventral sucking disk (4). Trophozoites encyst on transit toward the colon. The cyst is the stage found most commonly in non-diarrheal feces (5). The cysts are infectious when passed in the stool or shortly afterward and, if ingested by another person, the cycle begins again. Courtesy of the Centers for Disease Control, Atlanta, Georgia. Image is found in the Public Health Image Library #3394. Additional photo credit is given to Alexander J. da Silva, PhD, and Melanie Moser who created the image in 2002.

PERSON-TO-PERSON SPREAD

EPIDEMIOLOGY

Cysts can remain dormant for up to 3 months in cold water. Spread is through ingestion of contaminated food and also via the fecal–oral route (hands and fomites), although water is probably the main source. Contamination of public-drinking supplies has led to giardiasis epidemics. When children become infected, up to 25% of their family members also become infected. Individuals can shed cysts in their feces and remain symptom-free but are an important source of personto-person transmission (see Section 3). Sexual transmission of Giardia has been described in men who have sex with men. Giardia is found in a wide variety of different animal species and has been regarded as a zoonosis, although there is little evidence for animals being a significant source of human giardiasis.

Giardiasis is one of the most common water-borne diseases that infect humans and the most common enteric protozoal infection worldwide, with 280 million cases a year and up to 33% of individuals infected. Prevalence rates vary from 4% to 42%. It affects nearly 2% of adults and 8% of children in developed countries. Infection is linked to poor hygiene and sanitation and is more prevalent in warm climates. The prevalence within the US is estimated to be roughly 1.2 million, with the majority of cases not identified due to the carrier being asymptomatic. In 2012, the CDC reported 15223 cases. The most affected are children 0 to 4 years of age, with the largest percentage of cases being in the northwest US. Infection is most common in late summer and early fall due to outdoor water activities.

106

9780367696399_Lydyard.indb 106

13/02/2023 07:32

Case 12: Giardia intestinalis

HOST DEFENSES

Non-Immunologic Defenses Intestinal epithelial cells are shed and replaced every 3–5 days and therefore the trophozoites need to constantly detach and reattach to new epithelial surfaces. The mucus produced by the goblet cells impedes access of the trophozoites to the epithelium. It has been proved that certain commensal bacterial species enable mice to be resistant against Giardia colonization suggesting that differences in microbiotic composition between individuals and species could explain the variability in pathology and susceptibility to infection (see later).

IMMUNE RESPONSES

Figure 12.2 G. intestinalis attached to microvilli in the small intestine. This colored TEM (transmission electron micrograph) shows a G. intestinalis trophozoite attached by means of its ventral sucking disks to microvilli in the human small intestine. From CNRI / Science Photo Library, with permission.

Water-borne outbreaks appear to be the most common source of infection. In Canada, in 2016, there were 3818 reported cases of giardiasis with a similar seasonal variation to that seen in the US. In the Western world, giardiasis is more likely to be diagnosed as a cause of diarrhea that occurs or persists after travel to a developing country. This is due to its relatively long incubation period and persistent symptoms. Thus, the organism is a cause of “traveler’s diarrhea”, also called “backpacker’s diarrhea” and “beavers’ fever” (since it was originally believed to be transmitted from beavers to man). It is not unusual for outbreaks of Giardia to occur on cruise ships.

2. WHAT IS THE HOST RESPONSE TO THE INFECTION AND WHAT IS THE DISEASE PATHOGENESIS? In a study of prison “volunteers” in the 1950s given the same infectious dose, 50% of subjects developed asymptomatic infections, 35% self-limiting symptomatic infections, and 15% troublesome persistent diarrhea. Given that they received the same dose and strain this illustrates the impact of host susceptibility and resistance. However, host defenses against G. intestinalis are not well characterized but they are believed to involve both non-immunologic mucosal processes and immune mechanisms.

There appears to be little or no mucosal inflammation in human Giardia infection, which indicates that local defense must be occurring without systemic recruitment. Most of the data on immune responses to Giardia come from experimental animal models.

INNATE IMMUNITY Antimicrobial peptides such as defensins and cathelicidins secreted by intestinal epithelial cells have anti-giardial activity in vitro and may have activity in vivo. Nitric oxide (NO) produced by gut epithelial cells inhibits growth, encystation, and excystation in vitro, but has no effect on viability. However, Giardia has developed strategies to evade this host defense mechanism. Trophozoites down-regulate expression of iNOS in intestinal epithelial cells. This probably leads to the reduced expression of iNOS reported in pediatric patients infected with Giardia. Although monocytes/macrophages and polymorphs can kill trophozoites in vitro by oxidative mechanisms, very few are found in the human intestinal lumen during infection. Mast cells are currently recognized as effector cells of the immune response against several parasites. Interestingly, in tissues from the small intestine that have been infected with G. intestinalis, the most strongly induced transcripts are mast-cell proteases. Mast cells and NO could act together to induce peristalsis. The maturation and activation of dendritic cells (DCs) can be induced through Giardia lysates, excretory secretory products, and other specific proteins. This is seen as an increase in pro-inflammatory cytokines, for example IL-6, TNF and IL-12 and of surface molecules such as CD80, CD86, and MHC class II. Giardia may activate the maturation and migration of DCs to the site of infection with their subsequent release of immunomodulatory cytokines. The production of IL6 by DCs seems to be of major importance in the clearance of Giardia.

107

9780367696399_Lydyard.indb 107

13/02/2023 07:32

Case Studies in Infectious Disease

ADAPTIVE IMMUNITY

T-Cell Responses IL-6 produced by DC is important for T-cell differentiation and may determine the type of T-cell response that ensues. In both humans and animals, Giardia infection induces a strong protective adaptive immune response, in which the main player is CD4+ T lymphocytes. This includes Th-1, Th-2, and Th-17 cells. Recent findings in a murine model of Giardia has shown local elevations in the ratio between Th-17 and Treg lymphocytes within the small intestinal lamina propria and Peyer’s patches related to their increased resistance to infection. Interestingly, duodenal mucosal lymphocyte alterations are maintained for many months after the onset of infection in human patients. Th-17 cells producing IL-17 have been found in the blood of humans infected with Giardia. The evidence is clear that CD4+ T cells are important protection against Giardia infection and specific depletion of CD4+ T cells results in the development of chronic giardiasis. CD8+ T cells appear to play no role in host protection against Giardia.

Antibody Responses Although Giardia trophozoites do not usually cross the mucosal barrier, some antigenic Giardia products antigens must penetrate the barrier. This might be through the M cells of Peyer’s patches or the DCs with intraepithelial processes into the gut lumen. Antibodies to Giardia are found in both mucosal secretions and serum and it has been established that antibodies, particularly of the IgA isotype, contribute to the maintenance of protective immunity against giardiasis. Anti-Giardia secretory IgA can be detected in human saliva and breast milk. Antibodies of up to 16 immunogenic proteins in infected patients have been identified by serum IgG. These include variant-specific surface proteins (VSP: see below), cytoskeletal proteins unique to Giardia ( - and β-tubulin, - and β-giardin), and enzymes (e.g. arginine deaminase, orthinine carbamoyl transferase, and enolase) are found (see below). These enzymes have been found to be induced following contact of the parasite with the intestinal epithelial cells. IgG antibodies to Giardia have been shown to kill Giardia trophozoites in vitro through complement. Although it is unlikely that this mechanism could occur in the intestinal lumen, it might be one explanation as to why Giardia does not invade.

GIARDIA-SPECIFIC ANTIGENS AND ANTIGENIC VARIATION

108

Each trophozoite expresses only one of many highly immunogenic variant-specific surface proteins (VSP) on Giardia. The trophozoites being able to switch the VSPs in their surface coats. The mechanism of this variation is believed to be through interference RNA (iRNA) and mRNA. Disruption of this pathway generates trophozoites that simultaneously express numerous VSP. It is likely that the variation is driven by antibody. This antigenic variation (at least in mice) is

9780367696399_Lydyard.indb 108

thought to be a mechanism whereby the trophozoites can avoid the immune system. An alternative, but not mutually exclusive, biological explanation for antigenic variation is their adaptation to different intestinal environments. There is some evidence for this possibility in that different VSPs have different susceptibility to proteases. Interestingly, the repertoire of VSP antigens is much smaller than that seen in trypanosomes (see Case 37) and the mechanisms leading to their “switching” are different.

PATHOGENESIS The mechanisms by which giardiasis causes diarrhea and malabsorption are incompletely understood. Several have been postulated and include damage to the endothelial brush border, enterotoxins, immunologic reactions, changes in gut motility, and fluid hypersecretion via increased adenylate cyclase activity. Direct attachment of trophozoites to the epithelium has been demonstrated to cause increased epithelial permeability. Giardia-induced loss of intestinal brush border surface area, villus flattening, inhibition of disaccharidase activities, and eventual overgrowth of enteric bacterial flora appear to be involved in the pathophysiology of giardiasis but have yet to be causatively linked to the disease’s clinical manifestations. There is little evidence of any exotoxins producing epithelial damage. Regarding the damage to the brush border, which provides a large surface area for absorption, biopsies from only 3% of patients with infection showed villus shortening and there was little inflammation. In experimental infection of 10 human volunteers with Giardia type B genotype, only five individuals developed symptoms and only two of these showed any change to the brush border. Thus, microvillus shortening and inflammation are not directly correlated with the symptoms and indeed clearance of the organism from the intestinal tract. From in vitro studies, there is some evidence for Giardia inducing a change in the cytoskeleton of human duodenal cells with increased apoptosis and disruption of tight junctions in monolayers of intestinal cells. Cysteine proteases secreted by G. intestinalis may disrupt intestinal epithelial cell junctional complexes and degrade chemokines. Although disruption of tight junctions has not been confirmed by clinical observation, there is evidence for a correlation of infection with impairment of both absorption and digestive functions. In fact, there are varying degrees of malabsorption of sugars (e.g. xylose, disaccharides), fats, and fat-soluble vitamins (e.g. vitamins A and E) but these might contribute to substantial weight loss. Although CD8+ T cells do not contribute to protection against Giardia, there is some suggestion that they are involved in enterocytic damage. Giardia infections can produce symptoms that persist long after infection although, again, the mechanisms for this are unclear. Recent research has emphasized the importance of the intestinal microbiota in Giardia pathogenicity. Conventional, germ-free, or germ-free mice that were reconstituted with duodenal microbes from patients with symptomatic giardiasis,

13/02/2023 07:32

Case 12: Giardia intestinalis

were infected with G. lamblia trophozoites. The infected non-reconstituted conventional mice showed the intestinal pathology among the three groups, with the reconstituted germ-free mice showing moderate pathology. The infected germ-free mice, however, did not develop intestinal pathology compared with the other groups, suggesting a requirement for intestinal microbes to stimulate pathology. Colonization by Giardia in mice causes a dysbiosis with an increase of the Proteobacteria and a decrease in the Firmicutes. Segmented filamentous bacteria (SFB), found in many animals including humans, are present in the small intestine near the terminal ileum and are responsible for stimulating IL-17 production. Resistance to Giardia in mice occurs with a high intestinal density of SFBs and susceptibility with a low density. In humans, SFBs are scarce in neonates, who are prone to infection, and are more common in adults. A reduction in Clostridia has also been noted in infected animals and the presence of the organism is linked to effective Treg responses.

3. WHAT IS THE TYPICAL CLINICAL PRESENTATION AND WHAT COMPLICATIONS CAN OCCUR?

across the gut epithelium causes deficiency in lipid-soluble vitamins, which is an additional problem for children. Poor nutrition can also contribute to an increased risk of a person having symptoms with the infection. More serious infections, which can lead to death, are seen in people with a weakened immune system, such as patients with HIV/AIDS, cancer, transplant patients, and the elderly. Helicobacter pylori may predispose to Giardia due to hypochlorhydria. Patients may develop lactose intolerance. Giardia infection has also been associated with development of post-infectious complications including irritable bowel syndrome and chronic fatigue syndrome. Changes in the intestinal microbiota profile in children with Giardia might contribute to some of these complications.

4. HOW IS THE DISEASE DIAGNOSED, AND WHAT IS THE DIFFERENTIAL DIAGNOSIS? Clinical diagnosis is often difficult because the same symptoms can occur with a number of intestinal parasites. Giardiasis is therefore diagnosed by the identification of cysts (Figure 12.3) or trophozoites (Figure 12.4) in the feces, and this is still regarded as the gold standard method. Due

Most persons infected with G. intestinalis, on a global basis, are either asymptomatic or minimally symptomatic. The clinical effects of Giardia infection range from asymptomatic carrier status to severe malabsorption (see Section 2). Factors contributing to the variations in presentation include the virulence of particular Giardia strains (see Section 2), their genotype (A or B), the numbers of cysts ingested, the age of the host, and the state of the immune system (see later). Asymptomatic carriers often have a large number of cysts in their stools. If symptoms are present, they occur about 1–3 weeks after ingestion of the parasite. These include: • watery offensive-smelling diarrhea with abdominal cramps; • severe flatu lence; • nausea with or without vomiting; • fatigue; • and possibly fever. A slower onset may occur with development of yellowish loose, soft and foul-smelling stools – often floating due to the high lipid content. Stools may be watery or even constipation can occur. Initial symptoms usually last 3–4 days or can become chronic leading to recurrent symptoms, severe malabsorption and debilitation may occur. Other symptoms include anorexia, malaise, and weight loss. Children with malabsorption associated with Giardia infection often show failure to thrive and protein-losing enteropathy can be a complication leading to stunted growth of children, commonly seen in Africa. Reduced uptake of lipids

Figure 12.3 G. intestinalis cyst in a wet mount stained with iodine. Courtesy of the Centers for Disease Control, Atlanta, Georgia. Image is found in the Public Health Image Library #3741. Additional photo credit is given to Dr Mae Melvin who took the photo in 1977.

109

9780367696399_Lydyard.indb 109

13/02/2023 07:32

Case Studies in Infectious Disease

A “string” test (entero-test) can also be performed. This involves swallowing a weighted gelatine capsule on a piece of string. After the gelatine dissolves in the stomach, the weight carries the string into the duodenum. The string is left for 4–6 hours or overnight while the patient is fasting and then examined for bilious staining. This indicates successful passage into the duodenum and mucus from the string can be examined for trophozoites after fixation and staining. The polymerase chain reaction (PCR) detection of Giardia DNA is often restricted to laboratory use and mostly for subtyping of G. intestinalis (see Section 1). A duodenal biopsy can be taken and this may be the most sensitive test. This is often taken in cases of unexplained diarrhea.

DIFFERENTIAL DIAGNOSIS

Figure 12.4 G. lamblia trophozoite stained with trichrome. Courtesy of the Centers for Disease Control, Atlanta, Georgia. Image is found in the Public Health Image Library #7833. Additional photo credit is given to DPDX / Melanie Moser who created the original image.

to the intermittent and low levels of cysts, the fecal samples are usually concentrated before direct examination of wet mounts under the microscope. A number of methods have been used including formalin-ether and sucrose gradients for concentration. Samples can be stained with iodine, methylene blue or trichrome. Alternate methods for detection of the parasite in the stool samples include antigen detection tests by ELISA and by a direct fluorescence assay (DFA) (Figure 12.5). Commercial kits for both of these are available.

Other causes of gastroenteritis need to be considered including amebiasis, bacterial overgrowth syndromes, Crohn ileitis, Cryptosporidium enteritis, irritable bowel syndrome, celiac sprue, tropical sprue, strongyloidiasis, viral gastroenteritis, and lactose intolerance.

5. HOW IS THE DISEASE MANAGED AND PREVENTED? MANAGEMENT

With mild infection, giardiasis can resolve in 6 weeks or so. However, there are several drugs used in the treatment of giardiasis. They include: 1. Nitroimidazole derivatives (metronidazole (Flagyl®),

tinidazole, secnidazole, ornidazole,): most frequently used first-line drugs, especially metronidazole. 2. Benzimidazoles (albendazole and mebendazole): these

widely used anti-helminthic drugs are used to treat giardiasis but with variable efficacy. 3. Nitrofuran derivatives (furazolidone): these have been

reported to have high efficacy in first-line therapy. 4. Acridine compounds (mepacrine and quinacrine):

mechanism of action unknown but has been shown to be efficient in therapy. 5. Aminoglycoside: the oral aminoglycoside paromomycin

is the drug of choice for pregnant women as it is poorly absorbed and has no systemic effects. 6. Nitazoxanide: in controlled studies efficacies of between

110

Figure 12.5 Identification of cysts of G. intestinalis by fluorescentlabeled Giardia antibodies. A formalin-fixed preparation stained with commercially available fluorescent antibodies to Giardia and visualized under a fluorescence microscope. Cysts of Giardia are seen as large green ovoid objects (labeled C). Oocysts of Corynebacterium parvum are also seen in this preparation. Courtesy of the Centers for Disease Control, Atlanta, Georgia. Image is found in the Laboratory Identification of Parasites of Public Health Concern.

9780367696399_Lydyard.indb 110

44% and 91% have been reported. Different countries may have a preference for the use of different types of drugs. In addition, from a worldwide perspective, albendazole (a benzimidazole compound) is used, which has a much broader range of action than metronidazole and the other agents listed. It kills Giardia very well, but also

13/02/2023 07:32

Case 12: Giardia intestinalis

Entamoeba, Ascaris, Enterobius, and hookworms, and can do this in one single dose. In developing countries, a single-dose albendazole is being given to schoolchildren and has been associated with improved school attendance and educational attainment. They feel better for being cleared of protozoa and helminths.

the first trimester. Mildly symptomatic women should have their treatment delayed until after delivery. If left untreated, however, adequate nutrition and hydration maintenance is important. Paromomycin is now the only anti-Giardia drug considered completely safe during early pregnancy (see different drugs used to treat Giardia above).

TREATMENT OF REFRACTORY GIARDIASIS

PREVENTION

In general, efficacy of treatment with nitroimidazoles is 90%. However, nitroimidazole failure (especially with metronidazole) has been reported in up to 50% giardiasis cases in both travelers and in high endemic countries. A number of studies have indicated the use of a different class of drug, combination therapy or repeated courses with increased dose/ duration of the same drug can be used with variable efficacy.

• Good hygiene is very important. • Contaminated water should be avoided: untreated water should not be consumed. Outbreaks of giardiasis in developed countries are often traced back to breakdown in filtration systems of drinking water supplies. • Individuals traveling to warm climates where Giardia is found should take extra care with drinking water and consumption of raw food – boil drinking water, and so forth. • There is no known chemoprophylaxis for humans. • There is no vaccine for humans yet although there is an effective killed vaccine for dogs (Giardiavax®).

Pregnant Patients Treatment of pregnant patients with Giardia is difficult because of the potential adverse effects of anti-Giardia agents on the fetus. If possible, drug treatment should be avoided during

SUMMARY 1. WHAT IS THE CAUSATIVE AGENT, HOW DOES IT ENTER THE BODY, AND HOW DOES IT SPREAD A) WITHIN THE BODY AND B) FROM PERSON TO PERSON? ■ Giardia is a protozoan flagellate. It has two stages – a trophozoite and a cyst with a highly resistant wall. ■ Giardia consists of six species. Only one species infects humans and this is variously referred to as G. intestinalis, lamblia or duodenalis. There are at least eight genotypes or assemblages (A–H). Only A and B are human pathogens, with B being the most

commonly found in children where, in developing countries, it is estimated that up to 20% are infected. ■ Giardia is widespread in the US, with a prevalence of an estimated 1.2 million with the majority not identified due to the carrier being asymptomatic. Infection is most common in late summer and early fall due to outdoor water activities. Similar seasonal variation is seen in Canada. ■ In the Western world, giardiasis is a cause of diarrhea that occurs or persists after travel to a developing country – “traveler’s diarrhea”, “backpacker’s diarrhea”, and “beavers’ fever”.

frequent globally. Other genotypes are seen in other mammals and birds. ■ The main infectious stage is the cyst; cysts are ingested in contaminated water or food. They lose their cell wall in the duodenum and emerge as trophozoites, which attach to the intestinal wall through their ventral “sucking” disk and feed. They colonize large areas of epithelial surface. They rarely invade the epithelium and spread systemically. They become encysted again and both trophozoites and cysts pass out of the body in stools. ■ Contamination of public drinking supplies has led to giardiasis epidemics. When children become infected, up to 25% of their family members also become infected. Individuals can shed cysts in their feces and remain symptom-free. Sexual transmission of Giardia has been described in homosexual males. ■ Giardia is found in a wide variety of different animal species and

2. WHAT IS THE HOST RESPONSE TO THE INFECTION AND WHAT IS THE DISEASE PATHOGENESIS? ■ The host defenses are clearly effective since individuals infected with Giardia are often asymptomatic and some are able to clear the organism without treatment. ■ Host defenses are not well characterized but include both nonimmunologic and immunologic mechanisms. ■ Mucus prevents immediate access of trophozoites. The intestinal microbiota may also play a role in preventing attachment/ inhibiting proliferation. ■ Antimicrobial peptides such as defensins and cathelicidins secreted by intestinal epithelial cells have anti-giardial activity in vitro and may have activity in vivo. Monocytes/macrophages and

has been regarded as a zoonosis, although there is little evidence

polymorphs can kill trophozoites in vitro by oxidative mechanisms

for animals being a significant source of human giardiasis.

but very few are found in the human intestinal lumen during

■ Giardiasis is one of the most common causes of diarrhea worldwide. There are around 280 million cases a year. It is more

infection IL-6 produced by dendritic cells appears to be of major importance in the clearance of Giardia. Continued...

111

9780367696399_Lydyard.indb 111

13/02/2023 07:32

Case Studies in Infectious Disease

Continued...

■ Most information on immune responses to Giardia infections comes from animal models. CD4+ T cells including Th-1, Th-2, and Th-17 appear to play a role in Giardia infections in mice but their exact protective role in humans is unclear. CD8+ T cells do not play a role. ■ Antibody responses to human Giardia do have some protective role. Specific IgA antibodies in human saliva and breast milk can protect children against infection in early life. Immunodominant

■ Patients with a weakened immune system such as patients with HIV/AIDS, cancer, transplant patients or the elderly can develop more severe infections. ■ Infection has been associated with development of postinfectious complications including irritable bowel syndrome and chronic fatigue syndrome.

4. HOW IS THE DISEASE DIAGNOSED, AND WHAT

antigens include VSPs, cytoskeletal structures, giardin, and

IS THE DIFFERENTIAL DIAGNOSIS?

enzymes. Serum antibodies of IgG class to 16 immunogenic

■ Giardiasis is diagnosed by the identification of cysts or trophozoites in the feces which is still the “gold standard”. Direct mounts for microscopy as well as concentration procedures may be used. Samples can be stained with iodine or trichrome. ■ Commercial kits are available for antigen detection tests by ELISA and by immunofluorescence. PCR testing for Giardia DNA can also be used but is usually restricted to laboratory use. ■ A “string” test (entero-test) can also be performed. ■ Differential diagnosis for other causes of gastroenteritis includes amebiasis, bacterial overgrowth syndromes, Crohn ileitis, Cryptosporidium enteritis, irritable bowel syndrome, celiac sprue, and tropical sprue.

proteins of Giardia have been seen in humans, ■ Giardia shows antigenic variation with each trophozoite expressing one VSP expressed and switching of these probably providing escape from the immune system. ■ The mechanisms by which giardiasis causes diarrhea and malabsorption are unclear but have been postulated that include damage to the endothelial brush border, enterotoxins, immunologic reactions, changes in gut motility, and fluid hypersecretion

via

increased

adenylate

cyclase

activity.

Malabsorption of sugars (e.g. xylose, disaccharides), fats, and fat-soluble vitamins (e.g. vitamins A and E) might contribute to substantial weight loss.

3. WHAT IS THE TYPICAL CLINICAL PRESENTATION AND WHAT COMPLICATIONS CAN OCCUR? ■ The clinical effects of Giardia infection range from asymptomatic carrier status to severe malabsorption. ■ Factors contributing to the variations in presentation include the virulence of particular Giardia strains, their genotype (A or B), the numbers of cysts ingested, the age of the host, and the state of the immune system. Carriers often have a large number of cysts in their stools. ■ If symptoms are present, they occur about 1–3 weeks after ingestion of the parasite. These include: watery diarrhea with abdominal cramps, severe flatulence, nausea with or without vomiting, fatigue, and possibly fever. ■ Infection can become chronic leading to recurrent symptoms, severe malabsorption, and debilitation. Other symptoms include anorexia, malaise, and weight loss. Children with malabsorption syndrome often show failure to thrive. Patients may develop lactose intolerance.

FURTHER READING Goering R, Dockrell HM, Zuckerman M, Chiodini PL. Mims’ Medical Microbiology and Immunology, 6th edition. Elsevier, Philadelphia, 2018. Murphy K, Weaver C. Janeway’s Immunobiology, 9th edition. Garland Science, New York/London, 2016.

5. HOW IS THE DISEASE MANAGED AND PREVENTED? ■ Drugs used: nitroimidazole derivatives (e.g. metronidazole, tinidazole, ornidazole, and secnidazole); benzidamazoles (albendazole and mebendazole) nitrofuran derivatives (e.g. furazolidone), acridine compounds (e.g. mepacrine and quinacrine) aminoglycoside (especially for pregnant women) and nitazoxanide. ■ Metronidazole is the most common antibiotic treatment for giardiasis but more and more refractory cases are being described. ■ Aminoglycoside is the drug of choice for pregnant patients since it avoids the potential adverse effects of the other anti-Giardia agents on the fetus. ■ Prevention should include: good hygiene, avoidance of contaminated food and water, extra care during traveling to warm climates, boiling water, etc. ■ No known chemoprophylaxis and no human vaccine as yet. There is an effective vaccine for dogs.

REFERENCES Adam RD. Giardia duodenalis: Biology and Pathogenesis. Clin Microbiol Rev, 34: e00024–19, 2021. Buret AG, Caccio SM, Favennec L, Svard S. Update on Giardia: Highlights from the Seventh International Giardia and Cryptosporidium Conference. Parasite, 27: 49, 2020.

112

9780367696399_Lydyard.indb 112

13/02/2023 07:32

Case 12: Giardia intestinalis

Hooshyar H, Rostamkhani P, Arbabi M, Delavari M. Giardia lamblia Infection: Review of Current Diagnosis Strategies. Gasteroenterol Hepatol Bed Bench, 12: 3–12, 2019. Lopez-Romero G, Quintero J, Astiazaran-Garcia H, Velazquez C. Host Defences Against Giardia lamblia. Parasit Immunol, 37: 394–406, 2015. Maertens B, Gagnaire A, Paerewijck O, et al. Regulatory Role of the Intestinal Microbiota in the Immune Response Against Giardia. Sci Rep, 11: 10601, 2021. Morch K, Hanvik K. Giardia Treatment: an Update with a Focus on Refractory Disease. Curr Opin Infect Dis, 33: 355– 364, 2020. Touz MC, Feliziani C, Ropolo AS. Membrane-Associated Proteins in Giardia lamblia. Genes, 9: 404, 2018. Wang Y, Gonzalez-Moreno O, Roellig DM, et al. Epidemiological Distribution of Genotypes of Giardia duodenalis in Humans in Spain. Parasit Vectors, 12: 432–442, 2019.

WEBSITES Centers for Disease Control and Prevention, Parasites – Giardia: https://www.cdc.gov/parasites/giardia/index.html European Centre for Disease Prevention and Control, Giardiasis: https://www.ecdc.europa.eu/en/giardiasis/facts WebMD, Giardiasis, 2020: https://www.webmd.com/ digestive-disorders/giardiasis-overview

Students can test their knowledge of this case study by visiting the Instructor and Student Resources: [www. routledge.com/cw/lydyard] where several multiple choice questions can be found.

113

9780367696399_Lydyard.indb 113

13/02/2023 07:32

13

Helicobacter pylori

A 50-year-old advertising executive consulted his primary healthcare provider because of tiredness, lethargy, and an abdominal pain centered around the lower end of his sternum, which woke him in the early hours of the morning. The pain was relieved by food and antacids. His uncle had died of stomach cancer and he was worried that he had the same illness. On examination, his doctor noted that he seemed a bit pale and that he had a tachycardia. His blood pressure was low. He was slightly tender in his upper abdomen but there was no guarding or rebound tenderness. The doctor took blood and feces samples and organized for an upper gastrointestinal (GI) endoscopy. The full blood count showed a hypochromic normocytic anemia with a hemoglobin of 8.9 consistent with iron-deficiency anemia. The gastroscopy showed a 3 cm ulcer in the pre-pyloric region of the stomach (Figure 13.1). The fecal antigen test for Helicobacter pylori was positive. The patient was started on routine treatment for a duodenal ulcer.

1. WHAT IS THE CAUSATIVE AGENT, HOW DOES IT ENTER THE BODY AND HOW DOES IT SPREAD A) WITHIN THE BODY AND B) FROM PERSON TO PERSON? CAUSATIVE AGENT

Helicobacter pylori is a nonspore-forming, motile gramnegative bacterium with a helical shape measuring 2.5–4.5 × 0.5–1.0 μm. It has one to five unipolar sheathed flagellae. The organism has a high rate of polymorphism. The genome has at least five regions that it may have acquired from other organisms by lateral transfer of DNA. These are called “pathogenicity islands” (PAIs) and carry virulence genes. PAIs are found in many other bacteria. In H. pylori the cag-PAI (Figure 13.2) comprises about 30 genes that are responsible for the production of a type IV secretion system (T4SS) (Figure 13.3), which is used to transfer the CagA protein and other bacterial factors into host cells (see below and Section 2). Although the organism has a typical gram-negative cell wall, containing lipopolysaccharide (LPS) it is much less of an endotoxin compared with Escherichia coli. H. pylori grows in an atmosphere of 5–15% oxygen, 5–12% carbon dioxide,

Figure 13.1 Gastroscopy showing a duodenal ulcer in the prepyloric region of the stomach. Courtesy of Dino Varia, First Medical Clinical University of Bologna, Italy.

and 70–90% nitrogen (i.e. it is micro-aerobic) taking up to 5 days to grow on primary isolation and producing small (1–2 mm diameter) colonies on horse blood agar (5% horse blood in Columbia agar base). The colonies are domed, glistening, entire, gray or water-clear, and are sufficiently characteristic to suggest the presence of the organism (Figure 13.4). H. pylori can metabolize glucose, but its main carbon and energy source is from catabolism of amino acids. The organism has a urease, which is found both on the surface of the bacterium and in the cytoplasm and which is important for regulating the periplasmic pH. H. pylori can survive an acid environment for a short time but is not an acidophile. Many other Helicobacter species have been isolated from a wide range of animals. Some of these Helicobacter species can cause gastroenteritis in humans (e.g. Helicobacter cinedae) and some cause stomach ulcers in the animal host. Many are associated with the lower intestinal tract and particularly the hepato-biliary system where, in animals, the organisms are the cause of hepatoma.

ENTRY INTO THE BODY The route of infection is not clear but is either feco–oral or oro–oral. In some countries (e.g. Peru), there is some evidence that infection may be acquired from sewage contamination of water supplies or vegetables. Following entry into the body 115

9780367696399_Lydyard.indb 115

13/02/2023 07:32

Case Studies in Infectious Disease

A EPIYA Motif

Csk

PI3k

SHP1/2 key

key

E = Glutamic acid P = Proline I = Isoleucine Y = Tyrosine A = Alanine

P

EPIYA A - Csk EPIYA B - Csk PIk3 EPIYA C - SHP1 EPIYA D - SHP2

E

B Western/East Asian diversity

I

P

E

P

I

Y

Y

A

A

key

CM

CM = multimerization domain

Western A

B

C CM East Asian

A

C Common structural Western/East Asian diversity

key

B

D

A

B

C

A

B

C

C

A

B

C

C

A

B

D

A

A

B

A

B

A

A

B

D

1

2

Western

Binding to host cell membrane C

B

D

3

D

1

2

East Asia

3

Figure 13.2 The cytotoxin-associated gene (CagA) pathogenicity island (cag-PAI). This shows the general structure of the cag-PAI. Strains with a cag-PAI are called Type I strains and those lacking a cag-PAI Type II strains. Type I strains are more likely to be linked to severe disease than Type II strains. The cag-PAI comprises about 30 genes involved in the synthesis of the Type IV secretion system and a gene for the Cag A protein. The PAI may be complete, it may be separated into two sections (CagI and CagII) by an insertion element, as indicated in the upper Case_15_2 part of the diagram, part of the PAI may be missing or the strain may lack a PAI. Variation occurs in the 3′ end of the PAI and several sequence variations have been identified. The sequences belong to two groups, one found principally in strains isolated from Western countries (WSS) Type A, B, C and one from Asian (Eastern) countries (EASS) Type A, B, D. Adapted with permission from a map created by the Helicobacter Foundation and originally published at the following web address: http://www.helico.com/h_epidemiology.html.

116

9780367696399_Lydyard.indb 116

13/02/2023 07:32

Case 13: Helicobacter pylori

Figure 13.3 Type IV secretion system. The Type IV secretion system is a complex structure that acts as a microsyringe and is used to transfer material such as the CagA protein and part of the peptidoglycan of the cell wall of H. pylori into the host cell. The CagA protein is an oncoprotein and affects cellular signaling events, It is also important in evasion of the immune system by H. pylori. The T4SS is the largest of any bacterial species and up-regulates Toll-like receptor 4 (TLR4) on the host cell. Cag L, a component of the secretion system, binds to integrin receptor 5β1 on the gastric epithelial cell but also has a number of non-structural functions. Cag L stimulates epidermal growth factor receptor (EGFR) 4 and 5, stimulates cell spreading and increases gastrin secretion. -Helicobacter pylori also has other Type IV secretion systems: ComB transports free DNA, Tfs3 secretes CtkA (cell-translocating kinase A: activation of NF-kB) and along with Tfs4 is involved with conjugative DNA transfer. Various other proteins result in activation of NF-kB and a pro-inflammatory response. Adapted with permission from an image created by The Nobel Committee for Physiology or Medicine and originally published at the following web address: http://nobelprize.org/ nobel_prizes/medicine/laureates/2005/press.html.

Figure 13.4 Colonies of H. pylori on blood agar.

via the oral route H. pylori localizes in the stomach. Here, it is found mainly in the antrum but can also be found in all parts of the stomach and duodenum. In the micro-environment of the stomach, the H. pylori locates to different regions (Figure 13.5). Migration between the regions occurs and appears to be more frequent between the corpus and the fundus compared to the

9780367696399_Lydyard.indb 117

corpus and the antrum, which probably reflects the different physiologic environments of the antrum compared to the corpus/fundus, which is the main site of oxyntic cells secreting acid. On initial entry into the stomach, it can only survive for a short time before it is killed by the acid. However, the presence of the enzyme urease on its surface (by hydrolyzing urea and producing ammonium ion) protects it for sufficient time enabling it to penetrate the mucus layer. The spiral shape and motility of Helicobacter, and the production of phospholipases and the ammonium ion (which affects the tertiary structure of the mucus making it thin and watery) allows the penetration of the mucus layer very quickly. H. pylori can only adhere to gastric epithelial tissue, which is found in the stomach. Many adhesins on the surface of the organism include Sialic acid binding adhesin (SabA) and OipA but the principle adhesin is Blood Group Antigen Binding Adhesin (BabA), which binds to the Lewis b blood group antigen expressed on gastric epithelial cells. BabA is not only important as an adhesin but also anchors the bacterial secretion system to the host cell surface for efficient injection of bacterial factors into the epithelial cell cytosol. Once attached through BabA, the T4SS system host cell signaling is triggered to induce transcription of genes that enhance inflammation and leads to development of intestinal

117

13/02/2023 07:32

Case Studies in Infectious Disease

A

B

C

gastric pit

mucus cell fundus

cardia

parietal cell

body

chief cell

fundus

parietal (oxyntic cell) secretion canaliculi

mucous cell

pylorus

G cell Figure 13.5 Anatomic areas of the stomach with main cell types. The main areas of the stomach which bind H. pylori are shown in green and red. (A) Gastric ulcers typically develop on the lesser curve of the body and duodenal ulcers in the antrum and pylorus. (B) illustrates a gastric pit with associated cell types, (C) illustrates a parietal cell which secretes hydrochloric acid.

Case_4_3

Figure 13.6 Global prevalence of H. pylori infection. The prevalence of H. pylori infection correlates with local socio-economic status. H. pylori has been linked with the human population as far back as 100–150 000 years and different racial groups are colonized by different genotypes of the organism to such an extent that human migration patterns can be discerned from the strain of Helicobacter colonized by the person.

118

9780367696399_Lydyard.indb 118

13/02/2023 07:32

Case 13: Helicobacter pylori

decreased acid secretion

Figure 13.7 General scheme of infection with H. pylori leading to development of a peptic ulcer. (A) H. pylori and its antral location in the stomach; (B) H. pylori on the surface of the epithelium with inflammatory cells in the tissue (C) duodenal ulcer (D) the location of a duodenal and gastric ulcer. The former can be in the first part of the duodenum or the pre-pyloric region, as shown in (C) and the latter is usually on the lesser curvature of the corpus of the stomach.

metaplasia and associated precancerous transformation (see Section 2, Pathogenesis).

SPREAD WITHIN THE BODY The organism only grows within the stomach either only in the antrum or throughout the whole of the stomach lining. H. pylori can invade the gastric epithelium where they become sequestered and survive in phagosomes (see Section 2, host response). They can also be found in the vacuoles of yeasts present in the stomach. It is thought that this may be a source that could allow relapse of infection.

about 60–70% with little increase with age. Overcrowding and a poor public hygiene infrastructure are factors in the spread of Helicobacter. Spread within families has been documented by molecular typing techniques. In industrialized countries, the low rate in childhood and high rate in the elderly can be explained by improving social standards of housing (with less overcrowding), potable water supplies, and sewage disposal. The high rate in the elderly can be explained as a cohort effect, reflecting the housing and public health standards when they were children. Although some other species (see Causative agent, above) can infect humans, there is no evidence that H. pylori can be acquired from animals (i.e. it is not a zoonosis).

PERSON-TO-PERSON SPREAD Spread is via the feco–oral route or oro–oral route mainly, and it is believed to be from mother to child. Adults can become infected with H. pylori but the route is not clear and may involve the environment, food or water.

EPIDEMIOLOGY Globally, the prevalence is high in developing countries with the infection being acquired at a young age. Here, the incidence is 3–10% of the population each year compared with 0.5% in Case_15_7 countries. However, the incidence of H. pylori developed infection, gastric cancer, and ulcer disease are declining. Worldwide, more than 1 billion people are estimated to be infected with H. pylori. The global prevalence of H. pylori is shown in Figure 13.6. Sero-epidemiologic studies have identified several risk factors. Thus, infection is higher in Social Class IV and V compared with I and II. Infection in industrialized countries (Western Europe, North America) is about 5–10% in the first decade rising to 60% in the sixth decade of life. In non-industrialized countries (Africa, South America, Middle and Far East) infection in the first decade is

9780367696399_Lydyard.indb 119

2. WHAT IS THE HOST RESPONSE TO THE INFECTION AND WHAT IS THE DISEASE PATHOGENESIS? The overall sequence of infection of the host with H. pylori is shown in Figure 13.7, illustrating its macroscopic and microscopic location and the location of ulcer formation. On initial infection of the gastric tissue, the host responds strongly to the presence of H. pylori by both the innate and acquired immune systems.

INNATE IMMUNITY The mucus is the first barrier to H. pylori but having successfully dealt with that, it binds to the gastric epithelium using several adhesins in its outer membrane (see Section 1, entry into body). Although H. pylori does not normally penetrate the gastric epithelium it can invade epithelial cells and even enter the submucosa. The organisms can remain in vesicles in the epithelial cells when conditions are not optimum for the organism. The gastric epithelial cells express TLR2 and

119

13/02/2023 07:32

Case Studies in Infectious Disease

TLR4 that interact with microbial-associated molecular patterns (MAMPS) on Helicobacter resulting in production of defensins and other antimicrobial peptides. In addition, the epithelial cells produce a variety of cytokines (including IL-8) and chemokines that recruit monocytes, macrophages, and neutrophils to the submucosa resulting in an acute inflammatory response. These cells also have various patternrecognition receptors (PRRs) including TLRs that, through microbial products (including LPS) of H. pylori, result in their activation producing pro-inflammatory cytokines to further amplify the acute inflammatory response. Macrophages and neutrophils release reactive oxygen species (ROS) which damage the epithelial surface. This acute reaction is followed by a more chronic inflammatory reaction.

is able to evade the immune defenses. The inflammation mediated by H. pylori is shown in Figure 13.8.

MECHANISMS OF EVASTION OF THE IMMUNE SYSTEM BY H. PYLORI H. pylori has a number of mechanisms to avoid the immune system: 1. Evasion of recognition by PRRs through a number of

2. 3.

ADAPTIVE IMMUNITY Dendritic cells in the gastric mucosa are believed to be activated by H. pylori through their PRRs. They up-regulate their CD86, increase expression MHC class II, and release proinflammatory cytokines such as IL-12. This is consistent with a role of inducing Th0 cells to Th-1. Th-17 also plays a role by secreting IL-8 and other pro-inflammatory cytokines. Gastric inflammation, mediated by the adaptive immune system in H. pylori infection, is believed to be caused primarily by Th-1 cells through the production of IFN . Dendritic cells carrying processed antigen from H. pylori are likely to traffic to local draining lymph nodes where specific IgG and IgA antibodies are presumably made. It has been speculated that antibodies arising from molecular mimicry (autoantibodies) could contribute to epithelial cell damage during H. pylori infections. In addition, experimental models of H. pylori in mice have indicated that, instead of having a protective effect, specific antibodies facilitate bacterial colonization and counteract resistance against infection. Thus, although a good cellular and antibody immune response is mounted, the organism

Figure 13.8 Helicobacter pylori – mediated inflammation in the stomach. Hematoxylin and eosin (H&E) stain of the gastric mucosa showing in the mucus and on the epithelial surface (A). Helicobacter pylori stains poorly with H&E and is better seen with a silver or Giemsa stain. Infiltrating granulocytes and mononuclear cells can be seen in the epithelial layer (lamina propria) indicating that the infection is becoming chronic (B). With permission from Dr Lorraine Racusen, Johns Hopkins Medical School, Johns Hopkins University, Baltimore, MD.

120

9780367696399_Lydyard.indb 120

4.

5. 6. 7. 8.

mechanisms, e.g. Toll-like receptors, (TLRs), RIG-1-like receptors (RIRs), and C lectin-type receptors (CLRs). LPS: of low immunogenicity. CagA and other material transferred by the 4TSS: suppresses phagocytosis, decreases production of antimicrobial peptides by the epithelial cells, induces tolerogenic dendritic cells suppressing T-cell induction blocking T-cell effector responses. Vacuolating toxin A: suppresses phagocytosis, induces tolerogenic dendritic cells blocking T-cell effector responses. Gamma-glutamyl transpeptidase: induces tolerogenic dendritic cells blocking T-cell effector responses. Cholesterol- -glucosyl transferase: suppresses phagocytosis. Catalase superoxide dismutase: suppresses production of ROS and NO. Arginase: suppresses production of ROS and NO, blocks effector T-cell function.

PATHOGENESIS There are five main ways in which tissue damage can occur (Figure 13.9). Colonization by H. pylori leads to excess acid in the stomach (and peptic ulcers) and both a hypergastrinemia and hyperpepsinogenemia, unless gastric atrophy occurs. This destroys the acid-producing cells and leads to hypoacidity which is linked to gastric cancer. Hyperacidity is caused by inhibition of somatostatin by H. pylori. Somatostatin is a negative feedback for acid secretion by oxyntic cells. Vacuolating cytotoxin is a multimeric pore-forming protein consisting of a 55kD-binding protein and a 33kDa protein which forms a pore in membranes and induces vacuolation in cells by fusion of dysfunctional phagosomes to form megasomes. It has multiple cellular effects: disrupting the mitochondria inducing apoptosis, affecting autophagy, disrupting the tight junction between cells, blocking T-cell proliferation and thus the immune response, and disrupting acid production by oxyntic cells. After injection into the gastric cell by the T4 secretion apparatus, CagA is phosphorylated at a tyrosine residue and the activated CagA binds and activates SHP-2 and Csk. The activation of SHP-2 stimulates a change in the shape of the cell to a “hummingbird” phenotype. Activation of Csk which

13/02/2023 07:32

Case 13: Helicobacter pylori

Figure 13.9 Mechanisms of pathogenesis of H. pylori. There are five main ways in which tissue damage can occur. These are: (A) local damage caused by a vacuolating cytotoxin (VacA), the ammonium ion as a result of the urease activity and the production of phospholipase that contribute to the formation of a poor-quality mucus barrier; (B) alteration of gastric physiology with enhanced acid production; (C) bystander damage caused by activation of granulocytes; (D) autoimmunity; (E) alteration of the balance of cell division and apoptosis induced by CagA injected into the epithelial cell through T4SS.

Figure 13.10 The vacuolating cytotoxin. The toxin is activated by the acid of the stomach, and the monomers of the toxin oligomerize in the host cytoplasmic membrane, enter the cytoplasm and affect the normal endocytic cycle, producing large, acidified vacuoles that lead to cell death. Various polymorphisms are found in the gene. Its leader sequence varies with the strain: the S1 leader sequence is found in type I strains and the S2 in type II strains, which produces less cytotoxin. Polymorphisms within the S1 leader sequence, S1a, S1b, and S1c, are found in different geographic regions of the world, are associated with different racial groupings, and can be used as a surrogate marker for human migration patterns. S1a is found mainly in North America and Europe; S1b is found mainly in Iberia and South America, and S1c mainly in the Far East. In Mexico, a1b/m1 is common although S1d , a novel subtype of vacA has been found in children with recurrent abdominal pain. An intermediate region (i) between s and m has two subtypes i1, i2. A further region between i and m called d with one type called d1 (no deletion) and d2 (with an 81bp deletion) The role of i1/i2 in carcinogenesis is unclear because of conflicting results.

9780367696399_Lydyard.indb 121

121

13/02/2023 07:32

Case Studies in Infectious Disease

Figure 13.11 Model of the sequential steps in the development of gastric cancer mediated by H. pylori. This shows the multiple stages to development of gastric cancer with H. pylori being the initial insult but other factors such as salt intake, and a lack of vitamin C, also playing a role. Evidence from short-term studies suggests that eradication of H. pylori can lead to reversal of the histologic changes of atrophy, intestinal metaplasia, methylation changes, and epithelial-mesenchymal transition leading to a reduction in gastric cancer development. Long-term studies are not yet available.

then activates the Src tyrosine kinases to form a Csk-CagA complex acts as a negative feedback for the initial activation of CagA. The CagA-SHP-2 complex induce apoptosis of infected cells. Non-phosphorylated CagA interacts with PAR1 kinase (regulating cell polarity and tight junctions) and induces a “scattering” cell phenotype. The net effect of these interactions leads to a complex array of cellular alterations that induce stem-cell transformation of epithelial cells and gastric carcinogenesis. The route to duodenal ulcer (high acid, antral gastritis, low cancer risk) or gastric ulcer (low acid, pan gastritis, high cancer risk) may depend on the interaction between host and organism polymorphisms. The development of atrophic gastritis (atrophy of the mucosal epithelium) is a risk factor for the development of gastric cancer and the evolution to cancer proceeds via stages of metaplasia and dysplasia. Other factors are also important in the eventual development of cancer such as the amount of dietary antioxidants and salt consumed (Figure 13.11). Some of the extra GI effects of H. pylori (e.g. idiopathic thrombocytopenic purpura (ITP) are caused by the presence of autoantibodies.

3. WHAT IS THE TYPICAL CLINICAL PRESENTATION AND WHAT COMPLICATIONS CAN OCCUR?

122

Most individuals colonized by H. pylori will remain symptomfree. Antral gastritis is the most common manifestation in children. About 20% of adults will go on to develop peptic

9780367696399_Lydyard.indb 122

ulcer disease and about 1% gastric cancer. H. pylori is the prin­ cipal cause of peptic ulcer disease (gastric or duodenal ulcer) in adults but this is uncommon in children. Ulcer disease presents with epigastric pain, heartburn or dyspepsia or may be totally asymptomatic. Anemia (due to blood loss from the ulcer) and weight loss may also occur and signs and symptoms of perforation (acute abdominal pain, abdominal rigidity and guarding, rebound tenderness and shock). H. pylori is a good example of a “slow” infection, because infection occurs in childhood but related diseases occur in adulthood. Its role in gastric cancer formation is described as a “hit as and run” model since even if the original insult (H. pylori infection) is removed, the process leading to cancer is unstoppable. However, short-term studies have demonstrated histopathologic recovery to normal histology after Helicobacter eradication. The role of H. pylori in non-ulcer dyspepsia (NUD) is controversial but it may be that a subset of individuals who have NUD do benefit from eradication of the organism. In addition to GI diseases, it has been suggested that H. pylori may be related to a wide range of extra-GI disease such as coronary heart disease, stroke, migraine, rosaceae, Behcet’s disease, and gallbladder disease. Some evidence exists for most of these but again there is contrary evidence. Confirmed extra-gastric conditions caused by H. pylori are iron deficiency anemia, ITP, and vitamin B12 deficiency. Also, H. pylori may be protective for reflux esophagitis, Barrett’s esophagus, and esophageal carcinoma and in extra-gastric conditions: asthma.

13/02/2023 07:32

Case 13: Helicobacter pylori

The most serious consequence of H. pylori infection is the development of cancer. H. pylori is a Class I carcinogen and the cause of the majority of cases of gastric adenocarcinoma (except that of the cardia of the stomach) and mucosal associated lymphoid tissue (MALT) lymphoma. Clinical presentation of carcinoma is very nonspecific and is usually associated with gastric ulcer rather than duodenal ulcer. It presents with abdominal pain or a mass with so-called “alarm symptoms” – weight loss and anemia. Gastric cancer originating at the cardia (gastro-esophageal junction) does not appear to be related to colonization by H. pylori.

4. HOW IS THE DISEASE DIAGNOSED, AND WHAT IS THE DIFFERENTIAL DIAGNOSIS? There are numerous ways in which H. pylori can be diagnosed. The first general method involves an endoscopy and biopsy. The biopsy can then be cultured under micro-aerobic conditions for H. pylori, which takes 5 days; histology can demonstrate the characteristically shaped organism on the surface of the epithelial cells (Giemsa or Genta stain) and the inflammatory cell type can be identified (hematoxylin and eosin); the urease of the bacterium can be detected using a rapid urease test (which involves putting one of the biopsies into a urea solution with a pH indicator); and finally H. pylori can be detected using the polymerase chain reaction (PCR) with appropriate primers. With the development of whole genome sequencing (WGS), rapid detection of antibiotic resistance and virulence markers become a possible option. An additional further development in molecular diagnostics is detection of miRNA. These non-coding 18–25 nucleotide RNAs are found in body fluids/tissues and are involved in regulatory systems. Deregulation can lead to panels of biomarkers to identify gastric cancer or H. pylori. One miRNA (miR-223 ) is higher with gastric cancer and H. pylori. Routinely, in the primary care setting, diagnosis is by noninvasive tests. These are: 1. the stool antigen test – these are immunoassay tests that

use either polyclonal or monoclonal antibodies to detect the Helicobacter antigen in the feces; 2. the urea breath test – this test is performed by giving the patient a drink containing labeled urea (13C ) and 20 minutes later collecting the breath and measuring the amount of labeled CO2 using an isotope ratio mass spectroscopy The basis of the breath test is that the Helicobacter urease, in the stomach, hydrolyzes the labeled urea to labeled CO2 which is exhaled in the breath. Either of these tests is recommended for use in the Maastricht Guidelines (a consensus document from gastroenterologists in Europe) in the cost-effective “Test &

Treat” policy – a person complaining of upper GI symptoms will be tested for Helicobacter and, if positive, will be treated. Serology is of no help in diagnosis of active disease but is useful for epidemiology studies.

DIFFERENTIAL DIAGNOSIS This includes: stomach ulcer disease caused by nonsteroidal anti-inflammatory (NSAID) drug usage, particularly as the global prevalence of H. pylori is decreasing; acute gastritis; atrophic gastritis; gastric cancer; gastrinoma; gastroesophageal reflux disease, non-Hodgkin lymphoma; peptic ulcer disease and stress induced gastritis.

5. HOW IS THE DISEASE MANAGED AND PREVENTED? MANAGEMENT

The main problem in treatment is the increasing and widespread multi-antibiotic resistance of the strains, leading to failure of the usual first-line regimen: a proton pump inhibitor (PPI) and two antibiotics (amoxicillin, clarithromycin, metronidazole). Otherwise, it should be bismuth quadruple (PPI + bismuth + 2 antibiotics, e.g. metronidazole, tetracycline) The choice depends on the local resistance pattern of the strains. Several other treatment regimens have evolved to cope with the resistance problem: the use of different antibiotics such as tetracycline, furazolidone, fluoroquinolones; quadruple therapy with addition of bismuth citrate (now suggested as first line); sequential therapy with a short course (10 days) of a PPI + amoxicillin followed by the addition of clarithromycin/metronidazole (final 5 days); hybrid therapy with a PPI + amoxicillin (10 days) followed by the addition of clarithromycin/metronidazole (final 7 days); and sensitivity testing of the isolate. Additionally, vonoprazan (a potassium channel blocker) can be substituted for the PPI and studies so far indicate higher eradication rates compared to a PPI. Additionally, a new regime of vonoprazan + amoxicillin is being tried. If treatment fails, a biopsy should be taken and sensitivity of the isolate determined. In summary, first-line treatment should be bismuth quadruple or concomitant therapy but in areas of low prevalence of clarithromycin resistance a two-week clarithromycin­ containing triple therapy can be used in macrolide naïve patients. Second-line treatments include levofloxacin­ containing triple therapy and bismuth quadruple therapy. Sequential therapy has reduced success owing to resistance and is going out of favor. Probiotic can be used along with the antibiotics to reduce antibiotic-related side-effects. Other approaches being assessed are the addition of probiotics to the antibiotic regimen, administering the antibiotic cocktail during endoscopy directly to the stomach lining, using plant derived phytochemicals: antibiofilm medication derived from Acorus calamus and inflammatory 7,8 123

9780367696399_Lydyard.indb 123

13/02/2023 07:32

Case Studies in Infectious Disease

dihydroxycoumarins from Changbai daphne. Finally, laboratory evidence suggests that H. pylori is sensitive to photodynamic therapy.

PREVENTION The prevalence of H. pylori globally is decreasing possibly due to increasing standards of hygiene. Currently, there is no effective vaccine. The main technical challenges for vaccine development are the ability of the organism to avoid the host response with no obvious protein(s), the inhibition of which could lead to the eradication of the organism. This difficulty is amplified by the extensive polymorphisms and sequence

variation present in H. pylori. The other barrier to an H. pylori vaccine is a reluctance of many pharmaceutical companies to invest in vaccine development owing to a lack of appreciation that, despite the falling prevalence of the organism, it is still a common infection worldwide with serious outcomes (ulcer and cancer) for many patients. Two alternative vaccine approaches are being tried. One is a vaccine against HtrA (a protein which affects cell junctions allowing H. pylori to bind to the basolateral surfaces of gastric epithelium where the oncogenic CagA protein is injected into gastric cells. The other approach is a vaccine against g-glutamyltranspeptidase, a protein that inhibits T-cell responses against H. pylori.

SUMMARY 1. WHAT IS THE CAUSATIVE AGENT, HOW DOES IT ENTER THE BODY, AND HOW DOES IT SPREAD A) WITHIN THE BODY AND B) FROM PERSON TO PERSON? ■ The cell wall is a typical gram-negative structure. ■ The lipopolysaccharide has considerably less endotoxin activity compared with other gram-negative bacteria. ■ H. pylori occurs in over 50% of the global population but prevalence globally is decreasing. ■ Colonization occurs in childhood. ■ Transmission is feco–oral or oro–oral. In some locations, transmission may be from water supplies. ■ Colonization is related to local social conditions and the public health infrastructure. ■ The organism colonizes the gastric tissue either in the stomach or the duodenum.

2. WHAT IS THE HOST RESPONSE TO THE INFECTION AND WHAT IS THE DISEASE PATHOGENESIS? ■ There is a strong innate immune response with infiltration by macrophages and neutrophils (acute inflammation). ■ Helicobacter has several ways of avoiding the immune response including inhibition of phagocytosis, reduction in ROS and NO and inhibition of T-cell effector function. ■ There is a strong acquired immune response with antibody production but this is generally ineffective. Th-1 cells are mainly responsible for inflammation. ■ Certain virulence markers, e.g. CagA, VacA, are associated with more severe disease. ■ Direct damage is brought about by the secretion of enzymes that destroy the mucus barrier and vacuolating cytotoxin that kills the surface epithelial cells. ■ Gastric regulation of acid production is disturbed by the inhibition of somatostatin caused by the LPS of Helicobacter. ■ Autoantibodies are induced by Helicobacter that kill the acid-secreting parietal cells and cause some extra-intestinal manifestations.

■ Gastric cell dynamics are affected by interference with normal cell signaling events caused by introduction of the CagA protein of Helicobacter. ■ Bystander damage is caused by release of free radicals from the granulocytes.

3. WHAT IS THE TYPICAL CLINICAL PRESENTATION AND WHAT COMPLICATIONS CAN OCCUR? ■ Helicobacter is a “slow” infection, with colonization occurring in childhood and disease occurring years later. ■ The vast majority of persons colonized by H. pylori remain asymptomatic. ■ H. pylori is the main cause of peptic ulcer disease and gastric cancers. ■ H. pylori may be associated with some extra-gastrointestinal diseases.

4. HOW IS THE DISEASE DIAGNOSED, AND WHAT IS THE DIFFERENTIAL DIAGNOSIS? ■ Diagnosis is by invasive (endoscopy) or noninvasive tests. ■ Invasive tests are culture, histology, rapid urease test, PCR. ■ Noninvasive tests are serology, antigen detection, and the urea breath test. ■ A cost-effective strategy is “Test & Treat”. ■ Recommended tests are the urea breath test and the fecal antigen tests. ■ Whole genome sequencing (WGS) and assays for micro inhibitory RNAs are emerging tests.

5. HOW IS THE DISEASE MANAGED AND PREVENTED? ■ The first-line treatment is a PPI plus clarithromycin and amoxycillin or metronidazole and amoxycillin for 7–10 days in areas with low levels of resistance. ■ There is a global increase in resistant and multi-resistant isolates making treatment difficult. ■ Rescue regimens should be guided by the sensitivity of the isolate to antibiotics.

124

9780367696399_Lydyard.indb 124

13/02/2023 07:32

Case 13: Helicobacter pylori

FURTHER READING Malik TF, Gnanapandithan K, Singh K. Peptic Ulcer Disease. StatPearls, 2021. Mims C, Dockrell HM, Goering RV, et al. Medical Microbiology, 3rd edition. Mosby, London, 2004. Olga P Nyssen, Bordin D, Tepes B, et al. European Registry on Helicobacter pylori Management (Hp-EuReg): Patterns and Trends in First-Line Empirical Eradication Prescription and Outcomes of 5 Years and 21 533 Patients. Gut, 70: 40–54, 2021. Parikh NS, Ahlawat R. Helicobacter pylori. StatPearls, 2021.

REFERENCES Alipour M. Molecular Mechanisms of Helicobacter pyloriInduced Gastric Cancer. J Gastrointest Cancer, 52: 23–30, 2021. Bedwell J, Holton J, Vaira D, et al. In Vitro Killing Helicobacter pylori with Photodynamic Therapy. Lancet, 335: 1287, 1990. Chisty A. Update on Indigestion. Med Clin North Am, 105: 19–30, 2021. Eslick GD. Helicobacter Infection Causes Gastric Cancer? A Review of the Epidemiological, Meta-Analytic and Experimental Evidence. World J Gastroenterol, 12: 2991–2999, 2006. Fischer W, Tegtmeyer N, Stingl K, Backert S. Four Chromosomal Type IV Secretion Systems in Helicobacter pylori: Composition, Structure and Function. Front Microbiol, 11: 1592, 2020. Ford AC, Delaney BC, Forman D, Moayyedi P. Eradication Therapy for Peptic Ulcer Disease in Helicobacter pylori Positive Patients. Cochrane Database Syst Rev, 4: CD003840, 2016. Karkhaha A, Ebrahimpour S, Rostamtabar M, et al. Helicobacter pylori Evasion Strategies of the Host Innate and Adaptive Immune Responses to Survive and Develop Gastrointestinal Diseases. Microbiol Res, 218: 49–57, 2019.

Kusters JG, Van Vliet AH, Kuipers EJ. Pathogenesis of Helicobacter pylori Infection. Clin Microbiol Rev, 19: 449–490, 2006. Moyat M, Velin D. Immune Responses to Helicobacter pylori Infection. World J Gastroenterol, 20: 5583–5593, 2014. Munoz-Ramirez ZY, Pascoe B, Mendez-Tenorio A, et al. A 500-year Tale of Co-Evolution, Adaption and Virulence: Helicobacter pylori in the Americas. ISME J, 15: 78–92, 2021. O’Morain C. Role of Helicobacter pylori in Functional Dyspepsia. World J Gastroenterol, 12: 2677–2680, 2006. Robinson K, Atherton JC. The Spectrum of HelicobacterMediated Diseases. Ann Rev Pathol, 16: 123–144, 2021. Saracino IM, Pavoni M, Sacconanno L, et al. Antimicrobial Efficacy of Five Probiotic Strains against Helicobacter pylori. Antibiotics, 9: 244, 2020. Zagari RM, Frazzoni L, Marasco G, et al. Treatment of Helicobacter pylori Infection: A Clinical Practice Update. Minerva Med, 112: 281–287, 2021.

WEBSITES European Helicobacter and Microbiota Study Group: www.helicobacter.org Nature Portfolio, Helicobacter pylori: https://www.nature. com/subjects/helicobacter-pylori National Cancer Institute, Helicobacter pylori and Cancer, 2013: https://www.cancer.gov/about-cancer/causes-prevent ion/risk/infectious-agents/h-pylori-fact-sheet The Helicobacter Foundation, 2006: www.helico.com United European Gastroenterology Federation: www.uegf. org WebMD, What Is H. pylori? 2020: https://www.webmd. com/digestive-disorders/h-pylori-helicobacter-pylori Students can test their knowledge of this case study by visiting the Instructor and Student Resources: [www. routledge.com/cw/lydyard] where several multiple choice questions can be found.

125

9780367696399_Lydyard.indb 125

13/02/2023 07:32

14

Hepatitis B virus

Figure 14.1 Jaundice as demonstrated by a yellow discoloration of the sclera of the eye. From the Centers for Disease Control & Prevention, Atlanta, Georgia. Image is found in the Public Health Image Library #2860. Additional photographic credit is given to Thomas F. Sellers and Emory University who took the photo in 1963.

1. WHAT IS THE CAUSATIVE AGENT, HOW DOES IT ENTER THE BODY AND HOW DOES IT SPREAD A) WITHIN THE BODY AND B) FROM PERSON TO PERSON? CAUSATIVE AGENT

Hepatitis B virus (HBV) belongs to the virus family Hepadnaviridae – that is hepa (referring to the liver), DNA (referring to the nature of the viral genome), viruses – each member of which infects the liver of its particular host species. HBV is the member of the family that infects humans. The Hepadnaviridae have an unusual partially doublestranded circular DNA genome (Figure 14.2). The genome is small – around 3200 bases – and therefore encodes only a small number of viral proteins, or antigens. There are four open reading frames (ORFs) identifiable within the genome, as follows. • ORF C encodes the core antigen (HBcAg), which forms a protective nucleocapsid around the genome. • ORF S encodes the surface antigen (HBsAg). This is composed of the large, middle, and small surface

A 28-year-old stockbroker has been feeling generally unwell for the last 4 days, and off his food. Although usually a smoker (10–15 cigarettes per day), he hasn’t been able to face “lighting up” since his illness began. Since yesterday, he has complained of a vague ache below his ribs on the right side. He noticed that his urine had become very dark, and his friends told him today that his eyes looked yellow (Figure 14.1). He has no relevant past medical history. On examination by his local primary care physician, he was pyrexial (38.5°C) and clinically jaundiced. The only other sign of note was some right upper quadrant abdominal tenderness, but no guarding. A clotted blood sample was sent to the laboratory, and 3 hours later, the laboratory called to convey the preliminary results, suggesting that the patient was suffering from acute hepatitis B virus infection.

proteins, the different sizes arising from use of different

start codons within the gene (large contains the pre-S1,

pre-S2, and S regions; middle contains the pre-S2 and

S regions; small contains just the S region – see Figure

14.2A), which are embedded in a lipid bilayer derived

from internal membranes of the infected hepatocyte,

surrounding the nucleocapsid.

• ORF P encodes the polymerase enzyme, which has both DNA- and RNA-dependent polymerase activities. • ORF X encodes the X antigen, which is known to act as a transactivator of transcription and is likely involved in carcinogenesis. The hepatitis B viral structure is shown schematically in Figure 14.2B. The replication cycle of HBV is unique among human viral pathogens. The virus attaches to the sodium taurocholate co-transporting polypeptide (NTCP, a bile acid transporter) expressed on the surface of hepatocytes which has been identified as the primary receptor for the virus. After entry into a susceptible cell and uncoating of the viral genome, the first step is synthesis of the missing part of the positive DNA strand, to form a complete double-stranded (ds) DNA molecule. Within the cell nucleus, the dsDNA is converted into 127

9780367696399_Lydyard.indb 127

13/02/2023 07:32

Case Studies in Infectious Disease

covalently closed circular (ccc) DNA, which is an extremely stable molecule, behaving like a mini chromosome. A number of RNA species are transcribed from the DNA, encoding the various viral proteins referred to above, but importantly, there is also a 3.5 kb RNA copy of the whole genome (pregenomic RNA). This RNA is packaged within newly synthesized core protein, together with the viral polymerase, to form immature new virus particles. The final step in viral maturation is then the reverse transcription of this pregenomic RNA into a DNA copy, followed by synthesis of an incomplete complementary DNA strand to yield partial dsDNA.

ENTRY AND SPREAD WITHIN THE BODY HBV enters a new host via the genital tract or following direct inoculation of virus into the bloodstream (see below). Once within the blood, virus travels to the liver, where it infects hepatocytes. Once within a liver cell, the virus replicates, and new virus particles are released directly into the bloodstream. From here, they gain access to every bodily compartment, including the genital tract.

PERSON-TO-PERSON SPREAD HBV infection is spread by three routes. 1. Mother to baby, or vertical transmission. Babies acquire

infection at the time of birth, through exposure to infected maternal blood and/or genital tract secretions. On a global scale, this is by far the most important route of infection. Over 90% of babies of carrier mothers become infected. 2. Sexual transmission. In an infected individual, both seminal fluid (male) and the female genital tract will contain virus. Unprotected sexual intercourse may therefore result in transmission of infection from one partner to another. This is especially the case with men who have sex with men, where the act of intercourse may also involve exposure to blood through mucosal tears. 3. Direct blood-to-blood transfer of virus (parenteral transmission). This is a somewhat artificial, but nevertheless important, route of transmission, as humans are not normally exposed to blood from each other (except during childbirth). The most obvious way in which this can arise is via blood transfusion – if the blood donor is infected with HBV, then the blood will transmit infection to the recipient. More subtle ways of achieving this include the re-use/sharing of contaminated needles/syringes when injecting medicinal or recreational drugs; exposure to contaminated needles, for example, via tattooing, ear or body piercing or acupuncture; needlestick injuries as suffered by healthcare workers, i.e. accidental stabbing of a needle derived from an infected patient into the healthcare worker’s own finger; or by sharing contaminated razors

Figure 14.2 (A) Diagram of hepatitis B virus (HBV) genome showing the partially double-stranded DNA genome, and the four open reading frames (S, encodes hepatitis B surface antigen; C, encodes hepatitis B core antigen; P, encodes DNA polymerase enzyme; X, encodes X antigen) from which mRNA is synthesized. (B) Schematic diagram of the structure of an HBV particle. The partially double-stranded DNA genome is enclosed within the core antigen/protein which is, in turn, surrounded by an envelope consisting of a lipid bilayer derived from internal membranes of the hepatocyte into which are embedded the large, middle, and small surface proteins, which together constitute the surface antigen.

or toothbrushes. Horizontal transmission between young children also occurs through inapparent blood contamination of cuts and scratches.

EPIDEMIOLOGY The World Health Organization (WHO) estimates that there are more than 290 million people chronically infected with HBV (see below). Rates vary across the globe (Figure 14.3). The WHO classifies countries into three groups according to their prevalence of chronic HBV infection – “high” means that over 8% of a country’s population are infected, “intermediate” equates to 2–8%, while “low” is 70%), the only really effective form of therapy being a liver transplant.

CHRONIC INFECTION A proportion of patients acutely infected with HBV will fail to eliminate virus from their liver, and therefore become

130

9780367696399_Lydyard.indb 130

13/02/2023 07:32

Case 14: Hepatitis B virus

chronically infected. In chronic infection, virus continues to replicate in infected hepatocytes, and is continually released from these cells into the bloodstream, but without interfering with the normal lifespan of the cells. Such chronic carriers serve as the source of infection for other individuals, via the routes of transmission discussed above. The chances of an acute HBV infection becoming chronic are dependent on a number of factors, most importantly the age and immune status of the patient. Babies infected from their carrier mothers almost always become chronic carriers themselves, reflecting the immaturity of the immune system at birth. About 10% of infected children and 5% of infected adults become chronically infected – in these individuals, it is believed that a failure of the IFN response at the time of acute infection results in chronicity. Immunodeficient patients (e.g. those with HIV infection) are also highly likely to become chronic carriers once infected, as the absence of an effective CTL arm of the immune response mitigates against clearance of HBV-infected hepatocytes. Chronic carriers may not be symptomatic – the liver has a large functional reserve, and liver function may be preserved even although there are virally infected hepatocytes present. However, such individuals are at risk of long-term chronic inflammatory hepatitis, as the infected cells are killed by the host CTL response. The usual response to death of host cells within the liver is the laying down of fibrous tissue. Over time, the continual death of liver cells, and their replacement by fibrous tissue, results in complete loss of normal liver architecture and function – a condition known as cirrhosis of the liver, which has a number of potentially life-threatening complications. The average time from acute infection to cirrhosis is around 20 years. It is estimated that about 20% of chronic HBV carriers will develop cirrhosis and its complications within their lifetimes. One further serious complication of chronic HBV infection is the development of hepatocellular carcinoma (HCC), that is, a malignant proliferation of hepatocytes. There are a number of molecular mechanisms underlying this, including a possible

role for the X protein of HBV, which may interfere with normal cell division regulatory mechanisms. Additionally, HBV DNA may integrate into host cell chromosomes. Dependent on precisely where this integration takes place (it varies between individuals), the control and function of cell cycle regulatory and other genes such as oncogenes may be disturbed, leading to cell division. Chronically infected individuals are about 300 times more likely to develop HCC than individuals who are not infected. Thus, in parts of the world where chronic infection is common, primary liver-cell tumors are one of the commonest forms of malignant disease – globally, this is the third commonest malignant cause of death. On average, HCC appears about 5 years later than cirrhosis. Note that the propensity for HBV infection of babies to become chronic, plus the fact that the life-threatening complications of chronic carriage do not usually become manifest for at least 20 years, provides a rational explanation of the epidemiology of HBV infection. Infected baby girls become chronically infected but reach child-bearing age without yet having suffered the serious consequences of disease. Thus, they in turn infect their babies. Passage of virus from generation to generation in this way results in very high rates of carriage within a population.

4. HOW IS THE DISEASE DIAGNOSED, AND WHAT IS THE DIFFERENTIAL DIAGNOSIS? Laboratory diagnosis of HBV infection depends on the detection of various markers of infection (Table 14.2). Potentially, the laboratory can detect three viral antigens (HBsAg, HBcAg, and HBeAg) and antibodies to these three antigens (anti-HBs, anti-HBc, and anti-HBe, respectively), and the availability of sensitive genome-detection techniques also means that HBV DNA levels can be measured. The first test to be performed is to detect HBsAg in a blood sample. This protein is excreted in vast quantities by infected

Table 14.2 Labratory markers and their interpretation

Status Acute HBV

HBsAg

IgM anti-HBc

IgG anti-HBc

HBeAg

Anti-HBe

Anti-HBs

+

+

+

+

-

-

Cleared HBV infection

-

-

+

-

+

+

Chronic HBV infection, high risk

+

-

+

+

-

-

Chronic HBV infection, low risk

+

-

+

-

+

-

Responder to HBV vaccine

-

-

-

-

-

+

infection

131

9780367696399_Lydyard.indb 131

13/02/2023 07:32

Case Studies in Infectious Disease

liver cells and is easy to detect. The presence of HBsAg in a blood sample means only one thing – on the day the blood sample was taken, the patient was infected with HBV. Of itself, this marker cannot distinguish between acute or chronic infection, as it will be present in both. All patients infected with HBV will make an antibody response to the core antigen (anti-HBc). This marker can be used to distinguish between acute (recent) and chronic infection, as IgM anti-HBc will be present in the former, IgM antibodies being a marker of recent infection. Patients diagnosed with acute infection (HBsAg-positive, IgM anti-HBc-positive) must be followed to see if they eliminate virus over the following 6 months. This will be manifest by the disappearance of HBsAg from the peripheral blood, followed shortly by the appearance of anti-HBs antibodies. If HBsAg persists for longer than 6 months, then the infection has become chronic, by definition. However, not all chronic carriers are equally infectious or equally at risk of chronic liver disease. The HBeAg/anti-HBe system distinguishes between two types of chronic infection. HBeAg­ positive individuals are extremely infectious – HBV DNA levels are usually in excess of 108 IU/ml blood (eAg is derived from the core gene and is essentially a surrogate marker of virus replication). Over time, some HBeAg-positive carriers will lose HBeAg from their blood, as virus replication slows down, and shortly afterward, it is possible to detect anti-HBe. An anti-HBe-positive patient is much less infectious, with HBV DNA levels between 10 and 103 IU/ml. Note that there are mutants of HBV that do not obey the above generalizations, particularly those that have stop mutations within the coding region of e antigen and are therefore incapable of synthesizing this antigen. These viruses may nevertheless be fully replication competent, and a patient infected with such a mutant may have high levels of viral DNA in the absence of HBeAg. Further discussion of these viral mutants is beyond the scope of this text. Anti-HBs arise not only in patients who clear infection, but also in individuals who are vaccinated (see below) – measurement of anti-HBs responses in vaccines is an important marker of protection. HBV DNA is a true marker of viral replication and infectivity. HBV DNA monitoring has become an essential part of the management of patients undergoing antiviral therapy.

DIFFERENTIAL DIAGNOSIS

132

There are many causes of acute hepatitis, and therefore the differential diagnosis of an acutely jaundiced patient is a long one. Acute alcoholic hepatitis can present in an identical fashion. There are also a number of other infectious agents that can damage the liver. In particular, there are a series of viruses that have a tropism for the liver, which are known by letters of the alphabet. Note that these are quite distinct viruses and not related to each other – the only thing they have in common is that they infect the liver.

9780367696399_Lydyard.indb 132

Other Hepatitis Viruses The important features of these viral pathogens are as follows: Hepatitis A virus (HAV) – this is a picornavirus and therefore has a positive single-stranded RNA genome. An infected individual excretes virus in the feces, and therefore the transmission route is fecal–oral through contaminated food or water. The possible outcomes of HAV infection include asymptomatic seroconversion (the majority), acute hepatitis (clinically indistinguishable from acute HBV infection), and fulminant hepatitis (less common than with HBV). Note that there are no chronic sequelae of HAV infection – as patients recover from infection, virus is eliminated from the body and the patient is then immune from further infection. Diagnosis of acute infection is by detection of IgM antibodies against HAV. Hepatitis E virus – also has an RNA genome (with unusual genomic organization – HEV is classified into the hepevirus family) and bears many clinical and epidemiologic similarities to HAV. Spread is via the fecal–oral route, and huge outbreaks involving thousands of individuals have been reported from fecal contamination of water supplies in various countries. Mortality from acute HEV is greater than with acute HAV, especially in pregnant women (>10%). Chronic carriage of HEV does not arise in immunocompetent hosts but may occur if the patient is immunodeficient. Diagnosis of acute infection is by detection of IgM antibodies against HEV, and/ or detection of HEV RNA in a blood or feces sample. Hepatitis C virus – this virus is discussed in detail in a separate case. Hepatitis D virus – this is an incomplete virus. It has a small RNA genome, encoding its capsid protein, referred to as the delta antigen. However, it requires an outer protein coat and it uses HBV as a helper virus to provide it with HBsAg to enable cellular egress and entry. Thus, HDV infection can only occur in patients already infected with HBV (superinfection), or at the same time as acute HBV infection (co-infection). The risk groups for HDV infection are therefore essentially the same as for HBV infection. Co-infection increases the risk of acute fulminant hepatitis, while patients who become chronic carriers of both HBV and HDV are at increased risk of the development of serious liver disease. Diagnosis is by the detection of delta antigen and antibodies to delta antigen, and chronicity of infection by detection of HDV RNA.

5. HOW IS THE DISEASE MANAGED AND PREVENTED? MANAGEMENT

There is no specific treatment for acute hepatitis B infection – advice is mainly for bed rest and avoidance of alcohol (a potent liver toxin). In cases of fulminant hepatitis, liver transplantation may be the only resort. Patients with chronic HBV infection should be advised that they are potentially infectious to others, particularly

13/02/2023 07:32

Case 14: Hepatitis B virus

their sexual partners and close household contacts. Under no circumstances should they donate blood, or share razors or toothbrushes. Blood spillages (e.g. from cuts) should be appropriately dealt with. The management of patients with chronic hepatitis B has changed dramatically over the last 15 years, as the convoluted replication cycle of the virus has been elucidated. In particular, the recognition that there is a reverse transcription (RT) step in this process has led to the use of RT inhibitors in chronic hepatitis B, the relevant drugs being developed initially as RT inhibitors for the treatment of patients with HIV infection. Options for therapy (see Table 14.3) therefore now include the following: 1. Interferon-alpha

(IFN- ) – this acts as an immunomodulatory drug in this context, not as an antiviral. It induces the expression of class I HLA molecules on the hepatocyte surface, and therefore facilitates recognition of virally infected cells by circulating virus-specific CTLs, which kill the infected hepatocytes and thereby reduce the burden of infected cells within the liver. This mechanism will only be effective in patients with an intact immune system whose T cells are capable of cytotoxic activity against HBV-infected hepatocytes – thus, it is unlikely to work in immunodeficient patients, or in patients infected with HBV at birth, in whom immunologic tolerance is induced such that their immune system does not recognize the virus as foreign. IFN- is given as a 6–12-month course of thrice-weekly injections. More recently, preparations of IFN- linked to polyethylene glycol have become available (pegylated- or PEG-IFN). This has a longer half-life than standard IFN, and therefore only requires once-weekly injection. The longer maintenance of plasma levels over time with PEG-IFN is also associated with a better therapeutic activity. IFNs may induce a plethora of unwanted side effects. Muscle aches, fatigue, and headache are common, but most patients become tolerant to these effects. Bone marrow suppression (low white cell count) and psychiatric manifestations such as depression and even suicide are more worrying. 2. HBV DNA polymerase inhibitors – these are collectively referred to as nucleos(t)ide analogs. Currently licensed ones in the UK include lamivudine (3TC, 3-thia-cytidine), tenofovir (as this molecule has a phosphate group attached, it is a nucleotide analog), and entecavir. These are effective against the polymerase activity of HBV, and therefore directly suppress viral replication. Note that their use does not eliminate virally infected hepatocytes, and therefore viral relapse after cessation of therapy is common. As with HIV, there is also a risk that mutations may arise within the viral polymerase (Pol) gene, which confer resistance to these drugs. Around 75% of patients treated with

lamivudine for more than 3 years will have such drugresistant mutants, but resistance rates to tenofovir and entcavir are substantially lower. Management of patients with chronic HBV infection is complex and should be performed by physicians with appropriate training and expertise (hepatologists). Difficulties include when and in whom to initiate therapy, with what therapeutic modality (IFN- or nucleoside analog-based), and when is it safe to stop therapy. Suppression of viral replication, as evidenced by reduction of peripheral blood HBV DNA, is a valuable therapeutic endpoint, as it is associated with the lowering of risk of progression of disease to the life-threatening complications of cirrhosis and HCC development. However, current research is aimed at the development of drugs acting in different ways to not only reduce viral replication but also eliminate virus from infected hepatocytes and thereby achieve cure. These novel approaches include entry inhibitors, small RNA molecules that interfere with viral-derived mRNA, drugs which interfere with capsid formation or viral exit from cells, and immunomodulatory drugs such as Toll-like receptor (TLR)-7 agonists designed to encourage the host immune system to eliminate infected cells.

PREVENTION HBV infection can be prevented by means of vaccination, using the HBsAg as the immunizing antigen. Original vaccines consisted of HBsAg purified from human plasma, and plasma-derived vaccines may still be used in some parts of the world. However, in Europe and the US, the current vaccines are subunit vaccines prepared by recombinant DNA technology. The gene encoding the HBsAg has been cut out of the virus and inserted into a yeast. As the yeast replicates in culture, large amounts of HBsAg are synthesized within the cytoplasm of the cell. Cell lysis followed by purification yields HBsAg, which is administered by intramuscular injection (usually as a course of three injections given at 0, 1, and 6 months). As the only part of the virus present in the vaccine is Table 14.3 List of agents available for treatment of patients with chronic HBV infection Interferons Standard interferon-alpha Pegylated interferon-alpha Reverse transcriptase inhibitors Lamivudine Adefovir Entecavir Telbivudine Tenofovir

133

9780367696399_Lydyard.indb 133

13/02/2023 07:32

Case Studies in Infectious Disease

HBsAg, the response to the vaccine can be easily measured by quantifying the anti-HBs in the vaccinee 6–8 weeks after the last dose. Protection against HBV infection is proportional to the amount of anti-HBs produced. Around 5–10% of adults do not mount an anti-HBs response to this vaccine and therefore remain susceptible to infection. A further 5–10% will make a rather weak response, which may offer only limited protection. Having developed an effective vaccine, undoubtedly a major medical advance, the question arises as to who should be vaccinated. The WHO unequivocally recommends that the vaccine be included as one of the routine immunizations in childhood, and almost all countries have now adopted this approach, although the UK only switched from a selective policy targeted at identifiable risk groups (see Table 14.3), to a universal policy as recently as 2017.

Vaccines for Other Hepatitis Viruses Note that there is no effective antiviral agent against HAV, but there is a vaccine – heat-killed whole virus. This is currently offered to individuals at risk of infection, for example, travelers to countries where HAV infection is endemic. There is currently no effective antiviral agent or licensed vaccine against HEV, although there are experimental vaccines undergoing clinical trials that have shown initial promise. Treatment and prevention of HCV infection is discussed in a separate case (see Case 14). There is currently no licensed effective antiviral treatment for HDV but, as with HBV, there are promising new drugs in the pipeline such as bulevirtide, which blocks HBsAg from binding to the NTCP receptor. However, successful vaccination against HBV also prevents infection with HDV.

SUMMARY 1. WHAT IS THE CAUSATIVE AGENT, HOW DOES IT ENTER THE BODY, AND HOW DOES IT SPREAD A) WITHIN THE BODY AND B) FROM PERSON TO PERSON?

■ Acute infection may be asymptomatic, that is there is insufficient

■ Hepatitis B virus is a hepadnavirus.

■ Patients may present with acute hepatitis – initially (pre­ icteric) the

■ Entry is via the genital tract, or through direct inoculation of the virus into the bloodstream. ■ Once virus has entered hepatocytes, viral replication results in release of new virus particles directly into the bloodstream, and from there into every body compartment. ■ The three routes of transmission are mother to baby (infected birth canal, exposure to infected maternal blood); sexual; and parenteral, for example, via contaminated needles.

2. WHAT IS THE HOST RESPONSE TO THE INFECTION AND WHAT IS THE DISEASE PATHOGENESIS? ■ IFN production within the liver renders neighboring noninfected cells relatively resistant to infection. ■ Exposure to viral antigens leads to the development of both humoral and cellular responses. ■ Cytotoxic CD8+ T lymphocytes are crucial in eliminating virally infected hepatocytes. ■ Antibody against the surface protein (anti-HBs) is vital to enable neutralization of released virus particles. ■ An inadequate immune response results in failure to eliminate the virus from the liver. The patient then becomes chronically infected. ■ HBV is not a cytolytic virus. Chronic liver damage arises from the host inflammatory response, particularly CD8+ T-cell destruction of hepatocytes.

3. WHAT IS THE TYPICAL CLINICAL PRESENTATION AND WHAT COMPLICATIONS CAN OCCUR? damage to the liver to cause disease. features are nonspecific – fever, malaise, lethargy, anorexia, right upper quadrant abdominal pain. ■ Liver damage results in failure to excrete pigments derived from hemoglobin, resulting in dark urine, pale stools, and clinical jaundice. ■ Liver damage may be so overwhelming that the patient goes into liver failure – fulminant hepatitis. ■ Recovery from acute hepatitis does not always equate with clearance of infection: 90% of neonates, 10% of children, and 5% of adults will become chronically infected (carriers). ■ Chronically infected individuals may be healthy or may suffer varying degrees of chronic inflammatory liver disease, leading eventually (i.e. after 20+ years) to cirrhosis of the liver. ■ Carriers are also at a 300-fold increased risk of developing hepatocellular carcinoma.

4. HOW IS THE DISEASE DIAGNOSED, AND WHAT IS THE DIFFERENTIAL DIAGNOSIS? ■ There are a number of laboratory markers of HBV infection. ■ The presence of HBsAg in a serum sample indicates that HBV infection is present but does not distinguish between acute and chronic infection. ■ The presence of IgM anti-HBc indicates recent infection. ■ Chronic infection is defined as the persistence of HBsAg for more than 6 months. Continued...

134

9780367696399_Lydyard.indb 134

13/02/2023 07:32

Case 14: Hepatitis B virus

Continued...

■ Chronic carriers can be split into those who are highly infectious or those much less infectious on the basis of the HBeAg/antiHBe system. ■ There are mutants of HBV in which HBeAg testing is misleading, with high levels of viral DNA in the absence of HBeAg. ■ Measurement of HBV DNA levels is useful in monitoring the effectiveness of antiviral therapy. ■ There are many causes of acute hepatitis. Of particular relevance, a number of distinct viruses have a hepatic tropism. ■ HAV is a picornavirus, is spread by the fecal–oral route, causes asymptomatic, acute, and fulminant hepatitis, but does not result in chronic disease. ■ HEV is a hepevirus, spread via the fecal–oral route, and has a higher mortality than HAV in the acute stage. Chronic infection only arises in immunodeficient individuals. ■ HCV is a flavivirus, spread via blood–borne routes, and usually results in chronic infection, with consequent risks of cirrhosis and hepatocellular carcinoma. ■ HDV is an incomplete virus that requires the presence of HBV in order to replicate. This virus often results in chronic infection with an accelerated progression of liver disease.

FURTHER READING Barer M, Irving W, Swann A, Perera N. Medical Microbiology, 19th edition. Elsevier, Philadelphia, 2019. Humphreys H, Irving WL, Atkins BL, Woodhouse AF. Oxford Case Histories in Infectious Diseases and Microbiology, 3rd edition. Oxford University Press, Oxford, 2020. Murphy K, Weaver C. Janeway’s Immunobiology, 9th edition. Garland Science, New York/London, 2016. Richman DD, Whitley RJ, Hayden FG. Clinical Virology, 4th edition. ASM Press, 2016.

REFERENCES Castaneda D, Gonzalez AJ, Alomari M, et al. From Hepatitis A to E: A Critical Review of Viral Hepatitis. World J Gastroenterol, 27: 1691–1715, 2021. Lin S, Zhang Y-J. Advances in Hepatitis E Virus Biology and Pathogenesis. Viruses, 13: 267, 2021. Nguyen MH, Wong G, Gane E, et al. Hepatitis B Virus: Advances in Prevention, Diagnosis, and Therapy. Clin Microbiol Rev, 33: e00046–19, 2020. Philips CA, Ahamed R, Abduljaleel JK, et al. Critical Updates on Chronic Hepatitis B Virus Infection in 2021. Cureus, 13: e19152, 2021.

5. HOW IS THE DISEASE MANAGED AND PREVENTED? ■ Management of acute infection is symptomatic only. Fulminant hepatitis may necessitate liver transplantation. ■ Carriers of HBV should be educated about their condition and advised as to how to prevent transmission. ■ Chronic HBV infection may be treated with IFN- , which acts as an immunomodulatory agent, or with HBV polymerase inhibitors such as lamivudine, tenofovir, and entecavir. ■ Long-term therapy may result in the emergence of resistant viral mutants. ■ Prevention is by means of a subunit vaccine consisting of purified HBsAg, given as a course of three intramuscular injections. ■ The WHO recommends universal vaccination in childhood. ■ Vaccine-induced protection is related to the titer of anti-HBs produced. Not all vaccinees generate adequate anti-HBs responses. ■ HAV can be prevented by vaccination using a killed whole virus vaccine, currently offered only to high-risk groups.

Roberts H, Ly KN, Yin S, et al. Prevalence of HBV Infection, Vaccine‐Induced Immunity, and Susceptibility Among At‐ Risk Populations: US Households, 2013‐2018. Hepatology, 74: 2353–2365, 2021.

WEBSITES All the Virology on the WWW, Specific Virus Information, 1995: http://www.virology.net/garryfavweb12.html#Hepad Centers for Disease Control and Prevention, Viral Hepatitis, 2021: https://www.cdc.gov/hepatitis/index.htm GOV.UK, Guidance: Hepatitis B: the green book, chapter 18, 2013: https://www.gov.uk/government/publications/hepatitis­ b-the-green-book-chapter-18 Virology Online, Hepatitis B: http://virology-online.com/ viruses/HepatitisB.htm World Health Organization, Hepatitis B, 2022: https:// www.who.int/news-room/fact-sheets/detail/hepatitis-b

Students can test their knowledge of this case study by visiting the Instructor and Student Resources: [www. routledge.com/cw/lydyard] where several multiple choice questions can be found.

135

9780367696399_Lydyard.indb 135

13/02/2023 07:32

15

Hepatitis C virus

A 25-year-old man attends a needle exchange service to access a supply of clean needles and syringes in order to support his drug injecting habit. As he is provided with a pack of equipment, he is advised to be tested for evidence of any blood–borne virus infection that he might have acquired during his injecting career. He accepts this advice, and a fingerprick blood sample is taken and sent to the laboratory. On his next visit to the needle exchange, one month later, he is given the results by a specialist hepatitis nurse attending the drug treatment centre (Figure 15.1). The nurse persuades him to have a venous blood sample taken, to confirm these initial results, quantify the HCV viral load, determine the HCV genotype and check on his liver function. She also performs a transient elastography test, which indicates a liver stiffness of 6.3 kPa. Subsequent results indicate he has chronic HCV infection, genotype 3, with a raised alanine aminotransferase. The specialist nurse arranges for him to receive a total of 12 weeks oral antiviral therapy. Three months after he has finished his course of treatment, a repeat blood test shows the absence of HCV RNA.

1. WHAT IS THE CAUSATIVE AGENT, HOW DOES IT ENTER THE BODY AND HOW DOES IT SPREAD A) WITHIN THE BODY AND B) FROM PERSON TO PERSON? CAUSATIVE AGENT

Hepatitis C virus (HCV) is an enveloped virus containing positive single-stranded RNA of around 9.2 kb. When first characterized in 1989, HCV was classified as a flavivirus, i.e. a very distant relative of yellow fever virus, on the basis of its genome architecture and similarities in the structure of the polymerase enzymes encoded by the two viruses. However, viruses from many different species having much closer similarity to HCV have since been discovered. Currently, these are classified into the genera hepaciviruses and pegiviruses. Many differences between these viruses and classical flaviviruses have now been described, and it is possible these genera will, in time, be reclassified into their own family. The HCV genome was first identified by molecular biologists in California in 1989. Shortly afterward, a Japanese group also sequenced a full-length HCV, but this showed around 15% difference in nucleotide sequence to the Californian report. In fact, HCV is extremely genetically diverse – 10-fold more

9780367696399_Lydyard.indb 137

Figure 15.1 Laboratory results.

so than HIV. This has led to the classification of hepatitis C viruses into eight distinct genotypes, numbered 1–8, with >30% difference in nucleotide sequence between genotypes. Within genotypes, subtypes, with more than 15% sequence difference, are labeled with letters of the alphabet – currently nearly 100 different subtypes have been identified. Even within a patient, the virus exists as a quasispecies, i.e. a series of closely related, but not identical sequences. The genome structure of HCV is shown in Figure 15.2. The 5’ non-coding region (NCR) contains a binding site for micro-RNA 122 (miRNA-122, the most abundant miRNA in hepatocytes) which is essential for virus replication. It also contains an internal ribosomal entry site (IRES) so the positive sense RNA can be translated into proteins in a capindependent manner. Three structural (i.e. present in mature viral particles) viral proteins are encoded at the 5’ end of the genome – a core protein which surrounds and protects the RNA genome, and two envelope glycoproteins which are inserted into the lipid envelope. The 3’ end of the genome encodes a number of non-structural (NS) proteins including two protease enzymes (NS2, and an NS3/4a complex), and an RNA polymerase (NS5b) enzyme. NS4b forms a membranous web within an infected cell which acts as a scaffold for virus replication, while NS5b interacts with a number of host proteins and is involved in viral replication and egress from the cell.

137

13/02/2023 07:32

Case Studies in Infectious Disease

kilobases 0

9.5 structural genes

NS genes

UTR

3’UTR SS + RNA Core capsid

E1

E2

envelope glycoproteins

P7

NS2

NS3

NS2/3 protease component

protease, helicase, NTPase

viroporin

(in vitro only)

NS4A

NS4B

NS5A

membranous web formation NS3 protease cofactor

NS5B RNA-dependant

RNA-polymerase

viral assembly and egress

Figure 15.2 The genome structure and encoded proteins of HCV. UTR = untranslated region; E = envelope; NS = non-structural.

The replication cycle of HCV starts with binding of virus particles to cell surface molecules such as heparan sulphate and the low-density lipoprotein receptor. Further interaction with a number of other receptor molecules including CD81, scavenger receptor B1, claudin-1 and occludin results in virus entry into hepatocytes via endocytosis and fusion of viral and clathrin-coated endosomal membranes. In addition to translation of the genome into viral proteins, full-length anti­ sense copies of the genome are synthesized, which act as a template for the synthesis of multiple positive-strand RNA copies. Replication takes place within a membrane-associated replication complex within a double-membrane vesicle (DMV). New viral particles bud through the endoplasmic reticulum and egress from the cell as a lipoprotein-associated “lipoviral particle” (LVP) via the Golgi secretory pathway.

ENTRY AND SPREAD WITHIN IN THE BODY Acquisition of HCV infection requires inoculation of infected blood. Once virus particles have entered the bloodstream, they gain access to the liver. Hepatocytes express all of the viral receptors listed above, hence the observed hepatotropism of this virus. There is little evidence that HCV is able to replicate in any other cell type. New virus particles formed within Case_14_2 hepatocytes are released into the bloodstream, rendering that person a potential infection source to any contacts exposed to their blood. While it is impossible to prove that HCV cannot be transmitted sexually, in marked contrast to other blood– borne viruses (hepatitis B and HIV) there is no increased rate of HCV infection in groups such as commercial sex workers, individuals with multiple sexual partners, men who have sex with men, who traditionally are at increased risk of a sexually transmitted infection. Mother-to-baby (vertical) transmission of HCV is also unusual – only around 1% of babies of HCVinfected mothers acquire infection. Again, this is in marked contrast to HBV (>90%) and HIV (20–25%).

PERSON-TO-PERSON SPREAD

Virus is present within the bloodstream of patients with chronic HCV infection. Infection therefore requires exposure to infected blood. This can occur through a variety of means. Most obviously, transfusion of blood or blood products derived from an HCV-infected donor will result in transmission. In the past, this led to a significant number of infections, particularly in certain groups of patients such as hemophiliacs who require factor VIII replacement. However, introduction of blood-donor screening since 1989 has reduced such transmission to very small numbers worldwide. More subtle blood-to-blood exposure can occur through sharing of contaminated needles and syringes by people who inject drugs, contaminated body- or ear-piercing needles, and acupuncture needles. Healthcare workers are at increased risk of HCV infection through accidental needlestick injuries from HCV-infected patients. Any re-usable hypodermic needles which are not properly sterilized between use can potentially transmit HCV infection.

EPIDEMIOLOGY The WHO estimate there are in excess of 70 million individuals with chronic HCV infection globally. These are unevenly spread around the world, and the underlying drivers of HCV epidemics are different in different countries. Thus, in the UK (0.5% of the population), the epidemic has been driven by recreational injecting drug use – the majority of HCVinfected individuals give a history of having injected drugs, maybe decades ago. In contrast, in Egypt, with a prevalence approaching 20%, the epidemic was driven by bilharzia eradication campaigns in the 1960s to 1980s which involved the injection of an anti-Schistosoma drug with inadequately sterilized re-usable needles and syringes. This “medically­ driven” means of transmission has also been a major route of HCV transmission in a number of other countries.

138

9780367696399_Lydyard.indb 138

13/02/2023 07:32

Case 15: Hepatitis C virus

East Europe

UK

1b, 3a North America

1a, 1b, 2a, 2b, 3a

South Europe

1b, 2a, 3a

1b, 2a/c 1b 4

Central America

1, 2

1a, 1b

North-east Africa

West Africa

1b

South America

Japan

3a, 3b 6

India

Indonesia

5a

1b

China

South-east Asia

3b

Australia

1b,3a

South Africa

Figure 15.3 Distribution of the major genotypes of hepatitis C virus around the world. Adapted from a figure courtesy of Dr Alexander Tarr.

The virus has clearly evolved into different genotypes within different host populations around the world. This is reflected in different genotype distributions in different countries – see Figure 15.3.

2. WHAT IS THE HOST RESPONSE TO

THE INFECTION AND WHAT IS THE

DISEASE PATHOGENESIS? INNATE IMMUNITY The presence of HCV in hepatocytes activates a number of pathogen recognition receptors, including Toll-like receptor 3, protein kinase R (PKR), retinoic acid-inducible gene 1 (RIG-I) and melanoma differentiation-associated protein 5 (MDA5), leading to activation of transcription factors such as interferon regulatory factor 3 (IRF3) and nuclear factor kappa B (NF-κB) which, in turn, enhance expression of interferon genes. In chronic infection, this results in a state of continuous innate immune activation, with increased expression of pro­ Case_14.3 inflammatory gene expression which will encourage the development of tissue fibrosis. Activation of innate immune cells such as NK cells and liver-resident macrophages (Kupfer cells) also contribute to this process. HCV has evolved a number of ways of subverting this response – intracellular viral replication takes place within a double membrane (comprised of the NS4b viral protein) bound vesicle thereby “hiding” away within the cell; the NS3/NS4a protease enzyme cleaves mitochondrial antiviral signaling (MAVS) protein, a key protein within the RIG-I signaling pathway; HCV also interferes with the downstream

signaling actions of interferons via STAT1, STAT2, IRF9, and JAK-STAT pathways. Host genetic studies have identified a number of single nucleotide polymorphisms (SNPs) within the interferon lambda (IFNL) locus which are associated with the likelihood of spontaneous clearance of infection, and also the likelihood of clearance of chronic infection following treatment with interferon- . These SNPs alter the activity of interferon-λ4, although the precise mechanisms linking the functionality of IFN-λ4, and viral clearance have yet to be elucidated.

ADAPTIVE IMMUNITY HCV infection stimulates production of antibodies, including those directed against the two surface envelope glycoproteins (E1 and E2). Most antibodies are directed at a region of the E2 protein named hypervariable region 1 (HVR1). The name reflects the fact that mutations in this part of the coding gene are common and lead to amino-acid changes in the protein, this being one way in which HCV evades the adaptive humoral response. Only a minority of anti-E2 antibodies are able to neutralize viral binding to target hepatocytes. Generation of broadly neutralizing antibodies (Nabs), defi ned as antibodies that can neutralize cell-culture infection with viruses of more than one genotype during acute infection, is associated with viral clearance. CD4+ and CD8+ T-cell responses are also generated during infection. In patients with chronic infection, these T-cell responses are weak or undetectable, whereas in patients who have achieved spontaneous clearance, T-cell responses are stronger and more broadly targeted (i.e. recognize a number of epitopes in different viral proteins). 139

9780367696399_Lydyard.indb 139

13/02/2023 07:32

Case Studies in Infectious Disease

PATHOGENESIS HCV infection of hepatocytes is not cytolytic – cell death arises from host immune responses, both innate and adaptive. Alcohol is a potent hepatotoxin and acts synergistically with HCV to cause liver damage – patients should therefore be advised to abstain, at least until their HCV infection is cleared. Age does not appear to influence spontaneous clearance of infection, but older age at infection is associated with more progressive liver disease. All the life-threatening complications of chronic infection can arise with any genotype. There is some evidence that disease may progress more rapidly with genotype 3 infection, which is also associated with more fatty liver disease than any of the other genotypes.

3. WHAT IS THE TYPICAL CLINICAL PRESENTATION AND WHAT COMPLICATIONS CAN OCCUR? ACUTE INFECTION The majority of acute HCV infections are asymptomatic – it is estimated that only about 15% will present with an acute hepatic illness. Initial symptoms of acute hepatitis are nonspecific – lethargy, fever, nausea, loss of appetite, with the only localizing symptom being right upper quadrant abdominal pain as the inflamed liver swells inside its innervated capsule. Damage to liver cells may impact on liver function, with resultant generation of jaundice, dark urine and pale stools (see Case 13). While theoretically possible, acute liver damage sufficient to cause liver failure – fulminant hepatitis – is rare. However, 50–75% of patients will fail to clear infection at the acute stage and will therefore progress to chronic infection (defi ned as persistence of HCV RNA for more than 6 months). Female sex, and the presence of a vigorous immune response

sufficient to induce symptomatic disease (jaundice) are factors associated with an increased chance of viral clearance. In contrast to HBV, age does not appear to be a determinant of clearance/chronicity.

CHRONIC INFECTION During chronic infection, there is continual virus replication and release of new virus particles into the bloodstream. Death of hepatocytes may arise through the detrimental effects of virus replication going on in the cell, or through immune recognition of infected cells by NK and cytotoxic T cells. The repair mechanism in the liver is to replace dead hepatocytes by fibrous tissue. Over many years, this process results in the generation of cirrhosis (see Case 13) with all its attendant lifethreatening complications. Also similar to HBV, patients with chronic HCV infection are at hugely increased risk of developing primary liver cell tumors (hepatocellular carcinoma, HCC). For HCV, this appears to be a consequence of the liver becoming cirrhotic – HCV does not possess an equivalent protein to the HBX protein, and also has no DNA intermediate which might integrate into the host cell chromosomes. It is estimated that between them, chronic HBV and HCV infections account for over 75% of the global burden of HCC, which is the third commonest cause of death from malignant tumors.

COMPLICATIONS In addition to chronic hepatitis, cirrhosis, and HCC, chronic HCV infection is also associated with a number of extra-hepatic complications (Figure 15.4), most notably mixed cryoglobulinemia. A cryoglobulin is a protein which precipitates out at cold temperatures. Circulating cryoglobulins in HCV infection include immune complexes of HCV and anti-HCV. Precipitation in blood-vessel walls at cold temperatures stimulates a vasculitis which typically affects

Alterations in glucose and lipid metabolism HCV

Cryoglobulinemic vasculitis, often in glove and stocking distribution

Endothelial damage and lipid accumulation leading to cardiovascular complications

B-cell lymphoproliferative diseases

Renal insufficiency from immune complex deposition

Renal insufficiency from immune complex deposition

Figure 15.4 Possible extrahepatic manifestations of chronic hepatitis C virus infection.

140

9780367696399_Lydyard.indb 140

13/02/2023 07:32

Case 15: Hepatitis C virus

Table 15.1 Risk factors for HCV infection for patients in the UK Current or past history of recreational drug use. People in prison (more likely to have history of injecting drug use). Recipients of blood (before Sept 1991) or blood product (before 1986) transfusion. UK recipients of organ or tissue transplants before 1992. People who have lived or had medical treatment in countries where HCV infection is common. Babies and children of HCV-infected mothers. Recipients of tattoos or piercing where equipment may not have been properly sterilized. Sexual partners, family members and close household contacts of people with HCV.

hands and feet – a so-called “glove and stocking” distribution, due to the lower temperatures of those limb extremities. Deposition in the kidneys can result in glomerulonephritis. There is also evidence that chronic HCV infection may be causally associated with some types of lymphoma. Genotype 3 infection specifically alters lipid metabolism within the liver and may predispose to insulin resistance.

4. HOW IS THE DISEASE DIAGNOSED, AND WHAT IS THE DIFFERENTIAL DIAGNOSIS? Any patient with risk factors for HCV infection (Table 15.1) should be tested for antibodies to HCV (anti-HCV). This is a straightforward assay usually done in an enzyme-linked immunosorbent assay format. Anti-HCV positivity means that the patient has, at some stage, been infected with HCV, but gives no indication as to when this might have occurred, or, more importantly, whether the patient succeeded in clearing the infection or became chronically infected. A subsequent test for HCV RNA (i.e. a genome detection test, usually the polymerase chain reaction [PCR] assay) distinguishes those individuals who have cleared the infection (HCV RNAnegative) from those who are chronic carriers (HCV RNApositive). The latter should then be further tested to determine which particular genotype and subtype the patient is infected with as this will, in turn, determine which is the optimal form of antiviral therapy. Genotyping is done by sequencing all or part of the viral genome and comparing the sequence with standard databases to identify which geno/sub-type the virus is most closely related to.

DIFFERENTIAL DIAGNOSIS

There is a long list of differential diagnoses of acute jaundice, including acute alcoholic hepatitis, and infection with one of many other hepatitis viruses – see Case 13 for diagnosis of acute HBV, HAV, HEV, and HDV. Similarly, there are many causes of chronic liver damage and cirrhosis of the liver. Common etiologies of cirrhosis include chronic HBV or HCV infection, alcoholic liver disease, non-alcoholic fatty liver disease, hemochromatosis, Wilson’s disease, and auto-immune hepatitis. It is beyond the scope of this text to go into detail about how each of these conditions is diagnosed.

5. HOW IS THE DISEASE MANAGED AND PREVENTED? MANAGEMENT

All patients with HCV infection should be informed about the potential synergistic liver-damaging effects of alcohol and advised to abstain if possible. The treatment of chronic HCV infection has undergone a revolution in the past 10 years. Initially, therapy consisted of a combination of interferon and ribavirin given for 6 or 12 months (depending on genotype). This was a difficult therapy for patients to comply with, and had considerable side effects and a cure rate overall of only around 50%. However, the development of directly acting antiviral agents (DAAs) targeted at the products of the viral NS genes NS3 (a protease enzyme), NS5a (a protein essential for several steps in the

Table 15.2 Combination therapies for the treatment of HCV infection Drug name

NS3 inhibitor

Harvoni

NS5a inhibitor

NS5b inhibitor

Ledipasvir

Sofosbuvir

Zepatier

Grazoprevir

Elbasvir

Maviret

Glecaprevir

Pibrentasvir

Epclusa Vosevi

Voxilaprevir

Velpatasvir

Sofosbuvir

Velpatasvir

Sofosbuvir

141

9780367696399_Lydyard.indb 141

13/02/2023 07:32

Case Studies in Infectious Disease

viral replication cycle), and NS5b (the viral RNA polymerase enzyme) has allowed the abandonment of interferon-based therapy. The NS3/4a inhibitors have the suffix -previr, the NS5a inhibitors have the suffix -asvir, and the NS5b inhibitors are -buvir. Some of these NS proteins are quite variable between different genotypes of the virus, and therefore some DAAs are genotype-specific, while others are pan-genotypic. Combination therapy, chosen on the basis of which geno/ subtype the patient is infected with, with two or more classes of DAA administered as tablets once a day for 8–12 weeks has been shown to achieve cure of chronic HCV infection in over 95% of patients, and is associated with very few adverse events. Commonly used combinations of DAAs marketed in single tablets are shown in Table 15.2.

PREVENTION It has proven extremely difficult to develop an effective vaccine for HCV. The major challenge is the huge amount of genetic diversity such that vaccine candidates may protect

experimental animals against infection with virus of the same genotype, but not against other genotypes. There has been only one large-scale phase 3 trial of a candidate vaccine, which had no demonstrable effect on the rate of infection in people who inject drugs in Baltimore. The story of HCV, from identification in 1989 to the development of drugs with almost 100% cure rates 25 years later, is a triumph of modern science and medicine – and has duly been recognized as such by the award of the 2020 Nobel prize for Medicine or Physiology to three of the major players, Harvey Alter, Charles Rice, and Michael Houghton. The outstanding success of DAA therapy has contributed to the notion that it may be possible to eradicate HCV altogether. While the WHO has declared a target of 2030 for the elimination of HCV as a public health hazard, and many countries have therefore adopted elimination strategies based around increased diagnosis and better linkage to treatment for infected individuals, it is unlikely that eradication will be achieved in the absence of a prophylactic vaccine.

SUMMARY 1. WHAT IS THE CAUSATIVE AGENT, HOW DOES IT ENTER THE BODY, AND HOW DOES IT SPREAD A) WITHIN THE BODY AND B) FROM PERSON TO PERSON?

■ The majority of infections become chronic, with continuous viral

■ Hepatitis C virus belongs to the Flaviviridae family, with a positive ssRNA genome. ■ Virus replicates in the liver and is released into the bloodstream. ■ Transmission is via contaminated blood, blood products or needles and syringes. ■ Sexual transmission is unusual, as is mother-to-baby (around 1%).

■ Chronic infection with cirrhosis carries a significant risk of

2. WHAT IS THE HOST RESPONSE TO THE INFECTION AND WHAT IS THE DISEASE PATHOGENESIS? ■ Recognition of pathogen-associated molecular patterns by pathogen-recognition receptors triggers potent innate immune responses, dominated by production of interferon and interferonresponsive gene products. ■ Clearance of virus at the acute stage is associated with certain SNPs in the IFN-λ4 gene. ■ Humoral and cellular adaptive immune responses are also critical in determining the outcome of acute infection. Viral clearance is associated with generation of broadly neutralizing antibodies and broad and potent T-cell responses. ■ Liver cell death is mediated by host inflammatory responses. ■ Hepatocyte loss leads to replacement with fibrous tissue, leading eventually to cirrhosis.

3. WHAT IS THE TYPICAL CLINICAL PRESENTATION AND WHAT COMPLICATIONS CAN OCCUR? ■ Most acute infections are asymptomatic; 15% of cases present as an acute jaundice.

replication, stimulating immune-mediated liver cell death. ■ Loss of hepatocytes leads to replacement of the normal liver architecture by fibrous tissue, leading to cirrhosis. development of hepatocellular carcinoma. ■ Chronic HCV infection is also associated with mixed essential cryoglobulinemia, glomerulonephritis, and development of some types of lymphoma.

4. HOW IS THE DISEASE DIAGNOSED, AND WHAT IS THE DIFFERENTIAL DIAGNOSIS? ■ The primary diagnostic test is for anti-HCV by ELISA. ■ There are many causes of acute jaundice including infection with other hepatitis viruses and acute alcoholic hepatitis. ■ There are many causes of cirrhosis of the liver, including chronic HBV infection, alcoholic liver disease, and non-alcoholic fatty liver disease. ■ Anti-HCV positive samples should be further tested for HCV RNA by a genome amplification assay.

5. HOW IS THE DISEASE MANAGED AND PREVENTED? ■ Treatment of HCV infection consists of combination therapy with two or more directly acting antiviral agents targeting key viral proteins (NS3, NS5a or NS5b). Such therapy is successful in over 95% of patients. ■ There is currently no vaccine available for prevention of HCV infection.

142

9780367696399_Lydyard.indb 142

13/02/2023 07:32

Case 15: Hepatitis C virus

FURTHER READING Barer M, Irving W, Swann A, Perera N. Medical Microbiology, 19th edition. Elsevier, Philadelphia, 2019. Humphreys H, Irving WL, Atkins BL, Woodhouse AF. Oxford Case Histories in Infectious Diseases and Microbiology, 3rd edition. Oxford University Press, Oxford, 2020. Murphy K, Weaver C. Janeway’s Immunobiology, 9th edition. Garland Science, New York/London, 2016. Richman DD, Whitley RJ, Hayden FG. Clinical Virology, 4th edition. ASM Press, New York, 2016.

REFERENCES Castaneda D, Gonzalez AJ, Alomari M, et al. From Hepatitis A to E: A Critical Review of Viral Hepatitis. World J Gastroenterol, 27: 1691–1715, 2021. Dennis BB, Naji L, Jajarmi Y, et al. New Hope for Hepatitis C Virus: Summary of Global Epidemiologic Changes and Novel Innovations Over 20 Years. World J Gastroenterol, 27: 4818–4830, 2021. European Association for the Study of the Liver. EASL Recommendations on Treatment of Hepatitis C: Final Update of the Series. J Hepatol, 73: 1170–1218, 2020.

Mazzaro C, Quartuccio L, Adinolfi LE, et al. A Review on Extrahepatic Manifestations of Chronic Hepatitis C Virus Infection and the Impact of Direct-Acting Antiviral Therapy. Viruses, 13: 2249, 2021. Schwerk J, Negash A, Savan R, Gale M. Innate Immunity in Hepatitis C Virus Infection. Cold Spring Harb Perspect Med, 11: a036988, 2021.

WEBSITES All the Virology on the WWW Website, Specific Virus Information: http://www.virology.net/garryfavweb12.html# Hepad Centers for Disease Control and Prevention, Viral Hepatitis, 2021: https://www.cdc.gov/hepatitis/index.htm Virology Online, Hepatitis B: http://virology-online.com/ viruses/HepatitisB.htm World Health Organisation, Hepatitis C, 2022: https:// www.who.int/news-room/fact-sheets/detail/hepatitis-c Students can test their knowledge of this case study by visiting the Instructor and Student Resources: [www. routledge.com/cw/lydyard] where several multiple choice questions can be found.

143

9780367696399_Lydyard.indb 143

13/02/2023 07:32

16

Herpes simplex virus

A 13-year-old boy was taken to the hospital’s emergency department by ambulance. His mother had called out the emergency services as he had had a generalized fit that morning. Since he stopped fitting, he had been very drowsy. His mother reported that he was well until 24 hours previously, when he started acting strangely – including wandering around the house not knowing where he was. He vomited once the previous evening, but otherwise has not complained of any specific problems. On examination, he was drowsy but responsive. He was unable to give a coherent history and had difficulty in understanding where he was. He was febrile (38.5°C) but had no other abnormal physical signs. An intravenous (IV) line was set up, and he was started on empirical antibiotic therapy, together with IV aciclovir. A magnetic resonance imaging (MRI) scan of his brain was organized for later in the day, which was reported by the duty radiologist as

1. WHAT IS THE CAUSATIVE AGENT, HOW DOES IT ENTER THE BODY AND HOW DOES IT SPREAD A) WITHIN THE BODY AND B) FROM PERSON TO PERSON? CAUSATIVE AGENT

Herpes simplex virus (HSV) type 1 is a herpesvirus. The Herpesviridae family of viruses is characterized by having a double-stranded DNA genome of 125–240 kb in size (i.e. enough genetic material to encode for 150–200 viral proteins), surrounded by an icosahedral capsid, and a lipid envelope. HSV genomes are around 150–155 kb and encode around 100 viral proteins. The space between the capsid and the envelope is referred to as the tegument. This contains a number of virally encoded proteins, some of which are thought to play a role in viral transport within nerves (see later). Embedded within the envelope are several virally encoded glycoproteins, which are important for binding of the virus to target cells and subsequent cell entry. Thus far, eight herpesviruses have been identified as pathogens of humans – see Case 9. The genome sequences of HSV-1 and HSV-2 share considerable homology and the biological properties of these two viruses are indeed similar. Disease arising from infection with each virus is clinically indistinguishable. However, HSV-1 infections tend to occur “above the waist”, that is oral,

9780367696399_Lydyard.indb 145

showing “an area of low attenuation in the right temporal lobe extending into the frontal lobe gray matter. There was mass effect with displacement of the right middle cerebral artery, appearances most compatible with herpes simplex encephalitis”. A lumbar puncture was performed that evening. The cerebrospinal fluid (CSF) was noted to be slightly bloodstained. The microbiology technician reported the presence of 500 ×10 6 red blood cells l –1, 57×106 white cells l –1, predominantly lymphocytes, a normal CSF sugar, and a CSF protein level of 4.8 g l –1, just above the upper limit of normal. No organisms were seen on a gram-stained film. A provisional diagnosis of herpes simplex encephalitis was made. This was confirmed 2 days later when a report was phoned through from the virology reference laboratory indicating that the CSF was positive for the presence of HSV DNA as tested by polymerase chain reaction (PCR).

while HSV-2 infections are classically “below the waist”, that is genital. Herpes simplex encephalitis (HSE) is almost always due to HSV-1 infection.

ENTRY AND SPREAD WITHIN THE BODY HSV infects at mucosal surfaces (oropharynx and nasopharynx, conjunctivae, genital tract). Intact skin is impervious to HSV, the difference being that skin is covered by a thick layer of keratin, while mucosa has nonkeratinized epithelium. Glycoprotein B in the envelope appears to be one of the molecules involved in attachment of HSV to mucosal epithelial cells. The interaction of HSV with epithelial cells is cytolytic, with cell death occurring 24–48 hours after infection. Virus is released from infected cells by budding, and one infected cell may give rise to thousands of progeny virus particles. These new virus particles infect neighboring cells, leading potentially to extensive local spread at the site of infection.

LATENCY OF HSV At some stage during this process, virus enters nerve terminals and travels in a retrograde direction up the nerve axon to reach the nerve-cell body, the site of latency of HSV (Figure 16.1). Following HSV infection in the mouth, the trigeminal ganglia are typically infected, with occasional extension to cervical ganglia; genital infection, in contrast, results in infection of the lumbosacral nerve root ganglia. Within a latently infected

145

13/02/2023 07:32

Case Studies in Infectious Disease

Figure 16.2 Eczema herpeticum. Virus is able to spread across nonintact skin in patients with eczema, giving rise to eczema herpeticum. From St Bartholomew’s Hospital, London / Science Photo Library, with permission.

Figure 16.1 Schematic diagram illustrating HSV latency. (A) During primary infection of the mouth, there is active viral replication within the oropharyngeal mucosa (red dots). Some of this virus enters nerve terminals and travels up the nerve axon toward the nerve cell body – in this case, the trigeminal ganglion – which is the site of latency for this virus. (B) After recovery from the acute infection, the patient enters the latent phase. There is no viral replication within the mouth or trigeminal ganglion (absence of red dots), but the viral genome lies latent within the latter (open red circles). (C) Following reactivation, latent virus (open red circles) within the trigeminal ganglion may start to replicate (red dots), and newly formed viral particles travel back down the nerve axon to reach the periphery (i.e. the mouth), where further virus replication takes place, although this is limited by the effects of the host immune system. The clinical manifestation of this recurrence, i.e. a cold sore, is therefore much more localized than in primary infection. Adapted with permission from the Wellcome Trust.

cell, viral genome is present as an episome within the nucleus, but there is no detectable production of virus proteins, and no production of new virus particles. Molecular mechanisms underlying entry into, maintenance of, and subsequent reactivation from latency are poorly understood, although well-recognized triggers for reactivation include local trauma, immunosuppression, ultraviolet light, and “stress”. While virus is in a latent state, there is no evident damage to the infected cell. Reactivation from latency occurs when the viral replication cycle resumes, resulting in new virus particles that travel down the axon to reach the periphery, where they may give rise to clinical disease. The fi rst exposure of an individual to HSV infection is referred to as the primary infection, while a reactivated infection is referred to as a recurrent, or secondary, infection. The clinical manifestations of these two forms of herpesvirus infection are often quite distinct as, in the former case, the infection is occurring in an immunologically naïve individual while, in the latter case, virus is replicating in a host whose immune system has seen the virus before. In patients with oropharyngeal or genital HSV infection, an additional risk is spread to other mucosal sites via direct self-inoculation of virus, for example, infection of the fi ngernail bed (known as a herpetic whitlow, or paronychia); virally contaminated fi ngers can spread infection to the eyes, with conjunctival infection and the risk of keratitis. Virus may be scratched into the skin, resulting in a local crop of vesicles. If skin is not intact, for example, in patients with a chronic dermatitis such as eczema, virus may spread across the skin and cause very extensive lesions – so-called eczema herpeticum (this is usually associated with oral infection, see Figure 16.2). This is potentially life threatening, as virus may gain access to the bloodstream through the skin abrasions, and thereby seed internal organs– such viremia is thus potentially life threatening. Spread of virus into the brain, an event which necessarily precedes the onset of HSE, is discussed as part of the disease pathogenesis raised under Question 2 below.

146

9780367696399_Lydyard.indb 146

13/02/2023 07:32

Case 16: Herpes simplex virus

PERSON-TO-PERSON SPREAD

Herpetic lesions present externally, for example, in the oropharynx or genital tract, and contain very large amounts of virus. In addition to the development of symptomatic recurrent disease, reactivation of virus may occur in the absence of clinical disease. Such asymptomatic recurrences result in transient (e.g. 24 hours) shedding of virus from the mouth or genital tract. Infection can then be transmitted to others through direct contact with infected mucosal sites, for example, through kissing or unprotected sexual intercourse. Oral herpes infection can also be transmitted in this way to the genital area via oro-genital sex. One very important route of person-to-person spread of genital HSV is from mother to baby. This may arise when a baby is born through an infected birth canal due to maternal primary or even reactivated infection and may have devastating consequences for the baby (see clinical features below).

EPIDEMIOLOGY Oral and genital HSV infections occur worldwide. Oral infec­ tion is usually with HSV-1, and genital infection with HSV-2, but these boundaries have become blurred with increasing safe-sex practices leading to oro-genital transmission of HSV-1 (and vice versa, i.e. oro-genital transmission of HSV-2). In the UK, up to half of all new genital herpes infections may be due to HSV-1. However, as genital HSV-2 infection is much more likely to recur than is genital HSV-1, HSV-2 remains the principal cause of recurrent genital herpes. Large-scale seroprevalence surveys looking for antibodies specific to HSV-2, mostly conducted within the US, at a time before genital HSV-1 infection was common, suggest that up to 20% of the adult population is HSV-2 seropositive. Seroprevalence increases with age and number of lifetime sexual partners and is higher in black as compared with white populations. However, only a small proportion of individuals with antibodies to either HSV-1 or HSV-2 give a history of primary or recurrent oral or genital herpes, demonstrating the largely asymptomatic nature of the infection.

2. WHAT IS THE HOST RESPONSE TO THE INFECTION AND WHAT IS THE DISEASE PATHOGENESIS? IMMUNE RESPONSE

Innate Immunity Innate immunity is key to prevention or control of a primary infection. The importance of intact skin has been referred to above. Initial infection at mucosal sites will lead to interferon (IFN) production, as HSV-associated patternassociated molecular patterns (PAMPs) are recognized by pattern recognition receptors (PRRs). Interferons bind to

9780367696399_Lydyard.indb 147

their cognate receptors leading to a signaling cascade which ultimately generates induction of interferon-stimulated genes, the products of which induce an antiviral state in the infected cells and their neighbors. Toll-like receptors (TLRs) are a large family of PRRs. Glycoprotein B of HSV binds to TLR-2 leading to expression of several pro-inflammatory cytokines. TLR-3 recognizes double-stranded (ds) RNA and has an important role in controlling HSV in neuronal cells – patients with TLR-3 deficiencies or mutations are more susceptible to developing HSE. TLR-9 is expressed in microglia, astrocytes, and antigenpresenting cells. It recognizes dsDNA and mediates early and rapid production of type 1 interferons. Mice deficient in this TLR are unable to mount an effective immune response to HSV and therefore die when infected. Another important cytosolic DNA sensor is cyclic GMP­ AMP Synthase Stimulator of Interferon Genes (cGASSTING). cGAS-STING signaling also leads to expression of type 1 IFN genes. HSV has evolved multiple ways of avoiding cGAS-STING responses – the HSV protein UL41 degrades cGAS mRNA; HSV protein VP22 interacts directly with cGAS and inhibits its enzymatic activity. HSV protein UL46 interferes with the downstream signaling pathway following cGAS-STING activation. HSV has also evolved ways of subverting activation of complement and the downstream effectors resulting from this process, suggesting that such processes are an important defense mechanism. Thus, HSV glycoprotein C (gpC) inhibits the binding of C5 and properdin to C3b, thereby blocking the alternative pathway of complement activation. gpC also accelerates the decay of the alternative pathway C3 convertase.

Adaptive Immunity Carriage of the virus by immature dendritic cells to draining lymph nodes leads to the development of an adaptive immune response. Both CD4+ T cells and cytotoxic CD8+ T cells are produced in response to infection and play a role in elimination of the virus. Memory T lymphocytes appear to be effective in response to local episodes of recurrent infection and home efficiently to sites of infection. In experimental mouse models of genital herpes infections, IgG antibodies appear to be protective but IgA is less so. Once virus has become latent in nerve-cell bodies, the adaptive immune response, particularly the T-cell arm, is vital in preventing, or at least limiting, disease caused by reactivation. Thus, patients with cellular immunodeficiency, such as HIV-infected individuals, or transplant recipients, are much more prone to symptomatic recurrent disease. Infection with HSV-1 does not induce protective immunity against HSV-2 infection, and vice versa. HSV infection is common and may give rise to disease at a number of distinct sites within the body – see Table 16.1. HSE is fortunately rare – estimated incidence is around 1–3 per million population per year. HSV-1 is the cause of encephalitis in more than 90% of cases. More than two-thirds of cases of HSE are the result of reactivation of latent virus in previously infected individuals,

147

13/02/2023 07:32

Case Studies in Infectious Disease

Table 16.1 Diseases caused by herpes simplex infection Orolabial infection Primary – gingivostomatitis Recurrent – cold sore Genital infection Keratitis (infection of the cornea) Herpetic whitlow – infection of the nail bed Eczema herpeticum

is cytolytic. The presence of replicating virus generates an acute inflammatory host response, with an infiltration of mononuclear cells, the characteristic immune response to a virus infection. Cell death arises from both virus infection and the host immune response, resulting in hemorrhagic necrosis, the pathologic hallmark of HSE (see Figure 16.3). Involved necrotic brain tissue becomes liquefied, and the disease spreads outward as new virus particles are released. Microscopic examination of brain tissue reveals damage extending much further than that visible macroscopically.

Herpes encephalitis Herpes gladiatorum – scratching of virus into the skin, e.g. in wrestlers

with the remainder due to primary infection in children and young adults. Neonatal HSE is distinct in that HSV-2 is the more usual pathogen because of maternal genital infection resulting in intrapartum transmission. Patients suffering from HSE undoubtedly have virus present within the brain substance itself, but it is unclear when and how virus reaches this site. The precise anatomic pathways whereby virus enters the brain substance are not known with certainty. Virus may travel via either or both the trigeminal and olfactory nerve tracts. In patients with reactivated disease, there is also controversy concerning the site of the latent virus that reactivates. Virus may reactivate peripherally (within the trigeminal ganglion or olfactory bulb) and enter the central nervous system (CNS) by retrograde transport or may reactivate centrally within latently infected brain tissue. HSV DNA has been demonstrated by sensitive genome-amplification assays within post-mortem brain tissue in up to one third of asymptomatic HSV seropositive adults. However, the possibility of reactivation of virus from this site remains a speculative hypothesis at the moment. Despite the above uncertainties, what is not in doubt is that virus is indeed present and replicating within the brain substance in a patient with HSE. HSV infection in this situation

Figure 16.3 Herpes simplex encephalitis: gross pathology. The arrow indicates a large area of hemorrhagic necrosis in the left frontotemporal region of the brain.

3. WHAT IS THE TYPICAL CLINICAL PRESENTATION AND WHAT COMPLICATIONS CAN OCCUR? HERPES SIMPLEX ENCEPHALITIS

HSE can arise in any part of the brain, and the neurologic manifestations are dependent on the precise anatomic site. Large case series reported from the National Institutes of Health in the US show that around 75% of patients demonstrate altered behavior or personality change – reflecting the commonest underlying anatomic site of disease in the frontotemporal regions. Also, 80% of patients complain of headache, 90% have fever, and almost all have evidence of decreased level of consciousness. Other presenting manifestations include seizures (67%), vomiting (46%), hemiparesis (33%), and memory loss (25%). The pathologic process underlying the disease is such that the only relevant “complication” of HSE is continuing and worsening brain damage, leading to death. Untreated, the case mortality of HSE is around 70%, and only a small fraction (10%) of survivors are able to return to normal function after recovery from the acute illness. These figures have changed with the advent of antiviral therapy, although mortality may still be as high as 20–30%.

GENITAL HSV INFECTION As explained above, most primary genital (or oral) infections with HSV do not give rise to clinical disease. The assumption must be that expression of disease is a result of the race between replication and spread of the virus on the one hand, and the ability of the innate immune response to control this on the other. In the minority of patients where disease does become clinically evident, primary genital herpes is not a trivial disease. Lesions are extensive and bilateral, extending from the labia to the cervix. There may be spread onto adjacent skin. There will be painful inguinal adenopathy, and a systemic response, (e.g. fever) reflecting the release of cytokines from the intense inflammatory reaction taking place. In females, passing urine may be exquisitely painful if there are lesions near the urethral meatus. It may take 2–3 weeks before the patient fully recovers.

148

9780367696399_Lydyard.indb 148

13/02/2023 07:32

Case 16: Herpes simplex virus

In marked contrast, recurrent disease, even when symptomatic, is usually mild, as the reactivation is taking place in an individual whose immune system has seen the virus before. Thus, the lesions will be much more localized, mostly unilateral, with no spread onto the adjacent skin; no, or only unilateral, inguinal adenopathy; and no, or only mild, systemic symptoms. The natural history of recurrent lesions is resolution within 5–7 days. Many recurrences are asymptomatic – and it must be individuals who are not aware of asymptomatic reactivation of disease who are the source of spread to their sexual partners. There are a number of possible complications. In primary disease, there may be meningeal irritation, such that the patient presents with a clinical diagnosis of meningitis. This may rarely happen even with recurrent disease – so-called Mollaret’s meningitis. HSV meningitis is more common in women. Irritation of the sacral nerve roots (radiculomyelopathy) may present with aching pain in the sacral dermatomes associated with parasthesiae or dysasthesiae in the lower limbs. Accidental inoculation elsewhere in the body has been mentioned above, for example, giving rise to herpetic keratitis (infection of the cornea). One disastrous complication of genital herpes in a female is spread to her baby, resulting in neonatal herpes. This usually arises in women suffering a primary attack of genital herpes in late pregnancy, of which she may be unaware. However, the birth canal is rich in virus, and there has not yet been time for the mother to generate and pass protective antibodies transplacentally to the fetus. The majority of neonates infected in this way acquire internally disseminated infection, including herpes encephalitis, pneumonitis or hepatitis, which have a high mortality and morbidity even with appropriate therapy. Only about half of these neonates have herpetic lesions evident on their skin or mucous membranes, making the diagnosis very difficult. It is only the babies whose infection is limited to the skin and mucous membrane who make a complete recovery – only about 10–15% of all neonatally infected babies. Fortunately, neonatal HSV infection is not common – up to 17.5 in 100 000 live births in the UK, but perhaps 30 in 100 000 in the US. In immunocompromised patients, for example with HIV infection, or those on cytotoxic therapy, HSV recurrences are not only more frequent, but may also be somewhat atypical, with extensive local lesions, and the potential for systemic and life-threatening spread. Genital ulceration of any cause, and certainly including that due to HSV infection, increases the risk of spread of HIV infection, most likely through multiple mechanisms, for example, damage to the skin barrier, attraction of susceptible T cells and macrophages to the site, and up-regulation of HIV replication. HSV-2 infection is associated with a threefold increased risk of sexually acquired HIV infection. Recent data suggest that one way to reduce the spread of HIV infection may be to treat individuals with anti-herpes drugs to prevent the occurrence of genital ulceration.

4. HOW IS THE DISEASE DIAGNOSED, AND WHAT IS THE DIFFERENTIAL DIAGNOSIS? HERPES SIMPLEX ENCEPHALITIS

A diagnosis of HSE should be considered in any patient presenting with altered consciousness, fever, and/or focal neurologic signs, including seizures. Confirmation of the diagnosis may require both virologic and neuroimaging investigations and may not be straightforward. Examination of CSF obtained via a lumbar puncture is a routine investigation when assessing a patient with possible encephalitis. However, the findings in HSE are not specific. The CSF typically demonstrates elevated protein and pleocytosis, with mononuclear cells of between 10 and 500 cells/mm3 and red cells predominating, but even a completely normal CSF does not exclude the possibility of HSE. The most sensitive and specific test on CSF is detection of HSV DNA using a genome-amplification technique such as PCR, but this may be negative, especially if the sample is taken less than 72 hours from onset of symptoms. This approach has more or less replaced the need for brain biopsy, which used to be regarded as the definitive test, and may still be necessary in cases of diagnostic difficulty. Note that despite the presence of HSV DNA in CSF, successful isolation of HSV in cell culture is very unusual. Testing of serum samples for the presence and titer (amount) of anti-HSV antibodies, whether taken in the acute stage, at a later stage, or both, is not particularly helpful in the diagnosis of HSE. Most patients with HSE already have serum antibodies at the time of onset of illness, as the disease represents a reactivation of virus rather than a primary infection. Even a rise in antibody titer taken a few days apart does not confirm HSE, as reactivation of virus, with increased antigen levels boosting an antibody response, may be a consequence of fever or general illness. However, measurement of antiviral antibody synthesis within the CSF may be helpful, provided that serial samples demonstrate a rise in this antibody titer, although such a rise may take several days if not weeks to occur. The cardinal feature of HSE revealed by neuroimaging is the focal nature of the disease process and its predilection for frontotemporal lobe involvement – as opposed to other causes of encephalitis, which may affect the brain more globally (Figure 16.4). Thus, whichever imaging technique is used, the demonstration of a focal lesion or lesions (disease may spread bilaterally, especially if therapy is delayed) should be interpreted as indicating HSE until proven otherwise. Techniques include (proceeding from the more historical to the most modern) electroencephalogram (EEG), radionuclide brain scanning, CT scan or MRI. MRI scans can often show evidence of HSE before the damage is evident on a CT scan. 149

9780367696399_Lydyard.indb 149

13/02/2023 07:32

Case Studies in Infectious Disease

OROLABIAL OR GENITAL HERPES

Figure 16.4 Neuroimaging in herpes simplex encephalitis: CT scan of a patient with herpes simplex encephalitis. The arrows point to bilateral areas of low attenuation in the frontotemporal regions. Originally published in Humphreys and Irving, Figure 42.1, page 191. With permission from Oxford University Press.

DIFFERENTIAL DIAGNOSIS

The differential diagnosis of HSE is that of any encephalitic illness, and there is therefore a potentially long and complex list of diseases that can mimic HSE. The more common of these are listed in Table 16.2. Some of these, for example Japanese encephalitis virus, West Nile virus, and rabies, are specific to certain geographic locations. Measles virus encephalitis has been virtually eliminated in those countries with adequate immunization rates. CMV and EBV encephalitis is restricted to patients with underlying immunodeficiency.

The clinical picture of a patient with extensive ulceration in the mouth or genital area is characteristic enough to make a diagnosis, although it is still imperative to confirm this by sending an appropriate sample to the laboratory, as a diagnosis of herpes has important implications both for the individual patient and for his/her sexual partners. In addition, typing of the virus has prognostic significance, as type-2 genital infections are more likely to recur than are type 1 infections. Note that, in any patient with one sexually transmitted infection (STI), it is also appropriate to test for the presence of other STIs. Recurrent disease, where there may only be one or two ulcers, is much more difficult, and laboratory confirmation should always be sought, as the differential diagnosis of genital ulceration is not straightforward. Samples sent to the laboratory should be taken by abrading the base of an ulcer or vesicle and breaking the swab off into viral transport medium (isotonic fluid containing antibiotic to prevent bacterial overgrowth). Most laboratories will make the diagnosis by demonstrating the presence of HSV DNA by a genome-amplification assay such as PCR. By use of appropriate primers it is possible to distinguish between HSV-1 and HSV-2 using this approach. Alternatively, immunofluorescence of the abraded cells with monoclonal antibodies against HSV-1 and HSV-2 can be used, or virus can be isolated in cell culture. There is even sufficient virus within vesicle fluid to be visualized by electron microscopy, although very few laboratories will perform that these days.

Table 16.2 Differential diagnosis of HSE Other viral infections that can cause encephalitis

Cytomegalovirus Enteroviruses Epstein-Barr virus Influenza viruses Japanese encephalitis virus Measles virus Mumps virus Nipah virus Rabies virus West Nile virus Varicella-zoster virus

Space-occupying lesions

Abscess/subdural empyema Bacterial infections (e.g. Listeria, Mycobacterium tuberculosis, Mycoplasma) Other infectious agents (e.g. Rickettsia, Toxoplasma gondii, Cryptococcus neoformans) Intracerebral tumor Subdural hematoma

Systemic diseases

Vascular disease Systemic lupus erythematosus Toxic encephalopathy

150

9780367696399_Lydyard.indb 150

13/02/2023 07:32

Case 16: Herpes simplex virus

The differential diagnosis is anything that can cause oral/ genital ulceration. In the mouth, the most common condition to mimic HSV infection is hand, foot and mouth disease caused by an enterovirus infection. In the genital area, ulceration may arise from physical trauma including excoriation of marked acute candidal vulvitis, chemical burns from disinfectants, pyogenic infection, fixed drug eruption, systemic disease (e.g. Behcet’s syndrome, erythema multiforme), other infections, for example the chancre of primary syphilis, chancroid, granuloma inguinale, lymphogranuloma venereum (see Chlamydia trachomatis), and dermatologic conditions (e.g. lichen sclerosis et atrophicus).

5. HOW IS THE DISEASE MANAGED AND PREVENTED? MANAGEMENT

The treatment of HSV disease is with aciclovir (acycloguanosine, see Figure 16.5) and its derivatives. Thymidine kinase, a virally encoded enzyme, is able to mono-phosphorylate aciclovir, which is then di- and tri­ phosphorylated by host-cell enzymes. Aciclovir triphosphate then competes with GTP for incorporation into the growing viral DNA chain. Once aciclovir is incorporated, the absence of a complete deoxyribose ring means that there are no hydroxyl groups available to participate in formation of phosphodiester linkages with any incoming bases, and therefore further synthesis of the DNA chain is terminated. The antiviral selectivity of aciclovir arises because: • the drug can only be phosphorylated in virally infected cells as host cells do not possess the necessary thymidine kinase enzyme; • not only is the drug only activated in infected cells, but because phosphorylation of the molecule effectively reduces the concentration of free aciclovir within the cell, more free drug diffuses into the cell from the extracellular space, resulting in concentration of the drug specifically in virally infected cells; • in addition to acting as a DNA chain terminator, aciclovir triphosphate binds to, and strongly inhibits, viral DNA polymerase with a much greater affinity than it does cellular DNA polymerases. Valaciclovir is a valine ester of aciclovir that is hydrolyzed after absorption into valine and aciclovir. However, it is much better absorbed when administered orally than is aciclovir. Penciclovir is a similar molecule to aciclovir, and works in a similar fashion, that is, it requires initial phosphorylation which can only be achieved by viral thymidine kinase, and the triphosphate derivative acts as a potent viral DNA polymerase inhibitor. It is marketed as famciclovir, a complex ester of penciclovir that is better absorbed orally. Viral resistance to aciclovir may arise through a number of mechanisms. Viral variants lacking a thymidine kinase

enzyme (TK–) exist in nature and are selected for when aciclovir (or a similar agent) is used. However, the TK gene is an important virulence factor for HSV, and TK– variants are not pathogenic. Mutations may arise in the TK gene (TK mutants) such that the enzyme no longer phosphorylates aciclovir, but again, many of these mutants have decreased virulence and are not a clinical problem. Mutations may also arise in the viral DNA polymerase gene such that the enzyme no longer binds aciclovir triphosphate. Such DNA pol mutants are fully virulent and are a clinical problem but have, thus far, only been described in heavily immunocompromised individuals who take prolonged courses of aciclovir because of frequent and severe recurrences. Untreated HSE has a high mortality and survivors are left with major brain damage. Aciclovir should be administered IV in high dosage, 10 mg/kg–1 every 8 hours and continued for 2–3 weeks. The longer the delay in starting antiviral therapy, the more residual damage the patient will be left with, so it is vital that therapy be given as soon as possible. Thus, if a diagnosis of HSE is thought to be a clinical possibility in any given patient, aciclovir therapy must be started immediately, long before the necessary diagnostic tests have been conducted to confirm or refute the diagnosis. In clinical practice, this means that many more patients receive IV aciclovir therapy than turn out actually to have HSE, but given the safety profile of the drug, and the catastrophic consequences of not treating the disease promptly, this is acceptable. Disease outcome varies from death to full recovery. Key poor prognostic indicators include older age, lower Glasgow Coma Scale score at presentation (i.e. decreasing levels of consciousness) and increasing length of time between onset of illness and start of antiviral therapy. In patients with symptomatic primary oral or genital HSV infection, antiviral therapy should be initiated as soon as possible. This results in rapid cessation of viral replication and resolution of lesions several days faster than in the absence of therapy. Management of recurrent genital herpes is, however, more controversial. This is, as explained above, a much less severe clinical condition, with a natural history of evolution of lesions to clearance of the order of 6–7 days. Oral therapy

Figure 16.5 Structure of aciclovir. Aciclovir is acycloguanosine – note the absence of a complete deoxyribose ring.

151

9780367696399_Lydyard.indb 151

13/02/2023 07:32

Case Studies in Infectious Disease

with aciclovir (or derivatives), even if initiated by the patient as soon as he/she is aware that a recurrence is imminent, results in shortening of this disease period by around 24 hours. Thus, in general, there is not a great deal of benefit to be gained by routine therapy of recurrent HSV infection. However, there are exceptions to this. Some unfortunate individuals suffer from rather atypical genital herpes, with frequent attacks (e.g. once a month), which may last for up to 2 weeks. The immunologic

reasons for this are poorly understood, but such disease can be managed by use of continuous prophylactic aciclovir. This may continue for several months or even years.

PREVENTION T here is currently no prophylactic vaccine available that protects against infection with HSV.

SUMMARY 1. WHAT IS THE CAUSATIVE AGENT, HOW DOES IT ENTER THE BODY, AND HOW DOES IT SPREAD A) WITHIN THE BODY AND B) FROM PERSON TO PERSON? ■ Herpes simplex viruses 1 and 2 are herpesviruses. ■ Herpesviruses carry a double-stranded DNA genome in an icosahedral capsid surrounded by a lipid envelope. ■ HSV enters at mucosal surfaces (oropharynx and nasopharynx, conjunctivae, genital tract) and replicates within epithelial cells. ■ Spread is local, through release of new virus particles from dying cells. Intact skin is an effective barrier to HSV infection. ■ Virus is also able to enter nerve cells and travel retrograde to the nerve-cell body, the site of latency of HSV. ■ Virus may be reactivated from latency, leading to the presence of virus at the same mucosal sites. Such shedding of virus most commonly is asymptomatic. ■ Person-to-person spread arises through direct contact with virus shed from mucosal surfaces gaining access to recipient mucosal surfaces.

2. WHAT IS THE HOST RESPONSE TO THE INFECTION AND WHAT IS THE DISEASE PATHOGENESIS? ■ Innate immunity is important in early responses to HSV. Glycoprotein B of HSV binds to TLR-2 leading to expression of several pro-inflammatory cytokines. TLR-3 recognizes doublestranded (ds) RNA and has an important role in controlling HSV in neuronal cells. Mice deficient in TLR 9 are unable to mount an effective immune response to HSV and therefore die when infected. Other PRRs are important. ■ An antibody response develops, which is first detectable about 7 days after onset of infection, initially IgM only, followed by IgG. ■ T-cell responses are key to preventing symptomatic recurrent disease, as patients with impaired T-cell-mediated immunity are at risk of frequent and severe recurrences. ■ Routes of entry into the brain substance are not clear but include via the trigeminal nerve and the olfactory nerve. ■ Timing of viral entry into the brain is also controversial – this may occur at the time of primary infection, or at the time of peripheral reactivation of virus in the trigeminal ganglion or olfactory bulb. ■ Virus infects brain cells in a cytolytic fashion, resulting in cell death.

■ Virus particles released from dying cells spread to adjacent brain tissue. ■ The host acute inflammatory response is characterized by an influx of mononuclear cells. ■ The pathologic process is one of hemorrhagic necrosis with liquefaction of affected brain tissue.

3. WHAT IS THE TYPICAL CLINICAL PRESENTATION AND WHAT COMPLICATIONS CAN OCCUR? ■ Clinical presentation of herpes simplex encephalitis (HSE) is nonspecific, common symptoms and signs including altered behavior, headache, fever, seizures, and altered consciousness. ■ Less common manifestations include vomiting, paralysis (hemiparesis), and memory loss. ■ Untreated, the disease is progressive, with a high mortality (70%). Survivors may be left with considerable brain damage. ■ The majority of primary oral or genital HSV infections are asymptomatic. ■ When disease occurs, it is often extensive, bilateral, spread onto adjacent skin, with a systemic response evidenced by regional lymphadenopathy and a febrile response, lasting for 2–3 weeks. ■ Recurrent disease is milder, with fewer crops of lesions, unilateral, no spread onto adjacent skin; no, or only mild, systemic response; and lasting 5–6 days. ■ Complications of genital HSV include meningitis (usually women), lumbosacral meningeal irritation presenting as radiculomyelopathy, self-inoculation to other parts of the body, for example, herpetic paronychia, conjunctival infection. ■ Genital HSV in pregnancy (usually primary) may give rise to neonatal herpes, which has a high morbidity and mortality.

4. HOW IS THE DISEASE DIAGNOSED, AND WHAT IS THE DIFFERENTIAL DIAGNOSIS? ■ HSE should be considered in any patient with altered levels of consciousness, fever, and/or acute onset of focal neurology, including seizures. ■ CSF findings are not specific. The commonest CSF picture is an increase in red cells, around 100 white cells μl–1, mostly mononuclear, increased protein, and normal sugar. However, CSF may be completely normal. ■ The detection of HSV DNA in CSF by PCR is the most sensitive and specific diagnostic test, but this may be negative, especially early in the course of the disease. Continued...

152

9780367696399_Lydyard.indb 152

13/02/2023 07:32

Case 16: Herpes simplex virus

...continued

■ Neuroimaging, whether by EEG, or radionuclide, CT, or MRI scan, is helpful, the cardinal feature being the presence of focal, rather than diffuse, disease. ■ There is a long list of differential diagnoses, including other viral (many), bacterial, fungal or parasitic infections; intracerebral space-occupying lesions such as tumor; systemic disease, for example, vasculitis. ■ Oral or genital HSV is diagnosed by demonstration of HSV DNA in vesicle fluid or in a swab of an ulcer base by genome amplification (usually PCR). ■ Differential diagnosis includes physical trauma, chemical burns, pyogenic or other infections, fixed drug eruptions, systemic blistering diseases, dermatologic conditions.

FURTHER READING Barer M, Irving W, Swann A, Perera N. Medical Microbiology, 19th edition. Elsevier, Philadelphia, 2019. Goering RV, Dockrell H, Zuckerman M, Chiodini PL. Mims’ Medical Microbiology and Immunology, 6th edition. Elsevier, Philadelphia, 2018. Humphreys H, Irving WL, Atkins BL, Woodhouse AF. Oxford Case Histories in Infectious Diseases and Microbiology, 3rd edition. Oxford University Press, Oxford, 2020. Murphy K, Weaver C. Janeway’s Immunobiology, 9th edition. Garland Science, New York/London, 2016. Richman DD, Whitley RJ, Hayden FG. Clinical Virology, 4th edition. ASM Press, Philadelphia, 2016.

REFERENCES Banerjee A, Kulkarni S, Mukherjee A. Herpes Simplex Virus: The Guest That Takes Over Your Home. Front Microbiol, 11: 733, 2020. Kim HC, Lee HK. Vaccines Against Genital Herpes: Where Are We? Vaccines, 8: 420, 2020. Samies NL, James SH, Kimberlin DW. Neonatal Herpes Simplex Virus Disease: Updates and Continued Challenges. Clin Perinatol, 48: 263–274, 2021.

5. HOW IS THE DISEASE MANAGED AND PREVENTED? ■ HSV is sensitive to aciclovir (and derivatives). ■ Treatment is recommended for symptomatic primary disease. ■ Prophylactic therapy may be offered to patients with frequent, atypically aggressive recurrent disease. ■ Treatment of HSE comprises high-dose aciclovir therapy given intravenously for a minimum of 2 weeks is essential, administered as soon as the diagnosis is considered. ■ There is no prophylactic vaccine against HSV infection.

Verzosa AL, McGeever LA, Bhark S-J, et al. Herpes Simplex Virus 1 Infection of Neuronal and Non-Neuronal Cells Elicits Specific Innate Immune Responses and Immune Evasion Mechanisms. Front Immunol, 12: 644664, 2021. Whitley R, Baines J. Clinical Management of Herpes Simplex Virus Infections: Past, Present and Future. F1000Res, 7: F1000 Faculty Rev–1726, 2018.

WEBSITES All the Virology on the WWW Website, Specific Virus Information, 1995: http://www.virology.net/garryfavweb12. html#Herpe NHS, Genital herpes, 2020: https://www.nhs.uk/condit ions/genital-herpes/ Patient UK: http://www.patient.co.uk/showdoc/40000500/ Wikipedia, Herpes simplex virus, 2022: https://en. wikipedia.org/wiki/Herpes_simplex_virus World Health Organization, Herpes simplex virus, 2022: https://www.who.int/news-room/fact-sheets/detail/herpes­ simplex-virus Students can test their knowledge of this case study by visiting the Instructor and Student Resources: [www. routledge.com/cw/lydyard] where several multiple choice questions can be found.

153

9780367696399_Lydyard.indb 153

13/02/2023 07:32

17

Histoplasma capsulatum

Figure 17.1 Chest radiograph of the patient infected with H. capsulatum, revealing bilateral nodular infiltrates. Courtesy of the Centers for Disease Control & Prevention, Atlanta, Georgia. Image is found in the Public Health Image Library #3954. Additional photographic credit is given to M Renz who took the photo in 1963.

1. WHAT IS THE CAUSATIVE AGENT, HOW DOES IT ENTER THE BODY AND HOW DOES IT SPREAD A) WITHIN THE BODY AND B) FROM PERSON TO PERSON? CAUSATIVE AGENT

Histoplasma capsulatum is a dimorphic fungus which causes a systemic endemic mycosis called histoplasmosis (sometimes called Darling’s disease by the name of a pathologist, Samuel Darling who discovered the disease in 1905). The genus Histoplasma belongs to Ajellomycetaceae family (order Onygenales) and contains one species, Histoplasma capsulatum. There are three varieties: H. capsulatum var. capsulatum, which causes the common histoplasmosis, H. capsulatum var. duboisii, a cause of African histoplasmosis (histoplasmosis duboisii), and H. capsulatum var. farciminosum, which causes lymphangitis in horses.

A 60-year-old resident of Louisville (Ohio) had suffered from rheumatoid arthritis for 9 years and was currently being treated with 10 mg of methotrexate weekly and 8 mg of methylprednisolone daily followed by monthly injections of 3 mg kg –1 infliximab monoclonal antibody. Ten weeks after the start of infliximab, he felt severely ill and was hospitalized with the symptoms of dyspnea and cough, quickly followed by respiratory failure, requiring mechanical ventilation. A chest radiograph revealed bilateral nodular infiltrates (Figure 17.1). Bronchoalveolar lavage fluid contained yeast forms resembling Histoplasma capsulatum. Laboratory tests showed normal blood cell counts, but positive Histoplasma urine antigen (10.3 U, normal levels 40) Individuals over the age of 65 Children aged 2–16 Health and social care staff directly involved in patient care People who live in nursing homes and other long-term care facilities

SUMMARY 1. WHAT IS THE CAUSATIVE AGENT, HOW DOES IT ENTER THE BODY, AND HOW DOES IT SPREAD A) WITHIN THE BODY AND B) FROM PERSON TO PERSON? ■ Influenza A, B, and C viruses, carry a segmented (n = 8 for A and B) negative single-stranded RNA genome, are enveloped, and belong to the family Orthomyxoviridae. ■ Typing into A, B, C is according to the nature of the internal proteins. ■ Type A is subtyped according to the nature of the surface H and N proteins.

■ Entry is via inhalation, or droplet inoculation onto oropharyngeal mucous membranes. ■ Viral tropism is for respiratory epithelial cells, with no evidence of spread beyond the lungs (except perhaps for avian H5N1 infection of humans). ■ The viral ligand hemagglutinin binds to cell surface sialic acid receptors. ■ On entry, the virus uncoats and replication begins. ■ Epidemiology is characterized by pandemics arising from antigenic shift, and interpandemic epidemics, arising from antigenic drift.

Continued...

189

9780367696399_Lydyard.indb 189

13/02/2023 07:32

Case Studies in Infectious Disease

continued...

■ Antigenic drift is due to Darwinian selection of variants with mutations in key neutralizing epitopes within the surface glycoproteins in the face of immune selection pressure. ■ Antigenic shift is due to the emergence of a new influenza A subtype. ■ Possible mechanisms include genetic reassortment of human and avian influenza viruses within a mixing vessel, either pigs or humans, or direct trans-species transfer of avian viruses to humans, with subsequent adaptive mutations. ■ The last pandemic, due to A/H1N1 pdm virus, was in 2009. ■ Currently, A/H1N1, A/H3N2, and B viruses co-circulate, giving rise to annual interpandemic epidemics.

2. WHAT IS THE HOST RESPONSE TO THE INFECTION AND WHAT IS THE DISEASE PATHOGENESIS? ■ Viral infection results in lysis or apoptosis of the cell. ■ Influenza virus infection therefore effectively strips off the inner lining of the respiratory tract, including mucus-secreting and ciliated epithelial cells. ■ This predisposes to inhalation of particulate matter, including bacteria, into the lower respiratory tract. ■ Innate immune responses, triggered by recognition of viral associated pathogen associated molecular patterns (PAMPs) by host pathogen recognition receptors (PRRs), include potent induction of interferon and other cytokines. ■ Adaptive immune responses include neutralizing antibody production and T-cell responses.

3. WHAT IS THE TYPICAL CLINICAL PRESENTATION AND WHAT COMPLICATIONS CAN OCCUR? ■ There are two components to the clinical manifestations of influenza virus infection. ■ Respiratory tract symptomatology (e.g. rhinorrhea, cough) arises from local cellular damage and inflammation. ■ Systemic manifestations (fever, headache, pronounced myalgia) arise from the effects of circulating cytokines. ■ Life-threatening complications include primary influenzal pneumonia, secondary bacterial pneumonia, myocarditis, and post-infectious encephalitis. ■ Avian H5N1 infection of humans has a high mortality, due to an intense acute inflammatory reaction within the lungs (a cytokine storm).

■ Genome amplification, e.g. by reverse transcriptase real-time polymerase chain reaction assay is a rapid, specific, and very sensitive approach. ■ Other diagnostic approaches include antigen detection by indirect immunofluorescence, isolation of virus in cell culture (takes several days), or by demonstration of a rise in specific antibody titers (requires a blood sample taken several days after onset of illness). ■ Many other infections may present with an “influenza­ like illness” – the most common is with respiratory syncytial virus.

5. HOW IS THE DISEASE MANAGED AND PREVENTED? ■ The majority of patients infected with influenza virus infection can be managed symptomatically, for example, with appropriate analgesia and antipyretics. ■ Amantadine has activity against some (not all) influenza A viruses. It works by binding to the M2 protein and blocking an ion channel necessary for uncoating of the virus. ■ It has central nervous system stimulatory side effects and is not well tolerated, particularly in the elderly. ■ Resistance emerges rapidly due to mutations in the M2 protein. ■ The neuraminidase inhibitors (zanamavir, oseltamivir) have activity against all known influenza virus neuraminidase enzymes. In the UK, their use is focused on seriously ill hospitalized patients, or as prophylaxis in high-risk exposed patients. ■ Resistance to the neuraminidase inhibitors has been reported, due to point mutations within the NA gene. ■ Baloxavir marboxil, an inhibitor of the viral RNA polymerase complex, has recently been licensed for clinical use. ■ Vaccination is with either inactivated whole virus, a subunit derivative

containing

only

purified

hemagglutinin

and

neuraminidase, or live attenuated vaccines. ■ The vaccines are trivalent (i.e. contain antigens from all three co-circulating viruses). ■ Vaccine composition is adjusted annually to take account of antigenic drift. ■ Vaccine targeting may differ in different countries. In the UK, recommendations are to vaccinate high-risk subgroups within the general population, those over the age of 65 and children

4. HOW IS THE DISEASE DIAGNOSED, AND WHAT

aged 2–16.

IS THE DIFFERENTIAL DIAGNOSIS?

■ Targeted individuals require annual vaccination.

■ Diagnosis is by demonstration of influenza virus in a respiratory sample – for example, nasopharyngeal aspirate, throat swab.

■ An alternative to vaccination is prophylactic use of neuraminidase inhibitors.

190

9780367696399_Lydyard.indb 190

13/02/2023 07:32

Case 19: Influenza virus

FURTHER READING Barer M, Irving W, Swann A, Perera N. Medical Microbiology, 19th edition. Elsevier, Philadelphia, 2019. Humphreys H, Irving WL, Atkins BL, Woodhouse AF. Oxford Case Histories in Infectious Diseases and Microbiology, 3rd Edition. Oxford University Press, Oxford, 2020. Murphy K, Weaver C. Janeway’s Immunobiology, 9th edition. Garland Science, New York/London, 2016. Richman DD, Whitley RJ, Hayden FG. Clinical Virology, 4th edition. ASM Press, New York, 2016.

REFERENCES C hen X, Liu S, Goraya MU, et al. Host Immune Response to Influenza: A Virus Infection. Front Immunol, 9: 320, 2018. Harrington WN, Kackos CM, Webby RJ. The Evolution and Future of Influenza Pandemic Preparedness. Exp Mol Med, 53: 737–749, 2021. Lampejo T. Influenza and Antiviral Resistance: An Overview. Eur J Clin Microbiol Infect Dis, 39: 1201–1208, 2020. Nuwarda RF, Alharbi AA, Kayser V. An Overview of Influenza Viruses and Vaccines. Vaccines, 9: 1032, 2021.

Trifonov V, Khiabanian H, Rabadan R. Geographic Dependence, Surveillance and Origins of the 2009 Influenza (H1N1) Virus. New Engl J Med, 3612: 115–119, 2009.

WEBSITES All the Virology on the WWW Website, Specific Virus Information, 1995: http://www.virology.net/garryfavweb13. html#Ortho Centers for Disease Control and Prevention, Influenza (Flu), 2022: http://www.cdc.gov/flu/ GOV.UK, Influenza: the green book, chapter 19, 2013: https:// www.gov.uk/government/publications/influenza-the-green­ book-chapter-19 Virology online, Influenza Viruses: http://virology-online. com/viruses/Influenza.htm

Students can test their knowledge of this case study by visiting the Instructor and Student Resources: [www. routledge.com/cw/lydyard] where several multiple choice questions can be found.

191

9780367696399_Lydyard.indb 191

13/02/2023 07:32

20

Leishmania spp.

A 72-year-old man retired to the south of Spain but returned to the UK for the summer months. He began to develop fever, malaise, loss of appetite, and weight loss. He was admitted to hospital and had temperatures reaching 39°C. Both his liver and spleen were palpable. No lymph nodes could be felt. Blood tests showed a pancytopenia. Routine investigations for an infection were negative and he did not improve with broad-spectrum antibiotics. His condition deteriorated and the size of the liver and spleen increased (Figure 20.1). A bone marrow examination did not show any sign of hematologic malignancy. No organisms were seen on staining. His history was explored again. Four months before his illness he had been on a camping break in Spain to a coastal area. He recalled seeing many thin dogs in the vicinity. Part of his bone marrow sample was sent to a reference laboratory for Leishmania polymerase chain reaction (PCR). This returned positive. He was successfully treated with a course of liposomal amphotericin B and over the ensuing 3 months his liver and spleen became impalpable and his blood tests returned to normal. His diagnosis was visceral leishmaniasis probably due to Leishmania infantum. Figure 20.1 A child with visceral leishmaniasis. As in the patient described in the case history, the liver and spleen are enlarged, causing distension of the abdomen. Courtesy of World Health Organization, Special Programme for Research and Training in Tropical Diseases, http://www.who.int/tdr/index.html. Image #9706290. Additional photographic credit given to Andy Crump who took the photo in Sudan, 1997.

1. WHAT IS THE CAUSATIVE AGENT, HOW DOES IT ENTER THE BODY AND HOW DOES IT SPREAD A) WITHIN THE BODY AND B) FROM PERSON TO PERSON? CAUSATIVE AGENT

Leishmania are protozoan parasites. They have an intracellular form called an amastigote (Figure 20.2) and an extracellular, flagellated form called a promastigote (Figure 20.3). There is variety in the clinical diseases caused, geographic distribution, and animal reservoirs. The genus Leishmania is divided into groups, complexes and species. Classification was classically determined by isoenzyme typing; molecular methods (using DNA sequencing) are now more common. Table 20.1 lists species and the diseases they cause.

Figure 20.2 A skin biopsy showing Leishmania amastigotes (arrowed). Courtesy of the Centers for Disease Control & Prevention, Atlanta, Georgia. Image is found in the Public Health Image Library #331. Additional photographic credit is given to Dr Martin D. Hicklin who created the image in 1964.

193

9780367696399_Lydyard.indb 193

13/02/2023 07:32

Case Studies in Infectious Disease

ENTRY AND SPREAD WITHIN THE BODY

People are infected after the bite of a sandfly laden with Leishmania promastigotes. Under the skin, the promastigotes are rapidly phagocytosed by macrophages. For cutaneous disease, lesions are confined to the locality of the sandfly bite. For L. braziliensis and L. panamensis cutaneous spread can occur and later this can involve mucous membranes of the mouth or nose. L. donovani and L. infantum are capable of deeper spread within macrophages to the rest of the mononuclear phagocytic system, mainly present in organs such as the liver, spleen, and bone marrow. They are responsible for visceral leishmaniasis. In India, visceral leishmaniasis is called kala­ azar. Relapse of infection after an interval may be manifest as a widespread cutaneous form of disease, called post kala-azar dermal leishmaniasis (PKDL). This occurs in India and East Africa. The life cycle of Leishmania is shown in Figure 20.4.

PERSON-TO-PERSON SPREAD In areas with visceral leishmaniasis, sandflies can ingest protozoa when they feed from the skin. Numbers of Leishmania in the skin are even higher in PKDL. However, leishmaniasis is largely a zoonosis. Different animal reservoirs occur in different regions. They include rodents, gerbils, hyraxes, sloths, and the domestic dog. The sandfly vector is a Phlebotomus species in the Old World and Lutzomyia species in the New World. Sandflies are small, less than 5 mm in size, and bite at dusk or during the night (Figure 20.5). They are not capable of flying great heights above the ground and usually bite individuals sleeping close to the ground. In the case described above, the patient was probably infected through sandflies when he was lying near the ground on his camp bed. He normally lived in a flat. The sandflies will have carried infection from the local dog population.

Figure 20.3 Elongated Leishmania promastigotes. Courtesy of the Centers for Disease Control & Prevention, Atlanta, Georgia. Image is found in the Public Health Image Library #544.

Female sandflies bite and take blood from their target host. Any amastigotes ingested from the skin change into promastigotes. These pass into the sandfly midgut, proliferate, cause damage to the digestive valve system, and are regurgitated to the biting mouthparts and then onto the skin of the next host to be bitten. Another form of transmission for visceral leishmaniasis has been described among intravenous (IV) drug users in Southern Europe. Infection can be passed on with shared needles and equipment. In one study, about half of discarded needles in Madrid were positive by PCR for Leishmania.

EPIDEMIOLOGY Notification of cases of leishmaniasis is not universal. Numbers of people afflicted by the disease are therefore estimates. There are about 50 000 to 90 000 new cases of visceral leishmaniasis annually, and 95% of these are seen in Brazil, China, Ethiopia, India, Iraq, Kenya, Nepal, Somalia, South Sudan and Sudan. The annual incidence of cutaneous leishmaniasis is 0.6 to 1 million new cases. Ninety-five percent of cutaneous leishmaniasis occurs in the Americas,

Table 20.1 Species of Leishmania and the diseases they cause Complex

L. mexicani complex

L. donovani complex

Species

Disease

L. tropica

Cutaneous leishmaniasis

L. major

Cutaneous leishmaniasis

L. aethiopica

Cutaneous leishmaniasis

L. mexicani L. amazonensis L. venezuelensis

Cutaneous leishmaniasis Cutaneous leishmaniasis Cutaneous leishmaniasis

Subgenus Viannia L. [V.] guyanensis L. [V.] peruviana L. [V.] panamensis L. [V.] braziliensis

Cutaneous leishmaniasis Cutaneous leishmaniasis Muco/cutaneous leishmaniasis Muco/cutaneous leishmaniasis

L. donovani L. infantum (also known as L. chagasi in New World)

Visceral leishmaniasis

194

9780367696399_Lydyard.indb 194

13/02/2023 07:32

Case 20: Leishmania spp.

Figure 20.4 Life cycle of Leishmania spp. Leishmania promastigotes are inoculated by sandflies into human and other animal hosts at the time of taking a blood meal (1) Promastigotes are phagocytosed by macrophages (2) Within macrophages, promastigotes transform into amastigotes (3) Amastigotes can multiply in various cell types (4) Macrophages containing amastigotes are ingested by sandflies taking a blood meal (5) and the life cycle continues within the sandfly vector (6–8). Courtesy of the Centers for Disease Control & Prevention, Atlanta, Georgia. Image is found in the Public Health Image Library #3400. Additional photographic credit is given to Alexander J da Silva, PhD, and Melanie Moser who created the image in 2002.

the Mediterranean basin, the Middle East, and Central Asia. About 90% of mucocutaneous disease occurs in Bolivia, Brazil, Ethiopia, and Peru.

2. WHAT IS THE HOST RESPONSE TO THE INFECTION AND WHAT IS THE DISEASE PATHOGENESIS?

Figure 20.5 Phlebotomus sandfly. Courtesy of the Centers for Disease Control & Prevention, Atlanta, Georgia. Image is found in the Public Health Image Library #6274. Additional photographic credit is given as follows: World Health Organization (WHO), Geneva, Switzerland.

A s promastigotes enter the skin they are phagocytosed by macrophages and neutrophils. They change into amastigotes. Classically, any pathogen engulfed by a phagocyte is wrapped within host-cell plasma membrane. This forms a phagosome. Various membrane molecules are imported and exported as cytoplasmic vesicles fuse with or erupt from the phagosome. Eventually, lysosomes fuse with the phagosome and discharge their contents. Lysosomal enzymes lyse susceptible pathogens. On engulfment, phagocytes are activated to produce reactive oxygen species and reactive nitrogen intermediates. They secrete tumor necrosis factor- (TNF- ), which contributes to their activation state. Macrophages are activated further 195

9780367696399_Lydyard.indb 195

13/02/2023 07:32

Case Studies in Infectious Disease

by T-helper 1 lymphocytes (Th-1) through interferon- (IFN­ ). These are stimulated by antigen-presenting cells, most efficiently by dendritic cells. They secrete interleukin-12 (IL-12). When it takes time to deal with a pathogen, the combination of Th-1 cells and macrophages organize into granulomas. To survive, Leishmania need to subvert the above process. Various effects have been described but the mechanisms by which these occur are not altogether clear. Some leishmanial molecules that have been shown experimentally to play a part are lipophosphoglycan, a surface membrane metalloprotease (gp63), cysteine proteases, and a Leishmania homolog of activated C kinase receptor (LACK). The outcome is that macrophage activation and the generation of reactive oxygen and nitrogen intermediates are suppressed, Leishmania resist lysosomal attack, dendritic function is compromised, and LACK induces a Th-2 response. Variations in this interplay occur between different Leishmania species. Furthermore, this may be affected by simultaneous co-infections such as viruses infecting Leishmania parasites, viruses co-inoculated by sandflies and, importantly, by HIV infection in hosts. Some Leishmania species may use neutrophils as a “Trojan horse”. The appearance of neutrophils at the site of the sandfly bite is promoted by the pro-inflammatory effect of sandfly saliva. Cell entry is favored by complementmediated opsonization, provided that there is no complementmediated lysis of Leishmania. Leishmanial lipophosphoglycan promotes the former and inhibits the latter. Neutrophils fail to kill Leishmania after phagocytosis and undergo apoptosis. Apoptotic fragments may contain Leishmania. The neutrophils release MIP-1β, a chemokine that attracts macrophages. When macrophages phagocytose the apoptotic neutrophil fragments, Leishmania enter “silently” and continue to multiply. The macrophages release transforming growth factor-β (TGF-β), which is anti-inflammatory. In mice experimentally infected with L. major, there is a clear polarization of Th-1 and Th-2 responses. Some strains mount a Th-1 response and control infection, unlike others (BALB/c) which mount a Th-2 response and experience fatal disseminated infection. Humans infected with L. major experience localized cutaneous disease. Conversely, L. donovani, which causes visceral leishmaniasis in humans, can be controlled by BALB/c mice. L. donovani does not lead to a polarized Th-1 and Th-2 response between mouse strains. While there are differences between mice and humans in Leishmania infection, the experimental experience with mice indicates the important role of host genetics. In humans, markers of a Th-1 (IFN- ) and Th-2 (interleukin (IL)-4) response are both present at the same time. IL-4 downregulates Th-1 responses and so do other cytokines such as IL-10, IL-13, and TGF-β. These cytokines are more prominent in forms of infection that are not self-limiting like visceral leishmaniasis and PKDL. IL-10 seems to play a greater role in susceptibility to these infections than IL-4. Otherwise, a Th-1 response tends to heal other forms of infection. A n

exuberant inflammatory response mediated by Th-1 cells may paradoxically enable spread of cutaneous forms of disease.

3. WHAT IS THE TYPICAL CLINICAL PRESENTATION AND WHAT COMPLICATIONS CAN OCCUR? Infection may be asymptomatic. Leishmania may reside in the body for years and only cause clinical disease if the host becomes immunocompromised. Cutaneous leishmaniasis is seen on exposed parts of the body where the sandflies are likely to bite (Figure 20.6). Sandflies are unable to bite through clothing. Thus, lesions may be found on the face, arms, and lower legs. Lesions may be single or multiple. They are usually apparent 2–6 weeks after the bite. Initially there is a red papule. This gradually enlarges over a few weeks. The lesion may take on a raised painless, ulcerated form or is papulo-nodular. Secondary infection is possible and then lesions are more likely to be painful. Without specific treatment, lesions will usually selfheal, but over prolonged periods. This may take 6 months to a few years. Over this period, lesions may seem to regress and then relapse. All Leishmania species are capable of causing cutaneous disease, but the host immune response may alter the clinical picture. A weakened immune response with a high parasite burden causes diffuse cutaneous leishmaniasis (DCL) with multiple, spreading papular lesions. There may also be lymphatic spread with localized nodules along the track of lymphatics. A strong immune response with a low parasite burden causes a condition called leishmania recidivans (LR). The immune response effectively clears the initial site of infection. A series of small papules surround this central clearing and these in turn are cleared.

Figure 20.6 Cutaneous leishmaniasis of an ulcerating form. Courtesy of the Centers for Disease Control & Prevention, Atlanta, Georgia. Image is found in the Public Health Image Library #352. Additional photographic credit is given to Dr DS Martin.

196

9780367696399_Lydyard.indb 196

13/02/2023 07:32

Case 20: Leishmania spp.

Mucocutaneous leishmaniasis is associated with L. braziliensis and L. panamensis. As these species’ names suggest, this form of leishmaniasis is restricted to South America. Cutaneous lesions occur first as in purely cutaneous leishmaniasis. These can self-heal, but the parasite does not disappear from the body. After an interval, sometimes of several years, the parasite re-emerges in the mucous membranes of nose or mouth. Local inflammation results in nasal stuffiness. There is then progressive destruction of the anatomy of the nose or mouth and infection can progress backward toward the throat and larynx (Figure 20.7). Eating and drinking become difficult and secondary infections in the upper and lower respiratory tract often occur. These latter effects can prove fatal unless the infection is treated. There can be considerable scarring and disfigurement if treatment is delayed. Visceral leishmaniasis is associated with L. donovani in India and East Africa and with L. infantum around the Mediterranean and South America. A cutaneous lesion may not be apparent. After an incubation period of a few months, illness is heralded by fevers. These may continue for about one month before abating. The spleen progressively enlarges first and then the liver (Figure 20.1). Both may become massively enlarged. The enlarged spleen causes hypersplenism and consumption of blood cells, but infection within the bone marrow also causes a pancytopenia with anemia, leukopenia, and thrombocytopenia. In dark skins, the

anemia plus hormonal effects of chronic infection cause an altered appearance. In India, the graying of the complexion is called kala-azar. Leukopenia predisposes to secondary infections, which themselves may be life-threatening. Thrombocytopenia can predispose to bleeding and there may be life-threatening hemorrhage. On blood tests, there is also a fall in albumin levels. A drop in oncotic pressure can result in edema. This may be peripheral in the legs or ascites within the abdomen. There is also a polyclonal stimulation of IgG antibodies. The polyclonal stimulation of B lymphocytes can compromise their ability to respond to other infections. Visceral leishmaniasis runs a chronic and progressive course. Patients become wasted. It is invariably fatal unless treated. If treated, some parasites may escape killing and return later to cause post-kala-azar dermal leishmaniasis (PKDL). In India, this interval may be 2–3 years, but shorter intervals have been observed in Sudan. The host now has some immunity from the first spell of visceral leishmaniasis. The parasite is largely confined to the skin, with extensive papulo-nodular lesions starting on the face and peripheries and then spreading to most of the body surface. This may self-cure, only to relapse and remit at a later date. Leishmaniasis and HIV co-exist in many areas. The immunocompromised nature of HIV has caused more florid manifestations of leishmaniasis. Parasite burdens are higher. Species that may only cause cutaneous disease may become visceral.

4. HOW IS THE DISEASE DIAGNOSED, AND WHAT IS THE DIFFERENTIAL DIAGNOSIS?

Figure 20.7 Mucocutaneous leishmaniasis in a patient with progressive destruction of tissues around the lips and nose. Courtesy of World Health Organization, Special Programme for Research and Training in Tropical Diseases, http://www.who.int/tdr/index.html. Image #9106015. Additional photographic credit given to Andy Crump who took the photo in Sudan, 1997.

9780367696399_Lydyard.indb 197

In endemic areas, cutaneous and mucocutaneous leishmaniasis may be diagnosed on purely clinical grounds. The clinical picture of fever, splenomegaly, and anemia due to visceral leishmaniasis may also be caused by other diseases. Investigations are required to confirm the diagnosis. These could entail direct visualization of the parasite in a tissue sample, culture of samples, detection of antigen, detection of nucleic acid by PCR, or immunodiagnosis. The latter includes serology or, historically, a leishmanial skin test. Determining the exact species by culture or PCR can be important in choosing between treatment options. Deep-tissue samples may be obtained by bone-marrow aspirate, a splenic aspirate, lymph-node aspirate, or sometimes liver biopsy. Splenic sampling runs the risk of serious splenic hemorrhage. Cutaneous lesions may be squeezed fi rmly with fingers to exclude blood, superficially incised with a scalpel at their edge, and then tissue-fluid expressed and impressed onto a glass slide. On staining of tissue samples, intracellular amastigotes are sought. Their appearance is characteristic with a small kinetoplast body adjacent to the nucleus. This is called a Donovan body (Figure 20.8). The sensitivity of tissue sampling varies with the sample – >90% for splenic aspirate, 55–97% for bone marrow, and 60% for lymph

197

13/02/2023 07:32

Case Studies in Infectious Disease

of fever, splenomegaly, and anemia. The differential diagnosis includes malaria, schistosomiasis, typhoid fever, brucellosis, tuberculosis, rickettsial infection, sarcoidosis, Still’s disease, and hematologic malignancy.

5. HOW IS THE DISEASE MANAGED AND PREVENTED? MANAGEMENT

Figure 20.8 Characteristic Leishmania amastigote forms (Donovan bodies, arrowed) on an impression smear. Darkly staining small kinetoplast adjacent to nuclei. Courtesy of the Centers for Disease Control & Prevention, Atlanta, Georgia. Image is found in the Public Health Image Library #30. Additional photographic credit is given to Dr Francis W Chandler who created the image in 1979.

nodes. If facilities are available, culture can help but now in resource-rich settings PCR is applied with a sensitivity of >95%. Leishmanial antigen can be detected in urine. A latex agglutination technique called KATEX has shown sensitivities of 68–100% for visceral leishmaniasis. After successful treatment, antigen disappears from the urine. Serology is usually negative in cutaneous leishmaniasis and should be reserved for visceral disease. Serologic tests are unable to distinguish current from past infection. A commonly used test for anti-leishmanial antibody is the direct agglutination test (DAT). Promastigotes are formalin­ fixed onto slides and serum is placed on top. Agglutination is observed after 24 hours. DAT has a sensitivity of about 95% and its specificity is about 86%. A dipstick test has been developed with the K39 antigen impregnated on a reagent strip. Blood is added to the strip and a reaction noted after 20 minutes. The K39 dipstick has a sensitivity of about 94% and specificity of about 90%. The K39 antigen is also used in ELISAs. Historically, a test similar to the tuberculin skin test for tuberculosis was used for leishmaniasis. This was the Montenegro skin test. Leishmanial antigen was implanted in the forearm and the induration after 48–72 hours was measured. Now standardized antigen preparations are no longer available.

DIFFERENTIAL DIAGNOSIS In endemic areas, cutaneous and mucocutaneous leishmaniasis may have a characteristic appearance. Cutaneous lesions may have to be differentiated from other infected insect bites, tuberculosis, fungal infection, myiasis, and skin cancers. Mucosal sites may also be affected by syphilis, histoplasmosis, paracoccidioidomycosis, and leprosy. Visceral leishmaniasis may have to be differentiated from other causes

Small, single lesions of cutaneous leishmaniasis in immunocompetent hosts may be left to self-heal in geographic areas with L. major. Other cutaneous lesions, mucocutaneous disease, and visceral leishmaniasis require treatment. There are a number of treatment permutations dependent on form of disease, geographic location, species, and availability of agents. Originally, the mainstay of treatment was pentavalent antimony compounds. These include sodium stibogluconate and meglumine antimonate. They are administered by intramuscular (IM) injection on a daily basis for durations up to 28 days. The IM injections can be painful and there can be systemic toxicity. Alternative treatments are very welcome, as about 60% of visceral leishmaniasis infections in Bihar, India are resistant to treatment with pentavalent antimonials. Amphotericin B and the formulation of liposomal amphotericin B represent advances in treatment. However, they require IV administration, are also toxic, and are much more expensive, especially liposomal amphotericin B. An oral, tolerable agent is now available for visceral leishmaniasis. Oral miltefosine for 28 days was shown to be equally effective with amphotericin B for visceral leishmaniasis in India. Depending on geographic location, agents may be used singly or in combination. In certain situations, other options for cutaneous lesions include intra-lesional injections of pentavalent antimonials, topical paromomycin, and oral azoles.

PREVENTION Vaccines have been trialed for leishmaniasis but have not been encouraging to date. Prevention has therefore focused on sandflies and efforts directed at human and animal reservoirs. Targeting animal reservoirs is difficult, but it is important to identify and treat PKDL patients who are a key human reservoir for visceral leishmaniasis. As sandflies mainly bite at night, sleeping under a bed-net might afford some protection. But the sandflies are small and can get through the mesh of the nets. However, if the nets are impregnated with a pyrethroid insecticide the sandflies are killed. Insecticide-treated bednets are also a key component of malaria control programmes. Furthermore, indoor insecticide spraying on walls has been used both for malaria and leishmania control.

198

9780367696399_Lydyard.indb 198

13/02/2023 07:32

Case 20: Leishmania spp.

SUMMARY 1. WHAT IS THE CAUSATIVE AGENT, HOW DOES IT ENTER THE BODY, AND HOW DOES IT SPREAD A) WITHIN THE BODY AND B) FROM PERSON TO PERSON? ■ Leishmania are protozoan parasites. ■ There is a large number of species. ■ The extracellular stage is called the promastigote and the intracellular stage is the amastigote. ■ Spread is by sandflies either from animal reservoirs or humans with heavy skin loads of parasite. The latter occurs in a condition called post-kala-azar dermal leishmaniasis. ■ The ability to spread within the body is a function of both the species of Leishmania and the host immune response.

2. WHAT IS THE HOST RESPONSE TO THE INFECTION AND WHAT IS THE DISEASE PATHOGENESIS? ■ Leishmania are phagocytosed by neutrophils and macrophages. ■ T-helper 1 lymphocytes help macrophages to kill Leishmania. ■ Some species may be more successful at subverting the immune response and causing disseminated infection. ■ Subversion of the immune response involves suppression of macrophage activation and diversion toward a T-helper 2 type of response. ■ In mice infected with L. major, there is a clear polarization of T-helper 1 and 2 responses, depending on the genetic background of the mice.

3. WHAT IS THE TYPICAL CLINICAL PRESENTATION AND WHAT COMPLICATIONS CAN OCCUR? ■ Cutaneous leishmaniasis is caused by a large number of species with infection confined to one locality.

FURTHER READING Boelaert M, Sundar S. Leishmaniasis. In: Farrar J, Hotez P, Junghanns T, et al, editors. Manson’s Tropical Diseases, 23rd edition. Elsevier/Saunders, London, 2014: 631–651. Murphy K, Weaver C. Janeway’s Immunobiology, 9th edition. Garland Science, New York/London, 2016.

REFERENCES Aronson N, Herwaldt BL, Libman M, et al. Diagnosis and Treatment of Leishmaniasis: Clinical Practice Guidelines by the Infectious Diseases Society of America (IDSA) and the American Society of Tropical Medicine and Hygiene (ASTMH). Am J Trop Med Hyg, 96: 24–45, 2017. Rossi M, Fasel N. How to Master the Host Immune System? Leishmania Parasites have the Solutions! Int Immunol, 30: 103–111, 2018.

9780367696399_Lydyard.indb 199

■ Lesions enlarge gradually over a few weeks, become papulonodular or ulcerate. ■ Lesions may self-heal. ■ Mucocutaneous leishmaniasis is caused by L. braziliensis and L. panamensis. ■ After an initial cutaneous lesion, there is a later mucosal lesion, which is progressively destructive of nose or mouth. ■ Visceral leishmaniasis is due to L. donovani or L. infantum. ■ Infection spreads through the mononuclear phagocytic system with enlargement of spleen and liver, and bone-marrow infiltration. ■ Skin complexion changes giving rise to the Indian term, kala-azar. ■ After treatment of visceral leishmaniasis relapse may be confined to the skin with post-kala-azar dermal leishmaniasis.

4. HOW IS THE DISEASE DIAGNOSED, AND WHAT IS THE DIFFERENTIAL DIAGNOSIS? ■ Diagnosis of cutaneous or mucocutaneous leishmaniasis may be purely clinical. ■ Stained-tissue samples may show characteristic Donovan bodies. ■ Species-specific PCR has a high sensitivity. ■ Assays exist for leishmanial antigen or antibody.

5. HOW IS THE DISEASE MANAGED AND PREVENTED? ■ Simple cutaneous lesions may self-heal. ■ Parenteral pentavalent antimonial drugs have been traditional treatment for all forms of disease. ■ Parenteral amphotericin B, either conventional or liposomal, can be used for visceral leishmaniasis. ■ Oral miltefosine is easier to administer. ■ Combinations of agents may be required for treatment ■ Vector control and treatment of the human reservoir of PKDL patients are important preventive measures.

Torres-Guerrero E, Quintanilla-Cedillo MR, RuizEsmenjaud J, Arenas R. Leishmaniasis: A Review. F1000Res, 6: 750, 2017.

WEBSITES Centers for Disease Control and Prevention, Parasites Leishmaniasis, 2020: www.cdc.gov/parasites/leishmaniasis World Health Organization, Leishmaniasis: www.who.int/ health-topics/leishmaniasis

Students can test their knowledge of this case study by visiting the Instructor and Student Resources: [www. routledge.com/cw/lydyard] where several multiple choice questions can be found. 199

13/02/2023 07:32

21

Leptospira spp.

A Staff Sergeant was repatriated from an exercise in Belize and admitted to hospital in the UK. He was complaining of fever, headache, and myalgia. After an initial improvement, he began to deteriorate. He became jaundiced with signs of pneumonia and, on examination, had conjunctival inflammation and hepatosplenomegaly. A chest X-ray indicated bi-basal opacities. His

1. WHAT IS THE CAUSATIVE AGENT, HOW DOES IT ENTER THE BODY AND HOW DOES IT SPREAD A) WITHIN THE BODY AND B) FROM PERSON TO PERSON? CAUSATIVE AGENT

Leptospira spp. (leptospires) are motile, very thin, tightly coiled spirochetes measuring 0.1μm by 10–20 μm with a character­ istic curve at either end (Figure 21.1) and they have a typical gram-negative cell-wall structure (see Case 11, E. coli). They have two axial flagella located between the peptidoglycan and an outer-membrane layer in the periplasmic space (Figure 21.2). Ellinghausen McCullough Johnson and Harris (EMJH) liquid medium is used to isolate Leptospira and consists of bovine serum albumin, Tween 80, glycerol, sodium pyruvate, cyanocobalamin, and various salts (Mg, Fe, Zn, Ca) dissolved in ultrapure water. Growth is slow and may achieve 107 /ml of organism. In some cases, excessive growth can lead to lysis

Figure 21.1 Morphology of Leptospira. Reprint permission kindly given by the Centers for Disease Control & Prevention, Atlanta, Georgia. Image is found in the Public Health Image Library #1220. Additional photographic credit is given to Janice Haney Carr who took the image and the CDC NCID and Rob Weyant who provided the image for CDC PHIL website.

9780367696399_Lydyard.indb 201

blood count showed a neutrophilia with a thrombocytopenia. Liver function tests showed an elevated conjugated bilirubin with mild elevation of transaminases. He was oliguric and uremic. A diagnosis of Weil’s disease (leptospirosis) was made and he was started on benzylpenicillin and renal dialysis.

due to the lipases produced by the organism. The generation time can range from 6 to 16 hours depending on the isolate. The organism is aerobic but requires CO2 to stimulate growth. Ma ximal growth is between pH 7.2 and pH 7.6. Growth temperature depends on whether it is a pathogen (29°C–37°C ) or a saprophyte which can grow at 14°C but not at 37°C . I n addition to the growth temperature indicating its identity as a saprophyte or pathogen, growth in EMJH medium containing 8-azaguanine (or copper) allows the growth of saprophytes but inhibits pathogen growth. Recently in 2019, the taxonomy of the Genus Leptospira has undergone a major review. Previously there were 35 recognized species with many named serovars of one pathogenic species Leptospira interrogans. The Genus was divided into three divisions: the Saprophytes (e.g. L. biflexa), an Intermediate division (L. fainei, L. inadai) and the Pathogen division (e.g. L. interrogans). This latest study used whole genome sequencing (WGS) (NextSeq 500 (Illumina), Nextra XT Library preparation and CLC Genomics assembly platform) with additional analysis of Average Nucleotide Identity (ANI) and values of the percentages of conserved proteins (COPD). The study included 90 isolates of Leptospires from a variety of locations (e.g. Japan, France, Malaysia, Algeria) and identified 30 new Leptospira species giving a total of 64 named species. The taxonomy generated by this study identified two major clades and four subclades P1, P2, S1, S2 (see Table 21.1). A study of core gene sets identified genes and domains linked with each subclade. The two major clades are identified as saprophytes (S) and pathogens (P). Subclades P1 (Pathogens in humans and animals), P2 (those species previously classified as Intermediate), S1 (saprophytes) S2 a new subclade including L. idonii, L. kobayashii, L. ilyithenesis, and L. ognonensis. Species within this group grew at 14°C but not at 37°C and grew well in the presence of 8-azoguanine, both phenotypic characteristics of saprophytes.

201

13/02/2023 07:32

Case Studies in Infectious Disease

Figure 21.2 The axial internal flagella of Leptospira. Adapted with permission from Dr Samuel Baron and the University of Texas Medical Branch at Galveston, Department of Microbiology and Immunology. Table 21.1 Some representative species of Leptospira P1 L. interrogans L. noguchi L. alexanderi L. alstonii P2 L. licerasiiae L. hartskeetlii L. langatensis L. broomii S1 L. biflexa L. levettii L. perdikensis L. ellinghauseni S2 L. idonii L. ryugenii L. ilyithenesis L. kobayashii

ENTRY INTO THE BODY

202

Infection is acquired by contact with water or soil contaminated by one of the pathogenic Leptospira serovars. Entry is via skin lesions, or lesions in the mucosae of the respiratory and digestive tracts or conjunctivae. The organism may also be acquired from contaminated aerosols entering the respiratory tract. A number of animal species act as a reservoir of infection, the common ones being rodents, but dogs (L.

9780367696399_Lydyard.indb 202

canicola) and livestock (L. pomona) can also act as reservoirs. Infected animals excrete the bacterium in their urine thus contaminating water sources and soil. Human infection can be acquired directly from contact with the animals or from contact with contaminated water. Globally, Leptospira is the commonest zoonosis.

SPREAD WITHIN THE BODY After entry, the organism is spread around the body by the blood, entering all organs and thus giving rise to a wide spectrum of clinical presentations (see later).

EPIDEMIOLOGY Leptospirosis is a global disease, although it is primarily a disease of tropical and subtropical regions and is relatively uncommon in temperate climates. It is endemic in many countries but outbreaks are also associated with adverse weather. Examples of this include: an outbreak of leptospirosis in Nicaragua following Hurricane Mitch in 1995; an outbreak in Peru and Ecuador following heavy flooding in 1998; a postcyclone outbreak in Orissa, India in 1999. The precise number of human cases worldwide is not known but it is estimated that 1.03 million cases and 58 900 deaths occur annually. Incidences range from approximately 0.1–1 per 100 000 per year in temperate climates to 10–100 per 100 000 in the humid tropics; incidence may reach over 100 per 100 000. Reports of cases in the US and Europe are low. In the UK, it varies between 13 and 31 per annum and in continental France is in the order of 300, although in 2005 there were only 212 confi rmed cases. In the US, the incidence is between 43 and 93 per annum. Certain occupations are prone to infection, such as veterinarians, butchers, sewage workers, and farmers. Recreational exposure can occur through adventure holidays such as white-water rafting and other water sports.

13/02/2023 07:32

Case 21: Leptospira spp.

2. WHAT IS THE HOST RESPONSE TO THE INFECTION AND WHAT IS THE DISEASE PATHOGENESIS? The innate immune system constitutes the fi rst line of host defense, playing a crucial role in early recognition and elimination of leptospires.

INNATE IMMUNITY Pattern-Recognition Receptors, Neutrophils and Macrophages Microbes are recognized through their microbial-associated molecular patterns (MAMPs) through a set of different pattern-recognition receptors (PRRs). The MAMPS include many different components such as nucleic acids, flagellin, and lipopolysaccharide (LPS). PRRs also recognize endogenous molecules associated with cellular damage (DAMPs) produced through microbial infection, for example. PRRs are expressed on both immune cells and nonimmune cells (e.g. epithelial cells) and include members of the membrane Tolllike receptor (TLR), the cytosolic NOD-like receptor (NLR) families (Nucleotide-binding oligomerization domain-like receptors (including NOD-1 and NOD-2) and CLR (C-type lectin receptors). MAMP recognition results in a signaling cascade that leads to activation of transcription factors such as NF-kB and IRF3 that are involved in the production of cytokines, chemokines, and antimicrobial peptides. Pro-inflammatory cytokines produced include interleukins (IL-1β, IL-6, IL-12, interferons (IFNs) and tumor necrosis factors (TNFs), as well as chemokines. Both cytokines and chemokines lead to activation and recruitment of phagocytes, such as neutrophils, macrophages and dendritic cells to the infection site. Inflammation may not only lead to pathogen destruction but also, if unregulated, can result in a “cytokine storm” observed in patients with sepsis (see pathogenesis). TLR4 normally senses LPS on gram-negative bacteria but for Leptospira LPS, TLR2 appears to be the PRR which can potentially stimulate macrophages to release proinflammatory cytokines. Human monocytes are activated, in vitro, by Leptospira to produce pro-inflammatory cytokines as seen by up-regulation of TNF and IL1β genes. Intracellular uptake of leptospires through NLR triggers reactive oxygen species (ROS) and reactive nitrogen species (RNS). In addition, infection triggers TLR2-mediated production of IL-8 and NLRP3-dependent production of IL-1β. TLR2 on human macrophages also recognizes the outermembrane lipoprotein LipL32, the major lipoprotein of leptospires, It is thought that this occurs through dimers of TLR2 and TLR1. NOD-1 and NOD-2 (NLR family) are intracellular receptors that recognize peptidoglycan fragments of bacterial peptidoglycans (PGs) called muropeptides and are active against Leptospira PGs.

9780367696399_Lydyard.indb 203

There appears to be some controversy around the importance of neutrophils in the host response to Leptospira. Data suggest that direct phagocytosis of leptospires is rather poor although early studies on immunity indicated that serum from infected individuals but not “normal serum” results in phagocytosis. This indicates that antibodies were “opsonizing” the organisms for phagocytosis. It appears that neutrophils are mostly effective against saprophytic leptospires rather than pathogenic leptospires. Neutrophils also release myeloperoxidase and ROS that are antimicrobial. The antimicrobial peptide cathelicidin released by neutrophils has been shown to have anti-leptospiral activity among different Leptospira species (L. interrogans serovars and Leptospira biflexa). In addition, a particular strain of L. interrogans triggers the release of DNA extracellular traps (NETs) and kills leptospires through the process of NETosis – extrusion of the neutrophil DNA with bactericidal proteins which results in trapping and/or killing of the pathogens. It has been suggested that these DNA traps are crucial to preventing early leptospiral dissemination.

Complement The complement system has an important role in protection against foreign microbes. It is made up of many molecules (around 50) and is activated through three pathways, classical (CP), alternative (AP), and lectin (LP). Both LP and AP pathways are involved in the host innate immunity while the CP pathway is triggered by the IgM or IgG antibodies specifically bound to antigens. The lectin pathway is triggered when lectins, including ficolins or mannose-binding lectin, attach to carbohydrate moieties on the microbial surfaces. Activation of all three pathways leads to promotion of phagocytosis (opsonization), recruitment of inflammatory mediators and inflammation and lysis via the MAC attack complex. Recent data on using inhibitors of these pathways and survival in human serum of non-pathogenic or pathogenic serovars of Leptospira have suggested that both AP and LP have an important role in eliminating saprophytic leptospires. In all leptospires investigated, the deposition of the lytic MAC attack complex proteins on saprophytic Leptospira strains was more pronounced when compared to pathogenic species. The pathogenic species have many evasion mechanisms targeting complement activation – see below. T cells, that recognize a broad range of antigens without the presence of major histocompatibility complex molecules, have been shown to respond against leptospires. Identification of the protein antigens recognized will be important in understanding their role in the leptospiral immune response.

Adaptive immunity PRR activation is also important for the development of adaptive immunity and results in the expression of costimulatory molecules at the surface of macrophages and dendritic cells that are important for antigen presentation to naive T cells and the subsequent activation of B cells and the production of antibodies.

203

13/02/2023 07:32

Case Studies in Infectious Disease

Experimental animal models of leptospires have provided most of the evidence for antibodies playing a key role in both protection and clearance of infections. Several studies in leptospirosis patients have shown that there is a strong antibody response to leptospire LPS. Both IgM and IgG antibodies are produced against a variety of leptospire-specific antigens. These include LipL32, LipL41, and leptospiral immunoglobulin-like A (LigA) which are present only in pathogenic leptospires and have been shown to be exposed on their cell surface. The major protective role of IgG antibodies (which is increased in infections with pathogenic leptospires) seems to be opsonization. The role of T cells is unclear except for their help with the production of antibodies.

Host-response evasion mechanisms Most leptospire infections are asymptomatic and/or resolve fairly quickly, probably as the result of an effective immune response not compromised by extensive evasion mechanisms. Other strong pathogenic leptospires that do have extensive evasion mechanisms produce more severe and sometimes fatal illness – see pathogenesis below. PRRs: NOD-1 and NOD-2 receptors cannot sense Leptospira since its outer-membrane protein, LipL21 lipoprotein, binds to the PG and impairs its degradation into muropeptides, which therefore cannot signal through NOD-1 and NOD-2. Complement evasion: The complement system provides an early innate defense. Pathogenic leptospires use different sophisticated strategies to subvert or inactivate all three pathways (classical, alternative, and lectin) of the complement cascade. Pathogenic leptospires evade complement attack by binding Factor H (FH) and C4 binding protein (C4BP) of the alternative and classical pathways onto their surfaces. FH, a plasma protein, inhibits the alternative pathway of complement by preventing binding of Factor B to C3b, accelerating decay of the C3-convertase C4BP and acting as a co-factor for the cleavage of C3b by Factor I. LenA and LenB are leptospiral ligands for human FH. C4BP, a plasma glycoprotein, inhibits the classical pathway of complement by interfering with the assembly and decay of the C3-convertase C4bC2a and acts as a co-factor for Factor I in the proteolytic inactivation of C4b. LcpA is a leptospiral outer-membrane protein which interacts with human C4BP. Thus, complement activation is downregulated preventing opsonization and the formation of the lytic membrane attack complex on its surface. Phagocyte function: the outer-membrane proteins of L. interrogans serovar Copenhageni – LipL21 and LipL45 have been shown to be myeloperoxidase inhibitors.

Pathogenesis Leptospirosis starts with an acute phase that is self-resolving in most of the cases and is followed potentially by a chronic phase (especially kidney colonization) depending on the virulence of the strain, severity of the acute phase, and the overall immune defense of the host. As shown above, pathogenic leptospires have many evasion mechanisms to 204

9780367696399_Lydyard.indb 204

overcome the immune system. Around 10% of leptospirosis cases develop into severe forms. The organism migrates to the interstitium, renal tubules, and tubular lumen of the kidney causing an interstitial nephritis and tubular necrosis. Liver involvement is seen as centrilobular necrosis with proliferation of Kupffer cells. Leptospires also invade skeletal muscle, causing edema, vacuolization of myofibrils, and focal necrosis. In the lungs, leptospires induce intra-alveolar hemorrhages. A consistent pathologic finding in all organs is a vasculitis. Whether organ damage is due directly to secreted toxins of leptospires or is secondary to the vasculitis induced by cell-wall components of the organism such as collagenase or bystander damage is unclear. A study has shown that the peptidoglycan of leptospires can increase the adhesion of the organism to vascular endothelial cells and is, thus, an important virulence factor. The illness is characterized by a bleeding diathesis although its pathophysiology is uncertain. Infection causes a thrombocytopenia, but it is unclear whether this is due to a direct action of the leptospires on thrombopoiesis or secondary to disseminated intravascular coagulation or a specific antiplatelet antibody. During infection, the triggering of the inflammatory response through the production of pro-inflammatory cytokines is important for the early elimination of pathogens. However, unregulated cytokine production can result in a cytokine storm that might be followed by a state of immunoparalysis, which can lead to sepsis, associated organ failures, and subsequent death of some patients with severe leptospirosis.

3. WHAT IS THE TYPICAL CLINICAL PRESENTATION AND WHAT COMPLICATIONS CAN OCCUR? Leptopirosis was first recognized in sewage workers in 1883. The incubation period is about 10 days but can range from 5 to 30 days. Because the organism spreads to all organs of the body, the clinical presentation may vary. The infection may be asymptomatic and exposure is only recognized serologically. Symptomatic disease may present as a biphasic illness with an initial nonspecific phase where the patient complains of: •

a high temperature with rigors;



headache;



myalgia;



retro-orbital pain and photophobia;



conjunctivitis;



nausea, vomiting, diarrhea;



dry cough;



pre-tibial rash.

This is the leptospiremic phase of the illness lasting about 7 days. After the temperature falls, more specific system-based

13/02/2023 07:32

Case 21: Leptospira spp.

symptoms may develop within a few days clinically presenting as: • aseptic meningitis (headache, stiff neck, photophobia, lymphocytic CSF); • hepatitis (jaundice); • renal failure with jaundice and hemorrhagic features (Weil’s disease); • pulmonary infection (cough, hemoptysis); • systemic inflammatory syndrome or shock. The commonest presentation is anicteric, for example aseptic meningitis (80–90%) compared to icteric with renal failure (Weil’s Disease 10–20%). Laboratory abnormalities occur associated with specific syndromes, for example increased serum creatinine, thrombocytopenia, leucocytosis, and hyperbilirubinemia. On examination, the liver and / or spleen may be enlarged. This phase corresponds to the immune phase and may last from 1 to 6 weeks. In some cases, the two phases may not be apparent or the illness appears to start with the immune phase. The severity of the disease may vary from a mild illness to more severe disease (Weil’s disease) with complications including cardiac dysrhythmias, liver or renal failure, myelitis, and Guillain-Barré syndrome. Weil’s disease carries a high mortality.

4. HOW IS THE DISEASE DIAGNOSED, AND WHAT IS THE DIFFERENTIAL DIAGNOSIS? T he laboratory diagnosis is made with: • dark-ground microscopy; • culture; • serology; • genomics.

Dark-Ground Microscopy Leptospira spp. can be detected by dark-ground microscopy in blood or cerebrospinal fluid (CSF) during the first week of the illness and in the urine about 10 days thereafter. It has a low sensitivity and specificity and requires a concentration of 104 leptospires/ml for detection.

Culture Leptospires can also be cultured from the blood, CSF or urine using EMJH medium incubated at 30°C for 6–8 weeks. Because of the low sensitivity and specificity of dark-ground microscopy, and the long timescales of culture, the routine method of diagnosis is serologic. Culture, however, is important in epidemiologic studies with specimens obtained from soil, water, animals, and humans although, for the latter, it is not an important diagnostic test because of the long timescales.

Serology The microscopic agglutination test (MAT) is difficult to perform and relies on mixing serum from the patient with l ive leptospires from the different serogroups and looking for agglutination by dark-ground microscopy. This test also has a low sensitivity due to failure to seroconvert in a proportion of patients and cross-reactivity with a number of other infections and illnesses including other spirochetal diseases (Lyme disease, relapsing fever, treponemal disease), Legionella, HIV, and autoimmune diseases. Although the MAT is serogroup­ specific, cross-reactions between serogroups occur and the i ndividual serovars cannot be determined. A result of > 1:100 or a four-fold rise in titer is considered positive. An IgM ELISA directed against LipL32 is most often used for diagnosis and detects antibodies 7–10 after infection. The majority of IgM (95%) are directed against this antigen and it is common in pathogenic leptospires. It has a high sensitivity and specificity. Other formats such as latex agglutination, indirect hemagglutination and lateral flow tests (LeptoTek Lateral Flow) are also commercially available and have the advantage that can be used as point-of-care (POC) assays.

Genomics Various genomic assays are available: Polymerase chain reaction (PCR) directed against the sec Y-gene or the LipL32 gene have a high sensitivity and speci ficity. Other PCR modifications are also available, for example chip-based RT-PCR (Real-time PCR), loop-mediated isothermal amplification (LAMP). W hole Genome Sequencing (WGS) will probably be the standard genomic method of leptospire detection as it has more specificity of the identity of leptospire species. For example, PCR using 16S RNA is useless, as 16 different leptospire species all have the same 16S RNA profile. However, one gene that may prove useful to detect all leptospires is the ppk gene (polyphosphate kinase) as it evolves rapidly and gives the identical taxonomy to WGS.

DIFFERENTIAL DIAGNOSIS Leptospirosis is endemic in areas where the following are found: • Dengue virus (co-infection occurs in about 8% of cases); • malaria; • hantavirus; • scrub typhus. Patients presenting with the initial febrile illness may be diagnosed with Dengue fever which is more common than leptospirosis. If presenting with signs of meningitis, then leptospirosis may be misdiagnosed as viral meningitis or if with petechiae then meningococcal meningitis. A CSF white cell count would suggest a viral etiology rather than meningococcal, as the cell response in leptospirosis is 205

9780367696399_Lydyard.indb 205

13/02/2023 07:32

Case Studies in Infectious Disease

lymphocytic. If presenting with jaundice the differential diagnosis includes:

There is also some evidence for the benefit of plasmapher­ esis in severe leptospirosis.



viral hepatitis;



malaria;

Additional clinical support



schistosomiasis;



relapsing fever;



tularemia a nd if with jaundice and renal failure then it includes:



Legionnaires disease;



hemolytic uremic syndrome.

I f the pulmonary syndrome is prominent, it may be confused with hantavirus.

5. HOW IS THE DISEASE MANAGED AND PREVENTED? MANAGEMENT

Antimicrobials As previously indicated, the majority of cases of leptospirosis are self-limiting and do not require antimicrobial therapy. If the illness is severe enough to cause clinically recognized symptoms and diagnosis is made, antibiotic therapy needs to be started to reduce the duration of illness and shedding of organisms in the urine. Mild disease: patients are treated with doxycycline or azithromycin, ampicillin or amoxicillin, clarithromycin, fluoroquinolone such as ciprofloxacin or levofloxacin. These antibiotics also have activity against rickettsial disease, which can often be confused with leptospirosis. Severe disease: for hospitalized patients, IV penicillin, and oral doxycycline for up to 7 days. Ceftriaxone or cefotaxime can also be given. A Jarisch-Herxheimer reaction may occur following antimicrobial therapy for leptospirosis and is characterized clinically by fever, rigors, and hypotension. This reaction is due to the release of endotoxin when large numbers of organisms are killed by the antibiotics leading to increase in immune complexes and the release of cytokines, particularly TNF. This reaction also occurs typically with syphilis and Lyme disease. Supportive measures may be required, such as hemodialysis, in severe disease.

There is insufficient data evidence for the routine use of IV steroids but it has been proposed given the vasculitic nature of severe leptospirosis, especially in the setting of pulmonary involvement. Some data have indicated a benefit for the use of steroids as an adjunct to antibiotic therapy in severe disease but further study is needed.

PREVENTION Prevention includes avoiding potential sources of infection, prophylaxis for individuals at high risk of exposure, and ani mal vaccination. The most important control measures for preventing human leptospirosis include avoiding potential sources of infection such as stagnant water and animal farm water runoff, rodent control, and protection of food from animal contamination. I n a study of antimicrobial prophylaxis with doxycycline for individuals at high risk of exposure, fewer cases of clinical leptospirosis were observed in the antibiotic-treated groups of soldiers on jungle exercises versus the placebo group.

Vaccines The real challenge is to produce a universal vaccine against all of the >300 serovars of Leptospira. After many years of research, this has not yet been achieved. Inactivated whole cells (bacterins) are the only vaccines commercially available, primarily for veterinary use. Vaccines that have been produced are mainly serovar dependent and based on lipopolysaccharide antigens. In addition, inactivated vaccines do not promote long-term protection and require annual boosters. Safety has also been reported as an issue. Vaccination of domestic and farm animals against leptospirosis has been shown to give variable levels of protection with some immunized animals becoming infected and excreting leptospires in their urine. Many approaches are being made to produce effective vaccines and these have been helped by the WGS of leptospires. These include live vaccines, multiepitope vaccines against some OMP, DNA vaccines, and use of NOD-1 and NOD-2 agonists. The development of mRNA vaccines (recently used in SARS-2 vaccines) coding for multiple-surface proteins is also predicted to be an attractive future approach for creating a successful vaccine.

206

9780367696399_Lydyard.indb 206

13/02/2023 07:32

Case 21: Leptospira spp.

SUMMARY 1. WHAT IS THE CAUSATIVE AGENT, HOW DOES IT ENTER THE BODY, AND HOW DOES IT SPREAD A) WITHIN THE BODY AND B) FROM PERSON TO PERSON? ■ Two main identified species exist: L. interrogans and L. biflexa. ■ There are two clades – Pathogens and Saprophytes. ■ Leptospirosis is a zoonosis, the organism is carried by many animals and excreted in the urine. ■ Infection is acquired by contact with contaminated water or soil.

3. WHAT IS THE TYPICAL CLINICAL PRESENTATION AND WHAT COMPLICATIONS CAN OCCUR? ■ ■ ■ ■

The incubation period is about 10 days but can be 5–30 days. The clinical presentation is classically biphasic. The initial illness is nonspecific with fever and myalgia. The second phase of the illness may present as meningitis, pneumonia, jaundice or renal failure. ■ Complications include cardiac arrhythmia, Guillain-Barré syndrome and renal failure.

■ The organism enters through the conjunctiva, mucosa or skin abrasions. ■ Leptospires spread to all body locations by the blood.

2. WHAT IS THE HOST RESPONSE TO THE INFECTION AND WHAT IS THE DISEASE PATHOGENESIS? ■ TLR2 senses outer-membrane protein LipL32 and NODlike receptors sense peptidoglycan fragments of leptospires. Stimulation through these receptors results in production of proinflammatory cytokines TNF , IL1β and IL8 leading to recruitment of neutrophils, macrophages and dendritic cells to the site of infection. ■ The antimicrobial peptide cathelicidin released by neutrophils is effective against some leptospires. Neutrophils also release myeloperoxidase and ROS. ■ Complement is important as a host mechanism against leptospires, especially against the non-pathogenic serovars (saprophytes). ■ A strong antibody response is made against leptospire LPS. Both IgM and IgG antibodies are made against a variety of specific antigens including LipL32, LipL41, and LigA. The major role of IgG seems to be in opsonization. ■ Leptospires, especially those that are pathogenic, have many ways of evading the immune system including: inhibition of degradation of PG into muropeptides that are not recognized by NOD-1 and NOD-2 receptors; subversion or inactivation of complement pathways. ■ Pathogenesis: around 10% of leptospirosis cases develop into severe forms. Pathogenic leptospires cause vasculitis in many organs. Liver, lungs, heart, meninges, kidneys, and muscles are all affected; Thrombocytopenia can occur.

4. HOW IS THE DISEASE DIAGNOSED, AND WHAT IS THE DIFFERENTIAL DIAGNOSIS? ■ Leptospires can be detected in the blood, CSF or urine by darkground microscopy. ■ The organism can be cultured from the blood, CSF or urine using EMJH medium incubated at 30°C for 6–8 weeks. ■ Serologically, the disease can be diagnosed by the macroscopic agglutination test (MALT) or the IgM ELISA. ■ Various genomic assays exist: PCR, RT-PCR (real-time PCR), LAMP, and WGS. ■ The differential diagnosis includes Dengue virus, viral meningitis, viral hepatitis, malaria, hantavirus and Legionnaires disease.

5. HOW IS THE DISEASE MANAGED AND PREVENTED? ■ Most cases of leptospirosis are self-limiting. ■ Patients with mild disease are treated with doxycycline or azithromycin. ■ Hospitalized patients with severe cases are given IV penicillin, and oral doxycycline and ceftriaxone or cefotaxime is given for up to 7 days. ■ A Jarisch-Herxheimer reaction may occur following antimicrobial therapy and is characterized clinically by fever, rigors, and hypotension. ■ Prevention is by identifying risks, antimicrobial prophylaxis for individuals at high risk, and animal immunization (although shown to have variable levels of protection). ■ Vaccines pose a major challenge because of the many different serovars. There are many new approaches, helped by WGS, and include live vaccines against some OMP, DNA vaccines, and the use of NOD-1 and 2 agonists and mRNA.

207

9780367696399_Lydyard.indb 207

13/02/2023 07:32

Case Studies in Infectious Disease

FURTHER READING Bennett JE, Blaser MJ, Dolin R. Mandell, Douglas, and Bennett’s Principles and Practice of Infectious Diseases, 9th edition. Elsevier, Philadelphia, 2020. Goering R, Dockrell H, Zuckerman M, Chiodini PL (eds). Mims’ Medical Microbiology and Immunology, 6th edition. Elsevier, Cambridge, 2018.

REFERENCES Barazzone GC, Teixeira AF, Azevedo BOP, et al. Revisiting the Development of Vaccines Against Pathogenic Leptospira: Innovative Approaches, Present Challenges, and Future Perspectives. Front Immunol, 12: 760291, 2022. Bierquel E, Thibeaux R, Girault D, et al. A Systematic Review of Leptospira in Water and Soil Environments. PLoS ONE, 15 e0227055, 2020. Caimi K, Ruybal P. Leptospira, a Genus in the Stage of Diversity and Genomic Data Expansion. Infect Genet Evol, 81: 104241, 2020. Chin VK, Basir R, Nordin SA, et al. Pathology and Host Immune Evasion During Human Leptospirosis: A Review. Int Microbiol, 23: 127–136, 2020. Haake DA, Levett PN. Leptospirosis in Humans. Curr Top Microbiol Immunol, 387: 65–97, 2015. Jiménez JIS, Marroquin JLH, Richards GA, Amin P. Leptospirosis: Report from the Task Force on Tropical Diseases by the World Federation of Societies of Intensive and Critical Care Medicine. J Crit Care, 43: 361–365, 2018. Karpagam KB, Ganesh B. Leptospirosis: A Neglected Tropical Zoonotic Infection of Public Health Importance—An Updated Review. Eur J Clin Microbiol Infect Dis, 39: 835–846, 2020. Levett PN, Branch SL. Evaluation of Two Enzyme Linked Immunosorbent Assay Methods for Detection of

Immunoglobulin M Antibodies in Acute Leptospirosis. Am J Trop Med Hyg, 66: 745–748, 2002. Pinto GV, Senthilkumar K, Rai P, et al. Current Methods for the Diagnosis of Leptospirosis: Issues and Challenges. J Microbiol Methods, 195: 106438, 2022. Santecchia I, Ferrer MF, Vieira MR, et al. Phagocyte Escape of Leptospira: The Role of TLRs and NLRs. Front Immunol, 11: 571816, 2020. Vincent AT, Schiettekatte O, Goarant C, et al. Revisiting the Taxonomy and Evolution of Pathogenicity of the Genus Leptospira Through the Prism of Genomics. PLoS Negl Trop Dis, 13: e0007270, 2019. Wagenaar JFP, Goris MGA, Sakudarno MS, et al. What Role Do Coagulation Disorders Play in the Pathogenesis of Leptospirosis. Trop Med Int Health, 12: 111–122, 2007.

WEBSITES Centers for Disease Control and Prevention, Leptospirosis, 2019: https://www.cdc.gov/leptospirosis/ European Centre for Disease Prevention and Control, Leptospirosis: https://www.ecdc.europa.eu/en/leptospirosis GOV.UK, Leptospirosis, 2013: https://www.gov.uk/guid ance/leptospirosis Medscape, Leptospirosis, 2021: https://emedicine.med scape.com/article/220563-overview The Leptospirosis Information Center, 2008: http://www. leptospirosis.org/

Students can test their knowledge of this case study by visiting the Instructor and Student Resources: [www. routledge.com/cw/lydyard] where several multiple choice questions can be found.

208

9780367696399_Lydyard.indb 208

13/02/2023 07:32

22

Listeria monocytogenes

Figure 22.1 Colonies of Listeria monocytogenes on blood agar. From CC Studio / Science Photo Library, with permission.

1. WHAT IS THE CAUSATIVE AGENT, HOW DOES IT ENTER THE BODY AND HOW DOES IT SPREAD A) WITHIN THE BODY AND B) FROM PERSON TO PERSON?

A young woman was admitted to hospital with fever, headache, myalgia, and joint pains of 48 hours duration. The previous day, she had attended a lunch where she had eaten ham and a soft cheese. She was 26 weeks pregnant. The admitting physician thought the patient may have listeriosis and a blood culture was taken in addition to a full blood count. Gram-positive cocci were reported on the Gram stain of the blood culture and the patient was started empirically on teicoplanin. The following day, small hemolytic colonies were present on the blood agar (Figure 22.1) and a Gram stain revealed gram-positive rods. Further testing showed the organism was motile with tumbling motility and it was biochemically identified as Listeria monocytogenes. A diagnosis of listeriosis was made and the patient’s treatment was changed to ampicillin and gentamicin.

The most common cause of human disease is L. monocytogenes, although L. ivanovii can rarely cause disease. Listeria species grow on blood agar (see Figure 22.1) and L. monocytogenes produces a narrow zone of β-hemolysis often only seen beneath the colonies. The organism grows at 37°C but can also grow slowly at 4°C and this can be used as an

CAUSATIVE AGENT

Listeria monocytogenes is the cause of both animal and human disease and is widespread in the environment. As such, the organism is a good example of the “One Health” approach to medicine, as agreed at the G7 summit in 2016. Listeria monocytogenes is a gram-positive nonsporing motile bacillus (Figure 22.2). As of 2020, there were 21 species of Listeria identified and of these six species form a phenotypic homogenous group (Listeria sensu strictu); the remainder, Listeria sensu lato, may be better placed into a new Genus. They are phenotypically different from Listeria notably that they cannot grow below 7°C. The clade Listeria sensu strictu comprise L. monocytogenes, L. ivanovii, L. welshimeri, L. innocua, L. seeligeri, and L. marthii. However, in 2021, 27 isolates that grouped into five clusters representing five novel Listeria species were identified from soil and water samples across the US. Three of the new species, L. cossartiae, L. farberi, and L. immobilis belong to the sensu stricto clade and the other two belong to the sensu lato clade.

Figure 22.2 Scanning electron microscopic (EM) image of Listeria monocytogenes showing flagella. From AB Dowsett / Science Photo Library, with permission.

209

9780367696399_Lydyard.indb 209

13/02/2023 07:32

Case Studies in Infectious Disease

enrichment technique when examining foodstuff. Listeria can survive freezing which is relevant to the food-chain and infection. The organism shows a characteristic tumbling (end-over-end) movement at 25°C, which is diagnostic. L. monocytogenes has 14 serotypes based on cell wall (O) and flagellar (H) antigens and divided into three divisions I, II, and III. The majority of disease is caused by serotypes 1/2a, 1/2b, and 4b. Several molecular subtyping techniques (multilocus enzyme electrophoresis, pulse field gel electrophoresis, and ribotyping) have been found useful in epidemiologic investigations.

SOURCE OF INFECTION L. monocytogenes has been isolated from a variety of natural sources including soil, water, animals, and vegetables and is widely distributed in the animal kingdom, with over 40 species of wild and food-source animals including birds, crustaceans, and fish being colonized. The organism can also be carried in the intestine of about 5% of the human population without any symptoms of disease. Infection is acquired by consumption of contaminated food (Figure 22.3) such as fish, salad, pâté, soft cheeses, salami, ham, and coleslaw, where contamination rates as high as 70% may occur. Ingestion of Listeria probably occurs frequently and the organism can be carried in the intestine of the human population for short periods without any symptoms of disease. Disease develops mainly in specific groups of individuals including pregnant women, neonates, and immunocompromised patients (see Section 3).

ONE HEALTH In line with the One Health approach to disease, multi­ disciplinary research groups have investigated Listeria monocytogenes in horticulture, the Fresh Leafy Produce Supply Chain (FLPSC), animal welfare, and human disease. An environmental study showed that L. monocytogenes was detected in 0.7% of urban soils, 24% of soils on ruminant farms, and 9% in fruit and vegetable farms. Other studies have demonstrated Listeria was able to survive for at least 84 days in 71 different soil types, but not in 29 other soil types – the total set representing all the different soil types in France. Short-term survival (106 per ml for detection. Many decades ago, immunoelectron microscopy showing antibody seroconversion was used to prove that norovirus detected in feces was the cause of gastroenteritis (Figure 27.9A and 27.9B) and is still used today in the research setting to concentrate virus.

DIFFERENTIAL DIAGNOSIS T he other possible viral causes are given in Table 27.1 – notably outbreaks of diarrhea are commonly caused by rotavirus in young children and the elderly. Fecal samples should be tested for Salmonella, Shigella, and Campylobacter spp. In addition, Clostridium difficile can cause serious outbreaks with high morbidity and even mortality in hospitals.

5. HOW IS THE DISEASE MANAGED AND PREVENTED? MANAGEMENT

The illness is usually short and requires attention only to fluid and electrolyte replacement. Severe cases of persistent

13/02/2023 07:32

Case 27: Norovirus

Table 27.1 Viruses causing gastroenteritis and their epidemiologic features Family

Virus

Epidemiologic features Endemic Infection in children

Outbreaks of infection in all ages

Group A rotavirus

++++

Children and elderly in hospitals or residential homes

No

Group B rotavirus



Large outbreaks in China

No

Group C rotavirus



Occur but uncommon

No

Noroviruses

+++

Major cause of outbreaks

Yes

Sapoviruses

+/-



Yes

Adenovirus

Adenovirus types 40 and 41

++



No

Astrovirus

Human astrovirus

+

Family outbreaks

No

Caliciviruses

reduced risk of spread

Food- or water-borne

increased risk of spread virus variant/mutant

ward closure cohort nursing adherence to hand

affected patients

washing practices, soap and water

low infectoius dose high attack rate staff affected (reduced staffing)

elbow/automatic taps on handbasins single rooms with en suite

projectile vomiting

soft furnishings contaminated environment

surface finishes allowing decontamination

hand contact surfaces contaminated unrestricted visiting/admissions open wards

Figure 27.10 Risk of norovirus spread within the hospital environment. Courtesy of Dr Jim Gray.

diarrhea in the immunosuppressed may also require enteral or parenteral nutrition. There is no antiviral drug available for treatment.

OUTBREAK CONTROL

Figure 27.9 Immunoelectron microscopy to show antibody seroconversion. Noroviruses (size 27nm) from a patient with gastroenteritis visualized by electron microscopy (A) after incubation with serum taken from the patient in the acute phase of illness and (B) after incubation with serum taken from the patient in the convalescent phase of illness. Specific antibody molecules in the convalescent serum of the patient aggregate and coat the virus particles. Reprint permission kindly given by Dr Jim Gray of the Health Protection Agency (originator of the image) and by Elsevier.

As illustrated in the example given of an outbreak in a residential home, this is a difficult issue and outbreaks can grumble on for long periods and often do not end until most susceptible people have been infected. Because they lack an envelope, noroviruses are very resistant to adverse environmental conditions and even to commonly used disinfectants – this property facilitates their propensity to cause outbreaks (see below). Control measures (Figure 27.10) rely on:

Case_27_10

263

9780367696399_Lydyard.indb 263

13/02/2023 07:32

Case Studies in Infectious Disease

1. Limiting contact between infected and susceptible patients,

by isolation of those affected in single rooms in hospital or nursing homes, and using contact (enteric) precautions – gloves, aprons, and scrupulous handwashing with soap and water as alcohol hand rubs are insufficient. Closing the hospital ward or relevant section of a nursing home to new admissions may be necessary. If the ward is closed, it should not open until more than 48 hours (the longest incubation period) – usually 72 hours. 2. Infected staff should stay off work for at least 48 hours after

resolution of symptoms. This is particularly important for those handling food. Note – although the virus is shed in feces over about 3 weeks, the virus is most communicable in the acute phase of illness when the virus load is highest.

PREVENTION

This relies not only on clean drinking water and efficient sewage disposal but also on good standards of personal and food hygiene plus adequate cleaning arrangements in hospitals and residential homes. Raw shellfish should be cooked before consumption and fruit washed if to be eaten uncooked. Vaccine: An effective vaccine would give substantial economic and public health benefits. Currently, candidate vaccines are based on VLPs consisting of recombinant capsid proteins that are morphologically and antigenically identical to native virus and react specifically with convalescent human sera. Norovirus VLPs have been shown to be immunogenic when administered orally to human volunteers, inducing serum IgG and mucosal IgA. It seems likely that, as with the influenza vaccine, formulation would need to be changed as new genotypes of human noroviruses emerge.

SUMMARY 1. WHAT IS THE CAUSATIVE AGENT, HOW DOES IT ENTER THE BODY, AND HOW DOES IT SPREAD A) WITHIN THE BODY AND B) FROM PERSON TO PERSON?

■ Intestinal enterocytes appear to be the primary sites of viral infection. Viral antigen has been detected in macrophages, dendritic cells and T cells in the lamina propria cells; the significance of this is unclear.

■ Norovirus – an unenveloped single stranded RNA virus.

3. WHAT IS THE TYPICAL CLINICAL PRESENTATION AND WHAT COMPLICATIONS CAN OCCUR?

■ Noroviruses were formerly known as Norwalk or small, round, structured viruses. ■ Three norovirus genogroups, GI, GII, and GIV, subdivided into about 30 genotypes infect humans. ■ Norovirus infection has a seasonal incidence – “winter vomiting disease”. ■ New variants of norovirus genotypes regularly appear causing an increased number of outbreaks including summer out of season. ■ Noroviruses are spread by the fecal-oral route. ■ The main source of virus is another infected person, contaminated food, or water. ■ Norovirus outbreaks are common in healthcare settings and cruise ships.

2. WHAT IS THE HOST RESPONSE TO THE INFECTION AND WHAT IS THE DISEASE PATHOGENESIS? ■ Immunity is short-lived and re-infection is common. Very little is known regarding the pathogenesis of norovirus infection. ■ The cause of the delay in gastric emptying responsible for the high incidence of vomiting episodes is unknown.

■ Self-limiting vomiting and diarrhea. ■ Main complication is dehydration. ■ Causes persistent diarrhea in the immunocompromised.

4. HOW IS THE DISEASE DIAGNOSED, AND WHAT IS THE DIFFERENTIAL DIAGNOSIS? ■ The virus cannot be routinely grown in cell culture, so diagnosis relies on reverse transcription PCR to detect viral nucleic acid in vomit or feces. ■ Differential diagnosis includes infection with rotavirus, Salmonella, Shigella, and Campylobacter spp. and, in the hospital setting, Clostridium difficile.

5. HOW IS THE DISEASE MANAGED AND PREVENTED? ■ There is no specific treatment. ■ There is no vaccine. ■ Control of outbreaks includes reinforcing good hygiene, closure of hospital wards, and isolation of ill persons until 48 hours after symptoms have resolved.

264

9780367696399_Lydyard.indb 264

13/02/2023 07:32

Case 27: Norovirus

FURTHER READING Dolin R, Treanor JJ. Noroviruses and Sapoviruses (caliciviruses). In: Bennett JE, Dolin R, Blaser MJ, editors. Mandell, Douglas and Bennett’s Principles and Practice of Infectious Diseases, volume 2, 8th edition. Elsevier/Saunders, Philadelphia, 2122–2127, 2015. Goering RV, Dockrell HM, Zuckerman M, Chiodini PL. Medical Microbiology and Immunology, 6th edition. Elsevier, Philadelphia, 2019. Wobus CE, Green KY. Caliciviridae: The Viruses and their Replication. In: Howley PM, Knipe DM, editors. Fields Virology, 7th edition. Wolters Kluwer, Philadelphia, 2021.

REFERENCES Atmar RL, Ramani S, Estes MK. Human Noroviruses: Recent Advances in a 50-Year History. Curr Opin Infect Dis, 31: 422–432, 2018. Chong PP, Atmar RL. Norovirus in Healthcare and Implications for the Immunocompromised Host. Curr Opin Infec Dis, 32: 348–355, 2019. Ford-Siltz LA, Tohma K, Parra GI. Understanding the Relationship Between Norovirus Diversity and Immunity. Gut Microbes, 13: 1–13, 2021. Glass RI, Parashar UD, Estes MK. Norovirus Gastroenteritis. N Engl J Med, 36: 1776–1785, 2009. Green KY, Kaufman SS, Nagata BM, et al. Human Norovirus Targets Enteroendocrine Epithelial Cells in the Small Intestine. Nat Commun, 11: 2759, 2020.

Karandikar UC, Crawford SC, Ajami NJ, et al. Detection of Human Norovirus in Intestinal Biopsies from Immunocompromised Transplant Patients. J Gen Virol, 97: 2291–2300, 2016. Lopman B, Vennema H, Kohli E, et al. Increase in Viral Gastroenteritis Outbreaks in Europe and Epidemic Spread of New Norovirus Variant. Lancet, 363: 682–688, 2004. Roddie C, Paul J, Benjamin R, et al. Allogeneic Haematopoietic Stem Cell Transplantation and Norovirus Gastroenteritis: A Previously Unrecognised Cause of Morbidity. Clin Infect Dis, 49: 1061–1068, 2009.

WEBSITES Centers for Disease Control and Prevention, Morbidity and Mortality Weekly Report, 2004: http://www.cdc.gov/mmwr/ preview/mmwrhtml/rr5304a1.htm Guidelines for the management of norovirus outbreaks in acute and community health and social care settings, 2012: https://assets.publishing.service.gov.uk/government/uploads/ system/uploads/attachment_data/fi le/322943/Guidance_for_ managing_norovirus_outbreaks_in_healthcare_settings.pdf

Students can test their knowledge of this case study by visiting the Instructor and Student Resources: [www. routledge.com/cw/lydyard] where several multiple choice questions can be found.

265

9780367696399_Lydyard.indb 265

13/02/2023 07:32

28

Plasmodium spp.

A 26-year-old model went to see her doctor about 1 week after returning from a job in the Gambia. She complained of an abrupt onset of bouts of shivering and feeling cold, vomiting, rigors, and profuse sweating accompanied by a headache and nausea.

1. WHAT IS THE CAUSATIVE AGENT, HOW DOES IT ENTER THE BODY AND HOW DOES IT SPREAD A) WITHIN THE BODY AND B) FROM PERSON TO PERSON? CAUSATIVE AGENT

The organism causing malaria is Plasmodium, a eukaryotic protozoan that infects the erythrocytes of humans. It has the characteristics of eukaryotes, with a nucleus, mitochondria, endoplasmic reticulum, and so forth. There are approximately 156 named species of Plasmodium which infect various vertebrates. Five species of Plasmodium are able to infect humans: P. falciparum, P. ovale, P. vivax, P. malariae and P. knowlesi. P. falciparum is the most virulent species of malaria but P. vivax is the dominant malaria parasite in most countries outside of sub-Saharan Africa. All of these species have similar life cycles in which the organisms undergo both sexual and asexual reproduction in the vector and host and alternate between intracellular and extracellular forms. The female Anopheles mosquito is the vector for malaria. The risk of malaria transmission is therefore restricted to those areas where mosquitoes can breed and where the parasite can develop within the mosquito – see Epidemiology below.

On examination, she was noted to be pale with a temperature of 39.5°C and had tachycardia. She gave a history of having taken anti-malarial tablets before and during her stay in the Gambia but was admitted to hospital with a provisional diagnosis of malaria.

Liver Stage (Pre-Erythrocytic Stage) T he blood-borne sporozoites localize in the liver via the sinusoids, where through interactions between their surface circumsporozoite protein (CSP – most abundant protein on the sporozoite surface) and highly sulfated heparan sulfate proteoglycans (HSPGs), they cross the sinusoidal cellular bar rier through liver sinusoidal endothelial cells or Kupffer cells (KC). The sporozoites actively enter the hepatocytes (invade – rather than are taken up passively by endocytosis), and may traverse several hepatocytes before they switch to a replicative form. There, within a parasitophorous vacuole (to avoid lysosomal degradation), they increase in number and develop into schizonts. This asexual stage takes up to 2 weeks. Rupture of the liver cells releases the schizonts into the bloodstream as merozoites (with about 10–40 000 being released from the liver). P. vivax and P. ovale also produce a resting stage within the liver cell called hypnozoites. In this case, some sporozoites once in the liver do not develop immediately into schizonts, but remain at an uninucleate stage, in a quiescent form named hypnozoite, before resuming

ENTRY AND SPREAD WITHIN THE BODY The transmission stage of Plasmodium is the sporozoite, which is injected into the bloodstream of a human when the female Anopheles mosquito takes a blood meal (Figure 28.1). The detailed life cycle is shown in Figure 28.2. Following the mosquito bite, at least some of the sporozoites remain in the dermis for some time before crossing the endothelial cells and entering the bloodstream; some pass directly into draining lymph nodes. Only a few dozen sporozoites are transmitted during feeding but there is rapid translocation into the liver to begin the first stage of disease.

Figure 28.1 Anopheles funestus mosquito taking a blood meal from its human host. This mosquito species, together with Anopheles gambiae, is one of the two most important malaria vectors in Africa. Note the blood passing through the proboscis. From the Centers for Disease Control & Prevention, Atlanta, Georgia. Image is found in the Public Health Image Library #7192. Additional photographic credit is given to James Gathany, Dr Frank Collins and the University of Notre Dame. The image was taken in 2005 by James Gathany.

267

9780367696399_Lydyard.indb 267

13/02/2023 07:32

Case Studies in Infectious Disease

Figure 28.2 The lifecycle of Plasmodium. (1) The mosquito injects saliva containing sporozoites as it takes a blood meal and the parasite localizes in the liver (liver stage), where it undergoes a stage of development to produce a schizont which contains developing merozoites, in the infected liver cell (2). P. vivax and P. ovale also produce a resting stage within the liver cell called hypnozoites, which can persist in the liver and result in relapses months or even years later. The dead liver cell breaks open and the shizont ruptures producing small vacuoles (merosomes) which release their merozoites into the bloodstream. These invade erythrocytes (erythrocytic stage) and undergo developmental stages as trophozoites, which mature and produce schizonts, at which stage, the erythrocyte bursts rupturing the shizonts to release further merozoites (4). Further cycles of asexual development within uninfected erythrocytes occur, releasing more merozoites to infect further erythrocytes. Differentiation of the immature trophozoite into male and female gametocytes occurs in some erythrocytes (5) and these are ingested when a mosquito takes a blood meal. The male (microgametocyte – exflagellated) fertilizes the female macrogametocyte (6) to form a zygote within the intestine of the mosquito (vector stage) and this becomes an ookinete that invades the intestinal wall where it develops into an oocyte (7). The oocyte matures into sporozoites, which are released and migrate to the salivary gland of the mosquito. Here, they will be transmitted to a new human host when the mosquito takes a blood meal and the cycle starts again (1). From the Centers for Disease Control & Prevention, Atlanta, Georgia. Image is found in the Public Health Image Library #3405. Additional photographic credit is given to Alexander J da Silva, PhD and Melanie Moser. The image was created in 2002.

hepatic development induced by as yet unknown factors that cause relapses weeks, months or even years after the primary infection. The sporozoite has small vacuoles (micronemes) that release substances onto their surface in a co-ordinated way to ensure successful migration to and invasion of hepatocytes and production of merozoites.

Erythrocyte Stage

268

Merozoites (the smallest extracellular parasite form) invade and destroy erythrocytes giving rise to symptoms (see Section 3). Invasion is a multistep process that includes firstly

9780367696399_Lydyard.indb 268

binding of the merozoite, reorientation, discharge of secretory organelles (called rhoptries and micronemes), formation of an electron-dense “tight junction” between the merozoite apical end and the erythrocyte membrane. This leads to actinomyosin-powered entry into the erythrocyte with formation of a membrane-bound parasitophorous vacuole and resealing of the erythrocyte cell membrane. Within this vacuole, the parasite feeds on hemoglobin and multiplies. Invasion takes less than 30 seconds. Entry of the merozoites into erythrocytes is achieved through attachment of a number of surface molecules (merozoite surface proteins (MSPs) that

13/02/2023 07:32

Case 28: Plasmodium spp.

are anchored by glycosylphosphatidylinositol (GPI), and a number of erythrocyte binding ligands (EBL). At least four have been identified for P. falciparum merozoites. It is generally accepted that glycophorins A, B, C, and D are important “receptors” on erythrocytes and are known for P. falciparum Glycophorin A on erythrocytes. Another important structure on erythrocytes is band 3 protein that binds MSP1 complex on P. falciparum merozoites. P. vivax has a specific reticular binding protein to enable it to attach and invade reticulocytes but not mature erythrocytes. In addition, P. vivax has surface molecules (Duffy binding proteins – DBP-1) that bind to Duffy blood group antigens on the erythrocytes. Although initially thought that DBP-1 was essential for binding of P. vivax to erythrocytes, some cases have been seen where infection occurs in individuals that genetically lack DBP-1. The search is on for other receptors for P. vivax merozoites on reticulocytes. Within the erythrocyte, the merozoites undergo further development as a trophozoite (seen as a “ring” stage – see Figure 28.2) and then undergo asexual reproduction to produce schizonts, at which stage, the erythrocyte bursts releasing merosomes containing 16–32 daughter merozoites into the bloodstream. Each asexual cycle takes 44–48 hours and is followed by cell rupture and re-invasion steps that induce periodic waves of fever in the patient (see Figure 28.3 and Section 3). This erythrocytic cycle may continue for months or years. However, in some erythrocytes, the trophozoites differentiate into male and female gametocytes and a mosquito taking a blood meal will take up some of the gametocyte-containing erythrocytes, heralding the sexual developmental phase in the vector.

Vector Stage Within the intestine of the insect, male (exflagellated microgametocytes) and female gametes (macrogametocytes)

fuse to become a zygote. These become an ookinete, which then invades the intestinal wall where it develops into an oocyte. The oocyte develops into thousands of sporozoites, which then migrate to the mosquito’s salivary gland.

PERSON-TO-PERSON SPREAD The sporozoites are injected into an individual when an infected female Anopheles mosquito feeds and the whole cycle starts again.

NON-MOSQUITO SPREAD Malaria can also be transmitted through blood transfusion, hypodermic needle sharing or accidents, and from mother to fetus.

EPIDEMIOLOGY Malaria transmission occurs in five WHO regions. Globally, an estimated 3.4 billion people in 92 countries are at risk of being infected with malaria and developing disease (see map – the atlas project website), and 1.1 billion are at high risk (>1 in 1000 chance of getting malaria in a year). There was a marked reduction in global malaria case incidence and mortality rates between 2000 and 2019. The malaria case incidence rate (cases per 1000 population at risk) fell from 80 in 2000 to 57 in 2019. Total malaria cases declined from 238 million in 2000 to 227 million in 2019. The mortality incidence rate (deaths per 100 000 population at risk) was reduced from 25 in 2000 to 10 in 2019. The total number of deaths fell from 736 000 in 2000 to 409 000 in 2019. Children under 5 years of age are the most vulnerable group affected by malaria and in 2019 they accounted for 67% (274 000) of all malaria deaths worldwide. Africa continues to carry a disproportionately high share of the global malaria burden. In 2019, the region had 94%

Figure 28.3 Cyclical fever coincident with the release of merozoites. Adapted from Goering R, Dockrell H, Zuckerman M et al. (2008) Mim’s Medical Microbiology, 4th edition, Figure 27.11. With permission from Elsevier.

9780367696399_Lydyard.indb 269

269

13/02/2023 07:32

Case Studies in Infectious Disease

of all malaria cases and deaths. Six countries accounted for approximately half of all malaria deaths worldwide: Nigeria (23%), the Democratic Republic of the Congo (11%), United Republic of Tanzania (5%), Burkina Faso (4%), Mozambique (4%) and Niger (4%). In 2020, there was a slight increase in cases worldwide from 227 (in 2019) to 241 million cases. The estimated number of malaria deaths stood at 627 000 in 2020. The WHO African Region carries a disproportionately high share of the global malaria burden. In 2020, the WHO African region was home to 95% of the malaria cases and 96% of malaria deaths. With increasing international travel, prior to the SARS-2 (COVID-19) pandemic there continued to be a rise in the number of cases of malaria in travelers returning to non­ malarious areas from countries where malaria is endemic. In Europe, in 2018, 8349 malaria cases were reported, 8347 (> 99%) of which were confirmed. Among 7338 cases with known importation status, 99.8% were travel related. As in previous years, the overall rate of confirmed malaria cases was higher among men than women (1.6 cases and 0.7 cases per 100 000 population, respectively; male-to-female ratio 1.9:1). However, the travel-related cases were reduced during the SARS-CoV-2 pandemic due to limited travel. Pregnancy is associated with a higher risk of malaria. An estimated 10 000 pregnant women and 200 000 of their infants died annually in sub-Saharan Africa as a result of malaria infection during pregnancy. HIV-infected pregnant women are at increased risk.

HUMAN GENETIC FACTORS THAT DECREASE THE INFECTION RATES OF PLASMODIUM As already mentioned, the absence of DBPs in most West Africans prevents most infections by P. vivax, since it uses the Duffy blood group antigen-1 as a means of attachment. Sickle cell trait (heterozygous for HbS with HbA) gives an increasing amount of immunologic protection against malaria for young children during their first 10 years of life (a 29% reduction in malaria incidence). A number of mechanisms have been proposed to explain malaria resistance, including sickling of the infected red blood cells, increased splenic phagocytosis, premature hemolysis and parasite death, impaired hemoglobin digestion, weakened cytoadherence, acquired host immunity, translocation of HbS-specific parasite-growth inhibiting microRNAs, and induction of heme-oxygenase-1. Glucose6-phosphate dehydrogenase (G6PD) deficiency confers resistance to malaria. Although the mechanism is unclear, it has been found that when the parasites are at the ring stage, erythrocytes deficient in the enzyme were phagocytosed 2–3 times more intensely than normal ring-stage erythrocytes. There was, however, no difference when the parasites were at the more mature trophozoite stage.

The Impact of the SARS-CoV-2 (COVID-19) Pandemic on Malaria During the pandemic, a recent modeling analysis by the WHO predicted a >20% rise in malaria morbidity and >50% mortality in sub-Saharan Africa as a result of 75% reduction in routine malaria control measures. These might significantly increase the longer the pandemic goes on. Other indirect effects of the pandemic, particularly those that affect people’s lives and well-being, such as increased malnutrition, poverty, and social instability, may further influence malaria burden. Current guidelines by WHO at the time of writing have included the continuation of all routine malaria control measures while adhering to Covid-19 local personal and physical distancing guidelines established by the authorities.

2. WHAT IS THE HOST RESPONSE TO THE INFECTION AND WHAT IS THE DISEASE PATHOGENESIS? The understanding of the immune response to Plasmodium is far from complete. Some immunity does develop to the parasite with continuous infection (especially to blood-stage parasites) and children who survive early attacks become resistant to severe disease by about 5 years of age. Levels of parasites fall progressively until adulthood, when they are low or absent most of the time. This immunity, however, appears to be lost after spending a year away from exposure, presumably due to lack of repeated antigenic stimulation needed for its maintenance. IgG antibodies to blood-stage parasites are transferred across the placenta and limit parasitemia and severe disease in neonates. Thus, although immune responses do develop to different stages of the lifecycle in infected individuals, they are weak and cannot eradicate the parasite.

PRINCIPAL MECHANISMS THAT ARE THOUGHT TO BE RESPONSIBLE FOR IMMUNITY AT DIFFERENT STAGES OF THE LIFECYCLE OF PLASMODIUM Liver Stage • Initial entry of the sporozoites into the lymph nodes and bloodstream on the way to the liver: antibodies. • Evidence for antibodies to sporozoites and CSP to block binding to hepatocytes. • Activation of complement by the antibodies activates complement fixation, phagocytosis, and lysis by cytotoxic NK and NKT cells through their Fc receptors. It also recognizes parasite neoantigens at the surface of infected hepatocytes and kills through an antibody-dependent cell-mediated mechanism by KC and NK cells.

270

9780367696399_Lydyard.indb 270

13/02/2023 07:32

Case 28: Plasmodium spp.

• Since the liver stage of malaria infection is clinically silent, it has long been thought that parasites inside hepatocytes grow undetected by the innate immune system. However, a number of pattern recognition receptors (PRR) inside hepatocytes are stimulated leading to production of IFN- . DNA of the parasite probably acts as a (pathogen) microbe-associated molecular pattern (MAMP) in the hepatocyte cytosol. This results in recruitment of macrophages, neutrophils, and lymphocytes. • Cytotoxic CD8+ T cells that produce interferon- are mainly involved in killing of intrahepatic parasites. NK, NKT, and T cells also kill intrahepatic parasites through secretion of type I interferons and IFN- . T his stage is “silent” without any clinical symptoms.

Erythrocyte Stage • Merozoites in the bloodstream are targeted by antibodies to their erythrocyte attachment ligands, for example GPI anchoring MSPs (see Section 1) that can both opsonize and prevent entry. • GPI is also a major MAMP, recognized by the PRR, Toll-like receptor 2 (TLR 2) on macrophages that induces pro-inflammatory cytokines such as IL-1 and TNF . • At the intra-erythrocyte stage, antibodies can mediate cellular killing, block adhesion of infected RBCs to endothelium, and neutralize parasite toxins to prevent the induction of excessive inflammation. Also, complement can lyse infected RBC. • CD8+ T cells play little role at this stage but CD4+ T cells contribute to activation of macrophages. • NK cells and d T cells are involved at this stage. IFN- , perforins and granzymes produced by NK cells can also kill P. falciparum infected RBCs. Male and female gametocytes in the bloodstream are also thought to be targeted by antibodies. Recent studies have indicated a possible role for polymorphisms in the Fc receptor gamma (FcR ) genes since they have been associated with either susceptibility or resistance to malaria. FcR are present on many immune cells and have several functions particularly in relation to antibodymediated immune mechanisms. Some innate immunity is also induced in the mosquito to the gametes and sporozoites of Plasmodium.

Malaria Immune Escape Mechanisms Plasmodium has a number of “escape mechanisms” that allow it to avoid the immune response. The liver stage: a) In order to invade hepatocytes, the sporozoites have first to pass through KC and endothelial cells. Binding of CSP to KC surface proteins produces high levels of intracellular cAMP that prevents the formation of reactive oxygen species (ROS). b) Sporozoite contact with KC also down-regulates inflammatory Th-1 cytokines

9780367696399_Lydyard.indb 271

and up-regulates anti-inflammatory Th-2 cytokines. c) KC apoptosis may be induced and expression of major histocompatibility complex (MHC)-I reduced. d) Once inside the hepatocyte, the parasitophorous vacuole prevents lysosomal degradation. e) Host heme oxygenase-1 (HO-1) also enhances the development of intrahepatic parasites through modulating the host inflammatory response. The erythrocyte stage: a) The intracellular localization in the RBC escapes direct interaction with the immune cells. b) Lack of MHC-I molecule expression on the surface RBCs also avoids recognition by CD8+ T cells. c) Expression of variable antigenic surface proteins on infected RBCs helps the parasite to evade host immune responses. d) The phagocytic functions of macrophages are also hindered by P. falciparum malaria pigment or hemozoin. Hemozoin (released from ruptured erythrocytes) inside the macrophages reduces phagocytosis of infected RBC as well as reducing the production of radical oxygen intermediates.

PATHOGENESIS There are three major pathologic mechanisms that account for the pathology and clinical signs and symptoms in patients with malaria (see below). These are as follows: • The response of the mononuclear phagocytic system to the parasite leads to release of cytokines from host cells. Cytokines, such as IL-1 and TNF, act as endogenous pyrogens. • Cyclical fever is caused by erythrocyte rupture leading to toxins being released into the bloodstream such as hemozoin. Defective production of red blood cells probably induced by cytokines and toxins including hemozoin and, leads to anemia. Modification of the erythrocyte membrane makes it less deformable and the cells are removed by the spleen, leading to splenomegaly. Additionally, antibody-mediated lysis of the erythrocytes occurs as parasite antigens are expressed on the surface of the erythrocytes. • Obstruction of capillaries is caused by parasitized red blood cells. This is also caused by modification of the erythrocyte membrane as the surface molecules bind to adhesion molecules on the endothelium of the capillaries. This mechanism is particularly important in the pathogenesis of severe P. falciparum malaria such as development of cerebral malaria.

3. WHAT IS THE TYPICAL CLINICAL PRESENTATION AND WHAT COMPLICATIONS CAN OCCUR? In uncomplicated malaria, the fi rst symptoms are fever, headache, chills, vomiting, muscle pains and diarrhea that appear 10–15 days after a person is infected. The preerythrocytic stage in the liver is asymptomatic. If not treated promptly with effective medicines, malaria can cause severe

271

13/02/2023 07:32

Case Studies in Infectious Disease

illness that is often fatal. Patients may also have splenomegaly due to the sequestering of infected erythrocytes and anemia as the result of removal of erythrocytes. With time, the fevers take on a periodicity depending on the species of malarial parasite. Cyclical temperature fluctuations coincide with the rupture of erythrocytes and the release of merozoites into the bloodstream which, during the erythrocytic stage, occurs every 48 hours for P. falciparum, P. ovale, and P. vivax, and every 72 hours for P. malariae (see Figure 28.3). The cyclic temperature fluctuations are due to the release of toxins and hemozoin during erythrocyte rupture and GPI on fragments into the circulation. Through PRRs and other receptors, this induces the release of pyrogenic cytokines including TNF , IL-1β, and IL6 by macrophages that cause rapid elevation of core temperature by acting on the hypothalamus.

COMPLICATIONS In the most severe form of malaria (P. falciparum), there can be cerebral complications – thrombosis may occur due to occlusion of the cerebral vessels caused by the increased stickiness of the erythrocytes (see above). Multi-organ damage and renal failure can also result from the infection. The latter is uncommon in children with severe malaria who present with prostration, respiratory distress, severe anemia, and/or cerebral malaria. Blackwater fever can also occur due to intravascular hemolysis leading to hemoglobinuria and kidney failure. P. falciparum also causes pulmonary edema and P. malariae causes nephrotic syndrome. In patients with splenomegaly, the spleen may rupture due to its large size. The pattern of severe malaria is not fully understood, although genetic factors, age, and intensity of transmission determine susceptibility to severe anemia and cerebral malaria. Other infectious diseases can interact with malaria and modify susceptibility and/or severity of either disease. HIV infection with malaria increases the risk of both uncomplicated and severe malaria, and Plasmodium causes a transient increase in viral load, which might promote transmission of the virus. Plasmodium and Epstein-Barr virus are concurrent risk factors for Burkitt’s lymphoma in Africa.

under the microscope. The degree of parasitemia is noted and the morphologic appearance of the trophozoite (or gametocyte) can be used to identify the species of malaria (see diagnostic stages in Figure 28.2). P. falciparum: ring forms, often resembling stereo headphones, may be seen at the edge of the cell. The red cell is of normal size (Figure 28.4). P. vivax: irregular large thick ring. The red cells are enlarged. P. ovale: regular dense ring. The red cell is oval. P. malariae: dense thick ring or band form. The red cell is normal in size. Blood films may also be stained with acridine orange to iden­ tify the species. 2. The rapid detection test (RDT).

Malaria can be diagnosed serologically by the detection of antibodies or antigen, the latter being used for rapid identification of acute cases. A number of commercial kits are available for rapid diagnosis of P. falciparum. Although many are highly sensitive, some are not as sensitive as a blood film. They detect P. falciparum, histidine-rich protein (HRP), for example, ParaSight F® kits, or lactate dehydrogenase (LDH), for example Kat-Quick® kits. 3. Molecular diagnosis (PCR).

The detection of parasite nucleic acids using polymerase chain reaction (PCR), although slightly more sensitive than smear microscopy, takes longer and is often not available quickly enough to be of value in establishing the diagnosis of malaria infection. PCR is most useful for confirming the species of malarial parasite after the diagnosis has been established by either smear microscopy or RDT.

4. HOW IS THE DISEASE DIAGNOSED, AND WHAT IS THE DIFFERENTIAL DIAGNOSIS? Clinical diagnosis of malaria can be quite difficult because of the overlap of symptoms with other infectious diseases. There are three main diagnostic tests for malaria: 1. Microscopic diagnosis.

272

The gold standard for diagnosis is by a blood film. Thick or thin films are prepared from fi nger-prick blood. These are stained with Giemsa or Fields stain and viewed

9780367696399_Lydyard.indb 272

Figure 28.4 Blood film showing the presence of P. falciparum rings in human erythrocytes (×1125). Note that some red blood cells contain multiple parasites, which is more common with P. falciparum than other Plasmodium spp. From the Centers for Disease Control & Prevention, Atlanta, Georgia. Image is found in the Public Health Image Library #4884. Additional photographic credit is given to Dr Mae Martin who took the photo in 1971.

13/02/2023 07:32

Case 28: Plasmodium spp.

DIFFERENTIAL DIAGNOSIS

• Meningitis or encephalitis: Lower respiratory tract infection: COVID-19; • Influenza and other viral infections such as Epstein-Barr virus or cytomegalovirus; • Gastroenteritis: Urinary tract infection: Lymphoma: Sepsis: Viral hepatitis: HIV seroconversion: Legionellosis: Leptospirosis. Malaria may present with similar symptoms to other travelrelated infections, including: • Viral hemorrhagic fevers such as Lassa fever, CrimeanCongo hemorrhagic fever, Marburg, and Ebola – patients suspected of having a viral hemorrhagic fever require strict isolation. • Enteric fevers such as typhoid or paratyphoid. • Arboviruses such as Dengue, West Nile virus, and Japanese encephalitis. • Rickettsial infection such as scrub typhus and relapsing fever. • Trypanosomiasis. • Rabies.

5. HOW IS THE DISEASE MANAGED AND PREVENTED? Early diagnosis and prompt treatment are the basic elements of malaria control and this is crucial to prevent the development of complications and the majority of deaths from malaria. The two main approaches to malaria control are: • drug treatment of the patients with infection, and • prevention and control of the vector.

MANAGEMENT A ntimalarial treatment policies vary between countries depending on epidemiology of the disease, transmission, patterns of drug resistance, and political and economic contexts. H istorically, chloroquine has been the fi rst-line drug for treating malaria and is still used for P. ovale, P. vivax, and P. malariae except in Indonesia, Sabah, and Papua New Guinea where there is high-level chloroquine resistance. P. falciparum became increasingly resistant to chloroquine and sulfadoxine­ pyrimethamine combinations of drugs were used and are still used. However, there is now significant resistance to these drugs. Currently, the first-line drugs against P. falciparum and adopted by the WHO, are the artemisinin-based combination therapies (ACTs). Artemisinin, first isolated and developed in China in the 1980s from the plant Artemisia annua, has several derivatives including artesunate, artemether, and dihydro-artemisinin. ACTs work by combining a derivative of artemisinin, a fast-acting antimalarial endoperoxide, with

9780367696399_Lydyard.indb 273

a longer-lasting partner drug that continues to reduce the parasite numbers after the artemisinin has dropped below a therapeutic level. ACTs not only act on the asexual blood stages to alleviate symptoms but also on the gametes, therefore reducing the spread of the disease. From April 2019, the WHO recommended artesunate as the first-line treatment of severe malaria. Companion drugs for the artemisinin derivatives include lumefantrine, mefloquine, amodiaquine, sulfadoxine/ pyrimethamine, piperaquine, and chlorproguanil/dapsone. One drug that targets the pre-erythrocytic phase of the disease is primaquine and this is still used to kill the hypnozoites of P. vivax and P. ovale in the liver. The FDAapproved tafenoquine, an anti-plasmodial 8-aminoquinoline derivative is indicated for the radical cure (prevention of relapse) of P. vivax malaria in patients aged 16 years or older who are receiving appropriate antimalarial therapy for acute P. vivax. Sequencing of the parasite genome has aided and will continue to aid in the discovery of new drugs to treat malaria.

PREVENTION AND VECTOR CONTROL Personal prevention measures include the use of a) indoor residual spraying (IRS) of walls in dwelling places using orga nochlorines, pyrethroides, organophosphates, carbamates or neonicotinoids and b) window screens, repellents (such as DEE T) and wearing of light-colored clothes, long pants, and long-sleeved shirts. Insecticide-treated bed nets (ITNs) have been shown to reduce malaria illness, severe disease, and death due to malaria in endemic regions. In several African settings, ITNs have been shown to reduce the death of children under 5 yea rs from all causes by about 20%. Due to the resistance of mosquitos to DDT, only two insecticide classes are approved for net treatment – pyrroles and pyrethroids. These have been show n to pose very low health risks to humans and other mammals but are toxic to insects and kill them. Resistance is, however, already developing against pyrethroides. Nets, to be effect ive, have to be retreated every 6–12 months and now the WHO recommends that long-lasting insecticidal nets (LLINs) be distributed to and used by all people (“universal coverage”) in malarious areas, not only by the most vulnerable groups who are pregnant women and children under 5 years. These nets have been shown to be effective for up to 3 years. I n fection is also prevented by controlling the breeding cycle of the vector, which should significantly reduce the number of cases and rate of parasite infection. Control of the larval stage includes: • Oils applied to the water surface, suffocating the larvae and pupae. Most currently used oils are rapidly biodegraded. • Toxins from the bacterium Bacillus thuringiensis var. israelensis (Bti) can be applied in the same way as chemical insecticides. They are highly specific, affecting only mosquitoes, black flies, and midges.

273

13/02/2023 07:32

Case Studies in Infectious Disease

• Insect-growth regulators such as methroprene are specific to mosquitoes and can be applied in the same way as chemical insecticides. These approaches are generally considered to be environmen­ tally friendly methods of mosquito control.

PREVENTION FOR TRAVELERS TO ENDEMIC AREAS With travel to malaria endemic areas now very common, a number of drugs are given to prevent infection with the parasite, as well as steps taken to avoid mosquito bites. The drugs currently used in prophylaxis vary depending on which area you are travelling to. They include: Atovaquone-proguanil, doxycycline, mefloquine, tafenoquine and primaquine. The patient described in this case had been taking anti­ malarial tablets as a preventive measure against getting malaria. Possibilities as to why she still contracted malaria are: a) the patient was taking homeopathic tablets; b) the patient did not take the drugs on a regular basis; or c) the particular malaria species with which the patient was infected was resistant to the prophylactic treatment. Information as to the appropriate drugs for the area currently to be visited is available at several websites, for example https://www.cdc.gov/malaria/travelers/drugs.html.

VACCINES Development of highly effective and durable vaccines to prevent infection with Plasmodium falciparum and P. vivax

have been a key priority. Strategies for producing vaccines to P. falciparum have been focused on its life cycle. Vaccines include: • whole sporozoite antigens – using whole attenuated sporozoites, • liver-stage vaccines – using thrombospondin-related adhesion protein linked to a multi-epitope string (ME-TRAP) inserted into a chimpanzee adenovirus vector, • blood-stage vaccines – using immunodominant and non-polymorphic merozoite antigens (e.g. P. falciparum reticulocyte-binding protein homolog 5), • transmission-blocking vaccines (targeting the sexual stages to impact the parasite’s life cycle in the mosquito vector aiming to prevent sporozoite development and onward transmission) - using ookinete surface protein Pfs25 and the gametocyte antigens Pfs48/45 and Pfs230. The most extensively tested vaccine candidate for prevention of P. falciparum malaria is RTS,S/AS01 which has been under development for several years. This vaccine directs immune responses against the major circumsporozoite protein (PfCSP) covering the surface of the infecting sporozoite. The WHO is now recommending widespread use of the RTS,S/AS01 (RTS,S) malaria vaccine among children in sub-Saharan Africa and in other regions with moderate to high P. falciparum malaria transmission. It is recommended that this malaria vaccine be given in a schedule of four doses to children from 5 months of age for the reduction of malaria disease and burden.

SUMMARY 1. WHAT IS THE CAUSATIVE AGENT, HOW DOES IT ENTER THE BODY, AND HOW DOES IT SPREAD A) WITHIN THE BODY AND B) FROM PERSON TO PERSON? ■ The organism causing malaria is Plasmodium, a protozoan with a complex life cycle. ■ Five species of Plasmodium are able to infect humans: P. ovale; P. vivax, and P. malariae, P. knowlesi, and P. falciparum being the most virulent species of malaria. ■ The female Anopheles mosquito is the vector for malaria. ■ The sporozoite is transmitted from the mosquito into the blood during a blood meal and localizes in the liver. Schizonts are produced through asexual reproduction. ■ Liver schizonts rupture and release merozoites into the

■ The oocyte matures into sporozoites, which migrate to the salivary gland of the mosquito and are inoculated into another human when the mosquito feeds. ■ The malaria case incidence rate, total malaria cases and deaths fell from between 2000 and 2019. In 2020, there was an estimated 241 million malaria cases. The estimated number of malaria deaths stood at 627000 in 2020. The WHO African Region carries a disproportionately high share of the global malaria burden. In 2020, the region was home to 95% of all malaria cases and 96% of all malaria deaths. Children under 5 are still the most vulnerable group. ■ Asia, Latin America, the Middle East, and parts of Europe are also affected. ■ With increasing international travel, there continues to be a rise in the number of cases of malaria in travelers returning to

bloodstream, which invade and destroy erythrocytes giving rise

non-malarious areas from countries where malaria is endemic.

to symptoms.

However, since the start of the SARS-2 pandemic with reduced

■ Within the erythrocyte, the merozoite undergoes another round of schizogony to produce trophozoites. ■ Differentiation of the trophozoite into gametocytes occurs in some erythrocytes. ■ When another mosquito feeds it takes in the gametocytes, where they fuse to form a zygote, which becomes an oocyte.

global mobility it is likely that travel-associated malaria has decreased. ■ During the SAR-2 pandemic, a recent modeling analysis by the WHO predicted a >20% rise in malaria morbidity and >50% mortality in sub-Saharan Africa as a result of 75% reduction in routine malaria control measures.

Continued...

274

9780367696399_Lydyard.indb 274

13/02/2023 07:32

Case 28: Plasmodium spp.

...continued

■ Pregnancy significantly increases the risk of malaria. ■ Absence of Duffy blood group antigen in West Africans, sickle cell trait, and glucose 6 phosphate dehydrogenase (G6PD) deficiency are examples of genetic factors leading to reduced infection rates for malaria.

2. WHAT IS THE HOST RESPONSE TO THE INFECTION AND WHAT IS THE DISEASE PATHOGENESIS? ■ The parasite uses a number of “escape mechanisms” at both the liver stage and erythrocyte stages to avoid the immune response. These include different life cycle forms and direct immunosuppression by the parasite. ■ Some immunity does develop to the parasite with continuous infection but this is weak and does not eliminate the parasite. ■ Immune mechanisms that exist are different against the different stages of the life cycle. In the pre-erythrocyte phase: antibodies (and complement) against the sporozoites, phagocytosis, NK and cytotoxic T cells against the hepatocytes. Erythrocyte stage: antibodies against the merozoites and gametes. NK and T cells. CD8+ T cells play little role but CD4+T cells contribute to macrophage activation. ■ Pathogenesis occurs due to: (a) cytokine release leading to spikes of fever at regular intervals, (b) destruction of erythrocytes leading to anemia, and (c) obstruction of capillaries due to binding of erythrocytes through parasite encoded surface molecules to endothelium.

3. WHAT IS THE TYPICAL CLINICAL PRESENTATION AND WHAT COMPLICATIONS CAN OCCUR? ■ In uncomplicated malaria, the first symptoms are fever, headache, chills, vomiting, muscle pains, and diarrhea that appear 10–15 days after a person is infected. ■ Patients can develop splenomegaly and anemia. ■ Cyclical temperature fluctuations coincide with the rupture of erythrocytes and the release of merozoites into the bloodstream which, during the erythrocytic stage, occurs every 48 hours for P. falciparum, P. ovale, and P. vivax, and every 72 hours for P. malariae. This gives rise to fevers of differing periodicity. ■ The cyclic temperature fluctuations are due to the release of toxins and hemozoin during erythrocyte rupture and GPI on fragments into the circulation. Through PRRs and other receptors, this induces the release of pyrogenic cytokines including TNF , IL-1 , IL6 by macrophages that cause rapid elevation of core temperature by acting on the hypothalamus. ■ Complications include cerebral malaria where thrombosis may occur (with P. falciparum, severe anemia and blackwater fever). ■ Co-infection with HIV increases the risk of both uncomplicated and severe malaria, and Plasmodium causes a transient increase in viral load, which might promote transmission of the virus. ■ Plasmodium and Epstein-Barr virus are concurrent risk factors for Burkitt’s lymphoma in Africa.

4. HOW IS THE DISEASE DIAGNOSED, AND WHAT IS THE DIFFERENTIAL DIAGNOSIS? ■ There are three diagnostic tests. ■ Microscopic diagnosis: the gold standard is by a blood film; the morphologic appearance of the trophozoite or gametocyte can be used to identify the species. ■ The rapid detection test (RDT): malaria can be diagnosed serologically using the detection of antibodies or antigens; there are kits available. ■ Molecular diagnosis: PCR used to detect parasite nucleic acids; useful to confirm species after microscopic or RDT diagnosis. ■ Clinical diagnosis of malaria can be quite difficult because of the overlap of symptoms with many other infectious diseases, e.g. meningitis, enteric fevers, rickettsial infections, rabies, etc.

5. HOW IS THE DISEASE MANAGED AND PREVENTED? ■ The two main approaches to malaria control are drug treatment of the patients with infection and prevention and control of the vector. ■ Most drug treatments target the erythrocytic stage of the infection. ■ Chloroquine has been the first-line drug for treating malaria and is still in use for P. ovale, P. vivax, and P. malariae. P. falciparum has become increasingly resistant to chloroquine. ■ Drugs currently used against P. falciparum and adopted by WHO are the artemisinin-based combination therapies (ACTs). Companion drugs for the Artemisinin derivatives include lumefantrine, mefloquine, amodiaquine, sulfadoxine/ pyrimethamine, piperaquine, and chlorproguanil/dapsone. ■ Primaquine targets the pre-erythrocytic stage of the disease and is used to kill the hypnozoites of P. ovale and P. vivax in liver cells. ■ ACTs act not only on the asexual blood stages to alleviate symptoms but also on the gametes, therefore reducing the spread of the disease. ■ Sequencing of the parasite genome has aided and will continue to aid in the discovery of new drugs to treat malaria. ■ Personal prevention measures include the use of IRS for spraying internal surfaces and window screens, repellents (such as DEET), and wearing of light-colored clothes, long pants, and long-sleeved shirts. ■ Insecticide-treated bed nets (ITNs) reduce malaria illness, severe disease, and death due to malaria in endemic regions. In several African settings, ITNs have been shown to reduce the death of children under 5 years from all causes by about 20%. Due to the resistance of mosquitos to DDT, only two insecticide classes are now approved for net treatment – pyrroles and pyrethroids. ■ Nets have to be retreated every 6–12 months and now the WHO recommends that long-lasting insecticidal nets (LLINs) be distributed to and used by all people (“universal coverage”) in malarious areas. continued...

275

9780367696399_Lydyard.indb 275

13/02/2023 07:32

Case Studies in Infectious Disease

...continued

■ Mosquito larval control is also used. This is achieved by using oils applied to the water surface, toxins from Bacillus thuringiensis, and insect growth regulators such as methoprene. ■ Prophylactic drugs for international travelers depend on drug resistance in the endemic areas to be visited and include atovaquone-proguanil, doxycycline, mefloquine, tafenoquine, and primaquine.

FURTHER READING Goering R, Dockrell HM, Zuckerman M, Chiodini PL, (eds). Mims’ Medical Microbiology and Immunology, 6th edition. Academic Press/Elsevier, Cambridge, 2018. Male D, Peebles S, Male V. Immunology, 9th edition. Elsevier, Philadelphia, 2020. Murphy K, Weaver C. Janeway’s Immunobiology, 9th edition. Garland Science, New York/London, 2016.

■ There are a number of strategies for producing vaccines to P. falciparum including liver-stage vaccines, blood-stage vaccines, and transmission-blocking vaccines. The WHO have recently recommended the widespread use of a vaccine (RTS,S) against the major circumsporozoite protein (PfCSP) covering the surface of the infecting sporozoite. The vaccine should be used to treat children in sub-Saharan Africa and in other regions with moderate to high P. falciparum malaria transmission.

Milner, DA, Jr. Malaria Pathogenesis. Cold Spring Harb Perspect Med, 8: a025569, 2018. O’Leary, K. A Malaria Vaccine at Last. Nat Med, 27: 2057, 2021. Talapko J, Škrlec I, Alebic' T, et al. Malaria: The Past and the Present. Microorganisms, 7: 179, 2019. Thriemer K, Ley B, von Seidlein L. Towards the Elimination of Plasmodium vivax Malaria: Implementing the Radical Cure. PLoS Med, 18: e1003494, 2021. Tse EG, Korsik M, Todd MH. The Past, Present and Future of Anti-Malarial Medicine. Malar J, 18: 93, 2019.

REFERENCES A miah MA, Ouattara A, Okou DT, et al. Polymorphisms in Fc Gamma Receptors and Susceptibility to Malaria in an Endemic Population. Front Immunol, 11: 561142, 2020. Chu CS, White NJ. The Prevention and Treatment of Plasmodium vivax Malaria. PLoS Med, 18: e1003561, 2021. Cowman AF, Tonkin CJ, Tham W-H, Duraisingh M. The Molecular Basis of Erythrocyte Invasion by Malaria Parasites. Cell Host Microbe, 22: 232–245, 2017. Draper SJ, Sack BK, King CR, et al. Cell Malaria Vaccines: Recent Advances and New Horizons. Host Microbe, 24: 43–56, 2018. Gowda DC, Wu X. Parasite Recognition and Signaling Mechanisms in Innate Immune Responses to Malaria. Front Immunol, 9: 3006, 2018. Jaskiewicz E, Jodłowska M, Kaczmarek R, Zerka A. Erythrocyte Glycophorins as Receptors for Plasmodium merozoites. Parasit Vectors, 12: 317, 2019. Kurtovic L, Reiling L, Opi DH, Beeson JG. Recent Clinical Trials Inform the Future of Malaria Vaccines. Commun Med, 1: 26, 2021. Liu Q, Jing W, Kang L, et al. Trends of the Global, Regional and National Incidence of Malaria in 204 Countries from 1990 to 2019 and Implications for Malaria Prevention. J Travel Med, 28: taab046, 2021.

WEBSITES BMC, Part of Springer Nature, Malaria Journal: http:// www.malariajournal.com/ Centers for Disease Control and Prevention, Parasites – Malaria, 2022: https://www.cdc.gov/parasites/malaria European Centre for Disease Prevention and Control, Malaria, Annual Epidemiological Report for 2018, 2018: https://www.ecdc.europa.eu/sites/default/files/documents/ malaria-annual-epidemiological-report-2018.pdf Map – The Malaria Atlas Project: https://malariaatlas.org/ National Institute for Health and Care Excellence, Malaria, treatment: https://bnf.nice.org.uk/treatment­ summary/malaria-treatment.html World Health Organization, © Copyright World Health Organization (WHO) 2021. All Rights Reserved: http://www. who.int/mediacentre/factsheets/fs094/en/index.html World Health Organization, Malaria, 2022: https://www. who.int/news-room/fact-sheets/detail/malaria Students can test their knowledge of this case study by visiting the Instructor and Student Resources: [www. routledge.com/cw/lydyard] where several multiple choice questions can be found.

276

9780367696399_Lydyard.indb 276

13/02/2023 07:32

29

Respiratory syncytial virus

Figure 29.1 Acute bronchiolitis. Early radiographic appearance – flat diaphragm and hyperlucent lung fields indicate hyperinflation. From Di Nardo M, Perrotta D, Stoppa F et al. (2008) Journal of Medical Case Reports, 212. doi:10.1186/1752-1947-2-212. Published under Creative Commons Attribution License 2.0. A 2-month-old child presented to a doctor with a 2-day history of a febrile upper respiratory tract infection. His clinical condition deteriorated with worsening cough, wheezing and dyspnea, necessitating admission to hospital. On clinical examination in the hospital emergency department, the child had obvious evidence of respiratory distress with severe intercostal retraction and tachypnea (>70 breaths per minute) together with tachycardia. On auscultation of the chest, diffuse high-pitched wheezing was audible and there were fine inspiratory crackles. Pulse oximetry gave an oxygen saturation of 92% on air. Radiologic investigation

1. WHAT IS THE CAUSATIVE AGENT, HOW DOES IT ENTER THE BODY AND HOW DOES IT SPREAD A) WITHIN THE BODY AND B) FROM PERSON TO PERSON? CAUSATIVE AGENT RSV belongs to the paramyxovirus family in the pneumovirus subfamily. It is closely related to the recently discovered human metapneumovirus. Both are enveloped viruses with a single-stranded negative sense RNA genome with a helical nucleocapsid. The envelope contains the virus attachment glycoprotein, G, and the fusion glycoprotein, F.

Figure 29.2 Photomicrograph of respiratory epithelial cells taken from a child infected with respiratory syncytial virus (RSV) and stained with a monoclonal antibody specific for RSV. The antibody is labeled with fluorescein so that patchy (speckled) green fluorescence is observed under illumination with ultraviolet light. Note cytoplasmic staining and binucleate cell (result of RSV-induced cell fusion). This method was the mainstay of direct diagnosis for many years but is labor intensive and expertise and is unsuitable for large numbers of samples. Courtesy of Dr Jeremy A Garson and the Centre for Virology, Department of Infection, Royal Free and University College Medical School, Windeyer Building, London. revealed findings typical of bronchiolitis, with hyperinflation of both lung fields (Figure 29.1). A diagnosis of acute bronchiolitis was made and the child was admitted to hospital for further management. Respiratory syncytial virus (RSV) was detected in nasal secretions (Figure 29.2); the child was nursed in isolation on oxygen therapy and made an uneventful recovery.

The viral RNA-dependent RNA polymerase, an integral part of the virion, has no proof-reading mechanism. RSV evolves quickly by point mutations that allow for changes in viral virulence and avoidance of the immune response. The G protein is the most variable of the structural proteins, and the resultant antigenic differences form the basis of the two major groups of RSV strains, A and B, which circulate concurrently. At any one time, there may be multiple RSV variants within a population. In contrast, the fusion protein is highly conserved.

ENTRY AND SPREAD WITHIN THE BODY The RSV G protein is the primary mediator of attachment to the glycocalyx of ciliated airway epithelial cells and type 1 alveolar pneumocytes but the F protein can also facilitate 277

9780367696399_Lydyard.indb 277

13/02/2023 07:32

Case Studies in Infectious Disease

viral attachment although to a lesser extent than G. Virus attachment initiates a conformational change in F protein so that a fusion peptide is revealed and fusion of the closely juxtaposed viral and cell membranes ensues. Subsequently, the viral nucleocapsid is released into the cell’s cytoplasm and the viral RNA-dependent RNA polymerase, initiates virus replication with the synthesis of viral messenger RNA. Virus progeny from the first infected cell then spread to neighboring cells often involving cell-to-cell fusion mediated by the F protein to form syncytia, also known as multinucleate giant cells (Figures 29.3 and 29.4 – hence the name of the virus). Virus shed from the apical surface of infected cells infects more distal mucosal cells via respiratory secretions. In infants and the elderly, the infection is not limited to the upper respiratory tract and may involve the trachea, bronchi, bronchioles, and alveoli. In all cases, the incubation period is short, being between 2 and 8 days, since the infection is restricted to respiratory mucosa and does not become systemic.

PERSON-TO-PERSON SPREAD RSV infection is highly contagious, being shed in respiratory secretions for several days. Large droplets arising from coughs and sneezes of an ill person may transmit RSV to others within a radius of about three feet. Longer-distance spread by small-particle (droplet nuclei) aerosols seems to be much less likely. Touching objects (fomites) contaminated with infected secretions followed by self-inoculation into the eyes or nose, is an important mode of transmission. Virus in infected secretions is viable for up to 6 hours on nonporous surfaces, up to 45 minutes on cloth, and up to 20 minutes on skin.

EPIDEMIOLOGY RSV is responsible for about 80% of cases of bronchiolitis and is a leading cause of hospitalizations due to acute lower

278

Figure 29.3 Photomicrograph of RSV-infected respiratory epithelial cells as seen in tissue culture. Cultured respiratory epithelial cells on day 1 (A) and day 3 (B) following infection with the virus. After one day, the cells continue to appear healthy. After three days, many of the cells have fused, forming large syncytia (arrows), that is a mass of cytoplasm containing several separate nuclei enclosed in a continuous membrane. Magnification: ×400. From Domachowske JB & Rosenberg HF (1999) Clinical Microbiology Reviews 12. doi:10.1128/CMR.12.2.298. With permission from the American Society for Microbiology.

respiratory infection in infants and young children